You are on page 1of 333

$'edfidft'il

dv
ai,..Ifg.Nuffi$;,s,ydrc .,,',
,;'1,,'
t; tffpes'ofnfissues
."t'l'tr;'Mefi$ biil tt"ii.-"t,''
;. "" : ";
Ndw6
r;:
,:' i.'
1
5;';Atttoa'PtteniHs 1.
.
".,
4;,. The, Ne,ufomuSeillai.Iunetioni I l
ff 5:'J E $lt "' ;=,
fipt€1"',,;,,r,:,.'.",:
o*po*ng,'.,
us€ ,6;,ffi#bns:sy
i';c nti,btff Bb qiffi5r.,;i', 1lr,.. I

8..&eeeptCI. d'gen$orX,U gt,ri'

Pd
::: : : l

cCi FACCagCs er nnswet$l'

m
e.'.
R:;i E I M' *tI 'Ti'
Speciali#lH l{ ' ffi,;P#paration
Biotogy Nerve & Muscle Types Of Tissues

Types Of Tissues
Let's consider a series of discussions on cellular physiology. For example, we will
consider how muscle and nerve cells function. How does the chemical energy of
ATP (which was generated in glycoiysis, the Krebs cvcle, and oxidative
phosphorylation) become converted into the mechanical movement of, say,
muscle cells? How is it that the chemical energy of ATP is converted into an
electrical signal that al1ows various nerves to communicate with those muscles?

Before we can discuss the cellular mechanisms of muscles and nerves, we first
need to consider some of the general characteristics of cells, tissues, and organs.
The general body plan of an animal is fairly simple and can be divided into a
number of systems that represent a variety of organs working in concert with one
another. For example, one body system you are probably quite familiar with is
the skeletal system. Another is the muscular system. Others are the circulatory,
integumentary (skin), endocrine, nervous, and digestive systems, to name but a
few.

The digestive system is formed by an alimentary canal (gastrointestinal "tube")


that begins at the mouth and ends at the anus. This system is suspended within a
body cavity re-ferred to as the coelom. The coelom is separated into a thoracic
cavity (upper) and an abdominal cavity (lower). These two cavities are separated
by the dome-shaped mass of skeletal muscle called the diaphragm. Within the
thoracic cavity, one finds the lungs and the heart. The abdominal cavity contains
the liver, stomach, and iatestines.

As we examine l"hese various systems, we will find different levels of


organization. There are individual cells, and then there are cells of a particular
type which coalesce to form tissue. One example of a tissue is the layer of
epithelial cells that line one of the principal organs of the alimentary canal, the
stomach. some of the simple epithelial cells within the siomach secrete
hydrochloric acid (pH = 1) to aid in the digestion of food. Other epithelial cells of
the stomach secrete mucus to help prevent that acid from digesting the lining of
the stomach. Still other epithelial cells secrete enzymes. These epithelial cells are
just one type of tissue that is involved in forming the stomach. The stomach is
also composed of other types of tissue. For example, nervous tissue helps to
innervate the stomach, connective tissue helps to hold the stomach in its proper
position, and muscle tissue helps to propel food through the stomach. Thus,
these four groups of primary tissue (epitheliai, connective, muscle, and nerve)
have the abiiity to form the various organs of the body. An organ is n structure
that is composed of two 0r ffLlre tissues that act in such a way as to perform a specific
.function.

Epithelial Tissues
Let's examine the epithelial ce11s in a little more detail. The epithelial tissue that
constitutes the various organs of the body can be either simple epithelium
(consisting of a single layer of cells) or stratified epithelium (consisting of two or
more layers of cells). These epithelial celtrs come in a variety of shapes and sizes.
For example, there are squamous (flat), cuboidal, and columnar epithelial cells
(refer to Figure 1-1).

Copyright @ by The Berkeley Review i' The Berkeley Review


Specializing in MCAT Preparation
Biology Nerve & Muscle Types Of Tissues

On the lumenal side of the simple epithelial cells are projections cailed
microvilli (singular, microviilus--see Figure 1-1 and Figure 1-2). These
projections increase the total absorptive area of the cell (sometimes by as much as
25%). Sometimes you find specialized structures cailed cilia (singular, cilium)
projecting outward on the apical surface of these cells. For example, in the
respiratory tract these hair-like appendages move in a coordinated unidirectional
wave to move foreign particles out of the mucous lining of the lungs and
bronchial tubes.

Simple squamous
epithelial cel1s Simple columnar
epithelial cells

\-J Basal .--------\


lamina
Cuboidal and columnar Stratified squamous
epithelial cells epithelial cells (non-keratinized)

Figure l - l
Types of epithelial cells.

Thebe cells are !_l-undea by a number of specialized junctions. For example, tight
junctionS aet-fs a permeability barrier (see Figure 1-2). Not oniy do they prevent
the transport of protein molecules from the lumenal side of the cell towards the
basolaterai side of the cell, but they also act to hold neighboring cells together.

Epithelial cells are also held together by structures called desmosomes (see
Figure 1-2) One type of desmosome joins the epithelial cell to a structure on the
basal side of the ceil called the basal lamina (or basement membrane). The basal
iamina is in close contact with connective tissue that helps to anchor the cells in
place.

Gap junctions provide a means for water-soluble molecules to pass from the
cytoplasm of one cel1 to the cytoplasm of another cell (see Figure 1-2). These
lunclions allow for equilibration within the connected epithelial celis and
therefore allow those celis to function as a unit. For example, the beating cilia
appear to be coordinated by waves of calcium, which flow in the plane of the
juxtaposed epithelial cells.

Copyright @ by The BerkeleY Review 4 The BerkeleY Keview


Specializing in MCAT Preparation
Biology Nerve & Muscle Types Of Tissues

LUMEN
Apical surface

€ Microvillus

(=l Tightjunctions
Tight junctions

Gap
.lunctlon
Desmosome
Desmosomes
Basal lateral C)
surface
€f Basal iamina
(basement membrane)
BLOOD
Figure 1.2
Different components of an epithelial cell.

As we have mentioned, epithelial cells can secrete substances into a lumenal


space. For example, hydrochloric acid can be secreted into the lumen of the
stomach. If a cell secretes a substance into the lumen by way of a duct, it is
referred to as an exocrine gland. Endocrine glands secrete substances into the
blood. For example, insulin is a protein hormone secreted into the blood by
clusters of specialized epithelial cells in the pancreas.

Dead skin cells


&r
\t-
(keratinizedl

I
I noio..-ur
f cells

Basement
)
membrane
] conn..,,u.
tissue
Collagen fibers t) J
Figure 1.3
Stratified squamous epithelial ce1ls.

Stratified squamous epithelium usualiy has a protective function. Your skin is


composed of many layers of stratified squamous epithelial cells. The outer cells
of your skin are dead, and they contain a large amount of the fibrous protein
keratin (Figure 1-3). These cells are constantly being lost and replaced, as cells
begin to move toward the surface from beiow.

Copyright @ by The Berkeley Review The Berkeley Keview


Specializing in MCAT Preparation
Biology Nerve Er Muscle Types Of fissues

Consider a segment of skin. This organ comprises about 15o,i, of r-our total body
weight. The epidermal region contains stratified epithelial cells that act to
protect the deeper layers of the skin. Belou'the epidermis is the dermis. Within
the dermis are a variety of structures. Surrounding the hair follicles are erector
muscles, which act to straighten the hair shaft. This causes the skrn close to the
hair follicle to become depressed and gives the characteristic appearance of
"goose pimples." Those erector muscles are innervated biz nerves which cause
them to contract at specific times (e.g., when it is cold outside). The skin is also a
highiy vascularized organ. When it is hot outside, the blood is shunted towards
the surface of the skin where it can dissipate some of its heat to the outside
environment. Below the dermis is the subcutaneous tissue. This is where one
finds adipose deposits.

Connective fissues
This type of tissue helps to anchor and support the various structures of the
body. There are a variety of types of connective tissues, a few of which are
structural, blood cells, mast cells, adipose cells, and melanocytes.

Many of the proteins that make up structural connective tissue are secreted by
cells cailed fibroblasts. Collagen, reticulirl and elastin are structural proteins
which are secreted by these cells. Collagen is a triple-stranded, insoluble, fibrous
protein (see Figure 1-3) that is highly ooss-linked, a feature that makes these
fibers quite strong and rather flexible. Besides having a very high tensile
strength, collagen is also the most abundant protein found in mammals.
Reticulin is a thin fiber found in the spleen and lymph nodes. It is not as highly
coiled as collagen. Elastin is also a highly cross-linked protein found associated
with organs that require some degree of elasticity (like the lungs, skin, and blood
vesseis).

Another type of structural connective tissue, cartilage, is secreted by a


specialized fibrobiast cell called a chondrocyte. There are different types of
cartilage, but in general it is found in places where there is a certain amount of
stress placed on the body. For example, cartilage can be found in the nose, on the
articulating surfaces of bones (including the intervertebral discs of the vertebrai
column), and in the external ear.

Bone is also a structural connective tissue. About one-third of the weight of bone
comes from organic materiai such as collagen, while the remaining two-thirds is
inorganic material such as calcium phosphate and calcium carbonate. The
collagen found in bone matrix is secreted by specialized fibroblast cells called
osteoblasts. Collagen lends flexibility to bone, while the inorganic crystals lend
rigidity. Within the centrai cavity of bone, we find a spongy marrow where red
blood cells and white blood cells are formed. Towards the surface of bone the
ceilular arrangement is more compact. [As a comparison, the main structural
component of chitin (found in the exoskeleton of insects) consists of specially
modified glucose residues linked to one another to form long polymers.
Associated with these polymers is calcium carbonate (CaCO3). This combrnation
adds rigidity to the exoskeleton, but offers little in the way of flexibility.l

We mentioned that blood cells and mast cells are kinds of connective tissue. We
will discuss blood cells in a separate lecture. Mast cells can be found in the
respiratory tract, as well as in the gastrointestinal tract. Mast cells can release

Copyright @ by The Berkeley Review 6 The Berkeley Review


Specializing in MCAT Preparation
Biology Nerve 8r Muscle TYpes Of Tissues

histamines in response to an allergic reaction, an infection, or even an injury.


Histamine causes an increase in blood flow to the blood vessels of the affected
region.

Other types of connective tissue involve adipose cells and melanocytes. Adipose
cells are simply ceils that store fat whereas melanocytes are cells which store
pigments.

Muscle fissues
We will be discussing various types of muscie in future lectures. For example,
r,vhen we examine skeletal muscle we will find that it is voluntary muscle. That
is, we can generally control its action. Cardiac muscle and smooth muscle are
examples of involuntary muscles.

Nervous fissues
The nervous systems allow one to adapt rather quickly to external stimuli. For
example, consider a simple reflex arc. If someone were to tap on your knee with a
rubber hammer, your lower leg would extend outward. As the hammer
impinged upon the patellar tendon in your knee, an eiectrical impuise was
generated and traveled via a sensory nerve to your spinal cord. That sensory
neuron svnapsed with a motor neuron, which returned the impulse to the
muscle that was initialiy stimulated and caused it to contract. We will come back
io this exampie and examine it in a bit more detail later. First, let's consider some
terrninology.

Dendrites Neurotransmitters
are released from
sYnaPtic Uulbs --i

A Typical Neuron

Ceil body

Figure I-4
1r :nrjor components of a neuron.

\en-e cellsand associated supporting cells make up the newous system. Nerve
-s
:= are also called neurons, and ihey are the basic structural unit that make up
--:r: r.ervous system. The major anatomical features of a neuron are the cell body
-:.-,-oh'ed in integration of information), the dendrites (involved in receiving and
::a:.smitting information towards the cel1 body), and the axon (involved in
:,:.ducting information away from the cell body). When a neuron becomes
:\.-ri€d and receives electrical information in the form of a stimulus, the celi body
::it.esses that information and transmits it down the axon in the form of a nerve
rnpuise called an action potential. When that action potential reaches the end*o{
-j-.. a\on (referred to as the synaptic bulb or bouton terminal), it causes the

,,::" nght O by The Berkeley Review The BerkeleY Review


Specializing in MCAT Preparation
Biology Nerve & Muscle Types OfTissues

release of a chemical substance called a neurotransmitter (see Figure 1-4). The


neurotransmitter diffuses across the synaptic cleft and induces an identical
action potential in an adjoining neuron, muscle cell, or gland cell. The junction
between two such cells is called a synapse.

Copyright @ by The Berkeley Review a The Berkeley Keview


Specializing in MCAT Preparation
Biology Nerve & Muscle Membrane Potentials

The generation of electrical signals in the nervous system is concerned with the
diffusion of ions from a high to a low concentration (see Figure 1-5). in other
words, charged ions diffuse down their concentration gradient. In the
extracellular space of vertebrates the concentration of Nao is about 150 mM,
while that of Ko is about 5 mM. The concentration of Cle is about 130 mM,
while ihat of HCO3e is about 25 mM. Note that the concentrations of the cations
(Nae and KCI) and the anions (Cl€ and HCO3O) balance one another. In other
words, we find 155 mM of the cations and 155 mM of the anions. This represents
electroneutrality.

Inside Cell Outside Cell

K* = 120 mM to 140 mM K+=5mM

Na* = 10 mM to 15 mM ]..,,"", Na* = 150 mM

Cl-=5mMto40mM Cl- = 130 mM

HCO3- = 12 mM to 25 mM HCO3- = 25 mM

Proteins
-
l^","",

Figure I -5
I. :.::r ceiiular concentrations of the common ions.

Ke (about 120 mM to 140 mM)


r--r'n the cel1, we find a high concentration of
"
=,:. a lorv concentration of Nao (about 10 mM to 15 mM). We also find a lower
: -:.::nfation of Cle (about 5 mM to 40 mN! and usually a lower concentration
: :-rCO3 3 (about 12 mM to 25 mM). There are many negativeiy charged proteins
,' :--:ir. the cell. Electroneutrality can also be found on the inside of the cell as
: -.

: the distribution of ions across the cell's membrane, you will find it
. - .. -ook at
- r: :.::.-mrnetri.cal. Let's consider a resting nerve (i.e., a nerve that is waiting to
:: .=:;-:: an action potential) that has a permeabiiity to potassium which is much
.r:-:i:r :han its permeabiiity to sodium. In other words, PKe >>> PNao (where
F :=:::s to permeability). Because the concentration of Ke is higher inside the
,: --r.;r: outside the cell, potassium will diffuse down its concentration gradient
.: : -.:'" e 'Jee ceil (see Figure 1-6). As the positive Ko cation leaves the cell, there
: -::::>:cndrngiy less positive charge remaining inside the cell. In other words,
-- = ,: s:ie oi the cell is now more negatively charged with respect to the outside
. ---- ::--. This sets up a voltage that is positive on the side of the celi to which
::::::-'':. is trving to diffuse to (i.e., the outside of the celi). As that positive
:-,:i- ::gins to build up, it tends to push potassium back into the cell
:;::::,:.:. irke charges repel each other).

I The Berkeley Keview


Specializing in MCAT Preparation
Biology Nerve & Fluscle Membrane Potentials

Those tn'o forces (chemical and electrical) do not exactly match each other. It
lums out that the force of diffusion is a little larger than the electrical force. This
results in a little bit of leakage of Ko out of the cell, as well as a little bit of
Ieakage of Nao into the cell. The Ke that leaked out of the cell has to be pumped
back into the cell, and the Nao that leaked into the cell has to be pumped out of
the cell. The pumping action of these two ions is provided for by the Nao/K@
ATPase pump. This pump is responsible for the generation of the asymmetrical
concentration gradient of Nao and Ke across the cell's membrane.

Cell
membrane

Diffusion
Inside Outside
cell cell
Where
Na+ P* ))) P"u
-87mV + 87 mV and the
Na* anions are
impermeable

Figure l-6
Cellular gradients where PK >>> PNa.

We can calculate the membrane voltage (the potential difference) across the cell's
membrane using the Nernst equation as shown in (L-1). In this equation, V is the
voltage in miilivolts (1 mV = 10-3 volts), i refers to inside, o refers to outside, R is
the gas constant, T is the temperature in Kelvin, Z is the ion's valance, and F is
the Faraday constant. If we let the cell's membrane be permeabie to just Ko, we
find that the voitage is '87 mV inside the cell with respect to outside the cell.
Remember, this is if potassium is the only permeable ion. It is ihe gradient of
potassium alone across the cell's membrane that is able to generate this potential.

V'^ = 2.3 RL ,on [K*]o


"
ZF [K*]r (t-1)

Vio = 60 log [5 mM]" (1-2)


[140 mM]r

Vio=-87mV (1-3)

Copyright @ by The Berkeley Review lo The Berkeley Review


Specializing in MCAT Preparation
Biology Nerve & Muscle Membrane Potentials

If we stimulate a nerve, it leaves its resting state and enters an active state in
r,r'hich the cell's membrane is more permeable to sodium (Nuo) than it is to
potassiurn. In other words, PNa >> PK. Since the concentration of sodium is
higher outside the ce1l than inside the cel1, sodium will tend to diffuse down its
concentration gradient and into the cell.

Once again, we can establish a separation of charge. As the Nao ions enter the
ce11 ivith their positive charges, there is that much iess positive charge outside the
.e11. The outside of the cell becomes more negativeiy charged than it was before
\a3 started to diffuse into the cell. Similarly, as the Nao ions diffuse into the
-e11, the inside of the celi begir-rs to accumulate more positive charge. The inside
:- rhe cell becomes more positively charged than it was before the Nao ions
:t.:ied to diffuse in (see Figure 1-7). As the Nao ions diffuse into the cell down
--:-:ir concentration gradient, a chemical and an electrical equilibrium is being
=r:a:rished.

Inside
cell
Where Outside
P*, )) F cell

" Na+
Voltage

:x,qLu'e l. /
: . - -r :-:- :1::. 'ihere P\a >> PK

: -:-- -:-,:-:ie:he magnitude of the potential across the ceil's membrane by


==..- €equation. We find that the potential difference is +60 mV (inside
-- - - : I\-:llL,<l
.
----r.
r :e::eci to outside the cell)
: -

[Na*]o
[Na*]i (1-4)

11 i 150 mMlo (1-s)


[15 n-A4]i

(1-6)

I -:: . f:.: B:rielev Review lt The Berkeley Review


Specializing in MCAT Preparation
Biology Nerve & Muscle Membrane Potentials

suppose we were to measure the potentiar across the plasma membrane of a


neuron (nerve cell). We can do this by using two electroi"es. A microelectrode is
inserted into the neuron itself while another electrode is placed in the fluid that
surrounds the neuron (see Figure 1-g). If we connect these two electrodes to a
voltmeter that can read the potential difference between the two environments,
we will find that the inside of neuron registers about -g0 mv (with respect to the
extracellular space). Note that this voltage is quite close to tire voltage that we
would get (-87 mv) if this plasma membrane were permeable only to ro. Ttu
reason that the inside of the neuron is not -gr rny is because the plasma
membrane is not exclusively permeable to K@.

Figure l-B
Measuring electrical potential across a plasma membrane.

Copyright @ by The Berkeley Review t2 The Berkeley Review


Specializing in MCAT Preparation
Biology Nerve & Muscle Action Potentials

We can reduce the plasma membrane potential by transiently increasing the


membrane's permeability to Nao. This can be accomplished by stimulating the
nerve. The transient reduction in the membrane potential is referred to as a
depolarization of the membrane. If this stimulus is strong enough and the initial
depolarization exceeds a specific minimum value characteristic of the cell (called
the threshold potential), then within milliseconds a burst of Nao ions lr'iil enter
the cel1 and generate an action potential (see Figure 1-9). As the depolarization
of the membrane continues, its permeability to Nao becomes much greater than
its permeability to Ko (i.e., PNa >> PK). What we find is that the membrane
potential goes from a resting membrane potential of about -80 mV to a
membrane potential that approaches that for Na@. In this example, at the height
of the wave of depolarization, the membrane potential would be about +40 mV.
[Why isn t the membrane potential +60 mV as would be calculated using
equation (1-4)?l

+60 Na+ equilibrium potentiai

+40 (A) Stimulus


,.I {BtDepolarization * Px >> PNo
-=
.- (C t Repolarization
't *
PNr>PK
(D) Hyperpolarization
t)
(E)Relractory

=
! Threshold potential

- 8t) Resting potential


-z (E)
Q7
K+ equilibrium potentiai

Tirne (msec)

Figure I -9
- :-:.. .ction potential.
._"

-: :- :'.e membrane potential reaches the equilibrium potential for Nao there is
r- , ::-:::e influx of Nao ions into the celi. At this point (indicated by the peak of
-,- : ,'.::,... poiential) the membrane potential, which is +40 mV inside the cell, is

:: ,-.-=j r\ the concentration gradient of Nae. Recall that the concentration of


,, = - ':=r,je the cell is greater than the concentration of Nao inside the cell.

,. ::.-,-rsecond after the burst of Nao into the cell, the ion channels
-::phorest that let Na@ into the celi close and become temporarily inactive.
-:: -:,- \a3 channels are temporarily inactive they are said to be in a
_:z':r::rin' period, which usually lasts for several milliseconds. During this time a
-:*r-r n-il1 not be able to generate another action potential, because the Nao
-" .::.-. ::-nnot open to allow the cel1 membrane to depoiarize.

The Berkeley Review t3 The Berkeley Review


Specializing in MCAT Preparation
Biology Nerve 6r Muscle Action Potentials

once the Nao channers have closed, the Ke channels begin to open
and Ko
begins to leave the cell. This phase of the action potent"ial is
referred to as
repolarization. As the ceil membrane's permeability to ro increases,
a massive
amount of Ko flows out of the cell and the membrane potential
passes its origrnal
resting state of about -80 mv. This event is cailed hyperpolarization
(see Filure
1-9). Eventually the ce1l wilr reach equilibrium, u.,i'o.,i"
again an appropiiate
stimulus will generate an action potential.

The generation of an action potential is said to be an all-or-none


phenomenon.
What does this mean? If.we apply a stimulus to a nerve (see point (A)
in Figure
1-9), the plasma membrane of the neuron wil become depolarizea.
membrane potential will become less negatively charged during
ine
the increase in
the permeability of the plasma membrane to Nab io.,.]R"*"rnber,
Nao wants to
flow down its concentration gradient and into the cell. As Na@
enters the cell, the
membrane becomes more deporafized.. This increase in
depolarization tends to
induce neighboring Nao channels to open, thus retting
more Nae ions into the
cell' However, we know that the celr is dynamic rathe"r than
static; Ko ions are
leaving the celI and flowing down their concentration
gradient. This action has a
tendency to repolarize the prasma membrane. In other-words,
the generation of
an action potential depends on the ratio of membrane
permeabilitlito these two
ions. If there is not a sufficient depolarization (i.e., influx
of Nab ions) of the
membrane, then there will be no action potential. Flowever,
if there is a sufficient
depolarization of the membrane because there are more
Nao ions entering the
cellthan Ko ions leaving the ceil, then an action potential
will be generated if that
depolarization exceeds some threshold var'e. once an action
potentiar is
generated in a nerve cell it witt always haae the same magnitude.
Thus, an action
potential is either all-or none. rt either happens or it doeJn't.

In Figure 1-9 we have indicated how an action potentiar is generated. we


mentioned that in order to generate an action potential there
must be a
depolarization of the membrane above some threshoid value,
a reporatization of
the membrane, a hyperpolarization, and a refractory p"rioa
that ailows the
membrane potential to return to its resting value before a new
action potentiai
can be generated. Now let's examine how an action potential
propagates arong a
neIVe.

Consider the charge separation across the plasma membrane (see Figure
1-10) of
a nerve as an action potential moves from, say, the center
of the axon outwards.
Suppose we have a stimulus that exceeds a given threshold value.
As the action
potential is generated a separation of charge will be established. Because
Nae
ions are rapidly entering the cell, the inside of the cell will become
more
positively charged (+) with respect to the outside of the cell. Since there
is a
deficit of positive charge on the outside of the cell (compared to the original
resting condition), we say that the outside of the cell is io." negati"vely
charged (-) with respect to the inside of the cell. In other words, -or"
at thai pirticuta.
piace on the axon, where the action potential has been generated,
there is a
reversai in the membrane's polarity. This is shown in Figure"l-10a.

The depolarization of_the central region of an axon spreads outward in


both
directions as more Nae ions enter the cell. In Figure 1-10a the arrows represent
a
locai flow of current between the area of the
flur*u membrane that has been
depolarized and the adjacent areas of the membrane which are still at their

Copyright @ by The Berkeley Review t4 The Berkeley Keview


Specializing in MCAT preparation
Biology Nerve & Muscle Action Potentials

resting potential. The flow of positive charge within the ceil causes the adjacent
areas to become more depolarized.In other words, the polarity of the membrane
is being decreased in front of the propagating action potentials. Because the
membrane potential in these adjacent areas is becoming depolarized, more Na@
channels begin to open. This lets more Nae into the ce1l, which increases the
depoiarization until an action potential is generated.

+ + + + + + + '_j*t_j +++++++ +

Propagation f'l = Na+ Channels


of
Action
Potential
(b)
+++ ++++++++
Axon
+++ ++++++++

Figure l-lO
iction potential propagation.

\ote that as the action potentials spread outward, they create a region which is
refractory (see Figure 1-10b). Recali that the refractory period is due to
nactivation of the Nao channels. If the plasma membrane has been depoiarized
rom an action potential, then another action potential cannot be generated until
rat membrane has once again reached its resting membrane potential.
)urrng the state of hyperpol arization, the K@ channels are wide open and K@ is
:orrriig out of tne iett. Even though an action potential could be generated
,."urrng this time (of hyperpolarization), it is rather difficult unless a strong
..o.rgh stimulus is applied. Also, the action potentials in Figure 1-10b appear
:ackiards compared to the action potential in Figure 1-9, because the action
: otential in Figure 1-9 shows a fixed point on the axon and the membrane's
.ot".,trul variation with time at that point. The action potentials in Figure 1-10b
:ilo\\' a distribution of the membrane potential along an axon (not at a fixed
:ont) at a particular instant in time.

lirere are a many kinds of nerve fibers in the body, and they do not all conduct
:-::ion potentiali at the same rate. For example, large diameter neurons will

Jcrl'rrght O by The BerkeleY Review l5 The BerkeleY Keview


Specializing in MCAT PreParation
Biology Nerve & Muscle Action Potentials

conduct a depolarization or action potential further and faster than a small


diameter neuron. \Alhat this means is that the ability of a given neuron to conduct
a current (the flow of charge represents a current) depends on the cross-sectional
area of that neuron. The larger the cross-sectional area of a neuron, the more
cytosol il that neuron, and the more ions there will be to conduct that current.
The more ions available to conduct the current, the faster and farther that current
will spread.

The nerves which rve harre been discussing so far are referred to as
unmyelinated ner-"'es. Myelinated nerves have the ability to greatly increase the
rate at which action potentials are conducted. Myelin is a specialized membrane
that is composed of just a few types of proteins and a large proportion of
phospholipids. A specialized type of cell (called a glial cell) attaches itself to a
section of an unmyelinated axon and begins to rotate itself around that axon a
number of times. During this process a myelin sheath is deposited on the axon.
Sometimes this myelin sheath is thicker than the axon itself. The deposition of
myelin along the axon is interrupted by areas where there is t-ro
areas are called the nodes of Ranvier. See Figure 1-11.
-yeiitl. These

TJ:ln.'*1'l t\
+JL
Axon
\_-J

Plasma membrane Node of


(contains a high density of Na+ channels) R.anvier

Figure l- I I
Action potential propagation lrom node to node.

Later we wiii learn that the body's nervous system is separated into two divisions
(based on function). The central neryous system is macie up of the brain and
spinal cord whiie the peripheral nervous system represents the nerves in the
periphery. The point here is that the glia1 ce11s in the centrai nervous system are
called oligodendrocytes, while the gliai cel1s in the peripheral nervous system
are ca1led Schwann cells.

We mentioned that myelinated nerves increase the rate at which action potentiais
are conducied. The myelin that surrounds the axon acts as an electrical irsulator
and prevents the transfer of ions across the plasma membrane of the axon. The
only region of the axon where ions can pass across the plasma membrane is at
the nodes of Ranvier. These nodes contain a high density of Nao channels. As an
action potential is generated, there is a flow of current through the cytopiasm of
the axon (called the axoplasm) and the extracellular fluid from node to node. The
plasma .membrane at each of these nodes rs in turn depolarized enough to

Copyright @ by The Berkeley Review l6 The Berkeley Review


Specializing in MCAT Preparation
Biology Nerve 6e Muscle Action Potentials

generate an action potential. The nerve impulse seems to "jump" from node to
node along the axon. Thus, this type of impulse transmission is referred to as
saltatory conduction (from the Latin saltatorius--pertaining to dancing or
Ieaping). See Figure 1-11.

Nerve fibers can be grouped into different classes. The nerve fibers in some of
these classes are myelinated, whiie those in other classes are unmyelinated.
Some of the myelinated fibers that impinge on skeletal muscle can transmit
motor impulses quite rapidly, about 60 to 120 m/sec. In contrast, some of the
r:nmyelinated fibers which transmit sensory impulses of certain types of pain, do
so rather slowly (relative to a myelinated fiber), about 0.6 to 2 m/sec. As you can
see, myelinated nerve fibers greatly increase the rate at which information, in the
form of action potentials, is conducted.

So far, we have been considering the propagation of an action potential within a


single nerve cell. Let's see how this information is transmitted to other cells.

Cryyrlett @ by The Berkeiey Review t7 The BerkeleY Keview


Specializing in MCAT Preparation
Biology Nerve Et Muscle The Neuromuscular Junction

, tt |ll ffiuscunffffi ,,!,'u;,.''


At the end of the axon is a specialized area called the terminal bouton (or
syraptic bulb, synaptic knob, or even terminai foot). This specialized end region
of the axon can impinge or synapse on another axon of a different neuron, on a
dendrite, or even on some type of cell body (like a muscle ce11). The space
between this synaptic junction is called the synaptic cleft. The plasma membrane
at the terminal bouton is calied the pre-synaptic membrane while the piasma
membrane that the synapse is being made to is called the post-synaptic
membrane. See Figure 1-12.

Presynaptic Synaptic cleft Postsynaptic


membrane memhrane

-@\ ACh
ACh receptor
ACh complex

rcs +@
@,
X
aAC])
\

2 @
Terminal
bouton

Acetlycholinestrase

Figure I - l2
Presynaptic and postsynaptic membranes.

' Let's consider a neuromuscular junction. This is the synapse between the
terminal bouton of an axon and a muscle fiber. Within the terminal bouton of this
junction are hundreds of thousands of synaptic vesicles that contain the
excitatory neurotransmitter acetylcholine (abbreviated ACh). There may be
thousands of molecules of acetylcholine in each synaptic vesicle. Acetylcholine
itseif is synthesized in the cytosol of the neuron from the molecules acetyl-CoA
(which caries an acetate moiety) and choline.

As the action potential reaches the terminal bouton, it triggers the openrng of
calcium (Ca2o)channels" A transient influx of Ca2e into the terminai region
causes the synaptic vesicles to fuse with the presynaptic membrane and release
their neurotransmitters into the synaptic cieft (via exocytosis). Calcium is
pumped out of the cytosol and back into the extraceilular fluid. The membranes
of the synaptic vesicies that fused with the presynaptic membrane are recycled

Copyright O by The Berkeley Review la The Berkeley Keview


Specializing in MCAT Preparation
Biology Nerve & Muscle The Neuromuscular Junction

(via endocytosis). The released neurotransmitters diffuse ihrough the synaptic


cleft and bind to specific postsynaptic membrane receptors. Binding of
acetylcholine to this postsynaptic membrane receptor conformationally changes
that receptor into a channel (ionophore) that is large enough to allow cations
such as Nae through. [Note that this channel is not a voltage-activated channel
like the channels we examined in the axon itself. This channel is a ligand-
aciivated channel. In this case, the ligand (a molecule or ion bound to a protern)
rs acetylcholine.] As Nas enters the postsynaptic membrane, the muscle fiber
begins to depolarize and an action potential is eventualiy generated.

If acetylcholine were to remain in the synaptic cleft, then it would continue to


bind to the postsynaptic membrane and action potentials would continually be
qenerated. This cor,rid result in prolonged muscle spasms. However, the enzyme
acetylcholinesterase, which is bound to the surface of the postsynaptic
membrane, hydrolyzes acetvlcholine to acetate and choline. These two products
are then recycled as they are transported back into the presynaptic terminal
.,r-here they are used in the svnthesis of acetylcholine (Figure 1-12).

fhere are many different types of neurotransmitters. For example, the amino acid
*lutamate and the amino acid derivatives epinephrine, norepinephrine,
iopamine, and serotonin can act as neurotransmitters. There are also drugs that
;an act at the level of these postsynaptic receptors.

EPSPs and IPSPs


',\-e said that the binding of acetyichohne to its postsynaptic membrane receptor
:iicits an excitatory response that generates an action potential. These types of
:oientials are referred to as excitatory postsynaptic potentials (EPSPs). There can
aiso be inhibitory postsynaptic potentials (IPSPs) as we11.

Action
Potentials

Excitatory Post
Svnaptic Potential J
u
I Inhibitory
L,;*:',t:il",'"::l^,
The result of
synapse with
AandB {r=u t-fi]
' fi loo''l
Depolarization
,"r'liiili#iiiir::i," Hyperpolarizarron

Figure l-15
IPSPs and IPSPS.

Suppose neuron A synapses with neuron C, as shown in Figure 1-13a. If the


. -,'naptic connection betr.t'een these tr,vo neurons is excitatory, then the same

,Jopvright O by The Berkeley Revie*' l9 The Berkeley Keview


Specializing in MCAT Preparation
Biology Nerve & Muscle The Neuromuscular Junction

number of action potentials that pass down the axon of neuron A will also pass
down the axon of neuron C. In the case of an excitatory postsynaptic potential,
there is an increase in the permeability of the postsynaptic membrane to Nao
(i.e., neuron C will depolarize).

Suppose neuron B, which is inhibitory, also synapses with neuron C, as shown in


Figure 1-13b. In the case of an inhibitory postsynaptic potential, there is an
increase in the postsynaptic membrane to Ko and to Cl- ions. Neuron C will
hyperpolarize. Remember, becoming more hyperpolarized does not generate an
action potential. In fact, it will keep neuron C quiescent. In other words, if we just
excited neuron B alone, then nothing would happen to neuron C.

However, if we integrated the action potentials that neuron C receives from both
neuron A and neuron B, we find that neuron C only generates 2 action potentials
(Figure 1-13b). In this case, neuron B will decrease the excitability of neuron C to
the stimulation it is receiving from neuron A.

Consider two action potentials coming toward one another, as shown in Figure
1-14. When those two action potentiais meet what will happen? Wiil they
continue past one another? Wiil their amplitudes add together? The region in
front of each action potential is continually being depoiarized, while the region
behind the action potential is in its refractory period for a few milliseconds and
cannot immediately be depolarized. Therefore, when the two action potentials
meet one another ihey will stop their propagation along the axon.

+++++++++++++++
Axon :

+++++++++++++++

Figure l-14
Propagation of action potentials.

Copyright @ by The Berkeley Review 20 The Berkeley Review


Specializing in MCAT Preparation
Biology Nerve & Muscle The Sarcomere

The Sarcomere:
'!',-e mentioned that the synapse in Figure 1-12 involved a neuromuscular
'rnction. We have considered the neuronal portion of that junction already. Let's
:.or.,- examine the muscular portion and follow the fate of the action potential that
','.-a. generated.

-- -,-ou rvere to examine a typical muscle, you would find that in most cases it is
:::ached to a specific area of the skeleton by tendons at either end of the muscle'
f:.= region between the tendons is referred to as the belly of the muscle. A cross-
.=:rion of the beliy of the muscle shows a great number of multinucleated
n-ruscle fibers (ce11s), as shown in Figure 1-15. These muscle cells do not divide to
::r,iuce new muscle cells. However, if you were to exercise your muscles, those
:=Ls rr-ou1d just get iarger.

Individual muscle cells

Actin

it
!

l-llne H-zone

Sarcomere

il-g.ure 1- l5

- :,=: eramination of skeletal muscle fibers shows striations in both the


*l::-.::>e and longitudinal directions. The striations in the longitudinai

-,::: 3 b1' The Berkeley Review 2t The BerketeY Review


Specializing in MCAT PreParation
Biology Nerve & Muscle The Sarcomere

direction are due to myofibrils. The myofibrils contain the contractile units of
the muscle. Each contractile unit (Figure 1-16), bounded by a structure referred to
as the Z-line, is referred to as a sarcomere and each sarcomere contails tn'o types
of contractiie proteins. The thin contractile protein is called actin and the thick
contractile protein is called myosin. The myosrn thick filament comprises the A-
band and the region between two A-bands, where there is just the thin actin
filaments, is the l-band. The H-zone is that region in the center of the A-band
and between the ends of the actin filaments.

I-band

A-band Actin A-band


I
H"od'+>8ffi=%Quyosin

-
H-zone Z-line H-zone Z-line

Sarcomere Sarcomere

Figure I - 16
Sarcomere details

Myosin fiiaments are arranged toward the center of the sarcomere. They give rise
to the characteristic dark bands one sees when examining muscle tissue. Actin
filaments are attached to the Z-lines. The actin and myosin filaments interact
with each other through projections stemming from the myosin fiiaments. Those
projections are sometimes referred to as myosin heads. The myosin thick
filament does not have any of these head groups in its central region but rather
concentrates those head groups towards its terminal regions.

The actin thin filament is composed of a protein subunit called G actin ("G for
giobular), which is roughly spherical in shape. The actin filaments can grow by
the addition of G actin to the ends of the already existing filament. Each actin
fiiament is composed of two rows of G actin monomers wound around each
other to form a helix.

Actin and Myosin


Let's consider how myosin and actin interact with each other at the molecular
level. When a muscle is in its relaxed state, ATP is bound to the myosin head
groups. The myosin head is not bound to the actin filament, because ATP
reduces myosin's affinity for actin. \,Vhen ATP is bound to the myosin head, the
myosin head itself is at about a 45' angle with respect to the actin filament. Since
actin is not bound to myosin, the ATP on the myosin heads is hydrolyzed to
ADP and Pi (inorganic phosphate). These hydrolysis products remain on the
myosin head. More importantiy, the myosin head now undergoes a
conformational change, so that it is situated at a 90' angle with respect to the
actin filament. This high energy, stable, myosin'ADP-P1 head complex binds to
the actin filament. As we will see, this step is dependent on Ca2e being present.

Copyright O by The Berkeley Review 22 The Berkeley Review


Specializing in MCAT Preparation
Biology Nerve & Muscle The Sarcomere

The interaction between the actin filaments and myosin head groups causes the
reiease of Pi and then ADP from the myosin heads. This process causes a
conformational change iir the myosin heads, so that they shift by about 45' in a
direction that is away from the Z-lines. This step, in which the actin filaments
move relative to the myosin filaments, is cailed the power stroke, and the
product of this step is referred to as the rigor complex or rigor state. See Figure
1-17.

ATP binds and


causes myosin head

Mvosin

This is the
rigor complex
ADP

r f-ilament
:ioVes
Actin and
myosin bind

rxgure l.I/
-' a : -l:: -:Lrn Cf'Cle.

:-i.:: --:.. :TL\-osirl heads have swiveled and have pulled the actin filament past
: : '- : :-rr, tilament, the myosin heads remain bound to the actin filaments. This
'-"
:-: :=:..:-:alied rigor state. In order for the myosin heads to dissociate from the
i::*- 1ir1ents, ATP needs to bind to those myosin heads. Remember, ATP
'.:,::s:r-.-osin's affinity for actin. Without ATP, muscles remain in a state of
.".-:-: -:: : gir-en period of time. This is what happens to muscles in your body
.-.-- -.----. :ie (i.e., rigormortis). The myosin head groups can no longer bind
:lT :=-;use the metabolic pathways that generate this energy-rich nucleotide
",i = -=-..i to function.
lfi'u'oponin, Tropomyosin, Et Calcium
r" r: : ': .'ra::Lln€d actin, we saw that it was composed of two long rows of

*::- , ;::.:rica1 protein subunits, which polymerized together and then wound
: -r :;:: :--.Ler rn a helical fashion. If we look closely at the helical structure of
r--:-i .:.:ii:e tn-o grooves. Running the length of these grooves is a coiled
: *, ::*- :.--=j tropomyosin, composed of two helical polymers wound about
--:
=: :ee Figure 1-18). When tropomyosin resides in the actin groove, it
-; - : :u:ding sites for the myosin heads and prevents those head groups
-: -:':.1 :o the actin filament.

fhe Berkeley Review The Berkeley Keview


Specializing in MCAT Preparation
Biology Nerve & Muscle The Sarcomere

In the case of striated (skeletal) muscle, this is often referred to as the actin-based
regulation of muscie contraction. Cardiac muscle and smooth muscle are both
controlled by myosin-based regulation.

Troponin Protien
ca2+ Binding
Complex
Tropom yosin Site

/t Relaxation

"t llll f,l


Removal Addition ot
Calcium \1/ Calcium

li

Contraction

Myosin Head Binding Sites

Figure l-lB
Sarcomere detail.

Troponin, a multi-subunit binding protein, interacts with tropomyosin, actin,


and Ca2o. \A/hen Ca2o is bound to a particular subunit of the troponin complex,
it causes tropomyosin to shift its position and expose the myosin head binding
sites. The myosin heads then bind to the actin filaments (see Figure '1'-17) and
muscle contraction follows. [In Figure L-17, Ca2@ would bind at Step 3.] lf Ca2o
is not in the medium, there witl be none to bind to the troponin complex and
tropomyosin wili remain in the actin gloves and cover the myosin head binding
sites. This is the relaxed state.

Surrounding each myofibrii is a membranous structure, a modified version of the


endoplasmic reticulum called the sarcoplasmic reticulum. Calcium is
sequestered within this smooth membranous structure. Also surrounding each
mvofibril is an invaeination of the sarcoleuuna (i.e., the plasma membrane) called
the transverse tubule (abbreviated as T-tubules). These T-tubules follow lhe Z-
lines of each myofibril. Selected anatomical features of the structures associated
with the sarcoplasmic reticulum are shown in Figure 1-19.

After an action potential crosses the iast synaptic junction on its way to a muscle
cell, it passes down each T-tubule and somehow stimulates the release of Ca2e
from the sarcoplasmic reticulum.' [At the present time, it is thought that the
lumen of the T-tubules and the lumen of the sarcoplasmic reticulum are not

Copyright O by The Berkeley Review 24 The BerkeleY Review


Specializing in MCAT Preparation
Biology Nerve 8e Muscle The Sarcomere

continuous.] As the Ca2s flows from a region of high concentration (the


sarcoplasmic reticulum) to a region of low concentration (the cytosol), it binds to
its binding site on the troponin complex. Each sarcomere contracts
simuitaneously because of the way in which the nerve impuise is carried along
the sarcolemma and into the T-tubules.

+ Sarcolemma

Myofibrils

ransverse tubule

Figure l-19
- -
=,.:,'oplasmic reticulum.

- ,- :- :cntraction has taken place anC the nerve impulse has ceased, the Ca2e in
--= :-.:osol is pumped back into the sarcoplasmic reticulum by a Ca2o-ATPase
--::-: In the sarcoplasmic reticulum Ca2o associates with a specific binding
::,:: :.. \Vhen the next action potential passes down the T-tubuies, the cycle will
-:.larn and another muscie contraction will take place.

:.:e a number of ways ir-r which the strength of muscle contraction can be
The strength of contraction can be varied by (1) the size of the motor unit
:efined in a moment), (2) the number of available motor units, and (3) the
': of acth and myosin contained with each cell.

tor unit is simply a motor neuron and the muscle fibers that it innervates.
.. e already examined the muscle fibers. What is a motor neuron? Motor
. :-s are nerve cel1s whose ceII bodies are located in the centrai nervous
:- e.g., the spinal cord or brain stem), and whose myelinated axons
:,e skeletal muscle. Reca11 that myelinated axons allow action potentials to
::-,-::-,itted rapidly to the desired effector organ (in this case, a muscle)

,, anted to precisely move a muscle (e.g., the muscles associated with your
- :ngers), then you wouid need motor units which were rather small in
-:r smalier the size of the motor unit, the smaller the strength of
. : ; ::- rn, In contraSt the posturai muscles of the back or the legs require large

- . : .-::.:ts to be effective. Not only can strength be controlled but precrsion can

. : .--led as wel1.

::. : by The Berkeley Review 25 The Berkeley Review


Specializing in MCAT Preparation
Biology Nerve 6t Muscle The Sarcomere

The number of motor units also controls the strength of contraction. For example,
if you wanted to pick up a light object (e.9., u pencil), you would need to employ
only a few motor units. However, if you wanted to pick up heavier objects (e.g.,
gym weights), you would need to utilize more motor units. When you lift
weights regularly, the size of each muscle cell increases, because the amount of
actin and myosin in each muscle cell has increased. The more actin and myosin in
a muscle cell, the greater the strength o{ contraction that can be generated.

Even though the strength of muscle contraction can vary with the size of the
motor units, the number of motor units, and the amount of actin and myosin in
each muscie cell, the ultimate determinant of muscie contraction is the
concentration of ATP in your muscle cells. The energy for contraction comes
from the hydrolysis of ATP to ADP and Pi (inorganic phosphate). See equation
(1-7\:

ATP ----a ADP + P; + Energy


(r-7)
For muscie contraction to continue, ATP must be constantly regenerated. Recail
from basic biology that under aerobic conditions (when 02 is not limited)
glucose will be oxidized to CO2 and H2O. Energy can be extracted (in the form of
36 molecules of ATP--if we use the glycerol-phosphate shuttle) during this
catabolic process from glycolysis, the Krebs cycle, and oxidative
phosphorylation/electron transport. See equation (1-8):
Aerobic
(Slow)
Glucose CO2 + H2O + 36 ATP
(1-8)

During anaerobic conditions (when 02 is limited), glucose is metabolized to


lactate (the anion of lactic acid). Only 2 ATP molecules are extracted for the use
of energy in this process. See equation (1-9). [This equation is not balanced.]

Anaerobic
(Fast)
Glucose Lactate + 2ATP (1-e)

Even though the efficiency of ATP production is greater for aerobic metabolism
than for anaerobic metabolism (36 ATPs compared to 2 ATPs), ATP can be
produced much more quickly through anaerobic metabolism than through
aerobic metabolism, owing to enzyme regulation in the glycolytic pathway.

During anaerobic metabolism, the concentration of lactic acid begins to increase.


This means that the pH of the medium will decrease (i.e., become more acidic).
One of the key regulatory enzymes in the glycolytic pathway cannot function
weli below a certain pH value. This enzyme has some optimum pH range at
whictr it functions and if the pH falls below (or above) that range, the enzyme is
inJribitedQlycolysis comes to a halt and the ATP yield becomes rnsufficient to
/iatry out ndlmal metabolic processes. In other words, fatigue sets in.

Copyright @ by The Berkeley Review 26 The Berkeley Review


Specializing in MCAT Preparation
Biology Nerve Ef Muscle Nervous System Components

N.€ ti$,#$,ft€ i,,,.'r,,".,,,,' ,.. t, ..,.,.

The nervous system in its simplest form can be found among the cnidarians. All
the neurons in these creatures are linked together in a nerve net (much like a
rr.eb). stimulation of the nerve net causes muscle contraction. This simple
procedure is referred to as a reflex arc. In the case of the cnidarians there is no
associative activity, meaning that the transmission of the action potential is not
linfluenced by other neuronal activity.

-{ssociative neurons are {ound in higher animals (Platyhelminthes and above).


Here, different neurons can interact to produce a given response. when a
rrrlnber of these associative neurons are grouped together, the nervous system
erpands. A grouping of nerve cells is called a ganglion (or a nucleus or nuclei).
Groupings of neurons in higher animals also lead to more elaborate sense organs,
:jtterentiation into a central region of cells (the central nervous system, ot -Ns;
=ld a peripheral region of celis (the peripheral nervous system, or pNS), various
:rssociative areas, and a brain.

The three basic anatomicai divisions of the vertebrate brain can be seen in Figure
1-?0- Those divisions are: (1) the forebrain (prosencephalon), (2) the midbrain
:resorcephaion), and (3) the hindbr ain (rhombencephalon).

Rhombencephalon

rfignr l-2o
'Mlmcai Cirisions of the vertebrate brain.

erufurlcr icl rmeaning "below") the brain is the spinal cord. Recall that the brain
ffiie spinaf cord together are the central neraous system (CNS). From the spinal
''rlrmfi
r@!d* rrrmr-es extend into the limbs and extremities (the periphery) of the body.
ffime mru1:es represent lhe peripheral neroous system (PNS). If a neuron carries
into the spinal cord and brain, that neuron is said to be an afferent
mnffimnnnmrnn#.ncne

i{lm*.'*yt nenron- U a neuron carries the information away from the brain or
qryilltumf rmmd- that neuron is said to be an efferent (motor) neuron.

l[ffie Sunbrain has several anatomical subdivisions, including the cerebrum,


ilhihr md hrpothalamus (see Figure 1-21'.a). The cerebrum is divided into
@{ilfril mr,{ lc{t cerebral hemispheres, joined by the corpus callosum. The cerebral
lfirnlil'ilr1iql'ruttmsres are dirtded into the frontal lobes (associated with movement and

Wmrmnlilittmy ri" parietal lobes (associated with touch and stretch sensation),
niltntldmdtll ldes (associated with vision), and temporal lobes (associated with
rfti nirilmtwr,,os shonrr in Figure '1,-21b.

i{0fu@fh,@ @, Tbs Berkeley Review 27 The Berkeley Keview


Specializing in MCAT Preparation
Biology Nerve Er Muscle Nervous System Components

(a) (b)
Thalamus
Cerebral Central Sulcus
Cortex
Corpus
Callosum )/s \
/a
."o Parjetal
o
.)

Pons
The Cerebrum and its Four Lobes

\
Spinal Cerebe llum
Medulla
Cord

Figure l-2 I
Anatomical divisions of the vertebrate brain.

The outermost layer of the cerebrum is called the cerebral cortex. It consists of
gray matter, which is simply nerve cel1 bodies and their dendrites. Beneath the
gray matter is the white matter, or myelinated axons of the nerve cells. In the
spinal cord, the situation is reversed: The gray matter is more centralized, while
the white matter is more peripheral.

The cerebral cortex has many important landmarks, one being the central sulcus,
a prominent groove that separates the frontal lobes and the parietal lobes.
Anterior to this sulcus is the motor cortex, which controls the movement of
individual muscles. Posterior to this sulcus is the (somatic) sensory cortex, which
detects sensations in various parts of the body.

Somatic receptors send their information into the spinal cord via afferent nerve
fibers, which either cross over to the opposite side of the spinal cord and then
ascend to the sensory cortex in the brain, or ascend in the spinal cord and then
cross over in the brainstem before reaching the sensory cortex.

flomunculus
Wilder Penfield, a Canadian neurosurgeon, was able to rnap the sensory and
motor areas of the brain by electrically stimuiating certain regions of the brains of
his patients during sulgery and observing their actions. Throughout this
procedure his patients were conscious but were unable to feel any pain because
there are no sensory receptors for pain il the cerebrum. He was able to show that
the largest number of cortical neurons found in the sensory cortex register
sensation in the fingers, hands, lips, and tongue. This is represented as the
sensory homunculus (a schematic model of a human being mapped out on the
sensory cortex) shown in Figure 1-22.

Penfield was also able to show that the largest number of cortical neurons found
in the motor cortex control individual movement of the fingers, hands, and
speech. Groups of muscles are controlled by an area just anterior to the motor
cortex called the premotor cortex. This is an association area. Stemming from

2a The BerkeleY Review


Specializing in MCAT Preparation
Biotogy Nerve & Muscle Nervous System Components

this association area are neuronal connections with the thalamus and cerebellum.
Together these structures plan specific movements that the motor cortex then
executes. It is thought that cognitive functioning like speech, writing, and
reading are localized in the left hemisphere of the brain, whiie intuitive functions
are localized in the right hemisphere of the brain. This is not proven, only
theoretical.

-l

-J
Sensory receptors on the right side
of the body project to the
somatosensory cortex on the left hemisphere Tongue
of the brain and vice versa
Left i
L Right
Hemisphere J Hemisphere
, ,

Figure l-22
l:.: :.c,munculus.

.:. thalamus is a relay station for much of the visual and auditory information
-*-:: -,\-e receive from our environment.
The hypothalamus is concerned with the
-':t:a1 activities of the body. The pituitary gland is the master endocrine gland
,,' -:.: body. It receives information from the hypothalamus and sends out
::::rahon to regulate different parts of the body.
'' = brainstem contains such anatomical feafures as the midbrain, cerebellum,
: ::ls medulla, and the reticular formation. These areas coordinate motor and
-.:=:;-- activities. The midbrain detects movement and can direct the head and
: :! :iir-drds that movement. The midbrain can also sense pleasure and pain.
--- = :e:ebel1um is responsible for the bulk of regulation and coordination of
-: ..-.:-^iar activity. The pons and medulla coordinate visceral activities. The
-. * ----:-:: formation, which is the core of the brainstem, is essentially an activating
. :::::.;esigned to alert the brain. The reticular formation also inhibits motor
i.- - ::.*ior:\- impulses and can induce sleep. Below the medulla is the spinal cord.

br The Berkeley Review The Berkeley Review


Specializing in MCAT Preparation
Biology Nerve & Muscle Control Of Body Activity

G;dffitfdlrri drdyrfffifiiffii
Recall that when we first mentioned the nervous system, we iooked at a simple
reflex arc. If someone were to tap you on your knee with a rubber-headed
hammer, your lower leg would extend outward. The mechanism behind this is
quite simple. As the hammer impinges on the patellar tendon, sensory neurons
located in the tendon of the quadriceps muscle are excited. These impuises travel
along an axon that enters the spinal cord through the dorsal root gangiion and
synapses with two neurons (Figure 1-23).

This area, even though it is supposed


to be gray matter, is shown as being
clear so you can see the synapses.

Biceps ('hamstring")
muscie is a flexor
\*u. Ganglion
Dorsal
Grav Matter
I Root

Spinal Cord
Motor Nerve
Tibia Polysynaptic
Interneuron
Reflex Arc

Figure l-23
The reflex arc.

One of the synapses is to a motor neuron that immediateiy leaves the spinal cord
and returns to the quadriceps muscle, causing a contraction. As this muscle
contracts, the leg straightens (extends) at the knee joint. Because it elicits this
kind of action, the quadriceps muscle is termed an extensor muscle. The type of
synapse just described (making just one synaptic connection), is referred to as a
monosynaptic reflex arc.

The other synapse is made to an interneuron which, in this case, is inhibitory.


This interneuron in turn synapses with a motor neuron that innervates the bicep
("hamstring") muscle in the back of the leg. When the bicep muscle contracts, the
lower portion of the leg bends or flexes at the knee joint. We call this kind of
muscle a flexor muscle, and this type of synapse (because it makes at least two
synaptic connections) is referred to as a polysynaptic reflex arc.

If contraction of the biceps muscle is inhibited as the quadriceps muscle


contracts, then one observes a smooth and coordinated movement at the knee
joint, as the lower portion of the leg extends outward after stimulation by the
tapping of the hammer on the patellar tendon.

Copyright @ by The Berkeley Review 50 The Berkeley Kevibw


Specializing in MCAT Preparation
Biology Nerve & Muscle Control Of Body Activity

Neurovisceral Control
The autonomic nervous system, which is part of the efferent division of the
peripheral nervous system, can be subdivided into the sympathetic and the
parasympathetic systems. Nerve fibers from the autonomic nervous system leave
the spinal cord to irrrervate various glands, smooth muscle, and cardiac muscle.
Let's examine these two subdivisions.

Paras;rrmpathetic Division
The parasympathetic division of the autonomic nervous system has nerve fibers
which leave from the sacral portion of the spinal cord and from the midbrain
(mesencephalon) and medulla (part of the rhombencephalon), as shown in
Figure L-24. Parasympathetic nerve impulses tend to increase the rate of
digestion and lower the heart rate. The blood pressure is also iowered, and the
pupils constrict (contract). In general, the parasympathetic division conserves
energy and helps in the restoration of various bodily functions (".9., by aiding in
the digestion of food for later use in metabolic processes).

Ganglion
Neurotransmitter is
acetylcholine
Eve

Ganglion 1
Lacrimal glands

o \ur,
\Itrl.a
Postganglionic Heart
nerve fiber

lr
Lungs and
o
(,) Bronchi
Small intestines

I (tQ.i
\

\
Large intestines
I

1
Sexuai organs

Figure l-24
Parasympathetic nerves.

The parasympathetic division has both preganglionic and postganglionic nerve


fbers. The cell bodies of the preganglionic neurons are found in the sacral region

Copyright @ by The Berkeley Review 3l The Berkeley Keview


Specializing in MCAT Preparation
Biology Nerve & Muscle Control Of Body Activity

of the spinai cord and in the brainstem. The ganglia of the parasympathetic
division lie near or in the organs that are to be innervated. Therefore, the
preganglionic nerve fibers are rather long, while the postganglionic nerve fibers
are rather short. Both the preganglionic and the postganglionic nerve fibers in
the parasympathetic division release acetylcholine (A C h ) as the
neurotransmitter. INerve fibers that release acetylcholine as their
neurotransmitter are called cholinergic nerve fibers.]

The most prominent nerve in the parasympathetic division is the vagus nerve
[also called the (tenth) X cranial nerve]. Roughly three-quarters of all the neurons
in the parasympathetic division can be found in the vagus nerve. The reason for
this should become obvious, if you look at Figure 1-24 The vagus nerve
innervates the heart, lungs, stomach, liver (not shown), small intestine, large
intestine, and kidneys (not shown), among other organs.

Sympathetic Division
The sympathetic division of the autonomic nervous system has nerve fibers
branching off from the thoracic and lumbar regions of the spinal cord, as shown
in Figure 1-25. Sympathetic nerve fibers tend to condition the body for a "fight-
or-flight" response (a term first proposed by the Harvard physiologist Walter
Cannon in the early 1900s). The heart rate increases, blood pressure elevates,
pupils dilate (open wider), and the digestive functions decrease, all as a result of
sympathetic nervous innervation.

The sympathetic nerves that leave the thoracic and lumbar regions of the spinal
cord first enter chains of ganglia connected by nerve fibers on either side of the
spinal column. These chains of ganglia are collectively called the sympathetic
trunk. As these spinal nerves, called preganglionic nerve fibers, enter the
sympathetic trunk, they can either (a) pass through this collection of ganglia to
synapse with other ganglia outside the sympathetic trunk, (b) pass into the
sympathetic trunk and ascend or descend to synapse with ganglia at other levels,
or (c) pass into the sympathetic trunk and directly synapse with a given
ganglion.

The nerve fibers leaving a synapse in a given ganglion are referred to as post-
ganglionic nerve fibers. In the case of the sympathetic division the preganglionic
nerve fibers tend to be short, while the postganglionic nerve fibers tend to be
longer. The preganglionic fibers release acetylcholine, while the postganglionic
fibers release norepinephrine as their neurottansmitters. [Nerve fibers that
release norepinephrine (or epinephrine (adrenaline)) are called adrenergic nerve
fibers.l

One set of spinal nerves that originates in the lower thoracic region of the spinal
cord send long preganglionic nerve fibers to the adrenal medull4 a region
within the adrenal glands (located on the superior surface of the kidneys). These
nerve fibers synapse directly on the ganglia in the adrenal medulla. There are no
postganglionic nerve fibers. When the adrenal medulla is stimulated, both
norepinephrine and epinephrine are released directly into the bloodstream.
Because these substances are released into the bloodstream, we can refer to them
as hormones. Therefore, the adrenal medulla can be considered as an endocrine
gland. These hormones, distributed throughout the body by the circulatory
system, can increase the heart rate and cause the pupils to dilate.

Copyright @ by The Berkeley Review 32 The Berkeley Review


Specializing in MCAT Preparation
Biology Nerve & Muscle Control Of Body Activity

Postganglionic
Neurotransmitters are
nerve fiber
Eye acetylcholine and
norepinephrine

{Vi,
Preganglionic t( .&
LJ /^
Hearr
nerve flber

Thoracic

Lungs and
bo
Bronchi

Stomach
O
d)
Pancreas
o,
Small
(n
intestine
Kidney Large
intestine
'o
k
O

aa
Bladder

Figure I -25
. -:.:::irC nefves.

Somatic vs Autonomic Nervous System


-: , : :evierv the basic differences between the somatic nervous svstem and the
-: - ,-:r-ic nervous system. In the somatic nervous system, we find that (a) once
:
. :.=:r,'e fibers ieave the central nervous system, they do not make a synapse
:-- -:,ev have reached their effector organ. When the synapse is made at the
-:--: organ, (b) the neurotransmiiter that is released is acetylcholine. The
:--:-: nervous system (c) innervates skeletal muscle. Innervation of that
=:-'- nuscle (d) Ieads to excitation of the muscle itseif.

autonomic nervous system, we find that (a) once the nerve fibers leave the
:::- :,en-ous system, they synapse with a ganglion before they make the final
=:-= rvith their effector organ. The preganglionic fibers in both the
::':-rathetic and sympathetic divisions release (b) acetylcholine as the
-:,-:=.smitter. The postganglionic fibers in the parasympathetic division
=:'= a:eivicholine; in the sympathetic division, norepinephrine is released.
::nomic nervous system (c) innervates glands, and smooth and cardiac
l:'e ce1ls innervated by the autonomic nervous system can (d) be either
or inhibitory

The Berkelev Review The Berkeley Review


Specializing in MCAT Preparation
Biology Nerve & Muscle Control Of Body Activity

The adrenal medulla, a specialized ganglion in the sympathetic division of the


autonomic nervous system, is directly stimulated by a preganglionic fiber. This
nerve fiber releases acetylcholine, which causes the ceils of the adrenal meduila
to release two types of hormones into the biood. The major hormone released is
epinephrine. The minor hormone released into the blood is norepinephrine.
These differences are illustrated in Figure 1-26.

CNS
Skeletal muscle
Somatic
Nervous
System

Acetvlcholine
Giands, cardiac
Ganglion
and smooth muscle
Postganglionic nerve fiber
l-IE
Acetylcholine t'ts
>s
t4
Norepinephrine l5
Autonomic lo
Nervous
.*_|or-g".e F+ Biood r-) Organ
l-

)l6'
System
(Epineprhine)
Adrenal Medulla Glands, cardiac
Ganglion
and smooth muscle t
la
Preganglionic nerve fiber t:
IrJ
ID
J5
@

Figure l-26
CNS and PNS review.

Copyright @ by The Berkeley Review 34 The Berkeley Review


Specializing in MCAT Preparation
Biology Nerve & Muscle Receptors and Sensory Input

KeGeptof5,:,and,ri .S€n5,6ff ,.,Iffit


There are many different types of receptors receiving sensory information from
the environment and passing that information to the nervous system. For
erample, there are mechanoreceptors, which are concerned with pressure,
hearlng, balance and blood pressure. Nociceptors sense pain. Mechanoreceptors
:espond to a change in their configuration. Some can respond to a light pressure,
-,r-hile others respond to a deeper pressure. There are specialized regions on the

=:des of fish called lateral line organs that respond to a change in the pressure of
:re rvater. There are pressure receptors in the walls of the aorta that are able to
::nse an increase in blood pressure.

Thermoreceptors detect cold and warmth, while chemoreceptors can detect


-:--te, smell, oxygen, carbon dioxide, hydrogen ions, and blood giucose ievels.
-:-.e taste receptors in the tongue can distinguish between food which is bitter,
.:'.r, salty, or sweet. Olfactory cells rn the nasal cavity can distinguish hundreds
- ilfferent odors. Photoreceptors in the retina of the eye can respond to the
::.sence of a single photon of light. There are also receptors concerned with
:,ectricity and magnetism. For example, catfish have electrical receptors that
..=.r them detect prey, and birds have magnetic receptors that help them to
---:' 1gate.

- -.= sensory information that a receptor receives is specific for that particular
:=:E:tor. The stimulus that is received by that receptor changes (converts or
::ansduces) the potentiai (V*) across the receptor's membrane. This change in
=::'.biane potential is called a receptor potential. If this receptor potential were
',
='.:eed a specific threshold, an action potential would be generated. We also
:: -,rat if the pressure on this receptor is great and the change in membrane
: :=:.iial is large, then the receptor potential will exceed the threshoid level. The
:::*-: is an increase in the frequency of the action potentials being generated.
. . = ,:nplitude of the action potentials r,vi11 not be change, only the frequency.

Receptors and Transducers


.: . :onsider the receptor potential for a special type of mechanoreceptor ca11ed
:acinian corpuscle. A pacinian corpuscle has an unmyelinated nerve ending
. -::. is encapsulated in layers of connective tissue. However, the afferent nerve
-- :, . -=ar-es this encapsulated ending is myelinated.

Saltatory
Pressure

+
_/\:*v
+
\----l--\------r'

'','gUre 1.27
',, - .--. -'.rrpuscle.

.ii 0 by The Berkeley Review cc The Berkeley Review


Specializing in MCAT Preparation
Biology Nerve Et Muscle Receptors and Sensory Input

\zVhen the nerve ending is depressed, a local deformation causes sodium channels
to open and Nao ions to rush into the nerve ending. An action potential is not
generated in the receptor region. Instead, the establishment of this receptor
potential causes a localized flow of current to be propagated along the nerve
fiber. If the threshold has been reached, then this local current flow will jnitiate
an action potential at the first node of Ranvier, located at the outer edge of the
pacinian corpuscle. The action poiential can spread down the afferent nerve and
towards the central nervous system by saltatory conduction, as shown in Figure
7-27.

Suppose we were to depress the pacinian corpuscle just a smail amount. If the
threshold is not reached, an action potentiai will not be generated. This means
one would not feel the pressure that is being applied. If we apply a second
stimulus, we might depolarize the membrane even more. If we were to apply a
third stimulus that was even stronger, we could exceed threshold and an action
potentiai would be generated. If we maintain this pressure such that we are just
above threshold by a certain amount, we will continue to generate action
potentials at the rate of, say,2 action potentials every 10 milliseconds.

2 action potentials 3 action potentials


per 10 miliiseconds per 10 milliseconds

Threshold

o Time (ms)
lst
Weak [fr 3rd
tl
Strength
stimulus
Strong
of

4rh

Figure l-28
Acti0n potential frequencies.

What wouid happen if we apply a fourth stimulus, much stronger than the
previous stimuli? The membrane would become more depolarized, and we
would move further above the threshoid level. Once this happens, the frequency
of action potentials generated would increase, say, to 3 action potentials every 10
milliseconds. This is shown in Figure 1-28. Note that in each case the amplitude
of the action potentials remains the same. \Alhat changes is the frequency of the
action potentials propagating along the axon" It is the frequency of action
potentials that signals to the central nervous system the magnitude of the
stimulus being received. All receptors function on this basic principle.

Copyright @ by The Berkeley Review 36 The Berkeley Review


Specializing in MCAT Preparation
Biology Nerve 6r Muscle Receptors and Sensory tnPut

Adaptation
wheriyou wake up in the morning and take a cool shower, you {eel the coolness
of the water on your skin. When you get dressed, you feel the clothes touching
your skin. In each case, though, after a brief period of time you get used to the
water or the clothes touching your body. These are examples of sensory
adaptation. One type of receptor that undergoes sensory adaptation is the
pr"ir.r." receptor. if we plot the frequency of action potentiais (as they travel
Lack to the central ,r*tno.tt system aiong a given axon) as a function of the time
of a sustained stimulus, we would find that pressule receptors adapt rapidly'
F{owever, pain receptors do not adapt raprdly, as is shown-in Figure 1-29' If the
pressure ,u."ptott did not adapt to the touch of the clothes that we wear, it
rvould plove to be rather inconvenient. [On the other hand, the body does not
adapt ai readily to the sensation of pain--a good thing, evolutionarily speaking,
silce pain is a warning of potentiai damage to the body's tissue and not
something to be responded tb as a matter of convenience, but as a matter of
sunival.]

)\=
'r O

Time of sustained stimulus (sec)

Figure l-29
-.:.::aiion to pressure and to pain.

Tactile Discrimination
-"-=:J that the end of a neuron can be divided into many branches. Each of these
::::rches in turn can end at a receptor (such as a pacinian corpuscle). These
1.,--ra-l branches and their corresponding receptors constitute a receptive field.
there can be many
-.=:ending on which area of the body one is describing,
:=,::iiit'e f-ieids, some of which overlap, or there can be a few receptive fields,
:::-: o j tvhich do not overlap. Let's consider a set of receptive fields that overlap
: :,rier io d.etermine how iwo points in space can be distinguished from one
same touch.
ci- :
=er bv the
:-::l=e \\e have a set of receptive fields like the ones shown in Figure 1-30a.
,--.., ."" generally finds is thit a greater frequency of action potentiais will be
: r.:=:.:ed fass,tming threshold has been if the central region rather than
reached)
*: = :e:rphery of a glven receptive field is stimulated. In other words, there
l seem
,, , ," receptors in the central region of a receptive field than in itsperiphery.
-ore
0

3 r,,::: outd happen if receptive fields overlapped, as shown in Figure 1-30? If


-,,.

e ri: -.r-ere to sti.mulate the receptive field represented by (b) in Figure 1-30
n ,:r:t:-i.*- enough, then because of the field overlap, we would also stimulate
.e l::-= :::e::ive fields in (a) and (c). It would feel to a subject as if three different
.r::. _: e body were being stimulated when, in fact, just the receptive field in
b -. :eing stimulated. HJw can the information being received (b) from these
.:5s.-r-. a-ierent neurons be fine-tuned to let us know that only field is being

5/ The BerkeleY Keview


ra' - :' ::::: 3 b1' The BerkeleY Review
Specializing in MCAT PreParation
tn
Biology Nerve Ef Muscle Receptors and Sensory Input

stimulated, and not field (a) or field (c)? This is accomplished by a process called
lateral inhibition, mediated through interneurons within the spinal cord.

In Figure 1-30, the axon that leads away from field (b) has lateral connections to
interneurons that inhibit the impulses being sent down the axons from fields (a)
and (c). In other words, because of lateral inhibition the impulses generated in
receptive fields (a) and (c) are suppressed, allowing the impulses from the
receptive field in (b) reach the proper spinal tracts that ascend to the brain.

To Brain

(a) +

D
6
a (b)

(c) +
Lateral Inhibition
(interneuron)

Figure l-3O
Receptive fields.

Somatic Sensory Pathways


Once the sensory afferent nerve fibers enter the spinal cord they cross to the side
opposite from the side they entered, either in the spinal cord or in the brainstem.
In other words, the sensory input from the right side of the body witl be
represented on the somatosensory cortex of the left cerebral hemisphere, and
sensory input from the left side of the body will be represented on the
somatosensory cortex of the right cerebral hemisphere. [In Figure 1-39 which
side of the body are the receptive fields located, the right side or the left side?
Will the sensory input go to the somatosensory cortex of the right or left cerebral
hemisphere? How do you know?]

Where do these sensory neurons synapse as they ascend to the brain? As a


general rule, there are three neurons involved in sensory pathways. In the case of
pressure, we find that the first-order neurons carrying information from the
receptive field(s) enters the spinal cord and synapse with second-order neurons
that ascend on the opposite side of the spinal cord to the thalamus. Here, another
synapse is made with third-order neurons that continue to ascend, until they
reach a specific region of the somatosensory cortex of the cerebral hemisphere
opposite to the side of the body in which the sensation was perceived. The
cerebral cortex itself contains ceils that are organized into 6 horizontal layers.
The sensations of pressure would be registered in cells arranged in columns that
cross these different lavers"

Copyright @ by The Berkeley Review 3a The Berkeley Review


Specializing in MCAT Preparation
Nerrre & Musc[e
To Go
15 Passages

98 Questions

Time for All Passages Taken Together as a Practice Exam


125 Minutes

Passage Titles Questions


I. Types ofTransport 1-8
il. Autonomic Nervons SYstem 9-1.3
IIII. Action Potentials 14-20
IV. Local Anesthetic 2t-26
V. The Lens, the lris, & Associated Muscles
a l
/./- - JJ
VI. Resting Membrane Potential 34-40
VIII. Nicotine Replacement 4r-46
WII. RetinalProjection 47 -52
IX. Axonctl Transport 53-58
X. Huntington's Disease 59-64
XL Photoreceptors 65-71
XIII. Sound Transmission in the Ear 12-19
XIIII. T-r,-ptophan & Serotonin Experiment 80-85
XIV. Frog Muscle Experiment 86-92
XV. Skeletal Muscle Groups 93-98

,TftE

EIEru{EMY
L,g.p-y.1.fl-1ry'
Specializing in MCAT Preparation
Suggestions
The passages that follow are designed to get you to think irr a conceptual manner about the processes
of physiology at the organismal ievel. If you have a solid foundation in physiology, *u1y of these
answers will be straightforward. If you have not had a pleasant experience with the topic, some of these
answers might appear to come from the void past the Oort field of the solar system.

Pick a few passage topics at random. For these initial few passages, do not worry about the time.
Just
focus on what is expected of you. First, read the passage. Second, look at any diagrams, charts, or
graphs. Third, read each question and the accompanying answers carefully. Fourth, answer the
questions the best you can. Check the soiutions and see how you did" \A4rether you got the answers right
or wrong/ it is important to read the explanations and see if you understand (and agree with) whal is
berng explained. Keep a record of your results.

After you feel comfortable with the format of those initial few passages, pick another block of
passages and try them. Be aware that time is going to become important. Generally, you will have about
1 minute and 15 seconds to complete a question. Be a little more creative in how you approach this next
group. If you feel comfortable with the outline presented above, fine. If not, then try different
approaches to a passage. For example, you might feei well versed enough to read the questions first and
then try to answer some of them, without ever having read the passage. Maybe yor.un answer some of
the questions by just looking at the diagrams, charts, or graphs that are pr"r"rrtud in a particular passage.
Remember, we are not clones of one another. You need to begin to develop a format that works best for
you. Keeping a record of your results may be helpful.

The last block of passages might contain topics that are unfamiiiar to you. Find a place where the
level of distraction is at a minimum. Get out your watch and time yontr!ff on these parruges, either
individually or as a group. It is important to have a feei for time, and how much is passrng aslou try to
answer each question. Never let a question get you flustered. If you carurot figure ont *nui the answer is
from information given to you in the passage, or from your own knowledge-Lase, dump it and move on
to the next question. As vou do this, make a note of that pesky question ind come back to it at the end,
when you have more time. When you are finished, check your answers and make sure you understand
the solutions. Be inquisitive. If you do not know the answer to something, look it up. The solution tends
to stay with you longer. (For example, what is the Oort field?)

The estimated score conversions for 100 questions are shown below. At best, these are rough
approximations and should be used only to give one a feel for which ballpark they are sitting in.

Section I
Estimated Score Conversions
Scaled Score Raw Score

>12 86 - 100
l0- 11 19 -85
8-9 6s-78
7 59-64
6 54-s8
5 48-53
<4 0-47
Biology Types of Transport Passage I

Passage I (Questions l-8) The transport of substances across a cellular


membrane by a carrier protein occurs either through a
Animal ceils have membranes containing a vast array uniport, a symport, or an antiport mechanism.
of lipids and proteins. Some of the transmembrane
proteins in these membranes act as channels that allow for
the simpie diffusion (passive transport) of water-soluble Organelie
molecuies or ions of low molecuiar weight between the
intracellular and extracellular regions of the cell. The
ionic components of these two regions differ considerably o
i see Tabie l). O
ADP + P1
Selected Cellular Ionic Components O
k
Ion [Intracellular] [Extracellular] tEion (.)

Na+ 15 mM 140 mM +58 mV ATp Ca2+ ADp + pi Ca


K+ 135 mM mM
4 -92 mV 'o
o.
cl- 4mM 120 mM -89 mV rl
^ 1L
Lil-' 2 x 10-4 mM 4 mM +129 mV
\'lg2* 40 mM 2mM
'Eion = Equilibrium potential fbr the ion

Table 1

Other transmembrane proteins act as carriers to Figure I


r:nsport ions and molecules across the cell membrane. If
:"-.: substance is too large (e.g., glucose), it will require a
: =-ier protein. This type of carrier-mediated transport is
: ''i.et .faciLitated diffusion, and the rate of diffusion wiil 1. Compared to the extracellular fluid, the intracellular
:--r-ise until all of the carrier binding sites on the fluid will contain higher concentrations of all of the
::ir.brane transport protein are filled. following substances EXCEPT:

I- energy is required to move a substance across a A. protein


r::r,:iane, the process is called active transport. Primary B. HCO3-
r- -r-. i roDsport uses the energy of ATP directly to move a C. Po43-
,rS.ir,Ce against its concentration gradient (Figure 1). D. 11s
.--,: --arrier molecule is the membrane-bound protein
:- - F.::. Only three primary active transport systems
::'.. teen identified: one for Na@ and Ko ions, one for
- ,l i :rns. and one for Ho ions. A11 of the following will lead to inhibition of the
Na+/K+-ATPase EXCEPT a low:
!:--ondary active transport systems do not use the
; -r
--rtr oi ATP directly, but rather use the energy stored in A. intraceliuiar[ATP].
. :.-:inlration gradient established across a celluiar B. intracellularlNa@1.
- :;::::ne. C. extracellular [Ke].

-::nsmembrane proteins that act as channels in nerve


D. extracelluiar [Nae].
-: ceils can be electrically or chemically activated.
-"": -s--le
':'ting membrane potential (Em) in these ce1ls is
,' - - -i,j mV. The driving force that moves an ion Glucose can be carried across the cell membrane by
'lr'-:
-:-. ,:ne of these channels is the difference between all of the following EXCEPT:
! * ,- : :,on. as shown in the foliowing equation:
A. faciiitateddiffusion.
B. primary active transport.
Force Moving Ion - E* - Eion C. a symport mechanism.
D. secondary active transport.

::: 3 by The Berkeley Review 4l The BerkeleY Review


Specializing in MCAT Preparation
Biology Tylres of Tfansport Passage I

4. Glucose, despite its reiatively high intracellular All of the following characteristics are common to
concentration, can dissociate from its carrier protein gap junctions that exist between cells EXCEpT:
on the cytosolic side of the membrane and into the
cytoplasm. because rhe: A. cytoplasmiccontinuity.
B. rapid transmission of action potentials.
A. low intracellular [Na@] allows sodium to be C. coordination of muscle contraction in the
released from the carrier protein. heart.
B. high extracellular [Nao] allows glucose to be D. a high resistance electrical pathway.
released from the carrier protein.
C. affinity of the carrier protein for intracellular
sodium is high.
D. affinity of 8. Which of the following will NOT occur across a
the carrier protein for glucose
cell's membrane, if the eiectron transport chain is
decreases, once sodium binds to the carrier
inhibited?
protein on the extracellular side of the
membrane.
Decreased primary active transport.
4,
B. Decreased secondary active transport.
5. Which of the following graphs best represents the C. Increased sodium concentration gradient.
relationship between simple diffusion (SD) and D. Increased intracellular calcium levels.
facilitated diffusion (FD) ?

A. B.

/-- /:
FI) /," SD

/
/
IGradient] IGradient]

/to.
// ,,' st) / ./*
// -
/ //
lGradientl IGradient]

6. The acetylcholine receptor within the postsynaptic


membrane of a neuromuscular junction binds two
molecules of acetylcholine and allows Na@ and Ke
to flow through the membrane. The acetylcholine
receptor can best be described by its ability to allow:

A. Na@ to enter the cell, leading to repolarization.


B. more Na@ to enter the cell than K@ to leave
the cell.
C. more Ko to enter the cell than Nae to leave
the cell.
D. a change in the membrane potential to lead to
its activation.

Copyright @ by The Berkeley Review 42 The Berkeley Review


Specializing in MCAT Preparation
Biology Autonomic Nervous System Passage Il

Passage II (Questions 9-15) 10. Which of these situations would stimulate the SNS?

The autonomic nervous system (ANS) is concerned I. Extreme heat.


primarily with the regulation of visceral function in il. Extreme cold.
response to external and internal stimuli. The two ilI. Sleeping.
divisions of the ANS are the parasympathetic nervous W. Running a marathon on a sunny day.
system (PNS) and sympathetic nervous system (SNS).
A. I and II only
The SNS is associated with readying the body for B. I, II, and III only
stressful situations, while the PNS is involved in C. IIi and IV only
maintaining bodily functions at basal levels. These D. I, II, and IV only
subgroups use a two-neuron system. A primary neuron
rvill extend from the central nervous system (CNS) while
a secondary neuron wiil extend fiom a ganglion to the
nerve's target organ.
11. Which of the following symptoms would be
The primary and secondary neurons for the PNS considered a PNS response?
secrete the neurotransmitter acetylcholine (ACh). The
SNS, however, uses two different neurotransmitters. ACh A. Blushing
is secreted fiom the primary neuron, while norepinephrine B. Gastrointestinal immotiltiy
,NE) is secreled from the secondary neuron. C. Piloerection
D. Penile erection
The SNS has large coliections of ganglia that form an
intricate network. The most prominent of these ganglia
are the paravertebral ganglia, which are adjacent to the
vertebral column. The prevertebral ganglia lie along the
anterior surface of the great vessels of the abdominal The adrenal gland is considered an extension of the
:avity and on the adrenal medulla. The PNS has ganglia SNS. Stimuiation of the SNS will initiate the release
ihat are immediately adjacent to the organs that are being of the hormones epinephrine and norepinephrine
innervated. from the adrenai medulla. Cortisoi and aldosterone,
produced in the adrenal cortex, will also be released.
Which of the effects below can be attributed to the
hormones released by SNS stimulation of the adrenal
cortex ?
9. The controi of individual organs is considered an
important function of the ANS. However, the PNS I. Gluconeogenesis
component is thought to be more specific in its U. Retention of ions and water in the kidney.
regulation than the SNS. Which statement(s) best III. Proteolysis
explain this theory ? IV. Storage of glucose in the liver.

I. A response to stressful stimuli needs a general A. I and II only


stimulation of the body, thus the SNS is less B. I, II, and III only
specific. C. III and IV only
il. The SNS developed much later in evolution D. I, il, m, and IV
and thus is not as refined as the PNS.
III. The regulation needed to maintain the bodily
systems at equilibrium and after stress must be
more specialized to each organ innervated.
IV. The location of the gangiia of the PNS allows 13. The vagus (1Oth craniai) nerve is a component of the
it to regulate organ function more closely. PNS. Which of the following effects is LEAST
like1y to be associated with the PNS?
A. I only
B. II, III, and IV only A. Constriction of the pupils.
C. III and IV only B. Increased heart rate.
D. I, III, and IV only C. Increased gastric motility.
D. Increased secretion of HCI in the stomach

Copyright @ by The Berkeley Review 43 The BerkeleY Review


Specializing in MCAT Preparation
Biology Action Potentials Passage III

Passage III (Questions 14.2O) If a stimulus is strong enough so that the threshold
potential of a cell is reached (about 15 mV from the E,n),
Many animal cells have an intracellular fluid (ICF) an action potential (Figure 2) will be generated that will
that is electrically negative relative to the extracellular propagate along the nerve cell until it reaches the
fluid (ECF), creating a voltage difference across the ceil's presynaptic ending. A neurotransmitter is released and
membrane called the resting membrane potential (E^). diffuses across the synaptic cleft, where it binds to the
The E* in nerve cells and muscle cells is about -80 mV, postsynaptic membrane. A postsynaptic potential of equal
while in epithelial and red blood cells it is abour -30 mV. magnitude is generated in the postsynaptic cell and the
signal continues. The response of the action potentiai is
In nerve and muscle cells the concentration gradients all-or-none. The phases are shown in Table 2.
and the permeability of the membrane to both Na@ and
Ko determine the E6. The concentrations of these ions in
the ICF and ECF are shown in Table 1. They are
maintained by the Nao/Ke-ATPase pump, a pump that
transports 3 Nao ions to the ECF for every 2 Ke ions
transported to the ICF. At rest, the relative permeability
of the membrane to Ko is roughly 10 times greater than it
is to Nao.

Table 1
8-+l
Ion flntracellular] fExtracellular] fEion
Time (msec)
Na+ 15 mM 140 mM +58 mV
K+ 135 mM 4mM -92mY Figure 2
rEion = Equilibrium potential for the ion
Table 2
1. Resting State: The cell at its En'.
If the En-' transiently changes from its resting value, 2. Threshold: Potential that generates an Ap.
an electrical signal is generated in the nerve cell. There 3. Depolarization: Due to influx of Na+.
are two types of electrical signals: graded potentials and
action potentials (AP). Graded potentials are signals that 4. Overshoot: AP with a positive membrane potential.
operate over short distances, while action potentials are 5. Repolarization: Due to efflux of K+.
signals that operate over long distances. 6. Hyperpolarization: Membrane potential is more
negative than the E'n.
Along the plasma membrane of a nerve cell are 7. Absolute Refractory Period: AP cannot be generated.
voltage-sensitive Nae and Ko channels that contain 8. Relative Refractory Period: AP can be generated if the
gates. The Nao channel has an m gate that opens quickly stimulus is strong enough.
and an h gate that closes slowly upon depolarization. The
K @ channel has an n gate that opens slowly upon
depolarization. The Nae gate can be blocked by
tetrodotoxin (TTX), and the K@ gate can be blocked by
tetraethylammonium (TEA). In the resting state the Nao
and K@ channels have their gates arranged as shown in
Figure 1: 14. The threshold potential for the generation of an
action potential can be found at that value of the E1n
where the:

A. influx of K@ balances the efflux of Nao.


B. efflux of Ko is 10 times greater than the influx
of Nao.
C. efflux of K@ balances the influx of Nae.
D. influx of Nao is 10 times greater than the
elflux of Ke.
Figure 1

Copyright O by The Berkeley Review The Berkeley Review


Specializing in MCAT Preparation
Biology Action Potentials Passage Itr

15. The threshold potential for the action potential 20. Gamma-aminobutyric acid (GABA) is a major
shown in Figure I is: inhibitory neurotransmitter in the central nervous
system. It is synthesized from glutamate, a major
A. -50 mV. excitatory neurotransmitter in the brain.
B. -65 mV.
C. -80 mV.
@I
COOH
Ot
H
D. -92mY. H.N-C-H" . HrN- C- H
i Enzyme -l
CH.
I I

16. During depolarization, the quick opening of the CH,


t-
cH,
t-
voltage dependent m gates results in all of the COOH COOH
following EXCEPT:
Glutamic acid GABA
A. an influx of Nao ions.
B. an increase in the opening of fast m gates in The enzyme that catalyzes this reaction is best
described as:
neighboring Na@ channels.
C. an increase in the opening of fast n gates in A. GABA decarboxylase.
neighboring K@ channels. B. glutamic acid decarboxylase.
D. an absolute refractory period. C. GABA carboxylase.
D. glutamic acid carboxylase.

17. Which of the following voltage gated positions best


represents the repolarization phase?

A. m gate open; & gate closed; /, gate open.


B. m gate closed; h gate closed; r? gate open.
C. m gate open; I gate open; n gate open.
D. m gate closed; h gate open;n gate closed.

1& During the last half of the relative refractory period


shown in Figure 1, the membrane potential is
beginning to:

d repolarize.
B. hypelpolarize.
C. depolarize.
D. hypopolarize.

ft- If TIX and TEA are added to a nerve cell


preparation and a depolarizing stimulus is applied to
tire presynaptic ending, a postsynaptic potential will
te produced. This shows that:

-{- Nae and K@ ions are needed for the release of


the neurotransmitter to occur.
B Nao and Ke ions are not responsible for the
release of the neurotransmitter.
C. Tfi and TEA can amplify the postsynaptic
potential by allowing neurotransmitter release.
D. an increase in ICF K@ ions and an increase in
ECF Nae ions inhibit the release of the
neurotransmitter, unless a depolarizing
stimulus is applied.

I q'ryynght @ by The Berkeley Review 45 The BerkeleY Keview


tr Specializing in MCAT PreParation
Biology Local Anesthetic Passage IV

Passage IV (Questions 21-26) 23. Which of the following sratemenrs is LEAST


compatible with the effects of local anesthetics?
Local anesthetics provide a reversible block in the
conduction of impulses in nerve fibers. A11 local A. Local anesthetics slow the rise of the action
anesthetic drugs contain a lipophilic group, an potentiai in nerve fibers.
intermediate chain, and a hydrophilic group. The B. Local anesthetics slow the propagation of
intermediate chain may be either an ester (as in procaine) nerve impulses.
or an amide (as in lidocaine). The acriviry and potency of C. Local anesthetics decrease threshold for
any local anesthetic depends on several aspects, including eiectrical stimulation in nerve fibers.
its chemicai structure, and the pKu of the substance. D. Local anesthetics prevent the depolarization of
cell membranes.
Local anesthetics are weak bases, so they are kept in
an acid solution in which they form a water-soluble salt.
The base is lipid-soluble and can penetrate various tissue 24. The pKu of iidocaine is somewhere between 1.6 and
barriers. The concentration of base or cation in the 7.8. The pKs of procaine is between 8.i and 8.6.
solution depends on the pKu of the local anesthetic. A Based on this information, which of the followins
decrease in the amount of base facilitates removal of the statements is the MOST likely conclusion?
local anesthetics, resulting in a shorter duration of action.
The duration of action of a local anesthetic is also affected A. The onset of lidocaine is more rapid, because
by the local vasodilation which most of them produce. more exists in its base form at body pH.
Finally, experiments have shown that a nerve which has B. The onset of lidocaine is more rapid, because
been recently and repetitively stimulated is more sensitive more exists in its acidic form at body pH.
to a local anesthetic than a resting nerve. C. The onset of lidocaine is slower, because more
exists in its base form at body pH.

J
CH.
D. The onset of lidocaine is slower, because more
/-\ .. 'i cHz_ j,i_l'"'qHs
C
exists in its acid form at body pH.
\ ttr y- c- H
CH:
t< If acidosis develops after a locai anesthetic has
Lidocaine at pH = 9.0 penetrated a tissue barrier, it is most likely that:

F'igure I
A. the amount of ionized form will be decreased,
and the local anesthetic will freely cross the
tissue barrier.
21. Local anesthetics penetrate the blood-brain barrier B. the amount of ionized form wiil be decreased,
with great ease, most likely because of their: and the local anesthetic will be trapped in the
tissue.
A. large molecular size and high lipid solubility.
C. the amount of ionized form will be increased,
and the local anesthetic will freely cross the
B. large molecular size and low lipid solubility.
tissue barrier.
c. small molecular size and high lipid solubility.
D. small molecular size and low lipid solubility.
D. the amount of ionized form will be increased,
and the local anesthetic will be trapped in the
tissue.

ta Based on the passage, which of the following


statements is most likely true?
26. The structure of lidocaine is given in Figure tr.
Lidocaine is administered through injection, rather
than orally. This is because lidocaine:
A. The higher the pKu of the local anesthetic, the
lower the concentration of base in the tissue.
B. The lower the pKa of the local anesthetic, the
A. will be deactivated by the acidic conditions of
the stomach.
higher the concentration of acid in the tissue. B. will not be absorbed across the intestinal wall .
C. The higher the Ku of the local anesthetic, the C. is biotransformed by reactions occurring in the
lower the concentration of base in the tissue. liver.
D. The lower the Ku of the local anesthetic, the D. is biotransformed by reactions occurring in the
lower the concentration of acid in the tissue. spleen.

Copyright @ by The Berkeiey Review 46 The Berkeley Review


Specializing in MCAT Preparation
Biology The Lens, the lris, & Associated Muscles Passage V

Passage V (Questions 2'7 -33) 27. Fora person watching a meteor shower at midnight,
what are the contraction states of the circular muscle
Figure 1 shows the lens, the iris, and the muscles of of the iris, the radial muscle of the iris. and the
the eye. ciliary muscle?

A. The circular muscle of the iris is contracted,


the radial muscie of the iris is relaxed, and the
\ Retina ciliary muscle is relaxed.
B. The circular muscle of the iris is relaxed. the
Vitreous radial muscle of the iris is contracted. and the
Chamber
ciliary muscle is contracted.
Suspensory C. The circular muscle of the iris is relaxed, the
Ligaments radial muscle of the iris is relaxed. and the
Optic ciliary muscle is relaxed.
Nerve D. The circular muscle of the iris is relaxed, the
radial muscle of the iris is contracted, and the
ciliary muscle is relaxed.
Figure i
The opening of the pupil of the eye is controlled by
:'.;,o sets of muscles. The circularly arranged smooth
:uscle is under parasympathetic control, and the radially
.-an-sed smooth muscle is under sympathetic control.
F-qure 2 shows the relation between these muscles and the 28. As people age, the lens becomes less flexible and
:::ilof the eye. The size of the pupil reacts to the amount therefore less able to change shape during
: iight present. accommodation. What happens to a person's vision
with these age changes?

R.,lial Smooth Circular Smooth A. The lens is less able to focus the iight rays
\lrscle Fibers Muscle Fibers from near objects.
The lens is more able to fbcus the light rays
from near ob.jects.
Si'mpathetic Parasympathetic C. The lens is more able to focus the iight rays
Fiber
from far objects.
Normal D. The lens is less able to focus the iight rays
from far objects.

Dim Light Bright Light 29. Special solutions are used to dilate the pupil during a
retinal exan. so that the retina can be viewed. The
Figure 2 soiution causes an inability to focus on near objects,
reduces clear distance vision, and enlarges the pupil.
The lens of the eye is suspended within and What is the stimulation or inhibition required to
produce these changes in the eye?
-r:rnded by a ring of tissue called the ciliarir muscle.
: -,.r:nsory ligaments connect the lens to the ciliary
- '..:ie. When the ciiiary muscle is relaxed, the A. Sympathetic nerves are inhibited, and
ligaments pull the lens taut and flat. Since we parasympathetic nerves are stimulated.
-:::rsory
:-:: most of our time in far-away vision (20 feet or B. Sympathetic nerves are stimulated, and
- ',:: this is ideai. For closer work, the lens must parasympathetic nerves are inhibited.
r : . :-.modate and become thicker for focusing. In the C. Both are stimulated.
:. -:.ss. the ciliary muscle tightens, the suspensory D. Both are inhibited.
:::.3nls become slack, and the lens thickens and
- :: ,.:res mOIe COnVeX.

- - : .:ieht €l by The Berkeley Review 47 The Berkeley Review


Specializing in MCAT Preparation
Biology The Lens, the lris, & Associated Muscles

30. When light rays travel through the pupil and the
lens, upon which part of the eye does the image
focus?

A. Retina
B. Cornea
C. Optic nerve
D. Vitreous body

31. One eye exercise invoives focusing on an object


while it moves from arm's length to the closest point
upon which you can focus. This causes the lens to
undergo accommodation, that is, the lens must
change shape to keep the object in focus as it moves
cioser. As the object moves nearer, what changes
occur in the eye?

I. The pupil gradually contracts.


il. The ciliary muscle gradually contracts.
III. The suspensory ligaments gradually contract.

A. I only
B. II only
C. III only
D. II and III only

32. The iris is pigmented epithelial tissue. The color of


the eye is determined by the amount of pigment.
Biue eyes have the least pigment, brown eyes have
more, and black eyes have the greatest amount of
pigment. What coior are the eyes of a person who
has no pigment in her iris?

A. Black
B. White
C. Pink
D. Green

33. Astigmatism is the condition of having a non-


uniformly shaped lens or cornea. This means that
parallel light rays do not focus, so that a sharp image
is not formed. How could this eye condition be
corrected?

A. By providing a convex corrective lens.


B. By providing a concave corrective lens.
C. By providing a non-uniform conective lens.
D. By providing a uniform corrective lens.

Copyright @ by The Berkeley Review 4A The Berkeley Keview


Specializing in MCAT Preparation
Biology Resting Membrane Potential Passage VI

Passage VI (Questions 34-40) 34. The cell membrane acts as an insulator, because the:

Separated electric charges of opposite sign have the A. proteins making up the cell membrane are
capacity to do work if they are allowed to come together. charged.
This electric potential is more commonly known as B. proteins making up the cell membrane are
simply the potential. A11 cells in a resting state have a uncharged.
difference in potential across their cell membranes, the C. lipids making up the cell membrane are
inside being negative with respect to the outside. charged.
D. iipids making up the cell membrane are
The magnitude of the resting membrane potential is uncharged.
determined primarily by two factors. One is the typical
concentration of three kinds of ions within the
intracellular and extracellular fluids (Table 1):

Extracellular Intracellular
Ion (mmol/L) (mmol/L) 35. The resting membrane potential of a typical neuron
is:
Nao 150 15
cle 1i0 10 A. positive, with the excess charge representing a
very large fraction of the total number of ions
6o 5 150
inside and outside the cell.
B. positive, with the excess charge representing a
Table I very small fraction of the total number of ions
inside and outside the cell.
The second factor is the ability of each kind of ion to C. negative, with the excess charge representing a
penetrate the plasma membrane, an ability that fluctuates very large fraction of the total number of ions
frorn one moment to the next. In Table 1. notice that the inside and outside the cell.
sodium and chloride ion concentrations are generally D. negative, with the excess charge representing a
lower inside in the cell, while potassium ion concentration very small fraction of the total number of ions
is lower outside the cell. The concentration differences inside and outside the cell.
between sodium and potassium are produced by a plasma
membrane active transport system that simultaneously
purnps sodium out ofthe cell and potassium into the cell.

As an ion moves down its concentration gradient, an


electric force is established that opposes the movement of 36. Which of the following diagrams BEST represents
that ion. The membrane potential that exists when the the equiiibrium potential for potassium across a
electric force is equal in magnitude but opposite in nerve cell membrane?
direction to the concentration force is the equilibrium
potential for that ion. The equilibrium potential for each A. B.
ion species is different in magnitude--sometimes even in
direction--and can be calculated using the Nernst
equation:

e=#,,?
In a nerve cell at rest, the cell membrane is 50 to 75
times more permeable to potassium than to sodium.
During the resting potential, there is a net diffusion of
ions into and out of the cell. The concentration gradients,
rvhich would eventually dissipate, are maintained through
the Nao/Ke pump. If the concentration gradients remain B
fixed and the permeabilities of ions do not change, the
High
resting membrane potential remains constant.
11o

Copyright O by The Berkeley Review 49 The Berkeley Review


Speciatizing in MCAT Preparation
Biology Resting Membrane Potential Passage VI

37. The resting membrane potential lies above the


equilibrium potential for potassium, because a small
number of:

A. chloride ions diffuse into the cell.


B. chloride ions diffuse out of the cell.
C. sodium ions diffuse into the cell.
D. sodium ions diffuse out of the cell.

38. According to information in the passage, the


concentration gradients of ions are maintained
through the Nae/ I(9 pump. The ultimate source of
energy for this pump is the:

A. formation of ATP.
B. breakdown of ATP.
C. oxidation of nutrients.
D. reduction of nutrients.

39. The constancy of the resting membrane potential is


an example of:

A. an equilibrium state.
B. a dynamic state.
C. a steady state.
D. a static state.

40. The plasma membrane of many cells is permeable to


chloride ions and does not contain chloride ion
pumps. The membrane potential set up by other ions
thus acts on chloride ions. According to this
information and information in the passage, the new
resting membrane potential is:

A. changed only slightly by the chloride ion.


B. changed significantly by the chloride ion.
C. changed geatly by the chloride ion.
D. not affected at all by the chloride ion.

Copyright @ by The Berkeley Review 50 The Berkeley Keview


Specializing in MCAT preparation
Biology Nicotine Keplacement Therapy Passage VII

Passage VII (Questions 41-46) 42. The following graph shows the results of a binding
assay on the muscarinic acetylcholine receptor:
Acetylcholine is one of the body's most important
neurotransmitters, responsible for the transmission of
nerve impulses across synaptic junctions. There are two
Er
rnain classes of acetylcholine receptors. The nicotinic :t/
dl/
acetvlcholine receptor responds to nicotine as an agonist u l/
vi tt
and to curare as an antagonist. The muscarinic .) a li
acetvlcholine receptor responds to muscarine as an
f4|u
'n
=
ll
tl
:gonist and to scopolamine as an antagonist.
€t
z
\icotine is a psychoactive alkaloid extracted from IMuscarine]
::,bacco piants. It is a toxic substance, one that places a
:Iiiss on the heart and the entire cardiovascular system.
3:;ause this drug is quite addictive, several techniques Scopolamine, a competitive inhibitor of muscarine,
:-i; been designed to cure nicotine dependence. is added to the solution and the assay is repeated.
Which of the following graphs BEST represents the
In a novel nicotine replacement therapy, a results of the second assay?
: ,;::bination of physostigmine and scopolamine is
b:Trinistered. Physostigmine is an acetylcholinesterase A. B.
:,::'ritor at both muscarinic and nicotinic acetylcholine o ot
-i:E:iors. It must be noted that scopolamine is functional a al
OI
9x
lr '.n'e i€ntr?l nervous system, but receptors in heart tissue
,rr:: :nsensitive to scopolamine. Figure 1 schematically
oO EgF
Fo I /
'-
!d:r,;:-i the action of these substances in the autonomic 3o 3o
91./ | /'
iltf, -,ii-< S\-Stem, z
IMuscarine]

i:r:.i :npathetic division


+_ C. D.
^_-_ ...,--.J.
-2 :
:t
t aEt/
h t -'-
dl/dl/

/
Nicotinic
/'
Muscarinic =
ol/al/
s_6
oal
cE I //
I / vlll
aaLl ll
eC
receptor receptor 'cE
d.l II // .d llll
6u
ot/o[
6l/ail
mrr,::,eric dir ision = l/ = I
-1,
zvz
IMuscarine] IMuscarine]

=---- / ./'
\icotinic Adrenergic
ieceptor receptor

ll{i-i - -.-rr:i nen'ous system


43. The nicotinic acetylcholine receptor, which is made
of a pentamer of subunits, is essentially a sodium
!.l = -!.9T"{::i:1j-.3 : = Norepinephrine channel. Which of the following occurs directly
after acetylcholine binds to a nicotinic acetylcholine
Figure 1 receptor?

A. An action potential is generated in the


iltlll* 'trtlriir:: :i fte lbllowing ions is responsible for the postsynaptic cell.
1! ie:&:ce .-f acety'lcholine-filled vesicles from a B. An inhibitory postsynaptic potential is
mflbs' {T,artr J nen'e terminal? generated.
C. An excitatory postsynaptic potential is
& 5i+ generated.
t. a€ D. There is no change in the resting membrane
. LL,_:-
L-
potential of the postsynaptic cell.

X'.urummlruqilrm @l :1" lhe Berkeley Review 5l The Berkeley Review


Specializing in MCAT Preparation
Biology Nicotine Keplacement Therapy Passage VII

44. The novel nicotine treatment described in the


passage theoretically should be superior to
traditional nicotine addiction treatment, because it
offers patients:

A. no net nicotine excitation.


B. a net nicotinic and sympathetic nervous system
excitation.
C. a net nicotinic and parasympathetic nervous
system excitation.
D. a net nicotinic and neuromuscular excitation.

45. Cardiovascular stress is one common result of


ingesting nicotine by smoking cigarettes. This
supports the idea that:

A. nicotine stimulation of the nicotinic


acetylcholine receptor is dominant in the
sympathetic nervous system.
B. nicotine stimulation of the nicotinic
acetylcholine receptor is weaker in the
sympathetic nervous system.
C. nicotine acts directly on cardiac muscle to
promote overstimulation and stress.
D. nicotine acts indirectly through the central
nervous system to produce cardiac stress.

46. As part of an experiment, a molecule is introduced at


the synapses between heart muscle and nerve fibers
of both the sympathetic and parasympathetic
nervous systems. The molecule moves in a
retrograde fashion through both of these divisions.
By the time it reaches the next synapse, the
molecule will have traveled through:

A. a dendrite, moving a greater distance in the


parasympathetic nervous system than in the
sympathetic nervous system.
B. a dendrite, moving a shorter distance in the
parasympathetic nervous system than in the
sympathetic nervous system.
C. an axon, moving a shorter distance in the
sympathetic nervous system than in the
parasympathetic nervous system.
D. an axon, moving a greater distance in the
sympathetic nervous system than in the
parasympathetic nervous system.

Copyright @ by The Berkeley Review 52 The Berkeley Review


Specializing in MCAT Preparation
Biology Ketinal Projections Passage VItr

Passage VIII (Questions 47-52) 49. The blind spot, located at the optic disc, is called so
because:
The optic nerve is formed from the axons of all
retinal ganglion cells. The optic nerves from each eye this is the region where ganglion celis leave
the retina.
roin at fue optlc chiasm and eventually enters either the
.:ft or right bptic tract. The optic tract projects to three color vision is not available at this retinal
location.
s.rbcorti&l areas. One is the lateral geniculate nucleus,
C. this area is free of photoreceptors.
hich is responsible for processing visual information'
',,.

which produces pupillar reflexes D. light is unabie to reach this small area of the
'lne is the pritectal area,
-:sed on lnformation from the retina' Finally, the retina.
.:perior colliculus uses the information from the retina to
:::]erate eye movement.

\\'hen light is shone upon one eye, it causes


: , nsrriction of the pupil rn both eyes. Constriction of the 50. The neurotransmitter released by the axons in the
: . : in n,hich the light is shone is the direct response whlle Edinger-Westphai neurons is most likely:
: ,-:,striction of the other is known as the consensual
'.:.:,1nse. The pupillary reflexes are mediated through A. epinephrine.
-.:-:a, ganglion neurons that project to the pretectal area B. norepinephrine.
:--:h lies interior to the superior colliculus. The cells in C. acetylcholine.
'-.: F:3tecta1 area project bilaterally to preganglionic D. glutamine.
,.,.:., mpathetic n"urons in the Edinger-Westphal
-- l::s. This is also known as the accessory occulomotor
-
neurons in
-
-:-:*s. The preganglionic parasympatheticthrough the
-". =:inger-Westphal nucleus send axons
51. As part of a routine eye exam' the following is
. :: --::i-io1or n"tuJ to innervate the ciiiary gangiion' The
- .-- '. eangiion's postganglionic neuron innervates the notiled: If iight is shone directiy into the patient's
:: ., :,- nuscle of the pupillary sphincter. left eye, the patient exhibits a consensual but not a
direct response. Which of the following is a likely
explanation?

A. The optic nerve of the left eye is intact, but the


::' ,. .:s been determined that the frequency of action efferent limb of the left eye is damaged.
:, ::rdals increases dramatically in axons once they B. The optic nerve of the left eye is damaged, but
the efferent ofthe left eye is intact'
. :..: lefi the optic nerve. The most likely
.'.:-.nation for this increase is: C. The optic nerve of the right eye is intact, but
the efierent limb of the right eye is damaged'
\. ; hi-sher density of sodium channels are found D. The optic nerve of the right eye is damaged,
but the efferent limb of the right eye is intact'
;n the axons leaving the oPtic disc'
: lou er density of sodium channels are found
:i the axons leaving the oPtic disc.
. rl ine a\ons are myelinated by Schwann cells'
[] :he a.rons are myelinated by oligodendrocytes'
52. In another patient, the following is observed: Light
shone into ihe right eye does not elicit a response in
either pupil. Light shone directly into the left eye
causes a direct and consensual responsg' It can be
, l": concluded that there is a iesion in the:
11 ,-.:-t optic tract can be described as an:

A. left oPtic nerve.


B. right oPtic nerve.
C. Ieftpregangiionicparasympatheticneuron'
D. rightpregangiionicparasympatheticneuron'
nerve axons from

from

5J The BerkeleY Kcview


Specializing in MCAT PreParation
Biology Axonal Tfansport Passage IX

Passage IX (Questions 53-58) 56. Glucose does not simply diffuse over the inner
mitochondrial membrane. Glucose enters the
Movement of substances from the soma to the mitochondrial matrix as:
synaptic endings of a nerve cell by simple diffusion is an
inefficient process: because axons are often very long. A. glucose-6-phosphate.
However, components which originate in the soma must B. phosphoenolpyruvate.
be distributed along the axon. Axonal transport, a process C. pyruvate.
which costs metabolic energy] is a special trinsport D. oxaloacetate.
mechanism that accomplishes this function. Membrine-
bound organelles are transported rapidly by fast axonal
transport, while substances dissolved in cytoplasm are 57. Phaseolus vulgaris leucoagglutinin is taken up by
moved by slow axonal transport. Microtubuies provide a neurons and is transported anterogressively.
pathway along which membrane-bound organelles can ihis
marker substance is most likely injected at the:
move. These organelles may interact with the
microtubules through a Iinkage similar to that between A.
thick and thin filaments of skeletal muscle cells.
synapse to trace the pathway of neuronal
axons.
B. synapse to trace the location ofcell bodies.
Transport from the soma toward the axonal terminal C.
is known as anterograde axonal transport. Retrograde
soma to trace the pathway of neuronal
dendrites.
axonaL transport is transport in the opposite direction. In
D. soma to trace the synaptic endings of neurons.
order to trace neural pathways experimentally, marker
substances can be used that are transported either
retrogressively or anterogressively. 58. The graph shown beiow represents the response of a
postsynaptic cell to acetylcholine in the synaptic
cleft. Each vertical line (l) represents an action
potential.

53. A neurotransmitter traveling


presynaptic terminal will travel:
from the soma to the
|||ll||t
time [:)
A. anterogressively, via fast axonal transport.
B. anterogressively, via slow axonal transport. Which of the following graphs BEST represents the
C. retrogressively, via fast axonal transport. respo,nse following the addition of the enzyme
D. retrogressively, via slow axonal transport. acetylcholinesterase?

A.
54. Which of the following ions mosr likely triggers
interaction between organelles and microtubules?

A. Nao
an

ll||ll||t
Lime r-)
B. 6@
C. Ca2@ B.
D. Mg2e

hme L_2
55. Neuropeptide packaging into vesicles will include C.
all of the following EXCEPT the: llllllrt
tlme I )
A. signal recognition particle receptor.
B. signal peptide.
D.
C. complete translation of the peptide in the
cytoplasm.
D. transport through the Golgi apparatus.
trme I >

Copyright O by The Berkeley Review s4 The Berkeley Review


Specializing in MCAT Preparation
Biology Iluntington's Disease Passage X

Passage X (Questions 59-64) 61. The following molecule is referred to as GABA.


What is irs IUPAC name?
Huntington's disease is a relatively rare disease that
causes progressive degeneration of the cerebral cortex and
the basal ganglia of the brain. The basal ganglia are o o
masses of neuron cell bodies (gray matter) located deep H3N- CH2- CH2- CH2- coo
n,ithin the cerebrum. Within the basal ganglia, certain
types of neurons are destroyed, while others remain intact.
A. y-amino butyrate
Huntington's disease is a genetic disolder caused by an B. 3-amino butanoic acid
autosomal dominant genetic defect on the tip of C. 1-amino butyrate
chromosome 4. The symptoms of the disease are D. 4-aminobutanoicacid
choreiform movements (rapid, uncontrolied, jerky
novements), mental deterioration, and emotional
iisturbances. The disease is relentless and usually leads
to complete dehabilitation and death within 15 years of its
rnset. The age of onset is usually from 35-50 years,
usually after the patient has had children.

The proposed mechanism of the defect lies with 62. Which amino acid is the precursor to GABA?
:ertain neurotransmitters and is twofold. First of all, there
-s a decreased level of y-aminobutyric acid, an inhibitory A. Histidine
:eurotransmitter, and a decreased level of the enzyme that B. Leucine
.i'nthesizes it, glutamic acid decarboxylase. Secondly, a C. Tyrosine
:eliciency of the enzyme choline acetylase leads to D. Glutamate
jecreased levels of the excitatory neurotransmitter,
.;et,.lcholine. The delicate homeostatic balance between
j're r\\'o neurotransmitters is disrupted. These deficiencies
:re thought to lead to the observed characteristic
lovements and mental symptoms of Huntington's
:-sease.

63. After having one child, a 4}-year old woman with a


paternal family history of Huntington's disease is
diagnosed as having the disease. Assuming that the
father is not affected, what is the MOST probable
likelihood that the child will also develop the
59. Based on information in the passage, Huntington's disease?
disease is:
A. 100%
\. more common in men than in women. B. l5%o
B. more common in women than in men. C. 50Vo
C. equally common in both men and women, D. 25Vo

D. more common in children.

w"rt. Based on this passage, the basal ganglia most likely 64. In what structure of the neuron are neurotransmitters
stored?
-- rntrol:

A. pituitary gland secretion. A. The synaptic cleft.


B. speech.
B. The vesicles ofpresynaptic neurons.
C. the parasympathetic nervous system.
C. The receptors ofpostsynaptic neurons.
D. voluntarymovements. D. The vesicles of postsynaptic neurons.

; ::ght O by The Berkeley Review 5D The Berkeley Review


on Specializing in MCAT Preparation
Biology Photoreceptors Passage XI

Passage XI (Questions 65-71) 65. Which of the following rerinal cells is LEAST
responsible for vision in the dark?
The retina of the human eye contains two types of
photoreceptors known as rods and cones, both locited at A. Cone cells.
the back of the retina, Upon stimulation by a photon, the B. Rod cells.
sodium channels of these photoreceptor cells close. The C. Ganglion cells.
highest density of cones found on the retina will give the D. Bipolar cells.
greatest visual acuity, or highest visual precision. In
addition to the photoreceptor cells, the retina houses four
types of neurons. These cells are the bipolar, ganglion,
horizontal, and amacrine cells. The rods and iones
synapse with bipolar cells, which then go on to synapse
with ganglion cells. The axons of the ganglion cells 66. Stimulation of a photoreceptor by a single photon
converge and leave Lhe eyes as the optic nerve. will result in a:

The visual field is the view seen by the two eyes 4. monopolarization of the photoreceptor.
without movement of the head. As shown in Figure 1, ihe B. depoiarization of the photoreceptoi.
left visual hemifield projects ro rhe righr side of the brain, 9. hyperpolarization of the photoreceptor.
while the right visual hemifield projecrs to the lefr half of D. micropolarization of the photoreceptor.
the brain.

67. To view an object with greatest acutiy, one will


Left focus which of the foliowing structuies on that
Visual object?
Heniifield

A. Fovea centralis
B. Cornea
Temporal C. Optic disc
D. Choroid

68. An experiment involved the removal of the lateral


geniculate nucleus of the thalamus. To determine the
effects on processing of visual information,
electrodes were placed in which of the following
lobes ofthe brain?

A. Parietal
B. Frontal
C. Temporal
D. Occipital

69. Light entering the eye will pass which of these cells
Left Optic Tlact Right Optic Tract
first?

A. Rod cells
Figure I B. Cone cells
C. Bipolar cells
D. Ganglion cells

Copyright @ by The Berkeley Review 56 The Berkeley Review


Specializing in MCAT Preparation
Biology Photoreceptors Passage Xl

70. The region of the retina where the axons of the


ganglion cells converge and leave as the optic nerve
is best described as the region:

I. of highest visual precision, because of the


dense axonal population.
II. of the retina mbst sensitive to stereovision,
because ofa high density ofphotoreceptors.
UI. where no vision is possible, because of a lack
of photoreceptors.

A. I only
B. I and II only
C. III only
D. II only

71. According to Figure 1, light originating from the left


hemivisual field will strike the nasal hemiretina of
the:

A. right eye and the temporal hemiretina of the


left eye.
B. right eye and the temporal hemiretina of the
right eye.
C. left eye and the temporal hemiretina of the
right eye.
D. left eye and temporal hemiretinal of the left
eye.

lGqryngfu O by The Berkeley Review 5t The Berkeley Review


Specializing in MCAT Preparation
Biology Sound Transmission In The Ear Passage XII

Passage XII (Questions 72-79) 72. The human ear normally responds to a range of
sound covering about 120 dB. The loudest sound
Sound is created by disturbances within a medium pressure level that can be heard over this range is:
resulting in the production of pressure waves detected by
the ear. These pressure waves consist of aiternating A. ten thousand times less than the reference
compressions and rarefactions of the surrounding prgssure.
medium. The loudness of a sound is determined by the B. one million times less than the reference
amplitude of these pressure waves and is measured on the pressure.
decibel (db) scale. The relationship is as foliows: C. ten thousand times greater than the reference
pressure.
Sound Pressure Level (SPL) = 20 log1g P1 / Pp D. one million times greater than the reference
pressure.
where P1 is the test pressure and Pt is the reference
pressure. P, has the value of 20 micro-Newtons/m2. The
pressure waves travel through the external ear canal and
will cause the tympanic membrane of the middle ear to
vibrate.
73. According to Figure 1, the tympanic membrane has
A set of smail bones is responsible for transmitting an arga greater than the oval window. A result of
this vibration throughout the middle ear, and the last of this design is that the total:
these bones (the stapes) is attached to the oval window.
The oval window is the connection between the middle A. force acting on the oval window is increased.
and internal ear, which is composed of the cochiea and the B. forcelunit area acting on the oval window is
vestibular apparatus (Figure 1). decreased.
C. force acting on the oval window is decreased.
Scala Media D. force/unit area acting on the oval window is
Incus increased.
6lo c'

Malleus Stapes
Scala Vestibul
" \

Tympanic 74. The relationship between the speed of sound (C),


Membrane wavelength (7"), and frequency (v) is:
Scala
Round
Auditory Tympani C = (),Xv)
indow
Canal
Internal Ear where C = 340 m/s. What is the period of a sound
Middle Ear
wave with a wavelength of 3.4 x t0-4 km?
Figure 1
A. 1x10-6s.
The transduction of pressure waves into electricai B. 1x10-3s.
signais occurs in the internal ear. The vibration of the C. 1x103s.
stapes produces pressure waves in the fluid of the scala D. lx106s.
ueJnUuti and scala tympani. This wave motion in the
paralymph naturally sets up oscillations in the endolymph,
located in the scala media. The transduction organ (organ
of Corti) is located on the basement membrane of the
scaia media. Oscillating movements of this organ excite
and inhibit sensory transduction cells' which transmit 75. As a result of movement in the organ of Corti, the
impulses to the brain. sensory transducing cells most likely repond with a
potential that is:
Inducing a vibration in the temporal bone will cause
sounds to reich the cochlea. This sound transmission is A. depolarizing.
unique in that the middle ear is bypassed' While this is an B. hyperpolarizing.
inefficient method of energy transfer, such a method is C. oscillatin g depolarizing-hyperpolarizin g.
clinically useful for diagnosing auditory problems' D. returning to its resting membrane potential

Copyright @ by The BerkeleY Review 58 The BerkeleY Keview


Speciatizing in MCAT PreParation
Biology Sound Tfansmission In The Ear Passage XII

76, The following picture represents the basilar 78. A patient complains of auditory problems and so a
membrane uncoiied and stretched out flat. tuning fork is set into vibration and placed at the
patient's ear. When the patient indicates the sound is
inaudible, the fork is placed on the temporal bone.
n- If the sound becomes audible to the patient, the
i# damage can be narrowed down to the:

l',,,,,,,1;,,t",",',111'I-$;1il;;!ff o A. external ear.


B. middle ear.
C. inner ear.
D. auditory cortex.
region
Flexible
region

According to the resonance theory, different areas of 79. It has been discovered that every hair cell responds
the basilar membrane are atfected by varying sound maximally to a particular frequency. Which of the
frequencies. High fi'equency sounds resonate best in following graphs best represents this idea?
which region?

A. Region 1. A. B.

l/\
B. Region 2.
C. Region 3. -o -o
D. Region 4. !
o o

l/\
!
o. o.
E E
The foiiowing circuit model attempts to explain
away the problem that sound arrives at the two ears Frequency (kHz) Frequency (kHz)
at different times. Cells wiii fire with maxirnum
output when bilateral inputs anive at the same time.

C. D.
Input from
right ear
-o
13 .o
O
€O
o. o.
E E

Frequency (kHz) Frequency (kHz)

Input from
left ear

Ii sound to the right ear were delayed relative to the


-:ii. which of the following cell(s) would most likely

-{. Cells 1 and 2.


B. CelI3.
C. Cells 3 and 4.
D. Cell -5.

:'. ::rht O by The Berkeley Review 59 The Berkeley Review


Specializing in MCAT Preparation
Biology Tryptophan and Serotonin Experiment Passage XIII

Passage XIII (Questions 80-85) 81. Table 1 shows the study results for the animals
receiving CHO first (Trial 1). Serum Trp and Serum
The response in brain tryptophan (Trp) concentration LNAA are in pmol/i, and cortex Trp is in pmol/g.
and serotonin synthesis after the ingestion of two meals
was examined in rats. The synthesis and release of
serotonin, a neurotransmitter, by brain neurons is rapidly Serum Serum Cortex
influenced by the local Trp concentration. Group Trp LNAA Trp
No Food 96 482 28
Brain Trp concentration reflects uptake of Trp from
CHO - CHO 140* 3'76 36*
the blood into the brain. This is accomplished by a
transporter in the blood-brain barrier that is shared by the CHO - 67a Protein 1 50* 385 JO-
large, neutral amino acids (LNAAs): leucine, isoleucine, CHO - 127a Protein I 56* 43'/ 35*
tyrosine, phenylalanine, and tryptophan. CHO - 2470 Protein 1 60* 6'7 5 28

CHO - 40Va Protein 162* 9"t6 23


Food ingestion can alter levels of serotonin in the tp 0.05 versus no food
brain by altering blood concentrations of Trp and its =

LNAA competitors. The following study was designed,


Table 1
in which 2 sequential meals were fed to rats that fasted
overnight.
Table 2 shows the study results for the animals
The experimental diets contained varying ratios of
protein and carbohydrate, with fat held a[ a low and receiving protein first (Trial 2). Serum Trp and
constant percentage in ail the diets. The experimental
Serum LNAA are in pmol/l, and cortex Trp is in
pmol/g.
diets were CHO (a11 carbohydrate, no protein), 6Vo
protein, I27o protein,24Vo protein, and 40Vo protein. For
the sake of reference, standard rat chow is about 1070
protein, and the typical human eats about 12-l5%o protein. Serum Serum Cortex
Group T.p LNAA Trp
For the experimental design, the rats received either: No Food 142 51r 26
Trial 1 (the CHO diet followed by one of the protein diets CHO - CHO 162* 49',7 33*
2 hours later) or Trial 2 (one of the protein diets followed 6Va Protein - CHO lb)^ 533 31*
by the CHO diet 2 hours later). The rats were sacrificed
lTVa Protern - CHO 190* 621 27*
and the brain tissue examined for Trp concentration at 2
hours following the second meal. 247o Protetn - CHo 784* 743 28
407o Protein - CHO 194* 814 25
*p = 0.05 versus no food

80. In the following reaction, the molecular structures of Table 2


Trp and serotonin are shown. Trp is converted into
serotonin by first undergoing
Which of the following statements is FALSE
regarding the data in Table 1 and Table 2?
NH" NHt
t-
H.C_ C_ COOH H2C-
I

CH2 A. Serum LNAA concentration increases as


dietary protein content increases.

ffi"o .+""cd
Tryptophan Serotonin
B. A 24Va protein meal at 2 hours following a
CHO meal attenuates the increase in serotonin
due to Trp.
c. At higher levels of protein, the serum LNAAs
are converted to serotonin.
A. reduction followed by carboxylation. D. Serum Trp concentrations increase as dietary
B. hydroxyiation foliowed by decarboxylation. protein content increases.
C. amidation foilowed by hydroxylation.
D. amination followed by decarboxylation.

Copyright O by The Berkeley Review 60 The BerkeleY Keview


Specializing in MCAT Preparation
Biology Tryptophan and Serotonin Experiment Passage XItr

82, Which of the following statements is TRUE of the 85. What would be the metabolic effect of eating a large
data in Table I and Table 2? amount of pure Trp as a dietary supplement?

I. In Trial 1, the higher protein levels (24Vo and A. Dietary Trp alone would not be absorbed
40Vo) caused competition between Trp and the readily in the small intestine, so it would have
other LNAAs. no metabolic effect.
il. In Trial 2, the higher protein levels (lZVo, B. Dietary Trp alone would increase brain
24Vo, and 40Vo) caused competition between serotonin concentrations, since few LNAAs
Trp and the other LNAAs. would be competing.
In. In Trials 1 and 2, serum Trp increased with C. Dietary Trp alone would increase synthesis of
feeding. all classes of neurotransmitters.
D. Dietary Trp alone would cause more alert
A. I only behavior, due to the production of serotonin.
B. II only
C. II and trI only
D. I, II, and III

83. Which of the following statements is TRUE of Trp?

I. Trp is a neurotransmitter.
n. Trp is an essential amino acid.
IIr. Trp is an acidic amino acid.
A. I only
B. II only
c. I and II only
D. I, II, and III

84. As part of this experiment, researchers administered


an inhibitor of one of the enzymes that participates
in the conversion pathway of Trp to serotonin in the
brain. In this part of the experiment, what parameter
could be examined to estimate serotonin synthesis
rate?

A. Accumulation of an intermediate metabolite in


the brain.
B. Concentration of Trp in the blood.
C. Concentration of the inhibitor in the brain.
D. Accumulation of the inhibitor in the blood.

Copyright @ by The Berkeley Review 6l The Berkeley Keview


Specializing in MCAT Preparation
Biotogy Frog Muscle ExPeriment Passage XIV

Passage XIV (Questions 86-92) 86. The muscle is stretched more and more as heavier
weights are added to the experimental apparatus.
The gastrocnemius muscle along with the sciatic How is the intensity of the muscle stretch indicated
nerve from a frog's leg were dissected and attached to a by the activity ofthe nerve fiber?
device to measure action potentials and record them on
chart paper. Figure 1 is a diagram of the experimental A. A larger amplitude of the action potentials
setup: indicates greater intensity.
B. A larger ampiitude of the action potentials
indicates lower intensity.
C. A greater frequency of the action potentials
indicates greater intensity.
D. A greater frequency of the action potentials
indicates lower intensity.

87. Two more trials were performed on the same muscle


preparation. A 75-g and a 100-9 weight were used
to stretch the muscie. The printouts of the action
potentials looked exactly like the 50-g trial. What is
the explanation for this?

Figure 1
A. The action potentials had increased to a
maximum with the 50-g weight and increased
no further, due to the refractory period of the
neuron.
Werghts (1, 2, IO, 20, and 50 gm) were hung
5, B. The nerve and muscle were damaged by the
opposite the muscle, so that a known set of tensions were weights, and the nerve therefore slowed its
apptlea to it. The action potentials generated by each transmission of action potentials.
weight were recorded on the chart paper' This is C. The speed of the chart recorder could not keep
illustrated in Figure 2. Each vertical (l) line represents an up with the increased action potentials.
action potential. D. The action potentials were identical, due to a
neurotransmi tter defect.

I I | lsm
88. Figure 3 indicates the output of action potentials
lll llllllll lllll - 2sm after a 5-mg weight was added to the muscie
preparation. What is the explanation for this effect?
llllllllllllllllllllllllllllllllllll 5sm
5mg
lll ll ll llllllll lllll ll llll ll lllllllll ll llllllllllll 1ll I I I 10 gm
tlme L-)

Figure 3

A. The weight was not heavy enough to exceed


time t+ the action-potential threshold.
B. The weight was too heavy and suppressed the
Figure 2 aciion-potential threshold.
C. The weight must produce an action potential,
so the recording equipment must be damaged'
The purpose of this experiment was to learn more D. The size of the action potential was too small
about the intiraction between muscle and nerve, and how to read on the Printout.
muscle stretch is communicated.

62 The BerkeleY Review


Copyright @ by The BerkeleY Review
Specializing in MCAT PreParation
Biology Frog Muscle Experiment Passage XIV

89. Actions potentials are conducted by the nerve cell


through which structure?

A. Cytoplasm
B. Schwann cell membrane
C. Endoplasmic reticulum
D. Plasma membrane

90. When an action potential is communicated, what is


the sequence and direction of movement of sodium
and potassium?

A. Potassium moves from the intracellular fluid to


the outside of the cell, and then sodium moves
from the extracelluiar fluid into the cell.
B. Sodium moves from the extracellular fluid into
the cell, and then potassium moves from the
intracellular fluid to the outside of the cell.
C. Sodium moves from the intracellular fluid to
the outside of the cell, and then potassium
moves from the extracellular fluid into the cell.
D. Potassium moves from the extracellular fluid
into the cell, and then sodium moves from the
intraceliular fluid to the outside of the cell.

qI. \\-hat type of nerve function is studied using this


erperimental design?

.{. Efferenr function


B. Afferent function
C. Sympatheticfunction
D. Parasympatheticfunction

04" Tle sodium-potassium pump, an active transport


i\ of intracellular
stem. restores the proper balance
nnd extracellular ions. When an organism is
:oisoned with cyanide, which halts ATP production,
;hat happens to neural action potentials?

-4.- Action potentials stop immediately.


B. Action potentials will continue, even though
ATP production stops.
C. Action potentials continue briefly but stop
eventually, due to a lack of ATP.
D. -\ction potentials will generate ATP during
cvanide administration.

g by The Berkeley Review O.' The Berkeley Review


Specializing in MCAT Preparation
Biology Skeletal Muscle Groups Passage XV

Passage XV (Questions 93-98) il. Hamstring Group

\4uscles of the lower extremities aid in the movement Made up of three muscles. Al1 originate from the
r- bones such as the femur, tibia, and fibula. They help
ischium and run along the posterior side of the
irarntain an erect skeletal system and keep the body in femur. Two of the muscles of the hamstring group
balance by acting as an antagonist to gravity. These insert onto the medial condyle of tne tibia-, wtritl
muscles provide the pumping action that helps circulate the third inserts onto the lateral condyle of the
blood and lymph through the lower extremities. tibia and the head of the fibuia. The hamstrings
are innervated by branches of the sciatic nerve.
A diagram of the major bones found in the lower
extremity is shown in Figure i. III. Adductor Group
Several muscles comprise this group. All of these

o
peilor
Super
muscles originate from the pubic bone region and
extend to inserr along the medial and posterior
inferior shaft of the femur. The adduitors are
innervated mainiy by the obturator nerve.

€ Port.rio, AnteriorS
pubic bone

ischium femur

medial 93. Judging by where rhey atrach, which of the following


condyle
muscle groups would play the strongest role in
lateral flexing the hip joinr (i.e., pulling the kiee upwards
condyle
towards the chest)?
fibula
A.. Quadriceps.
B. Hamsrrrngs.
C. Adductors.
D. Quadriceps and hamstrings together"

Figure 1

A iist of the major muscle groups of the thigh region


94. The pumping action of muscles in the lower
of the lower extremity is provided below:
extremity is crucial in order for blood to properly
Muscles of the Thigh
flow back to the heart. Which of the following
statements must be true if blood is to be returned to
the heart effectively while a person is standing?
I. Quadriceps Group
Made of four muscles, three of which originate A. The veins involved must have inelastic walls.
from the upper, anterior portion of the femur and B. The arteries involved must have elastic walls.
one that originates from the anterior inferior iiiac C. The veins involved must have valves.
spine of the ilium. They all descend and insert into D. The arteries involved must contract under
the broad patellar tendon which in turn crosses the sympathetic control.
kneejoint and inserts into the tibial tuberosity of
the upper, anterior tibia. These muscles are
innervated by branches ofthe femoral nerve.

Copyright O by The Berkeley Review 64 The Berkeley Review


Specializing in MCAT Preparation
Biology Skeletal Muscle Groups Passage XV

95. If an individual contracts the hamstring muscles, the:

A. quadriceps must contract.


B. quadriceps must relax.
C. quadriceps are unaffected.
D. obturator nerve is responsible.

96. Paralysis of the f'emoral nerve would MOST


STRONGLY aff'ect the ability:

A. to stand on tip-toes.
B. to extend the hip posteriorly.
C. to kick a ball.
D. of the adductor muscles to contract.

97. Often when a person suffers a herniated vertebral


disk in the lower back, the sciatic nerve is
compressed and damaged. Which of the following
would most likely NOT be a symptom of such a
condition?

A. Difficulty walking.
B. Difficulty moving the lower leg posteriorly.
C. Pain radiating down the back of the leg.
D. Weakness in the adductor muscles.

98. Ciaudication is a medical condition in which arteries


which feed the muscles of the lower extremity
become partially occluded. In these patients,
moderate exercise might cause all of the following in
the affected muscles EXCEPT:

A. ATP deprivetion.
B. lactic acid buildup.
C. lower-than-normal CO2 leveis.
D. 02 deprivation.

Copyright @ by The Berkeley Review ttb The Berkeley Keview


Specializing in MCAT Preparation
Biology Nerve & Muscle Section I Answers

B is correct' HCo3e' Roughly 60va of the body by weight is due


to water, and all of the warer can be divided into
two compartments separated by a cellular membrane. orie compartment
contains the intracellular fluid (ICF) whiie
the other compartment contains the extracellular fluid (ECtr):
itr" tu.g". of the two compartments js the ICF
compartment, and it contains about2/3 of all the water in the
body. The ECF compartment t, ;;;;;;;;; il ,;;
types of fluids: interstitial fluid and biood plasma. Table
1 in the passage lists the major ions of the ICF
The first thrng to note is that the ICF has a higher concenrrarion and ECF.
oi Ko"than rh. ECF: i;i. utto*, us to eliminare
choice D.

The remaining three choices are not mentioned in the passage


and require a little thought. phosphate (po43e) is
required not only in celiular DNA and RNA, brt it is also Lquired
by a vast anay oiproteins that participate in
phosphorylation reactions within the cell. Simply_consider priffioryrating
ADp with p; to make ATp. Therefore,
we would expect the levels of phosphate to be higher in th.:
ICFlhun in tt.r, ECF, and they are. phosphate leveis in
the ICF are about 20 mM' while in the ECF they are about
4 *irrr. sti-inate choice C. we can use the same
reasoning for the relative levels of proteins. Ail of our
ceils are carrying out metabolic reactions, and to do
majority of these reactions enzymes (proteint) the
be employed. thl,
a cell than outside a cell' Eliminate choice A. Tr:J -"un, rhat there will be more prorein inside
cetts which are catabolizing rnot".ut"r-ur" proaucing
via the Krebs cycle) carbon dioxide can combine with water co2 gas (e.g.,
to form carbonic acid, which can then dissociate to
form the bicarbonate ion (HCo3e). Since Co2 is a wasre and
it needs to be eliminared via the blood, the cell wants
to get rid of it we find that the levels of bicarbonate are higher
in the ECF (i.e., the blood plasma) than in the icF.
Bicarbonate levels in the ICF are about 10 mM, while in trr"
iCn irr"f-ar" about 24 mM. The correct choice is B.
D is correct' extracelluiar [Nae]. Extracellular refers to what is happening
outside the cell and intracellular refers
to what is happening inside the cell. The Nae/Ko-ATPase
requires ATp in order to funcrion. If the levels of ATp
inside the cell are low,.the ATPase activity will decrease and
eventually be inhibited. The remaining ATp left
within the cell will be diverted to reactions ihut more important to the cell's survival. we can eliminate
A' A low intracellular concentration of Nao will^r" choice
also lead tolnhibition of the ATpase, because if
pump to the extracellular space, then the enzyme cannot there is no Nao to
catalyze the transport of sodium out of the cell and
potassium into the ceil' we can eliminate choice B. The
,a-e .earoning applies to low extracellular concentrations
of Ko ' This allows us to eliminate choice c. A low extracellular
concentration of Na@ will not directly affect the
ATPase' because the ATPase is located within the cell. It is
not found on the outside of the cell. The correct choice
is D.

B is correct, primary active transport. In the second paragraph


of the passage, we learned that glucose can cross the
cell's membrane by facilitated diffusion' At the bottom of nlgur.
1 in the passage, we see glucose and Na@ being
transported across the cell's membrane in the same directioti
carrier-were a uniport, only one molecule would.be crossing trre
rn. protein i. ;;G as a symport. If the
mlinrane."u.."i"i
An antipoJ;l;"*, two molecules to
cross the membrane' but each is crossing opposite to the Jther. Glucose
is entering the cell along with sodium
through a symport mechanism. The sodium was actively transported
out of the cell. sodium can re-enter the cell
down its concentration gradient, allowing glucose to b" t.unrpoit"J *;tt
it by a secondary active transport process.
A,primary active transport systen uses the energy of ATP directly to move
a substance across a cell,s membrane.
we do not observe this happening in the oiglu.ose, especiafiy since we read in the third paragraph that only
three primary active transport systems have"ot"
been identified: one for Nao and K@ ions, one for a;t6 ;;r,';;;;
for Ho ions. The correct choice is B.

A is correct, low intracellular [Nao] allows sodium to be released from the carrier
protein. A high extracelluiar
Nao concentration is what allows glucose to bind to the symport on the extracelluiar
side of the cell,s membrane and
be transported into the cytosol. we can eliminate choice B and D.
if the affinity of the carrier protein for Nao were
high on the cytosolic side of the membrane, then sodium and glucose could noi
be released into the cytosol. when
Nae is released into the cytosol from the carrierprotein, the carrier protein's affinity
for glucose is reduced" we can
now eliminate choice C. The correct choice is A.

A is correct' (see the graph below)' Simple diffusion (SD) involves a substance moving
from a high concentration
to a low concentration' Diffusion of lipid-soluble substances can occur rhrough
u lipid-til;t"r, while diffusion of

Copyright @ by The Berkeley Review


The Berkeley Review
Specializing in MCAT preparation
Biology Nerve 6r Muscle Section I Answers

water-soluble substances wili occur through a transmembrane channel. The larger the graclient, the more a substance
will diffuse across the membrane. We would expect to see a straight line showing that as the gradient of a substance
increases, the flux of the substance across the membrane increases as well-

ti= //'/: FD

/
IGradient]

Facilitated diffusion (FD) requires a carrier protein. These cauier proteins have a limited number of carrier sites for
the substance that is to be transported across the membrane. Therefore, at a higher concentration gradient ail of
these carrier sites will be filled. The carrier will then be saturated (think of Michaelis-Menten kinJtics). At this
point, the rate of diffusion of the substance across the membrane will no longer increase with an increasing
concentration gradient. We would expect to see a plateau in the curve as soon as all of the carrier sites become
saturated. The correct choice is A.

6' B is correct, more Na@ to enter the cell than Ko to leave the cell. The resting membrane potential of a
neuromuscular cell is about -80 mV. If a positively charged Nao ion enters the cell, it will not make the inside of
the cell more negative, but rather make it more positive. The resting membrane potential will be depolarized, not
repolarized. Eliminate choice A. Table 1 in the passage tells us that the concentration of Ko is higher in the cell
than it is outside the cell. It also tells us that the concentration of Nao is higher outside the cell than it is in the cell.
Therefore, when the acetylchoiine receptors open and allow for the flow of Ko and Nao ions, we find that Nao ions
enter the cell, while Ko ions leave the cell down their concentration gradients. This allows us to eliminate choice C.

The question tells us that two molecules of acetylcholine will bind to the acetylcholine receptor. This is a
chemicaliy mediated and activated response. The channel in the acetylcholine receptor is not activated (opened) by
an electrical stimulus (i.e., a depolarization). If that were the case, acetylcholine would not need to bind to the
receptor. We can eliminate choice D.

B.v the process of elimination, we arrive at choiceB. But how do we know that more Nae will enter the cell than
Ko will leave the cell? The electrochemical gradient for Nae is greater than the electrochemical gradient for Ke .
The electrochemical gradient that aliows an ion to pass through a membrane is simply the difference between the
membrane potential (E.) and the equilibrium potentials for the ion in question (Qon). The resting membrane
potential for our cell is about -80 mV. The resting membrane potential for Ko is about -92mY, whiie the resting
rembrane potenlial for Na@ is about +58 mV. Clearly, the electrochemical gradient for Nao is greater than that for
K3. The correct choice is B.
D is correct, a high-resistance electrical pathway. Gap junctions join the cytopiasm of one cell to the cytoplasm of a
r:ighboring cell through intramembrane proteins. These proteins (connexons) contain 6 subunits that form a central
:ore that allows molecules with molecular weight up to 1500 to pass through. Molecules iike ATP and ions like
\a3 can easily pass through these channels. This helps to establish a cytopiasmic continuity between the cells.
\\-hen an action potential reaches a gap junction, electrical coupling between the two cells occurs, and the action
propagates to the next cell. This type of coupling is important in heart tissue, where entire collections of cells must
contract in a coordinated fashion. Therefore, a gap junction must provide a iow-resistance (not a high-resistance)
electrical pathway that enables current to pass from one cell to the next. The correct choice is D.

5 C is correct, increased sodium concentration gradient. Inhibition of the electron transport chain means that the
amount of ATP being synthesized is substantially reduced. If the amount of ATP is reduced, then the amount of
energy transferred from ATP to the membrane ATPases is reduced. The primary active transport systems (those
involving the Nae/Ke -ATPase, Ca2o-ATPase, and He- e-ATPase) ali show adecrease in their activity. In secondary
active transport, the energy that was stored in the Nao concentration gradient across the cell's membrane (high
extracellular [Nae] and low intracellular [Nao]) is used to transport molecules like glucose and amino acids into

Sl right @ by The Berkeley Review 67 The Berkeley Review


Specializing in MCAT Preparation
Biology Nerve 8f Muscle Section I Answers

the cell and calcium out of the cell. If the synthesis of ATP decreases, the Nao concentration gradient across the
cell's membrane is not as great (i.e., it decreases, not increases), and the rate of secondary active transp ort decreases .
Low levels of ATP also mean that the Ca2 o-ATPase cannot adequately pump calcium out of the cell. Since calcium
is also transported out of the cell (against its concentralion gradient) by a secondary active transport system that
utilizes the sodium concentration gradient (which is now rather low), the levels of intracellular calcium begin to
increase. Calcium is able to diffuse back into the cell though calcium channels. The correct choice is C.

D is correct, I, m, and iV only. The sympathetic system is important in getting the body ready for a perceived
stressful situation. This response is a general response, affectrng ail parts of the body almost simultaneously. This is
essential to the survival of the animal. You wouldn't want to have to wait for each organ or muscle to detetmine that
this is a stressful situation while a lion is running at you! But the parasympathetic system has the luxury of
determining specificaily for each structure it innervates whether the job is done during this time of stress and when
it's time to return to basai levels. The parasympathetic system can do this type of individual monitoring, partialiy
because of the proximal location of its ganglia next to the structure in question. The correct choice is D.

10. D is correct, I, II, and IV on1y. All three situations are interpreted by the body as demands to increase particular
functions beyond their normai basal levels. In other words, they represent a stress to the body. Sweating on a wann
day is a response to release heat and cool down the external body. On cold days shir,ering and piloerection are
stimulated to increase heat in the body. Both of these reactions are instigated by the sympathetic system.
Furtherrnore, running is a stressor on the body. The correct choice is D.

11. D is correct, penile erection. Penile erection can be caused by direct stimulation invoiving penile
mechanoreceptors, or it can be caused by central nervous system activity stemming fiom sights and smells or even
thoughts and emotions. In either case, input reaches the neurons of the penis. Parasympathetic nervous system
activity is increased while sympathetic nervous system activity is decreased. When parasympathetic activity
increases and sympathetic activity decreases, vascular dilation of the arterioles occurs so that the tissues of the penis
become engorged with blood. As erection continues, the veins leaving the penis become compressed and little blood
is allowed to leave, thus maintaining the penis in the erect state. The correct choice is D.

12. B is correct, I, II, and III oniy. The adrenal gland sits on top of the kidney. Cortisol, a glucocorticoid produced by
the adrenal cortex, controls various aspects of metabolism. An increase in piasma concentrations of cortisoi will
lead to an increase in giuconeogenesis (i.e., the synthesis of glucose from precursor moiecules like iactate), a
decreased uptake of glucose by celis, an increase in protein catabolism, and an increase in triacylglycerol breakdown
(i.e., the release of free fatty acids). Aldosterone, a mineralocorticoid produced by the adrenal cortex, controls
electrolyte balance by stimulating sodium reabsorption and potassium secretion in the kidney. Sodium reabsorption
also leads to water reabsorption in the kidney. The correct choice is B.

13. B is correct, increased heart rate. The vagus (1Oth cranial) nerve is a component of the parasympathetic nervous
system. Think of the parasympathetic system as being passive. The iris muscle of the eye will contract, making the
pupii smaller. The motility and secretions (e.g., HCl from the parietal (oxyntic) cells) of the stomach will increase,
^especially
after a meal. An increased heart rate and contractility is due to the sympathetic nervous system (think of
the.figt'rt-or-flight response). The correct choice is B.

14. C is correct, efflux of Ke balances the influx of Nae. A small depolarization in the membrane will allow some
Na@ to enter the cytoplasm of the cell. At the end of the second paragraph in the passage we see that the relative
permeability of the membrane to Ko is roughly 10 times greater than it is to Nae . Since there is more Ke in the
iCp tt un Xa@ ifrom Table l), we see that more Ko will flow out of the cell than Na@ into the cell. Therefore, if
there is a smal1 depolarization, then the influx of NaO will be more than balanced by the efflux of Ko ' If the efflux
of Ko is greater than the influx of Nae , the explosive nature of an action potential will not resuit. What this means
is that the threshold potential for the generation of an action potential is that point where the influx of Nae exactly
matches the eff'lux of K@. Remember, there is a higher concentration of Na@ in the ECF and a higher concentration

Copyright @ by The Berkeley Review 6a The BerkeleY Review


Specializing in MCAT Preparation
Biology Nerve 8r Muscle Section I Answers

of Ke in the ICF. The nanosecond that there is more of an influx of Nae than there is an efflux of Ke. an action
potentiai will be generated. The correct choice is C.

15. B is correct, -65 mV. As stated in the fourth paragraph of the passage, the threshoid is about 15 mV away from the
E-. In Figure 2 of the passage, the E* is at -80 mV. The threshold is given by the dashed line labeled with the
number 2. Therefore, the threshold potential must be -65 mV. The correct choice is B.

16. C is correct, an increase in the opening of fast n gates in neighboring Ko channels. As depolarization takes piace,
Nae rapidly enters the cell (Table 2inthe passage). In order for Nae to enter the cell, the f'ast acting ar gate must
open to allow Nao to rush into the cell down its concentration gradient. As Nae enters the cell, it causes further
depolarization and allows more m gates to open. This is the explosive nature of the action potential. The absolute
refractory period begins at about the time the depolarization of the membrane takes place. Another action potential
cannot be generated at this time because of the opening of the m gates from the first action potential. This allows us
to eliminate choices A, B, and D. Depolarization of the membrane will lead to an eventual opening of the n gates.
The problem is that the /? gates are slow and not fast. The correct choice is C.

11. A is correct, m gate open; h gate closed; n gate open. Table 2 in the passage says that during the repolarization
phase Ko is flowing out of the cell and down its concentration gradient. If Ko is flowing out of the cell, the n gate
must be open. Looking through our choice of answers, we see that the iz gate is closed in choice D. Therefore, we
eliminate choice D.

We know that during repolarization, we do not want Nae entering the cell. If Nao enters the cell as Ko leaves, we
will not be abie to obtain the resting membrane potential of the cell as quickiy as we would like. Which one of the
Nao gates is closed, or are they both closed? The iast paragraph of the passage teils us that during depolarization
the ft gate begins to slowly close. Let's assume that the /r gate has closed as repolarization begins. If this is the case,
then we can eliminatb choice C.

However, all through depolarization Nae is entering the cell, which means that the m gate is remaining open. By
the time the peak of the action potential has occurred, the h gate has closed, the m gate is still open but beginning to
close, and the n gate is open. The correct choice is A.

ns_ .\ is correct, repolarize. An action potential will initially depolarize the resting membrane potential (E6) and cause
that potentiai to be less negative. As the Nao gates close and the K@ gates open, the membrane potentiai is being
repolarized toward the E* of the cell. Because of the high permeability to Ke during repolarization, the membrane
polential becomes even more negative than the E*. This is called hyperpolarization. Once the cell has completed
n;"'perpolarized, it will return to the En., value of the cell. This is what is occurring during the last half of the relative
r:;ractory period. Whenever a membrane potential returns to its true E,o, the process is referred to as a
:erolarization. The correct choice is A.

:l' B is correct, Nae and K@ ions are not responsible for the release of the neurotransmitter. The passage states that
TT\ blocks the Nao channels, while TEA blocks the Ko channeis. If this is the case, then the depolarization of the
:::re cell membrane will not occur. However, if the presynaptic ending is stimulated with a depolarizing signal, a
:,- srs\ naptic potential is produced. The last paragraph of the passage says that a neurotransmitter is released from
-:-- presynaptic terminal, diffuses across the synaptic cieft, binds to the postsynaptic terminal, and produces a
:,-sr:ynaptic potential. Since we are using a stimulus (signal) of depolarization to cause the reiease of a
:,=urorransmitter, it must mean that Nae and K@ are not responsible for the reiease of the neurotransmitter. If they
;:.e needed, then the stimulus that we applied would have no effect on the generation of a postsynaptic potential.
'-.--':
.-an eliminate choice A.

i'
nentioned above, TTX and TEA block Nae and K@ ion channels, respectively. They do not allow
3-it.rirrsrrritter release. If they did, we would not have needed the depolarizing stimulus. We can eliminate choice
-
{-- Because the channeis for Nae and K@ are biocked, it does not (necessarily) mean that the concentrations of ICF
!.: "nd ECF Nao are going to increase. Remember, the membrane is permeable to both Nae and Ko (see the
,"-;:r:l paragraph in the passage). These ions wili tend to diffube down their concentration gradients and the

ilrr '"T
li1 The Berkeley Review 69 The Berkeley Review
Specializing in MCAT Preparation
Biology Nerve Er Muscle Section I Answers

NaeiKe-ATPase pump will redistribute them again. Blocking the Nao and Ko channels is analogous to not having
a stimulus. The nerve cell will be at its resting state. We can elirninate choice D. The correct chJice is B.

20. B is correct, glutamic acid decarboxylase. The cx-carboxyl group of glutamic acid is being removed. When CO2 is
removed from a molecule, the reaction is called a decarboxylation and is carried out by a decarboxylase enzyme.
If
C02 were added to a molecule, it would be a carboxylation reaction, which wouli be carried out bv a carboxvlase
enzyme.
COOH H
@t Ot
HrN- C- H
Enzyme H3N - c- H
I

CH, CH"
t- t-
CH, CH,
t- I

COOH COOH

Glutamic acid GABA

Since the CO2 is being removed from glutamic acid, the enzyme that catalyzes this reaction is a
_ulutamrc acid
decarboxvlase. Note that once GABA is fbrmed, the numbering of the carbon atoms changes. What was the o,-
position becomes the y-position. There is no carboxyl group at the
1-position. Instead, there"is an amino group. If
we were to remove the amino group of GABA, the four carbon compound would be butyric acid. Hence
tt . ni.",
y-aminobutyric acid. The correct choice is B.

C is correct, small molecuiar size and high lipid solubility. The passage tells us that the base fbrm of the local
anesthetic, being lipid-soluble, can penetrate various tissue barriers. The blood brain baryier is certainly one of these
tissues. The reason lipid soluble substances are able to penetrate the blood brain barrier is that the leaflets of the cell
membrane, including those of the capillary endothelium, are composed of lipid molecules. Lipid soluble molecules
can thus diffuse through the two-dimensional liquid and through ihe membrane. Furthermorg it is logical
tliat the
smaller the molecule, the greater its ease in crossing a membrane. The correct choice is C.

7',' A is correct, the higher the pKs of the local anesthetic, the lower the concentration of base in the tissue. The
passage tells us that the concentration of base or cation in the solution depends on the pKa of the local
anesthetic.
Remember that local anesthetics are weak bases. Look at the structure of lidocaine. When it is protonated, it carries
a positive charge. In other words, the cation is the acid form. Therefore, when we have a high pKu, we have
a small
Ka. This, of course, means the reaction does not go very far to the right. In other words, we have a large
concentl'ation of acid and a small concentration of base in the tissue. The correct choice is A.

23. C is correct, local anesthetics decrease threshold for electrical stimulation in nerve tlbers. We are looking for a
statement that is least compatible with the way in which local anesthetics act. Remember that local anesthctics act to
produce a reversible, local nerve conduction block. Therefore, we are looking for a statement that contradicts this
goal. A decrease in the threshold value indicates it is now easier to initiate a nerve action potential. Crearing
conditions where action potentials are formed more easily does certainly contradict the goal of the locai anesthetic.
The correct choice is C.

2.1. A is correct, the onset of lidocaine is more rapid, because more exists in its base fbrm at body pH. We know from
the passage that the base form is lipid-soiubie, and therefore it penetrates the tissue. One can logically conclude that
the local anesthetic that exists more in its base form (as a percentage) at tissue pH will have a quicker onset of action
because more penetration of tissue will occur. That is why the pKos of these local anesthetiis are important. The
local with the lower pKu will have a higher percentage of molecules in the base form and thus have a more rapid
onset. Lidocaine has a more rapid onset because more of it exists in its base form. The correct choice is A.

25. D is correct, the amount of ionized lbrm will be increased, and the local anesthetic will be trapped in the tissue.
Again, the local anesthetic penetrates the tissue in its neutral, base form. If acidosis (lowering of the pH) occuned in
lissue after penetration of the local anesthetic, this would drive the base form back into its acidic form. In other
rvords, a lower percentage of the molecuie would exist in the base form. The problent with this is that the acidic

Copr ri_uht @ by The Berkeley Review 70 The Berkeley Review


Specializing in MCAT Preparation
Biology Nerve & Muscle Section I Answers

form is charged and thus cannot escape the tissue barrier. The trapping of the charged form of the local anesthetic
can contribute to an increase in the degree of toxicity. The correct choice is D.

26- C is correct, is biotransfbrmed by reactions occurring in the liver. Lidocaine is injected (rather than taken orally) to
avoid biotransformation reactions which occur in the liver. If the drug is taken orally, it is not destroyed by the
harsh conditions of the stomach. For example, the drug is not a protein, so there is no concern over the protease
content of the stomach. After absorption across the intestinal wall occurs (most wili probably occur in the smal1
intestine), the drug enters into the hepatic portal system and is transported to the liver. Remember that the liver
contains enzymes (we do not need to concern ourselves with the exact reactions) which are involved in modification
of substances. Such modification can render the iocal anesthetic inactive. Furthermore, we wish to avoid having a
local anesthetic circulating in the blood stream. The correct choice is C.

)1 D is correct, The circular muscle of the iris is relaxed, the radial muscle of the iris is contracted, and the ciliary
muscle is relaxed. Although this answer seems complicated, we can figure it out. First, figure out what the iris is
doing. In the dark, the pupii will be large to iet in light. The radial muscle will contract, and the circuiar muscle will
relax. This imrnediately eliminates choices A and C. We must now figure out what the ciliary muscle is doing to
choose between choices B and D. The ciliary muscle, as rve are told in the passage, is relaxed for vision beyond
about 20 feet. Since the night sky extends beyond 20 feet, the ciliary muscle is relaxed. Eliminate choice B. The
correct choice is D.

28. A is correct, the lens is less able to focus the light rays from near objects. Remember, the "default" shape of the
lens in the relaxed state is flatter, for far vision. This really does not change with age. Choices C and D are
incorrect. Near vision requires a flexible lens that can plump up into a convex shape to aliow focusing on near
objects. A less flexible lens is less able to focus on near objects. Choice B is incorrect. The correct choice is A

to B is correct, sympathetic nerves are stimulated, and parasl'mpathetic nerves are inhibited. What situation leads to
large pupils? The radial muscles must contract under sympathetic stimulation. The circuiar muscies must be
relaxed under parasympathetic inhibition. Choice A is backwards and thus incorrect. Two different stimuli are
required, so choices C and D are incorrect. The correct choice is B.

30. A is correct, retina. Rays converge and focus on the retina. The cornea is a clear, tough membrane covering the
front of the eyeball. Choice B is incorrect. The optic nerve is responsible for transmitting visual information to the
brain, but it does not focus the image. Choice C is incorrect. The vitreous body is the inert stuff inside the eyeball.
Choice D is inconect. The correct choice is A.

31. B is correct, II only. As the object moves closer, the lens must become more convex. This is accomplished by
contraction of the ciliary muscle. Choice II is correct. The ligaments do not contract, they merely pull the lens taut
by defauit. Choice III is incorrect. The pupil responds to changes in light, not in focusing. Choice I is inconect.
Since choice III is false, choice D is incorrect as well. The correct choice is B.

1) C is correct, pink. The condition of not having epitheliai pigment is call albinism. A person or animal with
albinism is called an albino. Most strains of laboratory rats are albinos. They have pink eyes, due to the color of the
blood vessels visible in their iris. Also, you may have seen white bunnies with pink eyes. Pigment covers up the
color of the blood vessels in people and animals who don't have albinism. Black eyes corrtain the most pigment.
Choice A is incorrect. Green eyes contain some pigment, as well. Choice D is incorrect. You may not know about
the lab rats, but a person would not have white eyes due to the presence of blood vessels in the iris. Choice B is
incorrect. The correct choice is C.

33. C is correct, by providing a non-uniform corrective lens. Since the person has an irregularly shaped lens or cornea,
the corrective lenses used should be shaped to correct these problems and allow focusing of the image on the retina.
This would require that the corrective lens have a compatible non-uniform shape. Convex lenses correct
farsightedness. Choice A is incorrect. Concave lenses correct nearsightedness. Choice B is incorrect. A uniform
lens would not help the person with astigmatism. Choice D is incorrect. The correct choice is C.

Copyright O by The Berkeiey Review 7t The BerkeleY Keview


Specializing in MCAT Preparation
Biology Nerve & Muscle Section I Answers

34. D is correct, lipids making up the cell membrane are uncharged. Cell membranes are made
up primariiy of lipids.
Lipids contain very few charged groups, cannot carry current, and have a high electrical resistance.
Materials with
high electrical resistance are known as insuiators. The lipid layers of the
flasma membrane are regions of high
electrical resistance separating two water compartments of low ."iistun.". The correct choice is D.

35. D is correct, negative, with the excess charge representing a very small fraction of the total number
of ions inside
and outside the cell. This problem requires outside knowledge about the resting membrane potential
of a typical
neuron. The typical re.stilg membrane potential lies somewhere between -40 mv to -75 mV. The
minus sign
indicates that the potential is negative with reference to the inside of the cell. In other words,
there is an excess of
negative charge inside the cell relative to the outside. This eliminates choices A and
B. Now, do these excess
charges represent a small or large fraction of the total ions in the cell? The excess
charges are but a very small
fraction of the total number of ions inside and outside the cell. The correct choice is D.

36. B is correct. We are looking for a picture that represents the potassium equilibrium potential.
The equilibrium
potential will exist where the fbrce due to the concentration gradient is equal
to tlie electrical force gradient.
Potassium ions have a higher concentration inside the cell, elimilnating choices C
and D. To discriminate between
choice A and choice B, we simpry iook at the magnitude of the arrows.

Again' at the equilibrium potential, the two forces will be equal. Answer choice A represents the
resting membrane
potential. The correct choice is B.

-'\t. C is correct, sodium ions diffuse into the cell. One can arive at this answer by thinking about
the membrane
potential and the direction in which it is going. We are told the resting membrane potentiailies
above that for the
potassium equilibrium potential. This means that some positive charge is coming
back into the cell, making the
potential higher. of the possible answers, the o-nly_ way this is going to -hupp"n (givln
the true ion concentratiois) is
to have sodium coming into the cell. This is indeed the case. There is a permeability to sodium
ions over the plasma
membrane, and this contribution causes a rise in the membrane potential. Chloride lons have
a higher conceniration
outside of the cell, so if they move down their gradient, they ;i11 decrease the membrane potential.
The correct
choice is C.

38. C is correct, oxidation of nutrients. The pump that transports sodium and potassium ions does breakdown ATp to
couple the energy of that reaction to the work it must perform in transporting the ions. However, the question
alludes to the ultimate source of the energy. Therefore, we need to think about how we are getting the ATp that is
used by this pump. This answer comes from our knowledge of metabolism and an undeistanOing that food is
oxidized in the course of glycolysis and the Krebs cycle. The electrons released are eventually transpoited down the
electron transport chain and are coupled to a proton gradient, which results in the fbrmation-of ATF. The correct
choice is C.

39. C is correct, a steady state. The constancy of the resting membrane potential is best described as a steady-state
potential. The answer really comes down to two choices. Is it an equilibrium, or is it a steady state? With an
equilibnum, no energy input is needed to maintain the state. Is this the case? No. We use the eneigy of ATp to run
the Nao/Ke ATPase pump that maintains the concentration gradients of the these ions. Since there is energy used,
this cannot be described as an equilibrium. When a state is constant, but energy is used to maintain that statE, this is
termed a steady-state system. The correct choice is C.

40. D is correct, the new resting membrane potential is not affectqd at all by the chloride ion. We are told that the
plasma membrane of many celis are permeable to chloride ions and do not contain chloride-ion pumps. Therefore,
in these cells, the membrane potential set up by other ions will act on chioride ions. The inside negativity moves

Copyright @ by The Berkeley Review 72 The Berkeley Keview


Specializing in MCAT Preparation
Biology Nerve & Muscle Section I Answers

chloride out of the cell until a concentration gradient develops. Looking at Table 1, we see the concentration of
chloride is larger extracelluiarly. This concentration gradient will force ihloride ions into the cell. However, the
diffusion fbrce will be exactiy equal to the electrical force pushing chloride ions out of the cell. The resuit is that the
equilibrium potential for chloride ions is equal to the resting membrane potential, and the chloride ion makes no
contribution to the magnitude of the membrane potential. The correct choice is D.

4I. D is correct, CaZ@. Calcium ions are the link between depolarization of the presynaptic membrane and
neurotransmitter reiease. Depolarization of the terminal causes voltage-sensitive calcium channeis in the membrane
to open, and calcium diffuses into the axon terminal from the extracellular fluid, including that in the synaptic cleft.
The increase in the calcium level in the terminal causes vesicles filled with neurotra*nsmitter to flse with the
presynaptic membrane and release their contents into the synaptic cleft. The correct choice is D.

42. C.is correct. In the question, we are given a graph of a binding assay for the muscarinic acetylcholine receptor,
where the substrate is muscarine. The question tells us that scopolamine, a competitive inhibiior, is added
to the
solution and the assay is run again. A competitive inhibitor will compete for the same site on the receptor as
muscarine. It will not remove or incapacitate any of the receptors (this would be a non-competitive inhibitor).
Since
the total number of receptors remain in place, the levei of maximum binding will not &ung". This eliminates
choices A and B. What will change is the apparent affinity of the receptor for muscarine, whicl can be represented
by half'-maximum binding. The apparent affinity of the receptor for muicarine will decrease, because the inhibitor is
competing for the same site. Therefore, we will see an increase in the half-maximum binding site of muscarine,
because it now takes a higher concentration of muscarine to reach the half-maximum binding ,It". R"-"mber, the
tell-tale sign of a competitive inhibitor is that if we increase the concentration of either substra"te. one of them should
be able to out-compete the other and reach maximum binding. The correct choice is C.

-13. C is correct, an excitatory post-synaptic potential is generated. We are told from the question that the nicotinic
acetyicholine receptor is essentially a sodium channel. Therefore, when acetylchoiine binds to a nicotinic
acetylcholine receptor, we are essentially activating a sodium channel. When a sodium channel opens, what
happens? Since sodium always has an extracellular concentration higher than its intracellular concentrati,cn, sodium
comes into the cell and causes a net movement of positive ions in the cell. At this point, we can eliminate choice D.
This causes a slight depolarization and is known as a excitatory post-synaptic potential (EPSP). Eliminate choice B.
This is a graded potential, and it is not an all-or-nothing action potential. in order to generate an action potential, the
depolarization must be beyond a threshold value set by that particular cell. Usuallyl several EPSps added together
will eventually generate an action potential at an area known as the axon hillock, where a hish igh concentration of
sodium channels are found. The correct choice is C.

14. C is correct, a net nicotinic and parasympathetic nervous system excitation. We want to offer the person who is
addicted to nicotine the "high," without the cardiovascuiar risks associated with nicotine. Therefore, we wiil want to
stimulate the nicotinic receptors in the central nervous system to achieve this sensation. The problem is that
nicotinic receptors are located in the pre-ganglionic synapse of both the sympathetic and parasympathetic nervous
systems. We might think that since both are stimulated, they simply cancel each other out. However, we know that
nicotine causes stress on the heart, so the nicotinic receptor in the sympathetic nervous system must be dominant.
With this in mind, let us look at the two drugs added. The first is physostigmine (eserine). This acetylcholinesrerase
inhibitor will cause increased levels of acetylcholine in both nicotinic and muscarinic receptors. This will give us
the nicotine "high" that we want. Yet we do not want to stimulate all the muscarinic receptors, so we add
scopolamine. This acts to block all muscarinic receptors. This is what we want. We get rhe familiar feeling caused
by nicotine and have blocked all muscarinic receptors. We are not done, though. We are still left to deal with the
cardiovascular stress. The physostigmine will increase acetylcholine (ACh) levels at the first synapse in the
sympathetic nervous system. But in the parasympathetic system, it increases ACh levels at both the nicotinic and
the muscarinic receptors (remember that scopolamine does not work directly upon heart tissue). Therefore, we have
two stimuiations in the parasympathetic system while we only have one in the sympathetic system. Therefore, we
are left with a net nicotinic stimulation with a parasympathetic excitation. The correct choice is C.

-t5. A is correct, nicotine stimulation of the nicotinic acetylcholine receptor is dominant in the sympathetic nervous
system. We would think that because the first synapse in both the sympathetic and parasympathetic are nicotinic that
nicotine would stimulate both of them equally, and therefbre there would be no stress on the heart. However, we

- rprright @ by The Berkeley Review /c The Berkeley Review


Specializing in MCAT Preparation
Biology Nerve & Muscle Section I Answers

know this not to be the case, as nicotine places considerable stress on the cardiovascular
system. Therefore, nicotine
stimulation of the nicotinic acetylcholine receptor in the sympathetic nervous system
must be dominant over the one
in the parasympathetic nervous system. There is no evidence in the passage fbr the
claim made in answer choice C.
Cardiovascular stress is not brought about by activity in the centrai n".uou, system,
but by activity in the peripheral
nervous system. we can eliminate choice D. The correct choice is A.

46. D is correct' axons' moving a greater distance in_ the sympathetic nervous system.
The question tells us we have a
molecule that will be moving in a retrograde (backward) fashion from the synapses
io the heart between both
sympathetic and parasympathetic nervous systems. The molecuie will thus ue movinj
*r.ougrr axons (eiiminating
choices A and B), because the axons, not the dendrites will be closest to the organ. "In
other words, an electrical
signal reaching the organ will travel through a nerve's axon to reach the synapsJbetw"""
tr,. nerve and the organ.
So the next question becomes one of distance. Remember that in the sympathetic
nervous system, the preganglionic
fiber is short while the postganglionic fiber is long. The molecule wili be traveling through
the postganglionic fiber
to reach the first synapi" (the iirst synapse is th{unction between the pre- and postganglionic
fibers). Therefore,
the molecule will travel a longei'distance in the sympathetic nervous system.
The correct choice is D.

47. D is correct, the axons are myelinated by oligodendrocytes. This question calls on our knowledge of the nervous
system outside of what is stated in the passage. We are looking for thb most likely
explanation for the increase in the
frequency of the action potential' Myelinated nerves have th*e ability to increase thl
frequency of action potential
conduction. Therefore, we can nalrow the options down to choices Ctr D. The question
tiren becomes: which celis
are responsible for the myelination? In both cases, glial cells are responsible
for laying down the myelin sheath. In
the central nervous system, these cells are called oligodendrocyter, r"itit" in the pNS tt"y
u." called Schwann cells.
Since we are talking about nerves located in the CNS, the best unr*", becomes choice D.
The correct choice is D.
48. D is correct, afferent pathway, containing nerve axons from both eyes. Again, this question requires
us to draw on
our knowledge of the nervous system, and in particular, the Cin. must remember that the information ieading
"y".
towards the brain is found in the afferent nerJes. Information leading away from the CNS is found in the efferent
nerves. Therefore, answer choices A and B can be eliminated. Now, the qlestion becomes whether
the optic tract
contains information from one eye or two. The answer is two. Information from the nasal
hemiretina (medial half
of the retina) of the left eye cross the optic chiasm and enters the right optic tract. The right optic tract
is also made
up of nerve fibers originating from the temporai hemiretina of the r'ight -eye. In that waylthe
right side of the brain
processes information from the ieft side of the visual world. Because the tract
contains informaiion from both eyes,
D is the correct answer. The correct choice is D.

49. C is correct, this area_is free of photoreceptors. The question is fairly straightforward. It asks for the best
explanation of the "blind spot." The biind spot is the area on the retina where the 6ptic nerve bundle leaves the eye.
That is not the reason why the area is termed the blind spot. The region is blind because it has no photoreceptors. If
there are no photoreceptors to be found, there can be no transduition of light into a visual image. The correct
choice is C.

50. C is correct, acetylcholine. We are told from the passage that the neurons which make up the Edinger-Westphal
nucleus are parasympathetic neurons. Therefore, this question is really testing one's knowLdge of the
neurotransmitter used by parasympathetic neurons. We cannot be expected to know from the question al6ne which
neurotransmitter these neurons use. However, we are supposed to be aware that neurons that are parasympathetic
use the neurotransmitter acetylcholine. The correct choice is C.

51. A is correct, the optic nerve of the left eye is intact, but the efferent limb of the left eye is damaged. The question
tells us we see a consensual but not a direct response. We can therefore conclude that ihe optic nJrve of the left eye
is intact because the optic nerve of the right eye is not involved in the response. The ."rponr" involves information
going down the left optic nerve to the pretectal area. From the pretectal area, n"u.ons project bilateraliy to the
Edinger-Westphal nucleus. Axons from neurons in the nucleus innervate the ciliary ganglion. We see a consensual
response in the right eye. We can therefore conclude that the bilateral projection and the efferent pathway to the
right eve are unharmed. In addition, we can conciude that there is some problem with the efferent patirway to the
Ieft eye. We are not seeing a constriction of the pupil in response to the light being shone. We aie left with the

Copyright O by The Berkeley Review 74 The Berkeley Review


Specializing in MCAT Preparation
Biology Nerve & Muscle Section I Answers

fbllowing conclusion: The ieft optic nerve is intact, but the efferent pathway of the left eye is somewhere and
somehow damaged. The correct choice is A.

it B is correct, right optic nerve. The light shone in the right eye elicits neither a direct or consensual response. This
most likely means that there is damage to the right optic nerve. The result of the damage is that the infbrmation
leading away ftom the eye is unable to elicit a reflex which results in the constriction of both eyes. This can be the
only explanation for the inforn'ration given in the question, because light shone in the other eye elicits both a direct
and a consensual response. The correct choice is B.

,{ is correct, anterogressiveiy, via fast axonai hansport. Recali from the passage that transport from the soma
ton'ards the axon terminal is known as anterograde axonal transport. Therefore, this eliminates choices C and D.
\ext. is the neurotransmitter rroving via fast or slow axonal transport? Recall that neuropeptides are packaged into
membrane bound vesicles as they are formed near the soma. From the passage, we know that membrane-bound
c.rganelles are transported rapidly by fast axonal transport. While the vesicle may not be technically considered an
orsanelle, given the choice between membrane-bound organelle and cytoplasm, a vesicle is best aligned with the
:rembrane-bound organelle system. Our best choice includes fast axonal transport. The correct choice is A.

C is correct, Ca2o. We are told from the passage that organelies interact with microtubules through a linkage
-'-nilar to that between thick and thin filaments of skeletal muscle cells. Recall that thick filaments refer to myosin,
'rile thin filaments refer to actin. Recall that calcium is needed 1br actin and myosin interaction, as calcium
:tltoves the troponin-tropomyosin complex which covers the actin binding sites. Since the interaction of the
',-:anelles with the microtubules is simiiar to that of actin/myosin, one can best conclude that calcium triggers the
..=.,:r;rction. The correct choice is C.

C is correct. complete translation of the peptide in the cytoplasrn. Recall that peptides that are produced by a ceil
- .:e purpose of secretion (neulopeptides are certainly an example) are processed through the rough endoplasmic
-.---:1um. The peptide being translated will have a signai peptide that attracts a cytoplasmic signai receptor particle
SRP . When the SRP attaches to the signal peptide, apause in translation will occur. The translation does not
.:.:r aqain until the SRP complex (SRP/signal peptide/ribosome) binds to an SRP receptor located on the rough
a-. \\-hen the SRP complex binds to the SRP receptor, translation begins aiong with transport of the new protein
:: .:.3 lumen of the RER. At that point, the signal peptide is cleaved off and the proteins are modified in both the
: :F. ::d the lumen of the Golgi apparatus. From the trans face of the Golgi, the peptide buds off as a vesicle.
--,:, l:rs to this hypothesis, there is no complete translation of the neuropeptide in the cytoplasm. The correct
:ir ,ice is C.

- is torrect. pyruvate. Glucose will go through the glycolytic pathway and the end product of glycolysis is
I -- :.3. Recall that giycolysis occurs in the cytosol of cells. This is a very straightforward question asking us to
':- :.i'.::: that pyruvate is transported over both the outer and inner mitochondrial membranes to enter the matrix.
T:c i,,rrrect choice is C.

- r: i,rrrect. so1.na to trace the synaptic endings of neurons. The marker substance moves anterogressiveiy.
- . - . - '_l :o the passage, the marker substance will move from the soma toward the synaptic ending. If we are
- :.: j.:r :r tracing a neural pathway, we should use this marker to identify synaptic endings. In other words, we
, - .,':-': this marker in a given cell body, and it will trace the axon and the synaptic endings of the neuron" If
: - : -: ::: narker into the soma, it will not trace the dendrites because of its direction of movement. Therefore,
; --- :.::-lnate choice C. Furthermore, we do not want to inject the marker by the synapse, because there is
' - ; -:. :r trace. The correct choice is D.
I * i rrect \\'e are told from the graph in the question that action potentials arise in a postsynaptic cel1 after the
... - . : '-'::.. lcholine. We then add acetylcholinesterase, which breaks down ACh. We must look at the change
: ::.:::r --ithe action potential response. Why? Recall that action potentials are all-or-nothing events, so they
- -:,- :3 rn amplitude. We can easily eliminate two of the choices. We then look at the pattern, and in this
, : -. .:----lencv of response. If we are going to have lower leveis of ACh due to the addition of
. : - - .:!.:iase. r.ve should see a less frequent train of action potentials in the postsynaptic cell. The correct
1 -:L:f)

:;:i Review /b The Berkeley Review


Specializing in MCAT Preparation
Biology Nerve & Muscle Section I Answers

59. C is correct, equally common in both men and women. The passage tells you the defect is an autosomal dominant
one. This means it is not sex-iinked, so eliminate choices A and B as incorrect. We also learn iiorn the passage that
the age of onset is from 35 years of age. Eliminate choice D. The correct choice is C.

60. D is correct, voluntary movements. The hypothalamus produces releasing hormones that aff'ect the pituitary.
Choice A is incorrect. The control of speech is localized in the cerebral cortex. Choice B is incorrect. The
parasympathetic nervous system (PNS) is not controlled by the basal ganglia. In the second paragraph, we learn that
symptoms of the disease include rapid, uncontrolled, jerky movements. The correct choice is D.

67. D is correct, 4-aminobutanoic acid. This molecule is a 4-carbon organic acid. The skeletal structure is called
butanoic acid. The carbon of the carboxyl group is designated 1. This means the amino group is attached to the 4th
carbon. This gives the name 4-aminobutanoic acid. The correct choice is D.

62. D is correct, glutamate. This information is given in the second paragraph of the passage. A key missing enzyme
that leads to GABA deficiency is glutamate decarboxylase. If we correctly named the previous molecule, GABA,
then we can imagine adding on a carboxyl group on the C-zl carbon to make glutamate. Choice A is incorrect:
histidine is the precursor of histamine. Choice B is incorrect. Choice C is incorrect: tyrosine is the precursor of
epinephrine and norepinephrine. The correct choice is D.

63. C is correct, 507o. Since the gene is autosomal dominant and the father is not affected, he is homozygous recessive
for this disease. The gene is rare, and only one of the woman's parents shows a tamilial history for the disease. The
woman is probabLy heterozygous. Therefore, she either passed on the dominant, disease-causing gene or the
recessive gene. The chance is 50i50, since she has 2 X-chromosomes and only one is affected. The correct choice
is C.

64. B is correct, the vesicles of presynaptic neurons. Neurotransmitters are synthesized in presynaptic neurons, stored
in vesicles there, and released by the arrival of a neural impulse. The released neurotransmitter crosses the synaptic
cleft and interacts with the receptors on the postsynaptic neuron's membrane. The correct choice is B.

65. A is correct, cone cells. This answer can be arrived at using previous knowledge, and not information specifically
from the passage. The fact is that cones are used for color vision, while rods are used for black-and-white vision.
The question asks about seeing in the dark, which definiteiy qualifies for a lack of color. Of course. one may see
sonre color in the dark and the cones would be responsible, but the question asks which cell is ntost likeb, y1s1
involved. Consider the other answers. We have already discussed that rods play a role in black-and-white vision,
making their role in dark vision likely. The other cells are involved in taking information from the photoreceptors to
the brain. It is stated in the passage that the photoreceptors synapse with the bipoiar cells, which then synapse with
the ganglion celis. Therefore, for both color and black/white vision, these cell types will be involved in the
transmission of visual information. The correct choice is A.

66. C is corrrect, hyperpolarization of the photoreceptor. It is stated in the passage that upon stimulation of a
photoreceptor cell by a photon, the sodium channels close. This implies that when there is no light stimulation, the
sodium channeis are open. When sodium channels are open, the sodium ion comes in (following its gradient) and
causes the cell to be depolarized. In an unstimualted cell, a very active Nae/K@ pump is constantly restoring the
sodium gradient. Again, the net result of this is that a photoreceptor cell in the dark is normally depolarized. When a
photon hits, the sodium channels close and the pump continues to work. Both of the these events cause a
hyperpolarization of the photoreceptor cell. This is a very special case for photoreceptors, and it is important to be
aware of this phenomonon. Based on this information, the other answers can easily be eliminated. The correct
choice is C.

67. A is correct, fovea centralis. The area where there is the highest density of cones on the retina will give the greatest
acuity, or precision. At this point, one must recall the anatomy of the eye. The area of highest cone density is the
fovea centralis, and when we look at objects, we move our eyes to focus light onto this region of the retina. This
answer could not be obtained from the passage, but we thought it would be nice to review some eye anatomy. Look

Copyright @ by The Berkeley Review 76 The Berkeley Keview


Speciatizing in MCAT Preparation
Biology Nerve Ef Muscle Section I Answers

at the other answers. The cornea is the transparent structure located at the front of the eye and is the structure
through which light rays enter. The optic disc is where the optic nerve leaves the eye and the retinal blood vessels
enter. Since there are no photoreceptors overlying this disk, it is known as the blind spot. The choroid is a pigmented
layer behind the retina, which contains many of the blood vessels that nourish structures in the eye. It also absorbs
light not taken up by photoreceptors. It absorbs rather than reflects, so as not to alter the photoreceptor responses to
other light. The correct choice is A.

68. D is correct, occipital. We need to recall our knowledge of the cortices of the brain. The experiment removed the
thalamus. The thalamus. in particular the lateral geniculate nucleus, is a relay station for visual information. From
this nucleus, information then goes on to the visual cortex, located in the occipital lobe of the brain. The occipital
lobe is located above the cerebellum. Based on this, the only possibie answer choice is D. As a very brief review,
one can associate the frontal lobe with association processes, the temporal lobe with hearing, and the parietal lobe
with high level sensory and motor control. The correct choice is D.

69. D is correct, ganglion cells. The answer can be obtained by either previous knowledge, or by reading the passage
carefully. It is stated that both photoreceptors are located at the back ofthe retina. It clearly states that the four other
types of cells (bipolar, etc.) are part of the retina. Since the photoreceptors synapse with the bipoiar cells, and the
bipolar cells synapse with the ganglia, one could create a picture in their head where the ganglion cells are furthest
away from the photoreceptor celis, or the first layer in the retina. Based on this thought process, light entering the
eye will first pass over the ganglion cells. The correct choice is D.

70. C is correct, III only. The place where the axons of the ganglion cells leave as the optic nerve is called the optic
disc. There are no photoreceptors at this region of the retina, because the axons are leaving and blood vessels are
entering. If there are no photoreceptors, then vision is not possible. This is ofcourse one's "blind spot." Consider the
other possibilities. The region of highest acuity is where one finds the highest density of cones. This is certainly not
the case. In the case of stereovision, it is not the structure of one eye that allows for depth perception, but the fact
that we have two eyes looking at one object from different angles. This gives us stereovision. Therefore, statement II
does not apply. Statement III is the only applicable statement. The correct choice is C.

7t. C is correct, left eye and the temporal hemiretina of the right eye. If light originates in the left visual hemifield, then
we known that our right side of the brain will be processing this information. In order to get to the right side of the
brain, we need to have light strike the temporal hemiretina of the right eye. If you follow the optic nerve, you will
see it goes to the right side of the brain. In addition, light from the left side will strike the nasal hemiretina of the left
eye. The optic nerve from this part of the retina will cross over at the optic chiasm and move on to the right side of
the brain. Draw some lines of light from the left visual hemifield; it may augment your understanding of how light
originating from one side of the visual field becomes processed in the opposite side of your brain. The correct
choice is C.

72. D is coruect, one million times greater than the reference pressure. The question asks for the relationship between
the loudest sound pressure level and the reference pressure. We know from the question that the upper available dB
range is 120. In order to arrive at this figure, we must have a test pressure that is 106 times as large as the reference
pressure, because 20 log 106 = 120. The conect choice is D.

73. D is correct, force/unit area acting on the oval window is increased. We know the sound wave is conducted through
the set of small bones in the middle ear. Since the passage does not mention any significant loss of force during this
transmission, we can assume the force associated with the wave does not increase or decrease. Well, how about the
pressure? Pressure is the force/unit area. The force does not change. However, the area becomes smaller.
Therefore, the total force/unit area acting on the ovai window is increased. The correct choice is D.

74. B is correct, 1 x 10-3 s. The question is asking us to carry out some calculations. First, let us convert the 3.4 x 10- 4
kilometers into meters. This gives us 0.34 m. Based on this value and the speed of sound, we get a frequency of
1000 Hz. Recall that the period of a wave is the inverse of its frequency. Therefore, the period of this wave is
1/1000 Hz, or 1 x l0-3 seconds. The correct choice is B.

Copyright @ by The Berkeley Review l/ The Berkeley Review


Specializing in MCAT Preparation
Biology Nerve & Muscle Section I Answers

C is correct, oscillatingdepolarizing-hyperpolarizing. The passage tells us that oscillating movements of the


organ
of Corti both excite and inhibit sensory transducingiells, bbcauie the hair cells attached-to
these cells are *oriing
back and forth' In one direction, these hair celis cause cation channels to open.
This will cause a depolarization.
However, in the other direction, the hair cells cause a closure of these channels.
This is associated with a
hyperpolarization. Therefore, the cell undergoes an oscillating depolarizing-hyperpolarizing potential.
The correct
choice is C.

76. A is correct, region 1. The question is asking which region of the basilar membrane would be
associared with high-
il-equency sounds. High-frequency sounds are associaied with shorr wavelengths
and wiil be more energetic than
lower-frequency sound waves. We can assume that the broad flexible ,egion-of the basilar
membrane will not be
able to resonate with these short energetic movements. In other words. thJ broad
flexibie region will not be able to
receive high frequency sound as well. Such movements are achieved by the short, stiff region of the membrane.
The correct choice is A.

77. D is coffect, cell 5. Let us look at the circuit model. We know that information is arriving
at the left ear at a tirne
before that of the right ear. we also know that these cells fire maximally when
the inputi from both sides arri'e
simultaneously. For example' if sound arrived at both ears at the same time, ccll : woutd
fire maximally, because
input would arrive at that neuron simultaneously. Based on that logic, if information
is ariving at the left ear befbre
the right' it will travel up the entire circuit before the information-from the right
ear will arrive. Il the tu,o inputs
arrive at any cell at the same time, that cell must be located in the upper region
of the model. 'rherefore, cell 5 is the
best answer. The correct choice is D.

B is correct, middle ear. The passage tells us that conducting sound through the temporal
bone is one way to bypass
the middle ear' The passage also tells us that this means of transmitting sound
is quite inefficient. Therefore, when
the fork is placed on the patient's temporal bone and the patient hears the sound (afier
they no longer heard the sound
with the fork at their ear), we can narrow down the p.oblem to conduction of air in the
middle ear. The correct
choice is B.

79. C is correct,
-eraph C. The Y-axis is ampiitude and th_e X-axis is frequency. If cells have a characteristic fiequency.
the data should reach only one point on the X-axis. Some graphs ,hor tr"o.
Choices A and D can be eiiminatecl.
Now that we know thai only one characteristic frequency point ihould be reached, which
direction will the curve go?
At a cell's best frequency, does the loudness of a sound need to be high or low to elicit a response?
It is low. The
amplitude of the sound does not need to be high, if we are at the ceil's characteristic frequJncy.
However, if we
move away fl'om that frequency in either direction, the amplitude of the sound needed
to elicit a response increases.
Based on this infbrmation, graph C is best. The correct choice is C.

80. B is correct, hydroxylation foilowed by decarboxylation. An -OH (hydroxyl) group is added to lrp
and a -COOH
(carboxyl) group is removed to form serotonin. The terms for this are-hydroxyiation and decarboxyiation.
Choices
A. C, and D are incorrect. The correct choice is B.

81. C is correct' at high LNAAs are converted to serotonin. We are looking lor the false answer. Serum LNAA
concentration does increase in both trials as dietary protein content increases. Choice A is true. Serum
Trp also
increases with dietary protein content. Choice D is true. In Trial 1,the24Eo protein meal does attenuate
the increase
in serotonin due to Trp (competition from LNAAs). Choice B is true. Otheiserum LNAAs are not converted to the
neurotransmittel serotonin. Choice C is False, and we are looking for the false answer. The correct choice is C.

82. D is correct, I, II, and III. In examining the data tables, look at how the serum LNAAs increased with the higher
protein diets' If these were competing with Trp for the LNAA receptor, then the brain Trp level should decrease as
protein levels increase. This is true for the higher protein diets. In trial 1, the levels niZ+Eo and,40Va cause this
competition. InTriai2,thelevelsof l27o,24Vo,and,40Vocausethiscompetition. Iandllarecorrect. Inalltr.ials,
the ievel of serum Trp increased over the fasted group. III is correct. Theiorrect choice is D.

B is correct, Ii only. Trp is not a neurotransmitter, but it can be converted into the neurotransmitter. serotonin. I is
incorrect. Trp is an essential amino acid and cannot be synthesized by the body. II is correct. As we are told in the
passage, Trp is a neutral amino acid. III is inconect. The correct choice is B.

Copyright @ by The Berkeley Review The Berkeley Review


Specializing in MCAT Preparation
Biology Nerve & Muscle Section I Answers

84. A is correct, accumulation of an intermediate metabolite in the brain. If one of the enzymes in a pathway is blocked
by an inhibitor, then some intermediate metabolite is not processed through to the final product. This means the
intermediate form may remain and accumulate in the tissue. In this case, some intermediate form between
tryptophan and serotonin will accumulate. We have no real interest in the concentration of the inhibitor itself either
in the blood or the brain, just whether it is working or nol. Choices C and D are incorrect. The Trp in the blood
would not provide any information about lhe Trp conversion pathway rate in the brain. Choice B is incorrect. The
correct choice is A.

85. B is correct, dietary Trp alone would increase brain serotonin concentrations, since few LNAAs would be
cornpeting. Trp is an amino acid, and it would be easily absorbed in the srnail intestine, either alone or in
combination with other foods. Choice A is incorrect. Trp would only increase synthesis of serotonin, since it is not
the precursor lor other classes of neurotransmitters, Choice C is incorrect. Actually, an increase in serotonin levels
leads to a feeling of relaxation, not alertness. Choice D is incorrect. The correct choice is B.

86. C is correct, a greater frequency of the action potentials indicates greater intensity. Action potentials act in an "all-
or-none" fashion. There is no amplitude modulation. Choices A and B are incorrect. Identical action potentials are
produced more frequentiy to indicate a stronger stimulus, such as the stretch caused by a set of increasing weights.
The correct choice is C.

87. A is correct, the action potentials had increased to a maximum with the 50-g weight and increased no further, due to
the refractory period of the neuron. The action potentials were limited by the nonconducting refractory period of the
neuron. Increasing the weight could not speed up the "recovery time." The nerve was not damaged by the weight.
Choice B is incorrect. Do not assume equipment f-ailure when a biological mechanism is being studied. Choice C is
incorrect. A neurotransmitter defect would probably mean no action potentials rather than slowed ones, exactl]'
identical to another weight. Choice D is incorrect. The correct choice is A.

88. A is correct, The weight was not heavy enough to exceed the action-potential threshold. 5 mg (1/200) of the 1 gram
weight caused only a few action potentials. Since there was not an action potential generated, this weight must have
been too small to overcome the threshold for generating action potentials. The weight was not too heavy, so choice
B is incorrect. The weight does not have to produce an action potentiai, if its stimulus is lower than the threshold.
Choice C is incorrect. Alt the action potentials were identical in size, so frequency was modulated, not the
amplitude. Choice D is incorrect. The correct choice is A.

89. D is correct, plasma membrane. The cytoplasm of a neuron is a poor conductor. Choice A is incorrect. The
Schwann cell acts to insulate nerve fibers and to provide gaps between the conducting regions. Choice B is
incorrect. The endoplasmic reticulum does not function as a conductor. Choice C is incorrect. The plasma
membrane is the site of conduction of action potentials. The correct choice is D.

90. B is correct, sodium moves from the extracellular fluid into the cell, and then potassium moves from the
intracellular fluid to the outside of the cell. This is an easy question, but all the words make it easy to make a
mistake. First, eliminate wrong answers based on ion location. Sodium is the principle extraceliular ion, while
potassium is the principle intracellular ion. Choices C and D are incorrect based on this fact. Then eliminate
based on orAer of acdon. The sodium channeis open first, fbllowed by the potassium channels. Choice A is
"hoi."r
incorrect. The correct choice is B.

91. B is correct, afferent function. Afferent pathways carry information from the tissues to the nervous system.
Information of muscle stretch was being conveyed back toward the (no-longer-present) nervous system. Efferent
function would be tested, if a nerve were stimuiated with electrodes to make a muscle twitch. Choice B is inconect.
Since the nerve and muscle under study are isolated from the intact frog, there is no parasympathetic or sympathetic
function. The correct choice is B.
o't C is correct, action potentials continue briefly but stop eventually, due to a lack of ATP. The Nao/Ke pump
requires ATP to perfoim active transport. The ceil has a smail reserve of ATP that would provide brief function of
the pump if ATP production stopped. Choice A is incorrect. Action potentials stop when all the ATP is gone.
Cholce B is incorrect. Action potentials require ATP, they do not generate it. Choice D is incorrect. The correct
choice is C.

- :'pr right @ by The Berkeley Review 79 The BerkeleY Review


Specializing in MCAT Preparation
Biology Nerve & Muscle Section I Answers

93.
has at reastone muscre that
rt attiches above.tiie hip joint u;h";";;i* j;-ff;;ffi'rr,""'""iil1'";;ifi#i:;crosses the hip
t;l:'"1::*,1'11'll:1"';"*:::*lf-t:,'l:i::':*.?t1"-3i:d'i'"t'c'9:p
j:f,-ilf.t":1v.
::::::,,:,1, 'l.,:r."{;1;;;;"-,#,;;",;;rr"# il;:;i*Ti';iJ;il;TT,?ifr':i"#ff"i'lj;'lfij,ffJ',','fl
. dru rr rre^run or lne nip by pulling the temur and tibia upwards
Refer to the skeleton diagram and try ro visualize-,''iexacrly how this works. -fhe
:"":,Td^t lli :!"tj' The other
orher answer
ili t,ii iii ri"l"J'iffi # :ff;
',ip' "-i
shorten fhev urnrrld nrrll rho f^^'. L^-t^,,^.")^ rhis
at:^ is r
fe1:i b.aclwar{. :^ hip
1,:,,
extJnsion,;;;'fi;;i";;;j;.i""#d;i*[:"i1,i1.i;Hi
ll,j^.,f1.i1:ll:ildf,rlrh:
11"^,1"::t;
choice is A.
when they contract, they pull the thigh inwards. rrris is'aaJJ.,-",'itirr""iffii::tfi: #::i
94. c is correct' the veins involved must have valves. How can blood be returned from the
gravity and under the low pressure that is normally found in lower extremity against
the venous system? Muscles of the thigh and lower leg
"massage" the elastic veins, pumping blood in onl direction:
,prurar. -Blood can only go in this direction because
veins in the lower extremity havJ one-way valves which p."u"it
very important in maintaining proper circulation. varicose u"inr
uu.tlo* of blood u;uj f-- rhe heart. These are
the other answer choices' Eiiminite choices B and D because
r"rullir"*^a"i*ir-l"i'tr.ri Let,s consider
arteries do not return blood to"u,""r.
the heart, veins do.
Eliminate choice A because if veins had inelastic walls, they .ouldnlt
b" squeezed by muscles and the process would
not work. The correct choice is C.

95. B is correct, the quadriceps must relax. The quadriceps and the hamstrings
are opposi.ng muscle groups with
opposite actions' The hamstrings extend the hip posterioity while
the quadriceps flex the hip anteriorly. In order for
one of these antagonistic muscle groups to woik, the otler must
relax. tf both li-ultaneously, nothing
would get accomplished because the two actions would cancel each "onr.u.i
other out. Knowing-rh"r" should lead us to
eliminate choices B and c. choice D can be ruled out because
the obturator n"ru", u-..o.ding to the passage,
innervates the adductor muscles, which aren't involved here.
The correct choice is B.
96. c is correct' to kick.1 .ba]l. The passage states that the femoral nerve innervates the quadriceps
quadriceps are responsible muscles. The
for flexing tnJrrip anteriorly (see prior question) and also to, ,riir.arrgthe
words, since the quadriceps crosses the knLejoint,.-ont.".iion putt, knee. In other
tt'r" lo*er leg and ioo, unt".iorly (a kicking
action' known as extension of the knee joint). Paralysis of the fem'oral-nerve
might tf,erefore afI-ect the ability to kick
a ball, among other things. Let's coniider the other answer choices.
choice A can be eiiminated because the
quadriceps muscle doesn't cross the ankle joint and hence can't plantar
flex the foot (i.e., cause a person to stand on
tip-toes)' Choice B can be ruled out because extendin-g the hil pori".io.ty
is the job of the hamstrings, not the
quadriceps' Choice D can be eliminated because the adductor
muscles aren't innervated by the 1emoral nerve; the
obturator innervates them. The correct choice is C.

97. D is correct' weakness in the adductor muscles. what would happen if an intervertebrai
disk herniated (squeezed
out of its sheath) and smashed the adjacent sciatic nerve? l rom the passage, we
learn that the sciatic nerve
innervates the hamstring muscles. These muscles are.normally responsible
fbr extending the hip posteriorlv and
flexing the knee (i.e., bringing the heel upwards posterioriy). These actions of the
hamstring group can be explained
by looking at where they attach. The himstringi attach to the ischium posreriorly, cross
the hip joint, and attach
inferiorly to the f'emur' tibia, and fibula. They thirefore cross both the hip
.loint uni ir," r.n"";oint. shortening of the
hamstrings results in flexion
of the knee and exte.nsio^n of the hip posterioriy. Since the hamsirings are innervated
by
the sciatic nerve, damage to this nerve would affect walking (i.e., hip extension),
difficulty moving the leg
posteriorly(knee flexion), and sensory stimulus along the back o1 tire thigtr
and lower ieg (which manifests"as painf
The combined conditions are known as sciatica. Choice D is our answer because
the adductor muscles are not
innervated by the sciatic nerve. According to the passage, they are innervated
by the obturator nerve. We would not
expect them to be affected by damage to the sciatic. The correct choice is D.

98. C is correct, lower-than-normal CO2 levels. What effect would reduced blood flow have
on exercising muscle
cells? To start,02 (which would be in high demand in active muscle) would not reach muscle fast enough due to
poor circulation. This would lead to a decline in aerobic respiration (oxidative phosphorylation,
etc.) reJulting in
ATP deprivation. In order to partially compensate, anaerobii respiration (glycolysis and lactic
acid f'ermentation)
would be stepped up, resulting in a buildup of the by-product lactic aciA
lpJssiUly ,"uere pain). Co2 levels
"urrlng
would actually rise because it could not be carried away effectively by the blood. The coriect choice is C.

Copyright @ by The Berkeley Review ao The Berkeley Review


Specializing in MCAT preparation
ffi&ffiKwffiy
A. The Fleart
See€,fr€>EB W l. CardiorrascnlarAnatomy
2. Cardiac Output
3. Poiseuille's Law
Fteart 4. Diffusion
& 5.
6.
LJSMOSIS
Lvmphatics
t uEegs 7. Blood Clotting
The Lungs
t. Cases
2. Gas Exchange

lF==== -,-''/

,T{te

ffiflffiffi€fltumY
AiK
Specializing in MCAT Preparation
Biology Ileart & Lungs Cardiovascular Anatomy

Within iiving creatures there are two types of circulatory systems. In an open
circulatory system the circulating fluid within the body is mixed with the body
fluid itself. In other words, the blood in these creatures does not circulate entirely
within the confines of vessels like arteries and veins. Most mollusks and
arthropods have an open circulatory system. Conversely, in a closed circulatory
system the blood that flows throughout the body is confined to vessels like
arteries and veins. It does not freely mix with the fluid of the body. Annelids and
mammals like ourselves have a closed circulatory system.

The function of the circulatory system is to bring nutrients and oxygen to the
tissues of the body while simultaneously removing waste producis from those
very tissues. Because the circulatory system is continually flowing it helps to
maintain body temperature. Also, the circulatory system can act as a means to
transport hormones to various locations within the body. Ultimately, though, as
the blood enters into the smallest vessels, the capillaries, there will be diffusion
between those capillaries and the cells in the immediate environment.

General Anatomical Features


Let's examine the general anatomical features of the mammalian cardiovascular
system. The foundation of the circulatory system is the heart. The heart can be
thought of as being divided into two halves. These two haives are often referred
to as the right heart and the left heart. \44ren we look at a picture of a heart we
must remember that we are viewing that heart in its anatomical position. In the
anatomical position the body is standing upright, the arms to the sides, hands
with plams up, and the face positioned forward. Thus, when we look at a
diagram of a heart on paper, the right side of that paper really represents the left
side of the heart and vice versa.

The right heart pumps blood to the lungs and back to the left heart. The left heart
pumps that blood to the remaining tissues of the body and back to the right
heart. The blood that is pumped from the right heart to the lungs and back to the
left heart is called the pulmonary circulation while the blood that is pumped
from the left heart to the rest of the tissues and back to the right heart is called the
systemic circulation. Both the pulmonary and systemic circulations lie in series
with one another.

We can pick a starting point in the circulatory system and follow a red blood cell
as it rnigrates through the pulmonary and systemic circulations. Let's start with a
red blood cell in the right heart. The right heart is composed of the right atrium
and right ventricle. As deoxygenatedblood passes from the right atrium into the
right ventricle it is pumped rnto the pulmonary artery and to the lungs where it
is oxygenated. The oxygenated blood returns to the left heart by way of the (left
and right) pulmonary veins. The blood enters the left atrium and passes into the
left ventricle where it is pumped out the aorta and to the branching arteries,
arterioles, and capillaries. It is at the level of the capiilaries that the blood
exchanges nutrients and oxygen for waste products created by metabolism.
Deoxygenated blood passes from the capillaries to venules and then to larger
veins and eventually to the superior and inferior vena cava which enter the right

Copyright @ by The Berkeley Review a5 The BerkeleY Keview


Specializing in MCAT Preparation
Biotogy fleart & Lungs Cardiovascular Anatomy

atrium. The circulation of our red blood cell can now start again. As you are
studying this circuit it is important to note that arteries carry iiood away from
the heart while veins carry blood from the tissues and lungs towards the heart.
This can be seen in the diagram shown in Figure 2-1.

Capillaries of Head and Arms

Superior Vena Pulmonary


Cava Artery

J O
\3
.c
bo
t
o
o
q o
a)
! -
(D

+ l-
O
0a

Right
-'
7
Atrium
tr Pulmonary
""*, Vein
lnferior Vena
Cava Left
Atrium
Left
Ventricle

Capillaries of Abdomen and Legs

Figure 2- I
Blood flow through the cardiovascular system.

Aorta and Arteries


As shown in Figure 2-L, blood ieaves the left ventricle of the heart and exits
through the aorta and passes to both the superior and inferior portions of the
body. The total cross-sectional area of the aorta is about s cm2. under resting
conditions the velocity of blood flow in the aorta is about 30 cm/sec. The blooJ
pressure fluctuates betrveen 120 and 80 mmHg with an average being about 100
mmHg. The volume of blood found in the aorta and arteries ui u.ty given time is
about'l.6oh to 20'/..

As the left ventricle contracts (systole) it propeis blood out the aorta and into the
arteries with a pressure of about 120 mmHg. At sea level this force is enough to
raise a column of mercury 120 mm above the ground. After contraction takes

Copyright O by The Berkeley Review a4 The Berkeley Review


Specializing in MCAT Preparation
Biotogy fleart & Lungs Cardiovascular Anatomy

irlace and the ventricles begin to relax (diastole) and fill r /ith blood, the pressure
n the arteries is about 80 mmHg.

\Ieasurement of Blood Pressure: When you have your blood pressure taken a
'-riessure cuff is placed around your upper arm and a stethoscope is piaced over
-re antecubital artery (the artery at the bend between your upper and lower
:.:rnl. The cuff is inflated above arterial systoiic pressure. This causes the artery in
, .ur arm to collapse thus stopping the flow of blood. The pressure in the cuff is
:e:reased and the pressure needle monitored. As soon as the pressure in the cuff
-' relou' the systolic pressure, blood will jet through the small opening in the
'::er\'. The flow through the artery is turbulent because of the great pressure
:ushing the blood through such a narrow opening. The "tapping" sounds that are
,-.=a:d in the stethoscope approximately correspond to the systolic pressure in the
,-.=:.rt, The cuff is continualiy deflated and the artery slowly regains its original
:j-.:le. The blood has an easier time passing through the vessel and the fiow
:=i:rr.s to change from turbulent to laminar (smooth). When the turbulent sounds
-rr,rfopear it means that you are recording the diastolic pressure of the heart.
---..d pressure readings are given as systolic/diastolic (e.g., 720 mmHg/8O
,:::'i{g). These are the approximate readings that the baroreceptors are sensing
-:: --.re\- monitor vour blood pressure.

- -.- rlood that moves through the arteries is under a lot of pressure. Therefore,
,: -.e arteries have to be rather durabie. They have thick walls and are composed
: smooth muscle and connective tissue that contain both collagenous and
"1,::tic fibers. The elasticity of the arteries prevents the blood pressure from
- :: -::ing too high when it is ejected out of the heart and it also maintains a high
: r.=r- jrl pressure between the systolic and diastolic phases of the heart. This
: - ,.': blood to flow to the rest of the circulatory system without a sudden loss of

- = -::nen of all the blood vessels in the body are lined with epithetial cells and
:: -: i>e these ceils are inside the cardiovascular system they are referred to as
.rl,rihelial cells. Damage to these endothelial celis b,v the pulsating arterial
- --::s.,:ie or even by abrasive substances in the blood can lead to the disease
:
--.,':-: as atherosclerosis. Once these ceils are damaged choiesterol can build up
:-- = site of the lesion and a plaque will form. During the later stages of the
- ,:: is- the arteries become "hardened" from layer upon iayer of deposit. This is
-:
-:-:= j to as "hardening of the arteries" or arteriosclerosis.

- :'.^-::ron of the circulatory system is controlled (in part) by the sympathetic


--r :::asvmpathetic divisions of the autonomic nervous system. The
:-::.:.eti.c division is the more important of the two. Besides nervous control of
: :- :-,.ir- there is also humoral control from the action of ions or hormones and
-.- ::rtrol at the level of the individual tissues from various metabolites.
:re metabolites important? If you were to start exercising a particular
".:--, "-ou would notice that the blood flow to that muscle would increase.
:' --,rrat u.hen muscles contract ATP is hydrolyzed io ADP and Pi, lactic acid
., ::',=i, and otirer such metabolites circulate in the tissue. When these
.;: -:;es diffuse from the tissue out to the arterioles the smooth muscle dilates
r
-- -: :::ases the flow of blood to that particular area. The blood brings not only
:-::.:s and oxygen to the working tissue, but it carries away the waste
. r-::i of metaboiism as well.

3 by The Berkeley Review The Berkeley Keview


Specializing in MCAT Preparation
Biology fleart & Lungs Cardiovascular Anatomy

Arterioles
After the aorta branches to form a variety of arteries, the arteries themselves
branch to form arterioles. The arterioles are important because they
represent the
major area of resistance in the cardiovascular system. Arterioles have
strong
muscular wails and are regulated as mentioned above.

Capillaries
Diffusion takes place at the level of the capillaries. The total cross sectional
area
of the capiliaries is about 3000 cm2. The velocity of blood flow has been
red.uced
to about 2 cm/sec. The pressure is roughly 25 mmHg and the total volume
of
blood is about 5o/o to7%. The wall of the capillaries are"composed of a
unicellular
iayer of endothelial cells. surounding tirese cells is a basement membrane.
Flowever, there is no connective tissue or smooth muscle. The capillary
itself is
just large enough for a red blood cell to squeeze through.
At the entrance to the
capillary bed is a precapillary sphinct"r .o*pos*d Lf smooth muscle
which
helps to regulates the flow of blood to the area.

Veins
once the blood flows through the capillaries and reaches the veins
the blood
pressure has been reduced to a value between 0 mmHg and
25 mmHg. The
velocity of blood flow through the veins is about 20 cm/Jec and the
total cross
sectional area is roughly T.cm2. Roughly s0'/. ofthe total blood volume
is present
in the veins at any given time.

Figure 2-2
A valve in a vein showing directionality.

since there is not much pressure in the veins the amount of smooth muscle
and
elastic tissue surrounding the veins is reduced. However, they are under
control
of the sympathetic nerves. Note that the rate of blood flow inihe veins is similar
to that in the arteries. This is due to specialized valves that allow the blood to
flow in only one direction (see Figure 2-2). As the muscles that surround the
veins in your body contract they squeeze the blood back towards the heart. Since
these valves are "one-way valves" blood is prevented from flowing backward. trf
the valves become damaged and blood is aliowed to flow backwarls, pressure in
the veins can increase. This has a tendency to cause varicose veins, which are
protrusions of the dilated veins beneath the skin.

Blood FIow in the tleart


We have mentioned that deoxygenated blood returns from the tissues and enters
the right atrium via the superior and inferior vena cava. As the right ventricle
begins to relax blood from the right atrium is pumped into the rig:ht ventricle.
The right ventricle contracts and forces blood out the pulmonary irtery and to
the lungs where it is oxygenated. Blood returning to the heart enters the left
atrium via the right and left pulmonary veins. As the left ventricle begins to relax

Copyight O by The Berkeley Review a6 The Berkeley Review


Specializing in MCAT preparation
Biology Ileart & Lungs Cardiovascular Anatomy

blood from the left atrium is pumped into the left ventricle. Contraction forces
the blood in the left ventricle out the aorta and to the tissues of the body. See
Figure 2-3.

Pulmonary Pulmonary
Artery Artery

Pulmonary
Veins
Superior
Vena Cava SA
Pulmonary Valves Aortic Valves
(semilunar) (semilunar)
AV
Kight L. Atrioventricular
Atrium Valves (mitral)
R. Atrioventricular
\ alves (tricuspid) Left
Ventricle
Inferior
Vena Cava Bundle of flis
gives rise to
.:->
L. E( R.
Chordae tendineae
Kight ::1. Bundle Branches
Ventricle (frrrther divides into
Descending
Papillary Purkinje Fibers)
Aorta
Muscle

ri.lre 2.5
- -.. -:ndmarks of the heart.
-
-: ::.e heart there are valves between the right atrium and right ventricle
: r l-- i atriorrentricular valve), between the right ventricle and the pulmonary
.:.. rulmonary valve, or tricuspid), between the left atrium and the left
-:. :,= 'ie left atrioventricular valve, or mitral), and between the left ventricle

-- .: r ird (the aortic valve). Once the ventricles are filied with blood and they
- =:, :ontract the valves between the atria and the ventricles close. This
: i: :r.\- backflow of blood into the atria and ensures that the blood will be
: :-- rhe pulmonary and systemic systems of the body. The closing of the
:.ricular valves between the atria and the ventricles as the ventricies are
.- : ::1\'es the characteristic "lub" sound when listening to the heart. As
,: :-:rrs out of the pulmonary artery and the aorta, the pulmonary valve
- :lrlic valve close, giving the characteristic "dub" sound. The closing of
-. as blood is pumped either from the atria into the ventricles or from
:-=s lo the puimonary or systemic tissues prevents backflow into either

fhe Berkeley Review The Berkeley Review


Speciatizing in MCAT Preparation
Biology Ileart & Lungs Cardiovascular Anatomy

the atria or the ventricles, respectivery. The valves between the atria and the
ventricles themselves do not invert because of tendinous cords, called chordae
tendineae, which hoid them in place.

Electrical Activity of the lleart


Located near the junction of the superior vena cava and the right atrium is a
specialized region of myocardium called the sinoatrial node (sA node) or the
pacemaker of the heart (Figure 2-4).

SA

AV
Node

Bundle of His

Figure 2.4
The SA and AV nodes of rhe heart.

The SA node is the point of origin for the electrical impulse that propagates
through the rest of the heart. Th[ electrical impulse spreads out over the atria
causing them to contract and filt the ventricles. Located in the lower portion of
the right atrium and near the right ventricle is the atrioventricular node (AV
node). Impulses from the sA node also spread to the AV node and then from the
AV node through a coliection of fibers called the bundle o{ His. Branches of the
bundle of His surround the ventricles, and when this bundle receives an
impuises it causes the ventricles to contract and eject blood to the pulmonary and
systemic systems. Anatomicaily speaking, ventricular contraction is from the
apex of the heart towards the base of the heart. It is interesting to note that if the
sA node is damaged, the AV node takes over and slows the heart down to about
40 beats per minute.

Cardiovascul.rr Control
The average blood pressure leaving the aorta is about 100 mmHg. This blood
pressure is monitored by baroreceptors and chemoreceptors located in the
carotid arteries and in the aortic arch. The baroreceptors (pressure receptors) are
continually monitoring how much the aortic and carotid arteries are eipanding

Copyright @ by The Berkeley Review 8a The Berkeley Review


Specializing in MCAT Preparation
Biology Ileart & Lungs Cardiovascular Anatomy

and contracting. Suppose we were to decrease the arterial pressure. There would
be less stretch on the aortic and carotid arteries. The baroieceptors would sense
this and send impulses to the medulla in the brain stem. The medulla responds
by activating the sympathetic nerves of the autonomic nervous system. Impulses
are sent via the sympathetic nerves and norepinephrine is released at the sA
node to increase the rate of the heart. This action helps to increase the contraction
of the heart, which in turn wiil elevate the blood pressrrre. Sympathetic nerve
fibers will also stimulate the adrenal medulla to releise epinephrine (adrenaline)
into the blood. This hormone acts to increase the rate of the heart, therefore
increasing the contraction of the heart. Both of these actions act in a negative
feedback manner to "negate" the initial loss of pressure due toi say,
hemorrhaging.

The sympathetic nerves will also cause constriction of the blood vessels that lead
to the gastrointestinal system and the kidneys. During hemorrhaging the brain
and heart receive first priority in terms of blood. The rest of the organ systems of
the body see a decrease in the flow of blood until the probiem is corrected.

Copyright @ by The Berkeley Review a9 The Berkeley Review


Specializing in MCAT Preparation
Biology Ileart & Lungs Cardiac Output

tffiffiffi :iiiiiijr;iilliijiiiti::

Every time your heart beats, a certain volume of blood is pushed out into the
circulatory system. The cardiac output is that amount of blood which is pumped
per minute by each of the two individual ventricles of the heart. We can define
the cardiac output (for either ventricle in liters/minute) as being equal to the
heart rate (in beats/minute) times the stroke volume (in liters/beat). This is
shown in equation (2-1). The stroke volume is simply the amount of blood ejected
by each ventricle during one beat of the heart.

If the average heart rate is T2beats per minute and the average stroke volume is
70 milliliters (or 0.07liters) per beat, then the cardiac ouput would be about 5
liters per minute. This value is for the average resting adult male. As we will see
later, the volume of the cardiac output can be influenced by the diameter of the
blood vessels in the periphery, the amount of blood returned to the heart by the
superior and inferior vena cava (i.e., return of the venous blood), and the heart
rate and force of ventricular contraction.

Cardiac Output = (Stroke Volume) (Heart Rate)


(2-L)

Copyright @ by The Berkeley Review 90 The Berkeley Review


Specializing in MCAT Preparation
Biology Ileart & Lungs Poiseuille's Law

Poiseuille's Law
Not all of the arteries and veins in the circulatory system have the same diameter.
Jean Poiseuille, a French physician studying the flow of biood in blood vessels,
established a relationship between the radius and length of a tube, the change in
pressure between the two ends of the tube, the viscosity, and the flow rate of a
fluid in that tube. This relationship, known as Poiseuille's law, is given in
equation (2-2).

Flow = AP ftRr = (P, - P, 'SnL


) 7IR*
SnL (2-2)

in this equation (2-2), AP is the pressure drop between the two ends of the tube
(i.e., P1 - Pz), R is the radius of the tube, eta (r1) is the coefficient of viscosity, L is
the length of the tube, and n/8 is a proportionality constant adjusting for the
cross-sectional area of the tube. What can we say about this equation?

(a) Notice that the fiow rate is proportional to R4. This tells us that the rate of
blood flow is extremely dependent on the radius of the vessel. If the radius
of the vessel were reduced by a factor of 2, then the rate of blood fiow would
be reduced by a factor of 16. similarly, if the radius of the vessel were
increased by a factor of '1,.5, then the flow rate would increase by a factor of
5.1.

(b) The flow rate is also inversely proportional to the length of the vessel. In
other words, the longer the vessel, the slower the rate of flow. The shorter
the vessel, the faster the rate of flow.

(c) The flow rate is also inversely proportional to the viscosity of the solution.
This tells us that a high viscosity gives a 1ow flow rate.

(d) The value of AP is provided for by the strength of the heart's contraction. In
other words, the difference in the pressure is what drives ihe blood in the
cardiovascular system.

Sopyright @ by The Berkeley Review The Berkeley Review


Specializing in MCAT Preparation
Biology Ileart 6e Lungs Diffusion

Diffusion is simply the process by which molecules randomly move from one
place to the next. The molecular weight of a moiecule and the temperature of the
medium have a lot to do with the velocity at which a molecule moves. Smaller
molecules tend to move faster than iarger molecuies. Similarly, a higher
temperature provides more energy to a system and therefore imparts more
thermal motion to the various moiecules in that system. you might think that
because smaller molecules move relatively fast they would have no problem
traversing their environment. However, don't forget that there are millions and
millions of molecules within a given system and that each of those molecules,
even thought they are moving at a given velocity, collide with their neighbors.
These collisions tend to alter the path o{ the molecules, thus confining them to a
random walk through their medium.

Consider an imaginary sphere of water with a given radius. suppose we place


some dye outside this sphere of water and ask how long it will take to reach the
center. The answer clearly depends on the radius of that sphere. If the radius
were 7 microns, it would take the dye about 5.4 seconds to reach the center.
Flowever, if the radius were L centimeter, it would take about 11,000 seconds or a
little more than 3 hours to reach the center. [As a reference a red blood cell is
about 7 microns in length. There are about 1,400 red blood cells end to end in 1
centimeter.l what this is telling us is that simple diffusion is a rather poor way
for a molecule to trek across long distances.

The Law that governs diffusion is given in equation (2-3) where J is the net flux or
net rate of diffusion (in moles per unit time, usually in seconds; that is, mol/sec),
D is the proportionality constant called the diffusion coefficient, A is the area of
the plane of interest (in cm2), and AC/Ax is the concentration gradient across that
plane (in mol/cm4, because concentration is in units of mol/cri3 and distance is in
units of cm). Since the net flux always proceeds down a concentration gradient,
from a high concentration to a low concentration, we need to add a minus sign in
front of this equation. [The minus sign indicates the direction of the flux or
diffusion.] This equation is sometimes referred to as Fick's law (after the German
physiologist who postulated it in the 19th century). In equation (2-3), what are
the units of the diffusion coefficient, D?

(DXAXCour - Cin)
r=-(DX^)f (2-3)
Ax

Copyright @ by The Berkeley Review 92 The Berkeley Keview


Specializing in MCAT Preparation
Biology fleart 8r Lungs Osmosis

Osmosis
In order to understand hydrostatic pressure we must first review osmosis. Recall
that osmosis is simply the net movement of water from a region of high
concentration to a region of 1ow concentration. General chemistry tells us that the
concentration of pure water is 55.5 moles/liter. If we had a beaker of pure water
and u,e added a solute to that beaker (say glucose), then we would decrease the
concentration of the water in the beaker. Remember, when we add a solute
nolecule (or molecules) we are occupying a volume of space that was once
occupied by a water molecule. The more solute molecules we add to our beaker
,.f pure water, the more water molecules we will displace and the lower r.t'ill be
re concentration of pure water.
Jlucose, when added to a solution of pure water, does not ionize. It stays as
:1ucose. Flowever, if we add a molecule of sodium chioride to a solution of pure
.,-ater, it u'ill ionize into a Nae ion and a Cle ion. Because we now have added a
.-.,lium chloride molecule which has dissociated into two ions in solution, we
:.ar-e displaced (iowered the concentration of) the water molecules twice as much
:-s rre would if we had added a glucose mo1ecu1e. \\hat this is telling us is that
--:.e concentration of water in a given solution depends on the number of solute
:a:ticles (e.g., glucose, Nae, or Cle) in that solution. We can define the total
.-ir-rte concentration in our solution as the osmolarity, where one osmol is
:-::prlv one mole of a molecule that does not ionize. If we had a 1M solution of
:-r.ose, it would have a concentration of 1 osmol per liter. If we have a 1M
.: -'.ihon of sodium chloride, we r,r,ould find that it wouid have a concentration of
I osmols per liter (one from the Nae ion and one from the Cle ion). If we have a
1i0mM concentration of NaCl, then after ionization we would have 150 mM of
:: rons and 150 mM of Cle ions or a total of 300 milliosmols (mosmols) per
,-:=:. Therefore, the osmolarity refers to the concentration of solute particles that
: :.ave in our solution.

to
I Ah
toa

L
Hzo- >.
oo
a

'a
)ao
a
a
Oa

lj .o
o
Permeable Membrane
o
Semipermeable Membrane

F,gure 2-5
- .: ::.ovement across a permeable and semipermeable membrane.

: -::ose rve have a U-tube with water and apermesble mernbrane as shown in
I fure 2-5a. Water is free to pass back and forth across this membrane and
| : - irS€ of this the height of the water in each of the columns of the U-tube will
r : --.: Sdrrl€. However, suppose we now replace the permeable membrane with a
''.'-'::'neable membrane and then add some protein to the right side of the U-
"" :: 1s shown in Figure 2-5b. \44rat will happen? The level of water in the right

:,:::ht O by The Berkeley Review 93 The Berkeley Review


Specializing in MCAT Preparation
Biology lleart & Lungs Osmosis

side of the tube will rise and the level of water in the left side of the tube will
drop. \Alhy?

In order to understand this we must examine the semipermeable membrane and


the solutes which we have added to the right side of the tube in a little more
detail. First, the concentration of solute on the right side of the tube is greater
than the concentration of the solute on the left side of the tube. This established a
solute concentration gradient in which the solutes on the right side of the U-tube
want to diffuse down their concentration gradient to the solution on the left side
of the U-tube. This cannot happen, because we have said that the membrane is
semipermeable. In other words, the membrane will not allow these solutes to
pass through only water. What about the concentration of the water? The
concentration of the water on the left side of the U-tube is greater than the
concentration of water on the right side of the U-tube. Again, a concentration
gradient has been established that will allow water to diffuse from the left side of
the U-tube to the right side of the U-tube. Since the membrane is permeable to
water we find that water diffuses down its concentration gradient (from left to
right). This leads to an increase in the volume in the right side of the U-tube and
a decrease in the volume in the left side of the U-tube. The effect is to decrease
the solute concentration in the right side of the U-tube. An equilibrium will
eventualiy be established when the concentrations of both the water and the
solute are equal on both sides of the semipermeable membrane.

Once equilibrium is reached no more water will flow from the left side of the U-
tube to the right side of the U-tube. This is because the pressure has increased in
the right side of the U-tube (because there is now a larger volume of solution
pushing on the semipermeable membrane). The amount of pressure that stopped
osmosis is referred to as the osmotic pressure (abbreviated as nosm). A direct
measure of the osmotic pressure is the difference in the levels of water in the left
and right sides of the U-tube. This difference, Ah, is referred to as the hydrostatic
pressure (or fluid pressure) which can be abbreviated as PH2O.

The osmotic pressure is proportional to the number of dissolved molecules in a


solution and is represented in Figure 2-6. As we increase the concentration of the
protein in solution (i.e., solute in solution) we find that the osmotic pressure
increases as well.

()O
ov
tr
q
o
[Protein]

Figure 2-6
Relationship between osmotic pressure and dissolved particles.

Now, let's return to the hydrostatic pressure generated by the heart which forces
fluid out of the capillaries and into the interstitial space. Since the hydrostatic
pressure in the capillaries turns out to be a bit greater than the osmotic "pulling
pressure" of the soiutes in the blood, there is a net movement of fluid from the
capillaries to the interstitial space and eventually into the lymphatic capillaries.

Copyright @ by The Berkeley Review 94 The Berkeley Review


Specializing in MCAT Preparation
Biology Ileart & Lungs Lymphatic System

Lymphatic System
The lymphatic system lies parallel to the systemic and pulmonary circulations.
Tl'Lis can be seen in Figure 2-7.The lymphatic system ioliects the excess fiuid
about 4 liters per day) that leaks into the interstitial space from the capillaries
and returns it by way of the vena cava back to the ciriulatory system. Lymph
:-.cdes located along the lymphatic system help to filter out foreign particles that
:ould potentially lead to disease. If the lymph flow through the iymphatic system
-"''ere biocked, edema wouid result.
This is simply an increase in the inteistitial
:-';id (because it cannot be reabsorbed by the 1ymphatic system). patients who
:-ar-e heart surgery usually have swollen legs and ankles. This is because the
--:art cannot pump the blood out fast enough, and as a result blood within the
---:art begins to back up. This translates to a back-up in the veins and eventually
- lne lvmphatic system. Since gravity pools fiuid toward the lower extremities,
: *::fla results in the legs and ankles.

Q t-ymphatic capillaries

Pulmonary
capillaries

Artery
.,:$
\
Vein Heart
\
Systemic
-.nph
:de capillaries

Q Lymphatic capillaries

i.r1Ule 2-7
: , *::=:lC SVStem.

..., 3 b1' The Berkeley Review 95 The Berkeley Review


Specializing in MCAT Preparation
Biology Ileart & Lungs Blood Ctotting

B[db i]iffiurfifig
Blood clotting occurs via a cascade prlcess. Thrombin, which is involved in bibod
clotting, is a serine protease. We can achieve amplification of a very weak signal
by a cascade process. If we consider blood clotting, we will find that there is an
intrinsic route (due to contact with some abnormal surface) and an extrinsic
route (due to trauma to the tissue), both shown in Figure 2-8.

In the outline shown in Figure 2-8 we wili use the Roman numerals to represent
the clotting factors. The subscript "a" means that we are dealing with the actiae
form of the molecule. Again, this would be some type of conversion of a
proenzyme to an enzyme. Factor IX will be converted to Factor IXa by some
intrinsic factor. Factor IXa wili be the trigger which will convert Factor X to
Factor Xa. From the extrinsic portion of this scheme we start with some tissue
factor which will convert Factor VII to the active form, Factor VIIa. Factor VIIa
can also bring about the conversion of Factor X to Factor Xa. It is Factor Xa which
is involved in the conversion of prothrombin (II) to thrombin (lla). Thrombin (IIa)
wili convert fibrinogen (I) to fibrin (Ia). These fibrin fibers are then crossed-linked
by Factor XIIIa (which is an enzyme called transglutaminase) to form the mature
cross-linked fibrin ciot.

Some of the serine proteases mentioned in Figure 2-8 are Factors VII, VIIa, X+
II, and IIa. In the blood clottingprocess there are more than 15 different factors
involved, and about 8 or so of them are serine proteases.

Intrinsic Pathway
(Damaged Surface) Extrinsic Pathway
(Trauma)
o n
o
r-------) vil
Factor IX Factor IXo Factor VII^ Factor VII

1'
I vrrr" n
il
1,'
Factor X t-----1 Factor Xu !--------------- tsactorX

{}v^
Prothrombin (II) Thrombin (IIo)

Fibrinogen (I)
vil
Fibrin (Io)

vfl *,,,. {transgturaminase}


Crossed-linked Fibrin Clot

Figure 2-B
The intrinsic and extrinsic pathways.

A lot is known about the biochemistry of blood clotting. In the conversion of


Factor X to Factor Xa by Factor IXa, we find that we need an additional factor
called Factor VIIIa. Individuals lacking Factor VIIIa have hemophilia (a sex-linked
recessive characteristic). Factor VIIIa is sometimes called the antihemophiliac
JLtLtUt.

Copyright @ by The Berkeley Review 96 The Berkeley Review


Specializing in MCAT Preparation
Biology fleart & Lungs Blood Clotting

o HO Carboxylase enzyme OH o
il lil requiring Vitamin K ill il
l\ L C_N_C
lt
H
-

CH. 0,
and HCO3
L__r'
.

tt
HC H2
l' I

CHr p C -H
I

C. y Y-Crrboxyglutamate o=c/ C:O


Oo// \o ' resrdue I

o^
I

^o
L-.-'/ l:,/
*Can
Preprothrombin lragment

Prothrombin fragment
chelated with calcium.

Figure 2.9
Cheiating action.

vitamin K is one of the fat-soluble vitamins that is found in green leafy


''-egetables. in addition, our intestinal flora can make a form of vitamrn K. \A{rat
is the importance of this vitamin? Prothrombin exists in even an earlier form
:ailed preprothrombin. In preprothrombin there are certain Glu residues which
are carboxylated by a carboxylase enzyme. This carboxylase enzyme has an
absolute requirement for vitamin K. The carboxylase enzyme adds another
carboxyl group lo the gamma position of the first ten Glu residues located in the
amino terminal region of preprothrombin. Thus, in the presence of vitamin K,
l{CO3 and the carboxylase enzyme, we will be able to convert preprothrombin to
prothrombin. This structure now has a great
ffinity (a good chelating agent) for
lir.alent ions such as Ca2o. This is shown in Figure 2-9. (Calcium ions are
essential for biood clotting.)

Platelet Membrane
following injury.

r", *Ca+ a *ca* o


ooo
0C)
rl rl
"-\ ,L-u u-L a=a)-
\ Aftet cur is made.
C-H C H thlombin is released.
I

_A_
NH:
CH.
t- i/\
A section of prothrombin o
Cut

Figure 2. I O
Binding of prothrombin to a membrane.

Prothrombin is next converted to thrombin in the presence of Factor Xa. The


:onversion of prothrombin to thrombin in the presence of Factor Xa can be pictured
as shown in Figure 2-10. Blood platelets have phospholipid molecules in their
:rernbrane. The head of the phospholipid is negatively charged. This will allow

Cop;,right O by The Berkeley Review 97 The Berkeley Keview


Specializing in MCAT Preparation
Biology Ileart & Lungs Blood Clotting

the y-carboxyglutamate residues of prothrombin to bind via ca2e. Factor Xa also


has y-carboxyglutamate residues on it, and can also bind to the membrane via
interaction with Ca2@. The enzyme Factor Xa wiil make cuts (at Arg-X residues)
in the prothrombin molecule as shown. The portion of the prothrombin molecule
that is cut away is called thrombin. Thrombin will drift away and convert
fibrinogen into fibrin in the vicinity of the damaged area. It is fibrin that wili
{orm the blood clot. There is also an auxiliary factor, Factor Va, which is
involved in this process. So, what you need for this clotting process to take place
is the (a) platelet membrane, (b) enzyme, (c) Ca2e ions, (d) an auxiliary factor,
and (e) the substrate prothrombin.

Thrombin can now act as a proteoiytic enzytne that converts fibrinogen to fibrin.
Fibrinogen is a large solubie protein. Its solubility is due to an excess of
negatively charged amino acids (Glu, Asp, Tyr-so4), particuiarly in the central
domain of the molecule. The net charge in the central domain of the fibrinogen
molecule is -8, while the net charge at the terminal ends is -4. If very large
molecules have a net charge of zero, they will tend to come together. However, if
we have an excess of negative charges or positive charges, the molecules will
repel one another.

Fibrinogen

Fibrinopeptides
h-
Fibrin

fl Aggregation of
ll ribrin monomers
t,

Fibrin Clot
o
Figure 2-l I
Excess charge aliows for aggregation.

Thus, one way to make things soluble is to have an excess of charge. In order to
convert a soluble protein to an insoluble protein one must remove that portion of
the molecule contributing all those negative (or positive) charges. When
fibrinogen is converted to fibrin we find that 4 Arg-Gly bonds are broken in the
central domain. This releases four peptides containing an excess of negative
charge called fibr inop ep tid e s.

Copyright @ by The Berkeley Review 9a The Berkeley Review


Specializing in MCAT Preparation
Biotogy fleart & Lungs Blood Clotting

Once these fibrinopeptides are released, the overall net charge in the central
domain now becomes +5. The fibrin nLonlmer that is now forir.ed (due to the
release of the fibrinopeptides) has the ability to interact with other fibrin
nonomers through electrostatic interactions between the terminai and central
domain regions of the polypeptide. This is shown in Figure 2-r2. This
aggregation of fibrin monomers leads to the formation of the fibrin clot.

lhe clot that is initialiy formed is called a soft clot. The hard clot involves a
:-rrther step, in which there is a cross-linking via the enzyme transglutaminase
-r'r Factor XIIIa). Cross-links are formed between the individual subunits that
:--ake up the aggregate. Those cross-iinks occur between Gln and Lys resid.ues as
!.-o\\'n in Figure 2-12. This type of reaction is referred to as a trnnsaminntion
- . :lian.
";

Fibrin- CHz CHz C- NH2 NH3- (CHr4-Fibrin


- -
GIn Lys
I
il XIIIa (transglutaminase)
U

Fibrin- cHz CHz C- N- (CHz)+-Fibrin


- -
Cross-linked
Fibrin Clot

F1Eure
--tng Ieads to a hard clot.

damaged area has been repaired, a serine protease called plasmin


;'Le
'.'zes specific regions
in the fibrin ciot in order to dissolve it into smaller
::'1: rragments (to remove the clot). Tissue plasminogen actipstor (TpA)
;;-,. plasminogen into this active protease.

:-; :::rirn to vitamin K for a moment. vitamin K is one of the fat-soluble


,:=,-:.s 'hat is found in green leafy vegetables. in addition, out intestinal flora
- :::-<e a certain form of vitamin K. what is the importance of this vitamrn?
: :-:. e already discussed the importance o{ this vitamin in connection with
. - : :---:iing. If you were to have a vitamin K deficiency, you could suffer from
:* r-j r irsorders. one way to have a vitamin K deficiency is a failure to absorb
::icerly. Another way is to take a lot of antibiotics. Antibiotics have a
:: ::. :-'- :o kill off the intestinal flora that help synthesize vitamin K. Finally,
: := --r t',' not be enough of this vitamin in the diet.

:,-: also antagonists to vitamin K, like dicoumarol. In the 1920,s many


. -:.- i\-isconsin were having problems with their cows. They were feeding
s -t'arious feeds,
rLLUD/ some vr which
Dvu.E of vvrrrLrr were hemorrhaging IIt
'" causing rrcrt[(-,rrrrcl6.tttB
wstE LduD[lB in the
LIte cows.
COWS.
:':: that dicoumarol is a naturaily occurring material in the rotting clover
:==is that the cows consumed. Cows that ate the dicoumarol had an
'. :'":ritrombin that did not bind Ca2o.

br The Berkeley Review 99 The Berkeley Review


Specializing in MQAT preparation
Biotogy tleart & Lungs Blood Ctotting

It occurred to some chemists that they might be able to find an inhibitor similar
to dicoumarol which would not be toxic to cows, chickens, or even people, but
might have an effect on rats or mice. One of the things that farmers didn't like
was rats and mice running around eating their chicken feed. The chemists at the
Wisconsin Agricultural Research Station developed a vitamin K antagonist called
warfarin. If you mixed warfarin wiih chicken feed, and that feed was eaten by
the rats and mice, then eventually they wouid die of hemorrhaging. it would take
much greater levels of warfarin to be toxic to humans. It turns out that warfarin
is much more active in mammals than in birds, so the chickens were not affected.
Warfarin is a Vitamin K antagonist that is used as rat poison. The structures of
vitamin K, Warfarin, and dicoumarol are shown in Figure 2-13.

HCHT
ttl \
CH2 - C= C- CH2+ H
e(x"
\"-Yri:e
I
Vitamin K2 C=O
Warfarin
I

CH:

Dicoumarol
CH;

Figure 2. l3
Chemical structures of vitamin K, Warfarin, and dicoumarol.

Copyright O by The Berkeley Review loo The Berkeley Review


Specializing in MCAT Preparation
Biotogy Ileart 6r Lungs Gases

Gases :

Giucose can be oxidized to carbon dioxide and water as shown by equation (2-4).
The majority of carbon dioxide is generated i'r the Krebs cycle. oxygln is used as
the final electron acceptor in the electron transport chain. Water ls generated in
that reaction. out of thjs metabolic pathway
-" at" able to extract energy in the
form of ATP and that ATP is ultimately usecl to keep the organism alive.

60z + C6H12O6 r-> 6CO2 + 6I{2O + Energy


(2-47

In general, respiration is the process by which oxygen is brought to the cells of


the tissues and carbon dioxide is removed ur u
-urt" product. As we breathe air
into our lungs it first enters our system by way of the nose or mouth. The air
passes form the oral cavity to the pharynx, into the larynx, and then
down the
trachea. At the end of the trachea the air passes into two tubuiar passageways
called bronchi. One bronchus enters into eich lung and continues to divide into
smaller passageways called bronchioles, ending eventually in the functional
units of the lungs which are the alveoli. Each alveolus consists of a single layer of
epithelial cells juxtaposed to a very thin basement membrane. Sulrou1,air1g
each single alveolus is a capillary network. The epithelial cell layer of eacfi
alveolus and the endothelial layer of the capillaries are separated from each
other by a very narrow interstitial space (if they are separated by an interstitial
space at all). This means that the air in each alveolus and the blood in the
capillaries are separated by a very small distance (about 0.2 to 0.3 pm, compared
to the average diameter of an erythrocyte, which is about 7 pm). Th" t*o irrrg,
are composed of millions of alveoli, and if they were all laid flat on a surfac"e,
their combined total surface area would be between 70 m2 to 100
-2 1*ni.h i,
about the size of a tennis court). Therefore, Iarge quantities of oxygen in the
:
alveoli of the lungs can quickly be equiiibrated wlth-the blood in the "capiliaries
bec.auss of the large surface area and the thin barrier to diffusion for gas
exchange.

The air that we breathe on a normal day is composed of roughly 7g"/o N2,21"/.
oz' 03% Co2, and 0.7"/o H2o. [These values vary slight from textbook to
textbook.] Because al1 o{ these gas molecules are (normally) quite far apart, they
tend not to interfere with one another. This means that the pressure exerted bv
one gas rs independeruf of the pressure exerted by ail of the other gases. hr oth&
words, the sum of each of these individual gas pressures equals the total pressure
of this mixture of gas. The partial pressure of a gas (e.g., pN2, po2, pco2, etc.) is
therefore a measure of the concentration of a given gas (such as 02) in a mixture
of gases (i.e., the air). If we add up all the partial pressures of the individual
gases in the atmosphere, we will come up with the atmospheric pressure. At sea
level the atmospheric pressure is 760 mmHg.

partial pressure of oxygen gas (Po2) at sea level? Roughiy 21o/, of the
\MFrat is the
total atmospheric pressure at sea levei is oxygen gas. Therefore, the partial
pressure of oxygen gas at sea level is about 160 mmHg (from 0.21 x 7G0 mmHg
=
16o mmHg). what happens to the parti.al pressure of oxygen as we ascend to the

Copyright @ by The Berkeley Review lol The Berkeley Review


Specializing in MCAT preparation
Biology Ileart & Lungs Gases

top of a high mountain? The concentration of oxygen in the air


on this mountain
top will still be about 21"/o.However, as we increaJe our altitude
the atmospheric
pressure begins to decrease. The result is a decrease in the
partial pressure of
oxygen.

How does a gas behave in a iiquid? Consider an open glass of water


sitting on a
table as shown in Figure 2-L4. The surface of that water is
constantly being
bombarded with gas molecules from the air.

'.,., "'l
,',,
',9j,,
............
Watei,,

Figure 2-14
Gas movement across a water surface.

when a gas molecure such as 02 comes in contact with that water


surface it can
dissolve in the liquid. The number of O2molecules which
dissolve in the water is
directly proportional to the partiar pressure of the 02 gas.
Just as 02 molecules in
the air can hit the surface of the water and dissolv;; th"
ltq*d so too can 02
molecules in the liquid hit the surface of the water and into the air.
Th.erefore, at equilibrium the number of 02 molecules "slcape
dissolving
will equal the number of 02 molecures leaving the water. In otherinwords, the water
the
partial pressure of oxygen in the gas phase (air) Is equar to the partial
pressure of
oxygen in the liquid phase (water), as shown in equation (2_5).

(Por)gu. = (Poz)liquia
(2-s)
W5 cl say that the pressure of the air acting on the membranes of
the epithelial
cells in the alveoli of the lungs is the sum puitial pressures of all the
gases in the
air. Thus, the total pressure for any given gas molecule is directly
proportional to
the concentration of that gas molecure in the air. For example,
-" .u1 account for
the many factors that a{fect the rate of diffusion of a gas into a liquid by
considering equation (2-3). In this case J is the flux or difiusion rate
of ihe gas
molecule across the membrane, D is the diffusion coefficient, A is the
area of the
plane the molecule is diffusing across, C is the concentration of the moiecule,
and
x is the distance of diffusion. Since the concentration of a gas is proportional
to
the pressure of the gas, we can write that as shown in equati"on (2-6).

(DXA)(C1O, - C2o') (DXA)elo, - p2o)


J_ s -
Ax Ax (2-6)

Copyright @ by The Berkeley Review t02 The Berkeley Review


Specializing in MCAT preparation
Biology tleart 6r Lungs Gases

The respiratory passages have a unique anatomical structure. Let's consider a


:ross-section of one of these passageways and work our way from the lumen
rutward. The epithelial cells which line the lumen of the passageways to the end
ri the bronchioles have cilia which are continualty beating towards the pharynx.
Scattered among these epithelial cells are glands which secrete mucus. As the
:ilia beat towards the pharynx the mucus is moved upward towards the oral
:ar"ity. Any foreign matter that has become trapped in the mucus is eventually
::ansported to the oral cavity where it is swallowed in a normal reflex action.
,.-\'re of the reasons that smoking is bad for you (besides its carcinogenic nature) is
::.at it decreases the activity of the cilia and lowers the body's defenses agarnst
-'-::rg infection by bacteria that can enter the respiratory tract on airborne dust
::rticles. Immediately beneath the layer of mucus is a iayer of fiuid in which the
:rlia operate. Air that flows within the passageways of the respiratory tract is
'..-arrned and moistened.

-:.: upper passageways of the respiratory tract maintain their opening by means
- - cartilage rings that surround most of the diameter of the passageways. By the
,-:-Le the bronchioles are reached the cartilage has disappeared. smooth muscle is
-l-,ji-rd in almost all areas of the respiratory tract where there is no cartilage. For
= '.anple, the walls of the bronchioles are mainly smooth muscle. The bronchioles
: :re lungs are innervated by nerve fibers from parasympathetic nerves (which
::-,'ei ir-t the vagus nerve).

{sthma is usually caused by an allergic hypersensitivity to airborne antigens


---ch have entered the respiratory tract. The direct result is to cause the mast
-=---' rr-ithin the bronchioles to release a number of different substances which
-,'--'e the smooth muscle surrounding bronchioles to spasm and constrict.

ltechanics of Breathing
-' = thoracic cage which contains the lungs is separated from the abdomen by a
:,-::i oi skeletal muscle and connective tissue called the diaphragm (see Figure
--i5. The lungs themselves are encased in a pleural membrane. The visceral
:-eura covers the lungs, while the parietal pleura adheres to the diaphragm and
: = - l-.est wall. Between the viscerai pieurai and parietal pleura is the intrapleural
.:ace rr'hich contains a watery fluid. As the muscles of the diaphragm contract
-: .- rull the diaphragm itself downward. Simultaneousiy the muscles of the rib
.:= ccntract and cause the rib cage to move upward and outward. Both of these
- "-::-s enlarge the area of the thoracic cage that contain the lungs.

:
-= the diaphragm is attached to the parietal pleura this pleura is also pulled
. , ard. The watery fluid in the intrapleural space is rather indistensible and
",-:,'i!
,-= : -:Iling of the parietal pleura translates through the intrapleural fluid. It also
--: 'rre visceral pleura downward as well. Enlargement of the thoracic cage
:::.ls the lungs and creates a subatmospheric pressure in the alveoli. Air
-,.:.=. dorvn its pressure gradient from 760 mmHg at sea level to whatever lower
r::.i jl€ i.s found in the lungs at inspiration.

: :- u1e muscies of inspiration stop contracting, the elastic tissue found within
'= --:.lracic cage and lungs returns to its normal length. Air within the alveoli is
":::esseci and forced out through the passageways of the respiratory tract
-:-:.i expiration.

-:it O bv The Berkelev Review r03 The Berkeley Keview


Specializing in MCAT Preparation
Biology tleart & Lungs Gases

If the lungs were to become separated from the visceral pieura, they would
collapse. This is because they have no anatomical structures io maintain
rigidity.
one way to separate the rungs from the visceral preura is to receive a very
ltrorrg
blow to the chest area (as often happens during football games).

To the pharynx and oral cavity


f
Larynx E)
Intrapleural
Cartilage r space
nngs

Parietal
pleural
Diaphragm

Figure 2'15
The thoracic cage.

Copyright O by The Berkeley Review to4 The Berkeley Review


Specializing in MCAT preparation
Biology Ileart & Lungs Gas Exchange

Gas Exchange
It is important to remember that molecules u'il1 diffuse from a site of high
concentration to a site of low concentration. As deoxygenated blood (a dark red
color) is coming back from the tissues it enters the right atrium via the superior
and inferior vena cava and then passes into the right ventricle, where it is
pumped out to the lungs. The PO2 is about 40 mmHg and the PCO2 is about 46
mmHg as this deoxygenated blood enters the capillaries surrounding the alveoli
of the lungs. The PO2 and PCO2 within the alveoli are about 105 mrnHg and 40
mmHg, respectiveiy. Passage of the blood through the capillaries is relatively
slow and an equilibration can be reached between the gas exchange in alveoli
and the capillaries. Oxygen will diffuse down its concentration gradient from the
alveoli to the capillaries, and carbon dioxide will diffuse down its concentration
gradient from the capillaries to the alveoli. As the oxygenated blood (a bright red
color) Ieaves the capillaries of the lungs and enters the left atrium of the heart, the
PO2 is about 100 mmHg and the PCO2 is about 40 mmHg. See Figure 2-16.

PO2 - 160 PCO2 = 0.3

PO2,= QQ PO2 = 100


P,COz= 46 PCOz = 40

Capiliaries at level of alveoli


I
H
Capillaries at level of tissues
i
PO2=40,- FO2=l[g
PCOr'='46 : FCO2=49

PO2 < 40

Figure 2- 16
Gas exchange at the level ofthe lungs and tissues.

Oxygenated blood will pass from the left atrium to the left ventricle of the heart
and then be pumped to the tissue capillaries. At the level of the tissues the PO2 in
the cells (depending on which cells you are considering) is less than 40 mmHg,
while the PCO2 in those cells is greater than 46 mmHg. Oxygen will diffuse from
the blood in the capillaries to the ceils while carbon dioxide wiil diffuse from the
cells to the blood in the capillaries. The deoxygenated blood that leaves the tissue
capillaries returns to the right atrium of the heart via the venous system.

Copyright O by The Berkeley Review l05 The Berkeley Review


Specializing in MCAT Preparation
Biology Ileart & Lungs Gas Dxchange

Oxygen can travei in the blood by being dissolved in the blood itself or by being
bound to a transport protein in the red blood cells (erythrocytes) called
hemoglobin (abbreviated as Hb). Since oxygen is rather insoluble in water, not
much is actuaily dissolved in the blood and transported in that manner.
However, since hemoglobin has such a high affinity for oxygen, more than 987o
of the oxygen in contact with hemoglobin is transported by this protein.

Hemoglobin is a protein that is composed of four polypeptide subunits. When


these subunits interact with each other to form the hemoglobin molecule, they
give hemoglobin a quaternary structure. Located within each of the four
polypeptide subunits is a heme prosthetic group that has an iron atom in the
center which is in the ferrous (Fe2e, oxidation state. Since one hemoglobin
molecule has four binding sites for oxygen, there is a potential for 4 02 moiecules
to bind to one hemoglobin molecule. Every time hemoglobin takes up 02 from
the blood more 02 can leave the gas phase in the alveoli and enter into the liquid
phase in the blood. This increases oxygen's solubility in the blood.

Oxygen Saturation Curve for tlemoglobin


In Figure 2-17 we have a plot of the percent saturation of hemoglobin with
oxygen as a function of the partial pressure of oxygen at various places in the
body. Recall that when the PO2 is about 100 mmHg, we are at the ievel of the
alveoli and when the PO2 is about 40 mmHg, we are at the level of the tissues. [It
is best to read this curve from right to left and from top to bottom.]

Venous Arterial
Blood pH7.4 Blood

02 released
to tissues
€80
b0
Extra 02
released
f;oo
qi to tissues

Shift due to:


'E 40 Decrease in pH o
(.)
!
Increase in temperature
d Increase in 2,3-DPG
a
*o ?O
Resting Muscle
Working Muscle

Figure 2- l7
Oxygen-hemogiobin dissociation curve

At the level of the capillaries (PO2 = 100 mmHg) surrounding the alveoli roughly
98'/" of the hemoglobin is saturated with oxygen. As the blood passes to the
tissues (PO2 = 40 mmHg) roughly 25"/' of the oxygen that was bound to
hemoglobin is given up to the tissues. This can be seen in Figure 2-17.

Copyright @ by The Berkeiey Review l()6 The Berkeley Review


Specializing in MCAT Preparation
Biology Ileart & Lungs Gas Exchange

Effects of ptl, Temperature, and 2,3,BP(i


The upper oxygen dissociation curve shown in Figure 2-17 is for normal
conditions in which the body temperature is about 37 "C and the blood pH is
about 7.4. If we were to decrease the pH of the blood (to a pH of 7.2) or increase
the temperature, then the oxygen dissociation curve would shift to the right and
downward as shown in the lower curve in Figure 2-17. This happens when you
exercise. The same iype of shift occurs when a by product of glycolysis, 2,3-
bisphosphoglycerate (abbreviated as 2,3-BPG), binds to hemogiobin. 2,3-BPG is
usually slnthesized in increased amounts when the body has been deprived of
oxygen for an extended period of time (e.g., when you visit high altitudes).

AII three of these interactions with hemogiobin (the lowering of the pH, the
increase in temperature, and the binding of 2,3-BPG) cause hemoglobin to release
more oxygen to the tissues at the same partial pressure of oxygen as in the
standard case. The oxygen dissociation curve can be shifted to the le{t and
upward by reversjng these interactions.

Carbon Dioxide
How is CO2 carried in the blood? It can be carried by (a) dissolving in the plasma
and the red blood ceiis, (b) binding to a specific site on the hemoglobin molecule,
or (c) in the form of bicarbonate ions (HCO3e), About 70% of the carbon dioxide
rs carried in the blood in the form of bicarbonate ions, roughly 20o/o is carried by
the hemoglobin itself, and about 10"/' is dissolved in the plasma and red blood
cells. See Figure 2-18.

7jVo
Plasma
Red Blood Cell

anhydrase ^ / ^
CO2, + H2O -:-+ H2CO3 H@ + HCO3e
20Vo Hb-COo, 57a CO,t

57a COo dissolved

Capillary
CO" from tissues

Figure 2-IB
,las exchange at the level of the tissues.

-\t the level of the tissues the PCO2 is greater than 46 mmHg. However, in the
:Iood of the capillaries it is about 40 mmHg. Therefore, CO2 will diffuse down its
:oncentration gradient and into the blood. Some of the carbon dioxide will
jissolve in the biood plasma, some will dissolve in the red blood cell, and some
,,, il1 bind to hemoglobin. The remaining CO2 will react with water and be

:onverted to carbonic acid (H2CO3) by the enzyme carbonic anhydrase. Carbonic


acid will ionize to the bicarbonate ion and a proton (H@). Bicarbonate will diffuse
rto the biood plasma and be carried by the circulatory system to the capillaries
-,: the lungs. See Figure 2-18.

-opyright @ by The Berkeley Review t07 The Berkeley Review


Specializing in MCAT Preparation
Biology tleart E( Lungs Gas Exchange

At the level of the capillaries in the lungs, the PCO2 in the blood is about 46
mmHg, while the qCOZ in the alveoli of the iungs is about 40 mmHg. Once
again, CO2 wiil diffuse down its concentration gradient and into the alveolar
space. The CO2 that is dissolved in the plasma and in the red blood cell diffuses
into the alveoli as does the CO2 that was bound to the hemoglobin. Bicarbonate
ion in the plasma will diffuse into the red blood cell and combine with a proton
to become carbonic acid. Carbonic anhydrase will convert carbonic acid to water
and carbon dioxide. The CO2 diffuses into the lungs. This is shown in Figure 2-
19.

Red Blood Cell

CO2 + HzO H2CO3 <--:Ho + HCO3o

-
CO.r dissolved

To the alveoli

Figure 2- l9
Gas exchange at the level of the alveoli.

Control of Breathing
The coordinated rhythm for breathing is generated by the medulia and the pons
in the brainstem. Nerve impulses from these centers cause the respiratory
muscles to contract. \Alhat is the stimulus that causes contraction? The major
stimuius is a change in the concentration of CO2 and Ho that affects chemo-
sensitive areas in the medulia. The concentration of 02 in the blood is sensed by
chemoreceptors in the carotid arteries and in the arch of the aorta. These
chemoreceptors transmit signals to the respiratory center in the brainstern,
informing it of the levels of 02 in the blood.

If you were to breathe into a container that collected your exhaled CO2, you
wouid soon find yourself breathing deeper and faster. The increase of CO2 in
your blood indicates to your respiratory center that CO2 is building up rn the
body. One way to eliminate CO2 wouid be to breathe faster and deeper. What
wouid happen if you were to breathe pure 02? The chemorecePtors for 02 would
relay a message to the respiratory centers indicating that there is too much
oxygen in the blood. Impulses from the brain stern would be sent to the
respiratory muscles in order to slow their action.

Copyright O by The Berkeley Review loa The Berkeley Review


Specializing in MCAT Preparation
Ffleart & Lungs
To Go
15 Passages

100 Questions

Time for All Passages Taken Together as a Practice Exam


125 Minutes

Passage Titles Questions


I. Mechanics of Blood Flow t -7
III. Enzymes in Blood Cloning 8-14
ilI. Cardiac OLttpnt & Venous Return 15 -2r
11 l0
IV. Circulatoryt Pressure, Area, Velocity, & Volume
V. Thoracic Cavitlt 29 -34
VI. Electrocardiogrant 35-40
VII. Folate Experiment 4t-41
VUI. Measurement of Blood Pressure 48 - 51,
IX. Aortic Compliance 55-60
X. Heart Muscle Action Potentials 61-66
XI. Capillary Filtration 6t -72
XII. Respiratorry Calculations 73 -78
XIIL Aspirin 19-85
XIV. Sickle Cell Anemict 86-92
XV. VentilationRegulatiott 93 - 100

Speiializing in MCAT Preparation


Suggestions
- The passages that follow are designed to get you to think in a conceptual manner about the processes
of physiology at the organismal level. If you i uv" a solid foundation in prryriotogy,
answers will be straightforward' If you have not had a pleasant experience
*uny of these
with the topic, some of these
answers might appear to come from the void past the Oort field of the solar
system.

. Pick a few passage topics at random. For these initial few passages, do not worry about the time. Just
focus on what is expected of you. First, read the passage.'s"coila, look at uny'diugru*s,
graphs' Third, read each question and the u""o*punying answers carefully. Fourth, charts, or
answer the
questions the best you can. Check the solutions and see how you did. \A4reth",
yt.r got the answers right
or.wrong/ it is important to read the explanations and see if you understand
being explained. Keep a record of your results. iur,a ugr"u with) what is

After you feel comfortable with the format of those initial few passages, pick another
block of
passages and try them. Be aware that time is going to become importani. Gerieral$, you will
have about
1 minute and 15 seconds to complete a question. Be a little more
creative in how yo, upprouch this next
group' If you feel comfortable with the outline presented above, fine. If riot, then try different
approaches to a passage' For example, you might feel well versed enough
to read the questions first and
then try to answer some of them, without ever having read the passage.
Maybe you can answer some of
the questions by just looking at the diagrams, charts, 6r graphs ttnut ui pr"r".rt"i
in a particular passage.
Remember, we are not clones of one another. You need-totegin to devetop
a format that works best for
you. Keeping a record of your results may be helpful.

The last block of passages might contain topics that are unfamiliar to you.
Find a place where the
levei of distraction is at a minimum. Get out your watch and time yorrr""lf
on these passages/ either
individually or as a gr9uP. It is important to have a feel for time, and how
much is passing as you try to
answer each question. Never let a question get you flustered. If you cannot
figure ort *hut the answer is
from in{ormation given to you in the passage, or from your own knowledge-Euru,
Jrr*p it and move on
to the next question. As you do this, make-a note of that pesky question
and come back to it at the end,
when you have more time. When you areanishe.l, check yolr, ur,r-"rs and
make sure you understand
the solutions' Be inquisitive. If you do not know the answer to something,
look it up. The solution tends
to stay with you longer. (For example, what is the Oort field?)

The estimated score conversions for 100 questions are shown below. At best,
these are rough
approximations and shouid be used only to give one a feel for which ballpark they are
sitting in.

Section tr
Estimated Score Conversions
Scaled Score Raw Score

>t2 86- 100


10- 11 19-8s
8-9 65-78
7 59-64
6 54-58
5 48-53
<4 0-41
Biology Mechanics Of Blood Flow Passage I

Passage I (Questions 1-7) 3. Which of the following equations BEST represents


blood flow for the entire vascular system acting as a
Flow can be used in three different contexts: (1) as unit?
flow, (2) as average velocity, and (3) as the linear velocity
of a small element of fluid. The total volume of fluid A. QxTPR=(CVP-MAP)
passing a given point per time is known as the flow rate. B. MAPxQ=(TPR-CVP)
The flow at a given point divided by the cross-sectional C. QxTPR=(MAP-CVP)
area is known as the average velocity of flow. The linear D. CVP x MAP = (Q _ TPR)
velocity is the distance traveled by small volume of blood
per unit time. The flow rate (Q) must remain constant
throughout the entire cardiovascular system. 4. The increased viscosity of whole blood relative to
piasma is best explained by the contribution of:
Blood flow through a blood vessel results from a
driving force in the form of a pressure gradient. The A. albumin.
petfusion pressure along the length of any blood vessel is B. glucose.
given by the differences between the pressure at the C. white biood celis.
proximal and distal end of the vessel. The following D. red blood cells.
relationship between blood flow and perfusion pressure
can be established:

Perfusion Pressure The structure of systemic veins differ from the


Blood Flow - structure of systemic arteries in that only:
Flow Resistance

The mean arterial pressare (MAP) is a time-weighted A. the inner surface of veins is lined with a single
average of the arterial pressure over the entire heart cycle. endothelial layer.
The net flow resistance of the systemic loop can be B. arteries contain a component of elastic tissue,
deflned as the total peripheral resistance (TPR) while the C. veins contain valves that favor one-way blood
entire venous pressure (entire vascular system acting as flow.
one unit) is termed the central venous pressure (CVP). D. arteries contain a layer of smooth muscle.

The viscosity of a fluid is a measure of the internal


u'ork necessary to make a fluid flow. The viscosity of 6. The most likely reason that the central venous
blood is determined by the composition of blood, the pressure (CVP) is not an average value similar to the
nature of the vessel in which it is flowing, and the mean MAP is because the:
flow velocity. Plasma has a viscosity of about 3, twice
that of water, while whole blood has a viscosity A. cross-sectional area of the veins is larger than
approximately three times that of water. the arteries.
B. venous pressure does not change significantly
1. Capillary walls provide the minimum possible over the heart cycle.
barrier to diffusion. Relative to the arteries and C. arterial pressure does change significantly over
veins, the blood in capillaries has a: the heart cycle.
D. arterial pressure rises rapidly with increased
A. low average velocity because of a high total volume while the venous pressure remains
cross-sectional area. within a few mmHg with the same increase in
B. low average velocity because of a low total volume.
cross-sectional area.
C. high average velocity because of a high total
cross-sectional area. 1 In the classical method of measuring blood pressure,
D. high average velocity because of a low total pressure from a cuff is applied to the brachial artery
cross-sectional area. in the arm to coliapse the artery and prevent blood
flow. The pressure of the cuff in this initial step
1
The diameter of a uniform blood vessel is reduced must exceed:
by a factor of 2. The resistance of this blood vessel
increases by a factor of: A. CVP.
B. MAP.
A. 2. C. arterial diastolic pressure.
B. 4. D. arterial systolic pressure.
C. 8.
D. 16.

Jopyright @ by The BerkeleY Review lll The BerkeleY Review


Specializing in MCAT PreParation
Biology Enz;rmes ln Blood Clotting Passage II

Passage II (Questions 8-14) Hemostasis is the process by which bleeding is


arrested. At the time of injury, enucleated blood celis
Figure 1. Blood Clotting Cascade cailed plateiets bind to the exposed subendothelium ofthe
damaged blood vessel. As platelets begin to aggregate to
Intrinsic Pathway one another, a platelet plug is formed. Vasoconstriction
Kallrkrern (___ ,] Plekallikrein of the blood vessel is stimulated by the release of
J_L
V
A thromboxane ,A2 and serotonin from the platelets. The
purpose of the platelet plug is to provide a surface that
allows for the activation of blood coagulation factors
(Table 1), many of which are synthesized in the liver.
Activation of these coagulation factors initiates the blood
clotting cascade (Figure 1). The end result of this cascade
is to form a blood clot (thrombus) that will seal the
damaged blood vessel.

Many of the coaguiation factors are glycoproteins


that can be ciassifled as being pre-enzymatic. They exist
in their inactive form as zyllogens of serine proteases,
proteolytic enzymes that have a common catalytic
mechanism based on a highly reactive serine resiclue at
the active site. Once activated these serine proteases
j .,.,oQ.Gu.,o. enhance the coagulation cascade.

ixtrx The blood clotting cascade can follow either the


"T"'x-"'ll
iL--------__l intrinsic pathway, initiated by contact of an appropriate
surface, or the extrinsic pathway, initiated by tissue
\.2 trauma. Both pathways converge at factor Xa, the
/?\
i! P.ottrro-olA activated form of factor X. Factor Xa initiates the finai
sequence of events in the common pathway that will lead
i-----a------- Thrombin to the formation of a clot.

vl
Factor--J-- 12 Vitamin K is essential for the synthesis of many of
the coagulation factors, including prothrombin. The
xilr \ XIIIa
- I.,-. Fibrinogen reduced form of this cofactor, vitamin K hydroquinone, is
Pibrin piurin involved in a carboxyiation reaction that converts many of
€ the glutamate residues iocated in the N-terminal segment
(hard clot) (soft ciot)
of these clotting factors into y-carboxyglutamate. Vitamin
K hydroquinone is regenerated in a two-step reaction that
Table 1. Biood Coagulation Factors
can be inhibited by antagonists like dicoumarol and
Common Name Factor HalfJifea warfarin.
Fibrinogen I 90-120
Prothrombinl * II 50- 120 y-carboxyglutamate is an excellent chelator of Ca2+
Tissue factor (thromboplastin) III N/A and, following injury, acts as an anchoring mechanism
Calcium IV N/A between calcium-dependent coagulation factors and the
Proaccelerin V 12-24 phospholipid membranes of platelets^ The functionai
Proconverrint * VII 2-6 significance of this event is that it brings specific factors
Antihemophilic factor VIIi 10-12 together that aid in the formation of a clot, thereby
Christmas factortf IX 18-30 accelerating clot formation many fold.
Stuart-Prower factort* X 25-60
Plasma thromboplastin antecedent+ XI 45-80 Defects in the blood clotting cascade may be assessed
Hageman factort XII 40-'74 using screening tests. Partial thromboplastin time (PTT)
Fibrin-stabilizing factor XIII '70-200 screens the intrinsic and common pathways while
Fletcher factor* Prekallikrein 48-52 prothrombin time (PT) screens the extrinsic and common
pathways. A deficiency in factor XIII cannot be detected
S Ca2*, Phospholipid Membrane A Reported in hours hy using lhese two lests.
f Dependent on Vitamin K
$ Undergoes activation to Serine Protease

Copyright @ by The Berkeley Review tt2 The Berkeley Review


Specializing in MCAT Preparation
Biology Enz5rmes In Blood Clotting Passage Il

8. Mature red blood cells obtain their energy from: 12. Hemophilia A results from a deficiency of factor
VIII while hemophilia B results from a deficiency of
A. electron transport. factor IX. Genes coding for both hemophilias reside
B. oxidativephosphorylation. on the X chromosome and are genetically
C. Krebs cycle. transmitted as a sex-linked recessive trait. What is
D. glycoiysis. the probability that a woman whose father has
hemophilia, and who marries a normal man, wili
have an affected son?

Glutamate is converted to y-carboxyglutamate in the A. 1.00


vitamin K dependent reaction shown below: B. 0.75
c. 0.50
HO
tlt
D. 0.25

-N-C-
tl I

HCH2H
I Glutamate
CHr
l6
coo 13. Which of the following compounds, when added
CO; stored whole blood, BEST prevents clotting?
- tlr-",',ll3illi"5"r
Y
A. B.
l\*
{,}
u,,u',"
Quinone " -) coo
6
^HO
elilo
HO I

lll CH,
r-o H"N-C-C-O
,t
-N-C-
tl I
HO - C-
I
COO CH"
HCH?H l-
-r CHt
a
y-Carboxyglutamate ro CH.
l6
-CH\9 coo coo
OOC COO
Citrate Glutamate
l-e :nzvme which BEST regenerates vitamin K
1r;.-rquinone is called a: C. D.
HO
A @tilo
-{- decarboxylase. coo H3N- C- C- O
K --arboxylase. I

(--. :e,luctase. CH"


t-
CH.
t'
nD. rrotease. CHt
IA o,cfo
coo OOC COO

Succinate y-Carboxyglutamate

tlilt Tr* ,:,111 coagulation factor listed below that DOES


I'u -'T :e,quire vitamin K is:

q., F';ror [.
S- -::omboplastin.
{:" Cr:istmas Factor. 14. The most common oral anticoagulant in use at the
0lr F::ror X. present time is warfarin. Administration of this
compound begins with a daily dose of about 5 mg
and adjustments are made 3 days later to maintain a
PT between I.2-I.5 times the control. This is most
LlililIl,L *,ill :i --::e tollowing mechanisms will limit clot likely attributed to a decrease in:
r'Tntu: L\CEPT:
A. prothrombin.
fl. :,:'r-.::rnFttion of leafy green vegetables. B. proconvertin.
il, =-r:.:,;al from circulation by the liver. C. Christmas factor.
d- n;'::iors of serine proteases. D. Stuart-Prowerfactor.
0u :r:r:r:,c 1-lo$' dilution.

tlMtryMpmlr"Wtn 3 1 ![he Berkeley Review I l5 The BerkeleY Review


Specializing in MCAT Preparation
Biology Cardiac Output & Venous Return Passage III

Passage III (Questions 15-21) If there is an increase in the venous pressure, then during
diastole there wili be more filling of the ventricles. Thi;
The blood returning to the right heart from the venous stretches the muscle fibers in the heart tissue and leads to
system must equal the blood leaving the left heart to the a greater force of contraction during systole. This is
arteriai system over any extended period of time. In a known as Starling's law of the heart, and simply stated
normal individual the venous return and the cardiac output says that an increase in venous pressure leads to an
is about 5 L/min. increase in cardiac output.

The vascular function curve in Figure 1 depicts a


relationship between venous pressure and venous return.
As the heart contracts, blood is removed from the vessels 15. The component of the circulatory system with the
of the venous system and pumped into the vessels of the LEAST total percentage of blood volume is the
arterial system. network of:

A. capillaries.
a10 B. arterioles.
! C. arteries.
e6 D. small veins and venules.
94
oL
o^
_4 0 4 8 16. The pH of the blood passing through the pulmonary
1216
Venous pressure (mmHg) artery, compared to the pH of the blood passing
through the renal artery, is:
Figure I
A. lower due to the loss of 02.
B. higher due to the loss of 02.
If the cardiac output and the venous return is increased, C. lower due to the accumulation of CO2.
the pressure in the arterial system will increase while the D. higher due to the accumulation of CO2.
pressure in the venous system will decrease. This is due
to the increased transfer of blood from the venous to the
arterial system.
17. Which of the following curves BEST represents an
If the heart were stopped and the cardiac output was increase in only arteriolar resistance to blood flow?
reduced to zero, the pressures in the venous and arterial
systems would soon equilibrate. However, the pressure A. B.
in the venous system will be at its highest value, simply
because the veins now contain more blood than when the a10 a10
heart was contracting. This pressure is referred to as the SU tr
J
mean circulatory pressure (MCP) and varies with g6
sympathetic stimulation and blood volume.
g6
E4 E4
The cardiac function curve in Figure 2 depicts a =)
o^ O^
relationship between venous pressure and cardiac output. >U
-4 0 4 8 1216 -4 0 4 8 1216
Venous pressure (mmHg) Venous pressure (mmHg)
410
C. D.

E6
o ?10 10
AA
J i! 8
a
c6 6

8o -4 0 4 I 12t6
94 o 4
a) 2
Venous pressure (mmHg)
o^ o
0
Figure 2
-4 0 4 8 1216 -4 0 4 8 1216
Venous pressure (mmHg) Venous pressure (mmHg)

Copyright @ by The Berkeley Review rt4 The Berkeley Review


Specializing in MCAT Preparation
Biology Cardiac Output & Venous Keturn Passage Itr

18. Changes in blood pressure or blood volume are 20. Sympathetic nerve fibers are supplied to all parts of
detected by stretch receptors in the atria and arteriai the heart while parasympathetic nerve fibers are
baroreceptors. Which of the following pairs of located primarily at the sinoatrial (SA) node and the
hormones act directly to increase blood volume? atrioventricular (AV) node. The neurotransmitter
released by sympathetic heart nerves is:
A. Renin and vasopressin
B. Ang)otensln II and aldosterone A. norep)nephrine, because it opens Na@ and
C. Renin and angiotensin I Ca2@ channels in myocardial cells.
D. Vasopressin and aldosterone B. acetylcholine, because it opens Ko channels in
myocardial cells.

19. Both the cardiac output (C.O.) from the heart and the
C. norepinephrine, because it opens Ko channels
in myocardial cells.
venous return (V.R.) to the heart depend on venous
pressure. This allows the curves in Figure I and D. acetylcholine, because it opens Nao and Ca2o
Figure 2 of the passage to be combined into one channels in myocardial cells.
graph as shown below:

c lU
:R 21. All of the following compensatory mechanisms
B6 would be expected to occur during a hemorrhage
a EXCEPT:
o+
o
da
E A. increased sympathetic discharge to the
arterioles.
-4 0 4 8 1216 B. decreased parasympathetic discharge to the
Venous pressure (mmHg)
heart.
C. increased cardiac output.

Which of the following graphs BEST represents


D. decreased arterial pressure.

activation of the sympathetic nervous system? (Solid


lines represent normal curves; dashed lines represent
changes.)

A. B.

410 410
SR
a
E6
a
O+ e4
o
d1 6a
E
I

UU
-4 0 4 8 12t6
5o -4 0 4 8 1216
Venous pressure (mmHg) Venous pressure (mmHg)

C. D.

al0 ai0
58 I
=o
q =O
q
a^ oa
a+
o
c
2 b
do -4 0 4 8 1216
d0 -4 0 4 8 1216
Venous pressure (mmHg) Venous pressure (mmHg)

Copyright @ by The BerkeleY Review ll5 The BerkeleY Keview


Specializing in MCAT PreParation
Biology Circulatory Pressure, Area, Velocity, & Volume Passage IV

Passage IV (Questions 22-28) The pulmonary veins return the oxygenated blood to
the left atrium of the heart which then passes it to the left
The total blood volume in an average human being is ventricle. Contraction of the left ventricle sends the blood
about 5.5 L. Blood flows through the circulatory system, into the aorta under high pressure.
which is divided into the systemic and pulmonary
circulatory systems. The systemic system supplies biood Biood passes down the descending aorta and into the
to all of the tissues of the body except the iungs. The systemic arteries, arterioles, capillaries, venules, and veins
pulmonary system supplies biood to the lungs. before returning to the right atrium by way of the superior
and inferior vena cava (the great veins).
The passage of biood through the circulatory system
can be examined in terms of differences in the pressure,
cross-sectional area, velocity, and total blood volume in
different segments of this closed system.

22. The flow of blood throughout the body is primarily


100
reguiated at the level of the:
480
:( A. veins.
Eoo B. capillaries.
O
! C. arterioles.
=40 D. aorta.
o
AIU

20 40 60 80
Total Biood Volume (%) 23. Which point on the graph of pressure versus total
blood volume BEST represents blood flowing
5000
through the pulmonary artery?

J 4ooo A. I
E
O B. II
II :ooo c.m
D. IV

1000

20 40 60 80
24. Which point on the graph of area versus total blood
Total Blood Volume (7o) voiume BEST represents blood flowing through the
lungs?

A. I
50
B. II
940
tr
c.m
O
D. IV

t0

0 20 40 60 80 100 25. Which point on the graph of velocity versus total


Total Blood Volume (7o) blood volume BEST represents blood flowing
through the vena cava?

Blood returns to the right atrium of the heart from the A. I


systemic system under low pressure and is passed to the B. II
right ventricle. Contraction of this ventricie pushes the C. ilI
blood into the pulmonary arteries where it flows to the D. IV
lungs.

Copyright @ The Berkeley Review I l6 The BerkeleY Review


Specializing in MCAT Preparation
Biology Circulatory Pressure, Area, Velocity, & Volume Passage fV

26. During circulation of the blood, turbulent flow could


increase with a decrease in the:

A. viscosity of the blood.


B. velocity.of the blood.
C. diameter of the blood vessels.
D. Reynold's number.

In the circulatory system, the vascuiar distensibilitv is


the LEAST for:

A. systemic veins.
B. pulmonary veins.
C. systemic arteries.
D. pulmonary arteries.

l8- \\hich of the following is NOT characrerisric of the


s\ stemic and pulmonary capillaries of an average
adult human?

{. The diameter of a capillary is the smallest of


any of the vessels.
B. The thickness of a capillary wall is the thinnest
of any of the vessels.
C. The capillaries contain between SVo to l\Vo of
the total blood volume.
D. The capillaries contain about 20Vo of the total
blood volume.

llxnrr r-r 3 The Berkeley Review tt7 The Berkeley Review


Specializing in MCAT preparation
Biology Thoracic Cavity Passage V

Passage V (Questions 29-34) .tr


,o
100
o
bo
The thoracic cavity of the body protects the heart and
cEo
lungs and is important in the mechanics of respiration. o
The thoracic cage is composed of the ribs, costal
cartilages, sternum, vertebral column, primary and o.^
troU
associated musc u I ature. o
d
The walls of the thoracic cage are constructed of the
(n
ribs. These are curved bones that articulate with the
vertebral column posteriorly and extend anteriorly and 9" 20
medially to articulate with the sternum. Out of the 72 X
pairs of ribs, only the upper six to seven pairs of ribs will
articulate directly to the sternum. The remaining ribs will sao
attach by costal cartilages. 20 40 60 80 r00
Partial Pressure of Oxygen
Spanning the distance between each rib are the
(external and internal) intercostal muscles. Attaching to Figure i
the most inferior ribs and costal cartilages, and extending
posterior to the vertebral coiumn, is a large, flat, dome-
shaped muscle called the diaphragm. These muscles,
along with several of the abdominal muscles, are tr 100
important in the expansion and compression of the ,.o
o
thoracic cage, which is necessary for the function of bo
o
respiration. c6U
+6)
Lining the internal wall of the thoracic cage is a sheet o60
of tissue referred to as the parietal pleura. Lining the o
external surface of the lungs is a similar sheet of tissue cc
k
called the visceral pleura. Between these two layers of :40
cd
tissue is the pleural cavity, a space containing a small (A
amount of fluid and registering a negative atmospheric
a0 2\)
pressure.
X

The functional unit of the iungs is referred to as the ra


a\0
alveolus, a small sac lined with epithelial cells. Each 20 40 60 80 100
alveolus is surrounded by a network of capillaries. The Partial Pressure of Oxygen
wails of the alveoli and the capillaries are quite thin,
allowing for maximal efficiency in gas exchange between Figure 2
the two structures.

Red blood cells are the primary means of gas


transport between the lungs and the tissues of the body.
The protein hemoglobin, a component of red blood ceils,
binds oxygen at the level of the alveoli and transports this
gas by way of the circulatory system to the tissues of the
body. Carbon dioxide, a waste product of metabolizing 29. As the intercostai muscles lift the thoracic cage, how
tissues is transported back to the lungs by the circulatory will pressure in the pleural cavity change?
system.
A. No change to pleural pressure.
The transport of both gases is dependent not only on B. Pleural pressure will become more negative.
their partial pressures but also on the temperature and pH C. Pleural pressure will become more positive.
of their environment. Figure 1 and Figure 2 represent a D. None of the above.
relationship between the partial pressure of oxygen and
the percent oxygen saturation of hemoglobin.

Copyright @ by The Berkeley Review tlB The Berkeley Review


Specializing in MCAT Preparation
Biology Thoracic Cavity Passage V

30. A pneumothorax is the presence of gas in the pleural 33. In the graph shown in Figure 1, curve I represents a
space of the thoracic cavity. This condition leads to a relationship between the partial pressure of oxygen
collapsed lung. In order for a collapsed lung to and the percent oxygen saturation of hemoglobin at a
occur, which structure associated with the thoracic pH of 7.40 and a temperature of 38 'C. If the pH of
cavity needs to be punctured? the blood is increased, curve II depicts the result.
Which of the statements below explains this
A. Internal intercostal muscles phenomenon?
B. Parietal pleura
C. Ribs I. A rise in pH requires a lower partial pressure of
D. Heart 02 to bind a given amount of 02.
il. A rise in pH requires a greater partial pressure
of 02 to bind a given amount of 02.
III. A rise in pH requires a lower partial pressure of
CO2 to bind a given amount of 02.

A. I only
B. II only
c. III only
31. Several muscles are involved in respiratory function. D. I and III only
If the external intercostal muscles are non-functional,
which muscle group will take over the primary
functions of normal respiration?

I. Abdominal muscles
il. Diaphragm
ilI. Internalintercostals

A. I and III only 34. In the graph shown in Figure 2, curve I represents the
B. II only relationship between the partial pressure of oxygen
C. III only and the percent oxygen saturation of hemoglobin at a
D. Normal respiration cannot occur. pH of 7.40 and a temperature of 38 "C. If the
temperature ofthe blood is increased, curve II depicts
the result. Which of the statements below explains
this phenomenon?

L A rise in temperature requires a lower partial


pressure of 02 to bind a given amount 02.
U. A rise in temperature requires a greater partial
32. The main gases that are exchanged in the lungs are pressure of 02 to bind a given amount of 02.
carbon dioxide and oxygen. This exchange depends III. A rise in temperature requires a lower partial
to a large extent on the partial pressures of these pressure of CO2 to bind a given amount of 02.
gases (pCO2 and pO2) in the capillaries and the
alveoli. Which situation is ideal for normal gas A. I only
exchange between the capillaries and the alveoli of B. II only
the lungs as blood enters the lungs from the C. III only
pulmonary arteries? D. II and III only
A. High pCO2 in the capillaries; low pO2 in the
capillaries.
B. Low pO2 in the capillaries; low pCO2 in the
capillaries.
C. High pCO2 pressure in the lungs; high pO2 in
the lungs.
D. Low pO2 in the lungs; high pCO2 in the lungs'

Copyright @ by The BerkeleY Review 119 The BerkeleY Review


Specializing in MCAT PreParation
Biology Electrocardiogram Passage VI

Passage VI (Questions 35-40) 36. The phase of contraction of the ventricies is called
systole. Based on the passage, to which part of,the
Potentiai differences within the heart rissue are ECG does systole best correspond?
conducted to the surface of the skin because the body is a
good conductor of electricity. A reading of these patterns A.P
provides a graph representing the electrical activity of the B. QRS
heart, called an eiectrocardiogram (ECG). C.T
D. T-P phase
During each heartbeat cycle, 3 distinct wave patterns
are produced: P, QRS, and T waves. The waves represent
changes in potential occurring across heart tissue as the
result of many action potentials inside the myocardial 37. Fibrillation involves random contraction of
cel1s. myocardial fibers at different times. It can be
monitored with an ECG. What is the effect on an
The P wave represents the spread of depoiarization individual who experiences ventricular fibrillation?
through the atria. The QRS wave represents the spread of
depoiarization into the ventricles. The T wave represents A. More heartbeats than usual.
the repolarization of the ventricles. The graph shown in B. Fewer heartbeats than usual.
Figure 1 indicates a standard cycle of these waves. C. Unaffected.
D. Death, if a normal rhythm is not introduced.

38. Which group of pacemaker cells provides the trigger


for depoiarization in the right atrium and initiates the
P wave?

- 0.5 A. Sinoatrial node


o B. Atrioventricular node
C. Bundle of His
D. Purkinle fibers

39. Why is the body a good conductor of eiectricity?

200 400 A. Tissue fluids contain ions that participate in


Milliseconds conduction.
B. Tissue fluids are free of ions that hamper
Figure 1 conduction.
C. Tissue fluids contain ions that hamper
conduction.
D. Tissue fluids are free of ions that participate in
conduction.

40. The ECG can also be used to count the number of


35. Which of the following conditions could be heartbeats per minute (the cardiac rate). Which
monitored with an ECG? person would most likely have the slowest cardiac
rate at rest?
A. Blood pressure
B. Cardiac output A. An infant
C. Arhythmias B. A marathon runner
D. Hematocrit C. A weight lifter
D. A sedentary adult

Copyright @ by The Berkeley Review r20 The Berkeley Review


Specializing in MCAT Preparation
Biology Folate Experiment Passage VII

Passage VII (Questions 4l-47) 42. A microbiological assay using a folate-dependent


strain of Lactobacillus casei is used to determine
Currently, the recommended dietary allowance for the levels of folate in the plasma and in the red blood
B-r'itamin fblate is 200 trtg/day fbr non-pregnant women. cells. What type of laboratory technique is used to
One group of scientists explored the adequacy of the 200 separate the plasma and the red biood cells?
,reidal' recommendation in the fbliowing experiment.
A. Extraction with an organic solvent
Erueriment I B. Centrifugation
C. Thin layer chromatography
30 non-pregnant women were assigned to 3 different D. Dialysis
Jietary groups: either a diet containing 200 pg/day, 300
-tgiday, or 400 rp"glday of folate. They foilowed the
:resclibed diet for 10 weeks. Levels of folate in the How would a microbiological assay for foiate
:iasma and in the red blood celis (RBCs) were measured performed using Lactobacillus casei ?
"t the beginning of the experiment and at the conclusion.
Results of these measurements are shown in Table 1. A. The subject's sample is added to a known
amount of bacteria, incubated for a time
period, and the turbidity of the medium is
Dosage Initial Initial Final Final measured.
per Day Plasma RBC Plasma RBC The subject's sample is added to a known
(pg) Folate amount of bacteria along with a known amount
Folate Folate Folate
(nglml) (ng/ml)
of folate, incubated for a time period, and the
(ng/ml) (ne/ml)
turbidity of the medium is measured.
200 3.6 444 3.1 381 The subject's sample is added to a known
amount of bacteria, incubated for a time
300 3.5 450 3.3 4t1 period, and the amount of folate produced is
400 3.6 449 6.7
measured by an enzyme acliviry assay.
610
D. The subject's sample is added to a known
Table 1:Folate concentrations in plasma and RBCs before and amount of bacteria, incubated for a time
aiter folate diets.
period, and the number of mutated colonies is
counted.
If plasma foiate drops below 3.0 ng/ml, then a folare
:eiiciency is indicated. Furrher deficiency will deplete
RBC stores and couid lead to anemia.
44. Based on the results in Table l, what conclusion
wouid you make about the leveis of fblate these
women were consuming in their diets before the
study began?

A. They were consuming between 300 and 400


pg/day.
11. In this study, the outcome variables of piasma folate
B. They were consuming more than 400 pglday.
and RBC folate were chosen to reflect the status of C. They were consuming between 200 and 300
two different body pools of folate. Which body pool
pg/day.
does each reflect?
D. They were consuming less than 200 ytg/day.
A. Plasma folate represents foiate intake over the
past year, while RBC folate represents fblate
intake over the past day.
45. When folate is deficient, red blood cell synthesis is
dramatically affected. Folate is required for
B. RBC folate represents folate intake over the
synthesis of the nucleotide thymidylate. Developing
past year, while plasma folate represents folate
cells will enlarge too much before they are mature
intake over the past day.
and are reieased liom the bone marrow. What type
C. Plasma folate represents folate intake over the
of anemia is this called?
past day, while RBC folate represents folate
intake over the past few months.
D. RBC folate represents folate intake over the
A. Microcytic anemia
past day, while plasma folate represents folate
B. Megacytic anemia
intake over the past few months.
C. Millicyric anemia
D. Macrocytic anemia

Copyright @ by The Berkeley Review t2t The Berkeley Review


Specializing in MCAT Preparation
Biology Folate Experiment Passage Vtr

46. What could you infer about the folate needs in


pregnant women?

A. Folate needs are still about 200 1tglday in


pregnancy.
B. Folate needs increase during pregnancy due to
synthesis of many new cells.
C. Folate needs decrease during pregnancy due to
synthesis offoiate by the placenta.
D. Folate needs of the mother are met by liver
stores of folate, since folate is a fat-soluble
vitamin.

47. Which of the following structures indicates the


pyrimidine nitrogenous base thymine?

A. B.
o NH"

.A I

HzN -t r")N ',t'

H
13);
C. D.

NH, o
H. cHr
\,
'"^ N

"L*) .A*)
;
'l

Copyright @ by The Berkeley Review 122 The Berkeley Review


Specializing in MCAT Preparation
Biotogy Measurement of Blood Pressure Passage VItr

Passage VIII (Questions 48-54) 49, If a person has a large amount of fat tissue on their
arms, how will this change the blood pressure
The first measurement of blood pressure, in the early reading?
1700s by Stephen Hales, used a cannula inserted into the
artery of a horse. The pressure of the blood was
determined by the height it achieved in a vertical tube A.. The reading will be falsely high.
attached to the cannula. Although this is a direct B. The reading will be falsely low.
technique, we currently use an indirect technique based on C. The reading will be accurate.
arterial sounds to measure blood pressure. D. One cannot tell from this passage.

A pressure cuff is wrapped around the arm and


inflated, compressing and closing the brachial artery.
Using a stethoscope, the reader listens for the beginning
of blood flow as the pressure is released from the cuff.
The tapping sounds she hears are spurts of blood, pushing 50. What is laminar flow?
through the partially obstructed artery with each heartbeat.
She also listens for the final sound and notes at what A. A fluid moves as a group of particles, each
pressure the first and last sounds occur. As the artery with its own random course.
becomes completeiy opened, no sounds are heard. The B. A fluid moves as a series of individual layers,
lirst sound is recorded as the systolic pressure, and the each of the same velocity.
final sound is recorded as the diastolic pressure. An C. A fluid moves as single particles, each with its
attached meter called a sphygmomanometer is used to own random course.
read the pressures. D. A fluid moves as a series of individual layers,
each of a different velocitv.
In Table 1 are some average arterial pressures
nm/Hg) at various ages for men and women.

Systolic Systolic Diastolic Diastolic 51. Gravity plays a role in reading blood pressure. For
Age Men Women Men Women this reason, blood pressure measurements are
20-24 23
conventionally made with the cuff approximately
16 76 72
level to the heart. For a 46-year-old woman, to what
25-29 25 I'7 '78 74 value would the systolic pressure change if it were
30-34 26 20 '/8 75 measured at the level of the ankle? Assume the
35-39 2'7 1^
80 78 ankle is 100 cm below the heart, and gravity
increases blood pressure 0.8 mm/Hg per cm.
40-44 29 t21 81 80
45-49 30 3l 82 82 A. 162
50-54 35 3l 83 84 B. 2t0
55-59 38 39
c. 211
60-64
84 84
D. 313
r42 144 85 85

Table 1

52. Astronauts on the space shuttle routineiy participate


in research experiments. Where should the blood
+8. As the pressure in the cuff is released, what does the pressure cuff be placed for a reading of blood
first sound, read as the systolic pressure, indicate? pressure on an astronaut in space?

A. The minimum pressure of the blood following A. The cuff must be located in the conventional
a heartbeat. position at the level of the heart.
B. The beginning of laminar flow in the artery. B. The cuff should be located at the ankle, as far
C. The maximum pressure of the blood following as possible from the level of the heart.
a heartbeat. C. The location of the cuff does not matter.
D. The heartbeat itself. D. The cuff wili not work in an anti-gravity
situation.

J:pyright @ by The Berkeiey Review 123 The Berkeley Keview


Specializing in MCAT Preparation
Biology Measurement of Blood Pressure Passage VIII

53. When during the blood pressure measurement is the


blood flow turbulent?

L Before the systolic measurement


il. Between the systolic and diastolic
measurements
IfI. After the diastolic measurement

A. I only
B. II only
C. II and III only
D. I, II, and iII

54. The puise pressure is the difference between the


diastolic pressure and the systolic pressure. It can be
used to calculate the mean arterial pressure (MAP)
during the cardiac cycle.

MAP = diastoiic pressure + (pulse pressure/3)

What is the MAP, on average, for a 27-yeat-old


man?

A. 51
B. 62
c. 18
D. 94

Copyright @ by The BerkeleY Review 124 The BerkeleY Review


Specializing in MCAT PreParation
Biology Aortic Compliance Passage IX

Passage IX (Questions 55-60) 55. Compliance is best represented by which of the


following relationships ?
To obtain curves shown in Figure 1, aortas were
obtained at autopsy from individuals in different age A. AD/D
groups. Successive volumes of liquid were injected into B. AV/AP
this closed elastic system and after each increment of C. AP/AD
volume the internal pressure was measured. The graph D. D/AD
:an be used to extract information about the elfects of ase
on aortic compliance.
56. The elastic modulus is most likely:

A. constant over a lifetime.


B. not related to aortic compliance.
C. directly proportional to aortic compliance.
D. inversely proportional to aortic compiiance.

:
6-.'
200 57. According to Figure 1, which curve most likeiy
C) represents the oldest age group?
tr
i 150
E A. Curve A.
B. Curve B.
; 100
C. Curve D.
d
o
o
D. Curve E.
-)U-^
58. As compliance diminishes, it is most likely that peak
arterial pressure occurs progressively:
0 50 100 150 200
Pressure (mmHg) A. earlier in systole.
B. later in systole.
Figure 1
C. earlier in diastole.
D. later in diastole.
l:eeffects of aging on the elastic modulus (Ep) is
. -- .'" 2. The elastic modulus is defined as Ap/
r, rn Fisure 59. According to Figure 1, aortic compliance in the
"l.-, D . AP is the aortic pulse pressure,
D is the mean youngest individuals is:
-,:-.-: diameter during the cardiac cycle, and AD is the
-:".,-::irm change in aortic diameter during the cardiac smaller over ail pressures when compared to
- :.: The fractional change in diameter (AD/D) of the other age groups.
- ,. :-.rnng the cardiac cycle is a reflection of the change greater over all pressures when compared to
.::n: during left ventricular contraction. Finaily, the other age groups.
"::,-, :s unable to eject its stroke volume into a rigid C. greatest at very high and low pressures and
-:--:--.- s'\'stem as rapidly as into a more compiiant system. least over the usual range of pressure
variations.
D. least at very high and low pressures ancl
:r greatest over the usual range of pressure
variations.

60. According to Figure 1, it can most likely be


concluded that compliance:

A. increases with age, a manifestation of


increased arteriai rigidity.
B. increases with age, a manifestation of
decreased arterial rigidity.
decreases with age, a manifestation of
40 60 80 100 increased arterial rigidity.
Age (years) D. decreases with age, a manifestation of
decreased arterial rigidity.
Figure 2

- bi The Berkeley Review t25 The Berkeley Review


Specializing in MCAT Preparation
Biology fleart Muscle Action Potentials Passage X

Passage X (Questions 61-66) Figure 3 represents the membrane potentials of a cell


from the sinoatrial node, which is located in the right
Contraction of cardiac muscle is triggered by atrium of the heart.
depolarization of the plasma membrane. A typical
ventricuiar action potential from a contractile myocardial
cell is iilustrated in Figure 1. In order to bring about this
action potential, there must be inherent changes in the
permeabilities of ions flowing into and out of the muscle
ce11.
61,. In passing from the right atrium to the right
(€
ventricle, blood wiil have passed through the:
c-
q)
A. mitral valve.
F. a- B. semilunar valve.
e5 -50 C. tricuspid valve.
-o D. pulmonary valve.
2
i00
0 0.15 0.30
Time (seconds)
62. Figure 2 depicts the permeabilities of ions during the
Figure 1 action potential in a myocardial contractile cell.
Graph C most likely depicts the permeability of
Figure 2 illustrates the membrane permeability which of the following ions?
changes which are depicted by the action potential.
A. Na@
10.0
B. 60
o C. Ca2@
h>' D. CIE
o-o
2 ;i t.u
srF
.ai
dLL
CJ
& 63. The capacity for spontaneous and rhythmic self-
0.1
excitation is best manifested by which region of the
action potential shown in Figure 3?
0 0.15 0.30
Time (seconds)
A. Region A.

Figure 2
B. Region B.
C. Region C.

Special cells located in the heart itself are responsible


D. Region D.

for the heart's spontaneous and rhythmic self-contraction.


These specialized cells of the sinoatriai (SA) node have
significant differences in their membrane potentials when
compared to contractile myocardial cells described in 64. A membrane potential plateau region, similar to the
Figure 1.
one shown in Figure 1, is best explained by:

A. inactivation of Nao channels.


qa'
B. a significant increase in the permeability of the
membrane to CaZ@.
C. the flow of positive ions out of the cell
-o equaling the flow of positive ions into the ceil.
o
D. both Nao and Ko channels becoming
Time (seconds) inactivated.

Figure 3

Copyright @ by The BerkeleY Review 126 The BerkeleY Review


Specializing in MCAT PreParation
Biology Ileart Muscle Action Potentials Passage X

65. The action potential initiated in the SA node spreads


throughout the right atrium from contractile cell to
contractile cell. The means by which this spread
takes place is most likely through:

A. tight junctions between cells.


B. neurotransmitter communication between
cells.
C. gap junctions between cells.
D. desmosomes, which hold adjacent cardiac cells
together.

66. The following graph depicts three different action


potentials from a SA nodal cell. Which of the
following statements is most likely true of the
graph?

50

o
aa-
a4 n
6*

C)

50
Time (seconds)

A. Graph B is the result of acetylcholine release


onto the heart muscle.
B. Graph B is the result of vagus nerve
stimulation.
C. Graph C is the result of norepinephrine release
onto ths heart muscle.
D. Graph C is the result of vagus nerve
stimulation.

Copyright @ by The Berkeley Review 127 The Berkeley Review


Specializing in MCAT Preparation
Biotogy Capillary Filtration Passage XI

Passage XI (Questions 67-72) 67. An increase intracapillary hydrostatic pressure


would:
The direction and magnitude of water movement
across a capillary wall are determined by the algebraic A. increase the concentration of osmotically
sum of the hydrostatic pressure and osmotic pressure that active particles within the vessel.
exist across that membrane (Figure 1). B. result from a decrease in the arterial pressure.
C. favor movement of fluid from vessel to
interstitial space.
D. increase the venous resistance.
Filtration

fii6#-"P*""'" 68. Studies using plastic capsuies implanted in the


Reabsorption
subcutaneous tissue indicate a P1 of -I
Io -7 mmHg.
If such values are accurate, which of the following
Lumen of Capillary
could most likely be concluded?

lnterstitial Space A. Pc Pi > Pc.


B. Pc Pi < Pc.
Figure 1 c. P; Pc> Pi.
D. Pc Pi < Pi.

The hydrostatic pressure gradient across the wall of a


capillary, P", depends on the arterial pressure (35 mmHg),
the venous pressure (15 mmHg), and the interstitiai fluid
pressure, P;, which is about 2 mmHg. Precapillary and
post capillary resistances are also important. A reduction 69. A reduction the diameter of a precapillary vessel
in the arterial or venous pressure reduces capillary would:
hydrostatic pressure. The interstitial fluid pressure
opposes capillary filtration, and the difference between A. favor movement of fluid from the vessel to the
the two provides the driving force for filtration. interstitial space.
B. result in a reduction of hydrostatic
Important factors responsible for fluid retention in the
pressure.
capillaries are the osmotic pressure of the plasma proteins, C. be equivalent to an increase venous
nO, and the osmotic pressure of the interstitial proteins, n1.
resistance.
The osmotic pressure exerted by these proteins is also D. significantly increase the concentration of
cailed the colloid osmotic pressure or the oncotic osmotically active particles in the interstitial
pressure. space.
While this oncotic pressure is low compared to the
totaL plasma osmotic pressure, it plays a major role in
fluid reabsorption. This is due to the fact that the
electrolytes that are responsible for the major fraction of
plasma osmotic pressure are equal in concentration on
both sides of the endothelium.
70- Fluid movement is most likely described by which
The protein albumin is dominant in determining of the following equations? Note: k is the filtration
plasma oncotic pressure. While aibumin is relatively constant for the capillary membrane.
impermeable to the capillary membrane, small amounts of
the protein do leak into the interstitial fluid and create a A. Fluid movement = k[(Pc + nil-(Pi+?rp)1.
very small osmotic force. B. Fluid movement = k[(Pc + P1)-(np-n1)1.
C. Fluid movement = k[(Pc Pf+(n1+rrp)1.
D. Fluid movement = k[(ni np)-(Pi+Pc)1.

Copyright @ by The Berkeley Review r28 The Berkeley Review


Specializing in MCAT Preparation
Biology Capitlary Filtration Passage Iil

71. Albumin exerts a greater osmotic force than can be


accounted for soleiy on the basis of the number of
molecules dissolved in plasma. Based on this
information, it can BEST be described that albumin:

A. may be replaced by inert substances with no


effect.
B. dissolves into more than one protein molecule
in plasma.
C. carries electrical charges at blood pH which
attracts various eiectrolytes.
D. is permeable to the capillary endotheiium.

72. Only a small percentage (ZVo) of the plasma flowing


through the vascular system is filtered, and of this,
about 857o is reabsorbed in the capillaries and the
venules. The remaining 157o of fluid:

A. acts to increase intracapillary hydrostatic


pressure.
B. remains in the interstitial fluid.
C. returns to the arterial circulation via the
lymphatic system.
D. returns to the venous circulation via the
lymphatic system.

Copyright by The BerkeleY Review 129 The BerkeleY Review


r @
Specializing in MCAT PreParation
n
Biology Kespiratory Calculations Passage XII

Passage XII (Questions 73-78) 74. A respiration physiologist measures a volume of air
to be 1L at 0 degrees Celcius and 1 ATM. What will
The bronchial circulation is the nutritive supply to all be the volume of this air at 25 degrees Celcius and
of the lung support structures. Such structures include lATM?
airways, connective tissue, and pleura. The pulmonary
circulation is the nutritive supply to the alveolar walls. A. 0.82 L
The pulmonary capillaries form an extensive network B. 0.91 L
within the alveolar walls. The maximum capillary C. 1.0 L
volume is about 200 ml, while the normal capillary D. 1.09 L
volume at rest is about 70 ml. This capillary volume can
be increased by opening compressed or closed capillaries. /5. 02 in systemic arterial blood
The partial pressure of
Such opening is callled recruitment. - l0 mmHg less than that in alveolar
is normally 5
gas. The difference in partial pressure exists
For a given volume, the total gas pressure of all because:
molecular species is the sum of the individual pressures.
This law of partial pressures makes the assumption that A. a fraction of 02 is directiy dissolved into the
gas molecules do not interact with each other. The dry pulmonary blood flow.
room atmospheric pressure is given the value 760 mmHg. B. a fraction of 02 is replaced by the partial
This air contains 2IVo 02,79Vo N2, and }Vo CO2. Blood pressure of water.
that leaves the pulmonary capillaries has come into C. the venous blood from bronchial venules and
equilibrium with all of the alveolar gases. In addition, heart vessels contaminate the pulmonary
small quantities of venous biood from bronchial venules venous outflow.
and vessels from the heart join the pulmonary venous D. the partial pressure of gases dissolved in liquid
outflow. is not equal to the partial pressure of the gas
phase at equilibrium.
When air is inspired, it becomes saturated with water
vapor at 37 degrees Celcius in the nose, throat, and
trachea. The source of the heat and water vapor are the 76. Pulmonary physiologists are often interested in only
pulmonary and bronchial blood flows. The water vapor anhydrous gas volumes and partial pressures. Taking
exerts a mandatory partial pressure, with the pH2O = 47 this into consideration, the partial pressure of
oxygen in dry inspired air at the trachea is:
mmHg. The total quantitiy of water in expired gas over a
24-hour period accounts for nearly one-half of the
A. 102 mmHg.
obligatory daily water loss from the body.
B. 115 mmHg.
C. 150 mmHg.
In the alveoli, some oxygen diffuses directly into the D. 170 mmHg.
pulmonary capillary blood. In fact, 3.0 ml of 02 are
dissolved in iL of blood for a pOz -- 90 mmHg. This
reduces the fraction of 02 in alveolar gas to 0.143. Most
77. Based on the information in the passage, the
anhydrous partial pressure of oxygen in alveolar gas
oxygen binds to the protein hemoglobin, which is able to
is:
bind 1.34 ml O2lg, with the normal hemoglobin
concentration in blood being 150 g/L ofblood. A. 102 mmHg.
B. 115 mmHg.
C. 150 mmHg.
D. 170 mmHg.

73. Besides recruitment, capillary volume can be


78. At an oxygen partial pressure of 90 mmHg,9'77o of
hemogiobin is fully saturated with oxygen. At 90
increased by enlarging open capillaries as their
mmHg, which of the following is the BEST
internal pressure rises. Such distension is most
calcuiation of the TOTAL concentration of oxygen
likely caused by: (ml O2lL) in the systemic arterial blood?
A. increased cardiac output.
A. 145 mlO2/L.
B. increased left atrial pressure.
B. 195 ml O2/L.
C. decreased left ventricular pressure.
D. right heart failure. C. 197 ml O2lI-.
D. 200mlO2/I-.

Copyright @ by The Berkeley Review 130 The BerkeleY Review


Specializing in MCAT Preparation
Biotogy Aspirin Passage XItr

Passage XIII (Questions 79-85)

Aspirin is a non-steroidal anti-inflammatory agent that O


works by inhibiting the initial enzyme in prostaglandin
synthesis pathway, cyclooxygenase (CO). Aspirin //
:lialCells
Endothelial Cells
acetylates a serine residue in the active site of CO, IJO

inactivating it. This makes aspirin a useful drug for >0) /


modifying prostaglandin synthesis. <=
o
*i
The diagram in Figure I shows the pathways of c\

prostaglandin synthesis from the fatty acid, arachidonic


acid. 123
Days Following Aspirin Dose
Leukotrienes
I
I

I
Lipoxygenase Figure 2.

Arachidon ate

l'" looxygenase

Prostaglandins Thromboxanes
79. Which of the following statements BEST describes
Figure 1. what is happening in Figure 2?

A. After aspirin treatment, platelets produce new


Low-dose aspirin treatment is often used to reduce risk CO, but endothelial cells do not.
of heart attack or stroke in people with a history of B. After aspirin treatment, endothelial cells
cardiovascular disease. Aspirin lowers the chance of produce new CO, but platelets do not.
blood clots traveling through the vessels and obstructing C. After aspirin treatment, both platelets and
blood flow through the coronary arteries of the heart or endothelial cells produce new CO.
through the carotid arteries leading to the brain. Aspirin D. After aspirin treatment, neither platelets nor
*'orks by altering the ratio of various prostaglandins to endothelial cells produce new CO.
reduce clot formation.

Specifically, the important alteration is in the ratio of


prostacyclin to thromboxane ,{2. Platelets, which have a
life span of four days, are important in the clotting
process. Platelets secrete thromboxane A2, which
promotes platelet aggregation and vasoconstriction. The 80. How many degrees of unsaturation are present in the
endothelial cells of the blood vessels secrete prostacyclin, following structure, thromboxane 82?
ir hich inhibits platelet aggregation and causes
rasodilation. The balance between the two causes OH

appropriate clotting: a strong clot that does not cover an


ercessive area, and that permits unrestricted blood flow
around the clot.

Aspirin inactivates CO permanently in both platelets OH


and endothelial cells. Endothelial cells rapidly produce
more CO enzymatically, but platelets cannot manufacture A. One degree of unsaturation.
co. B. Two degrees of unsaturation.
C. Three degrees of unsaturation
The model shown in Figure 2 indicates CO levels in D. Four degrees of unsaturation.
platelets and vascular endothelial cells after the ingestion
aspirin:
".f 300 mg of

Copyright @ by The Berkeley Review l5l The Berkeley Review


Speciatizing in MCAT Preparation
Biology Aspirin Passage XIII

81.. When aspirin modifies CO, inactivating it, which 84. Low-dose aspirin therapy is used every other day to
structure indicates the product amino acid residue? prevent heart attacks in some people with heart
disease. Why does this dosing regimen help?

A. B. A. The dosing regimen keeps platelet aggregation


HO HO low.
til lil
_NH,-C-C-O_ B. The dosing regimen keeps platelet aggregation
-NHr-C-C-O* I
-t
CH" CH"
high.
t' t' C. The dosing regimen keeps endothelial cells
o S
I I
proliferation low.
a
/l-\ C D. The dosing regimen keeps endothelial cells
/t\
o cHj o cHl proliferation high.

c. D.
HO HO
I lt Iil
85. Although the mechanisms behind the birth process
-NHr-c-C-o- -NHr-C*C-O-
CH.
I I

CHt are not well understood, it is clear that


t' t- prostaglandins play an important role in initiating
o S
lo la and promoting parturition. What does this indicate
o= p- o" o=P-o" for late pregnancy (months 7-9)?
I I
OO OO
A. Aspirin would probably stimulate an early
labor.
B. Aspirin would have no effect on pregnancy.
C. Aspirin should not be used in late pregnancy.
D. Aspirin would probably cause birth defects.

82. What symptoms would accompany an intake of


aspirin that was too high?

I. Easy bruising.
II. Prolonged bleeding time.
III. Rapid clotting.

A. only
B. and II only
C. and III only
D. II, and trI

83. The fatty acid molecule that is the precursor to


prostaglandins and the substrate for cyclooxygenase
has the common name of arachidonic acid. What is
the IUPAC name?

A. 5, 8, 11, 14 arachidonic acid.


B. 6, 9, 12, 15-arachidonic acid.
c. 6, 9, 12, 1S-eicosatetraenoic acid.
D. 5, 8, 11, 14 eicosatetraenoic acid.

Copyright @ by The Berkeley Review 132 The Berkeley Review


Specializing in MCAT Preparation
Biology Sickle CelI Anemia and IIbF Passage XIV

Passage XIV (Questions 86-92) 87. In the blood cells of an adult heterozygote with the
sickle cell trait, how are the hemoglobin types
Sickle cell anemia is caused by a singie substitution of distributed?
one amino acid on the B chain of the adult hemoglobin
(HbA) molecuie. This change produces the form of A. All cells contain both HbS and HbA.
hemoglobin called HbS. Under conditions of low blood B. Some cells contain exclusively HbS, while
partial pressure of oxygen (pO), HbS comes out of others contain exclusively HbA.
solution and forms a cross-linked crystaiiine structure C. All cells contain HbA only.
inside the red blood cell. This leads to the characteristic D. All cells contain HbS only.
sickle shape of the red blood cells. Complications can
arise because the sickied red blood cells cannot properly
fold up enough to pass through small capillaries of the
circulatory system. Tissues are deprived of some of their
blood supply by blockages caused by sickled cells.

Homozygotes are severely affected by the sickling 88. Treatment with hydroxyurea in homozygotes
cells, but heterozygotes rarely experience sickling under promotes fewer crisis events. HbF is increased to
normal oxygen levels. Heterozygotes may experience about 20Vo of the cell's hemoglobin content by
problems at high altitudes (such as in a depressurized treatment with hydroxyurea. What beneficial role
piane) or under some types of anesthesia. does HbF play inside the cell?

Certain drug treatments for sickle cell anemia focus on A. HbF binds and inactivates HbS inside the cell.
increasing levels of fetal hemoglobin (HbF), which does B. HbF promotes a lower oxygenation state inside
not sickle. The following chart shows types of Hb present the cell.
at various ages: C. HbF promotes a higher oxygenation state
inside the cell.
r00
g chain D. HbF binds and activates HbA inside the cell.

b\
;80
o
o-^
!. ttu
!o
-o
{)
+O
89. Sodium metabisulfate (Na2O5S2) functions as an
antioxidant in many medical preparations. It is
q
transformed to sodium persulfate (Na2OgS2) in its
role as an antioxidant. How could sodium
Birth 3 6 metabisulfate be used to test for sickle cell trait
Gestation Age when combined with hemoglobin from red blood
(months) (months) cells?

Figure I
A. Sodium metabisulfate would promote
reduction of the hemoglobin to a rusty color,
which would identify affected individuals.

86. Sickle cell anemia is usually diagnosed by 3 months


B. Sodium metabisulfate would oxygenate
hemoglobin from red blood cells, and the
of age, but not always at birth. This is most likely sickling pattern would be noted in affected
due to: individuals.
A. a decrease of HbF and an increase of HbA
C. Sodium metabisulfate would promote
oxidation of the hemoglobin to a rusty color,
with age. which would identify affected individuals'
B. persistent maternal hemoglobin until age 3
D. Sodium metabisulfate would deoxygenate
months. hemoglobin from red blood cells, and the
C. a decrease in HbF and an increase in HbS with
sickling pattern would be noted in affected
age. individuals.
D. persistent maternal antibodies to HbS until age
3 months.

Copyright @ by The Berkeley Review tJc The BerkeleY Keview


Specializing in MCAT Preparation
Biology Sickle Cell Anemia and HbF

90. What would happen if an adult had too much HbF,


as is the case in some of the thalassemias, in which
too much of the y chain of hemoglobin is produced?

A. Decreased delivery of oxygen to the tissues,


compared to other adults.
B. Increased delivery of oxygen to the tissues,
compared to other adults.
C. The red blood cells would sickle, bur at high
pO2levels.
D. The red blood cells would sickle, but at low
pO2levels.

91. Which diagnosric technique would distinguish


heterozygotes, homozygotes with sickle cell anemia,
and wild-type homozygotes?

I. Restriction fragment length poiymorphisms


(RFLPs) on red blood cell DNA.
U. Gel electrophoresis of hemoglobin samples
from the red blood cells.
rII. Centrifugation of hemoglobin samples from
the red blood cells.

A. I only
B. II only
C. I and II only
D. I, II, and III

92. Which of the following srarements is TRUE of HbF?

I. HbF does not bind 2,3-BPG.


I HbF has a higher oxygen saturation at a given
pO2 than HbA.
rfl. HbF contains two s and two p chains.

A. I only
B. I and II only
C. II and III only
D. I,II, and trI

Copyright @ by The Berkeley Review t34 The Berkeley Review


Specializing in MCAT Preparation
Biology Ventilation Kegulation Passage XV

Passage XV (Questions 93-100) Figure 2 shows the results of the same experiment with
the variable being arterial pCO2. The reflex involves the
A group of inspiratory neurons located in the medulla peripheral and central chemoreceptors and is closely tied
is responsible for controlling ventilation. Inputs to these to changes in the proton concentration.
neurons from both the peripheral and central
chemoreceptors are important in regulating involuntary An increased arterial pCO2 wili cause an increase in
control of ventiiation. The peripheral chemoreceptors the arterial concentration of protons. The peripheral
consist of two different bodies of receptors located in the chemoreceptors are stimulated by this iowered pH. At the
carotid artery of the neck and in the arch of the aorta. The same time, the elevated arterial pCO2 causes an increase
nerve terminals are intimate with the arterial blood and in the pCO2 of extracellular brain fluid. This causes a rise
respond to changes in pCO2 , pO2, and Ho. Impulses in the proton concentration of the fluid, stimulating the
from these nerve terminals travel up afferent fibers toward central chemoreceptors. It is the role of the central
the brainstem and eventually synapse with inspiratory chemoreceptor which is most important in mediating this
neurons of the breathing center. The central ventilation reflex.
chemoreceptors, located in the medulia, monitor the
brain's extracellular fluid for changes in the concentration
of protons. Fibers from this receptor synapse with
inspiratory neurons.

^30 93. To assess the effects of a changing arterial pO2 on


J the ventilation rate accurately, which of the
q?O foliowing conditions should exist?
d
A. The arterial pCO2 should decrease at a
O constant rate.
>10
o B. The arterial pCO2 should increase at a constant
rate.
2,
(l C. The arterial pCO2 should be maintained at 40
0 20 40 60 80 100 mmHg.
Arterial pO2 (mmHg) D. The arterial pCO2 should be maintained at 46
mmHg.
Figure I

Figure 1 shows the relationship between arterial pO2


.:J the ventilation rate. The graph indicates that a 94. According to Figure 1, a drop of 30 mmHg from the
:'.', ering of the arterial pO2 will stimulate the rate of normal resting value of pO2 does NOT radically
:nlilation. This reflex is mediated by the peripheral increase the ventilation rate. This occurs most likely
-:3;noreceptors which respond to a lower arteriai pOZ by because the:
:.easing their rate ofdischarge to the breathing center.
A. pCO2 is increasing.
16 B. total amount of oxygen transported is
relatively unaffected.
C. body responds with a decreased use of oxygen.
<1' D. pCO2 is decreasing.
J

=6
o 95. The inspiration of carbon monoxide gas will result
O in:

A. an unchanged arterial pO2.


B. a lowered arterial pO2.
40 44 48 C. an increased arterial pO2.
Arterial pCO2 (mmHB)
D. an increased percentage of hemoglobin
saturated with 02.
Figure 2

Copyright @ by The Berkeley Review t35 The BerkeleY Review


Speciatizing in MCAT Preparation
Biotogy Ventilation Regulation Passage XV

96. According to Figure 1 and Figure 2, changes in 100. During times of sleep, the body's ventiiation
arteriai pCO2 are: rate
decreases more than the consumption of oxygen by
ceils. The result ofthis situation is:
A. augmented by reflexes regulating ventilation to
a greater degree than are equivalent changes in A. an increased arterial pCO2 and a increased
arterial pO2.
arterial pO2.
B. augmented by reflexes regulating ventilation to
B. an increased pCO2 and a decreased
a lesser degree than are equivalent changes in
arterial pO2.
arterial pO2.
C. resisted by reflexes regulating ventilation to a
C. a decreased arterial pCO2 and an increased
greater degree than are equivalent changes in arterial pO2.
arterial pO2. D. a decreased arterial pCO2 and a decreased
D. resisted by reflexes regulating ventilation to a arterial pO2.
lesser degree than are equivalent changes in
arterial pO2.

97. Which of the following statements provides the


BEST support for the type of ventilation found early
on in metabolic acidosis?

A. Hypoventilation is caused by increased neural


_ gutput from the peripheral chemoreceptors.
B. Hypoventiiation is caused by increased neural
output from the central chemoreceptors.
C. Hyperventilation is caused by increased neurai
gutput from the peripheral chemoreceptors.
D. Hyperventilation is caused by increasid neural
output from the central chemoreceptors.

98. Which of the staremenrs below is FALSE regarding


pCO2 during exercise?

A. The alveoiar pCO2 determines arterial pCOZ.


B. Alveolarvenrilationincreases.
C. Venous pCO2 increases.
D. Arterial pCO2 increases.

99. In order to hold their breath for a longer duration,


swimmers often hyperventilate immediately before
competing in a swimming event. This activity can be
dangerous because even though the:

A. low pCO2 is permitting one to hold their


breath, the exercise may lower the pO2 to
levels which may induce unconsciousness.
B. Iow pCO2 is permitting one ro hold their
breath, the exercise may raise the pO2 to levels
which may induce unconsciousness.
C. high pCO2 is permitting one ro hold their
breath, the exercise may lower the pO2 to
levels which may induce unconsciousness.
D. high pCO2 is permitting one ro hold their
breath, the exercise may raise the pO2 to levels
which may induce unconsciousness.

Copyright @ by The Berkeiey Review 136 The Berkeley Keview


Specializing in MCAT preparation
Biology tleart & Lungs Section II Answers

1. A is correct, low average velocity because of a high total cross-sectional area. This is according to the continuity
equation describing fluid flow. This equation states that since blood flow must be equal throughout an entire closei
system, the cross-sectional area times the average velocity equals the cross-sectional area times the velocity. The
total cross-sectional area of aii the capillaries is very large, the largest of all types of yessels seen in the
cardiovascular system. Therefore, the blood flow through the capiliaries will have a low average velocity because of
their high total cross-sectional area. The correct choice is A.
.,
D is correct, 16. The problem requires that we be farniliar with the relationship between resistance of a vessel and
the radius. The resistance of the vessel varies inversely with the radius to the fourth power. Therefore, decreasing
theradiusbyafactorof2leadstoanincreaseintheresistancebyafactorof 16. TheiorrectchoiceisD.
3. C is correct, Q x TPR = (MAP - CVP). We are looking for an expression to represent blood flow. The general
equation is given in the passage. Biood flow (Q) = Perfusion Pressure/Flow R.eslstance. We are asked about the
entire vascuiar unit. The perfusion pressure is therefore going to be the mean arterial pressure minus the central
venous pressure. The resistance for the entire unit is given by the total peripheral resistanie. Placing these values in
the formula and multiplying both sides by TPR will give the solution. The correct choice is C.

4- D is correct, red blood cells. One can arrive at this answer by thinking about the composition of biood. Blood is
made up of plasma and cells. The question is asking about the increase in viscosity ieen in whole blood versus
plasma. Knowing the composition of biood ieads one to believe that increase must come from the cellular
contribution' It then becomes a matter of WBCs or RBCs. At this point, one must be aware that there are millions
of red blood cells in whole blood. In other words, there are many more red cell than white cells. Therefore, the
major contribution must come from the presence of red blood cells. The correct choice is D.
5. C is correct, veins contain valves that favor one-way biood flow. We are required to draw on knowledge about the
structure of blood vessels. Both arteries and veins have a layer of endothelial ceils, a layer of elastic tissue, ancl a
layer of smooth muscle. Therefore, we can eliminate answer choices A, B. and D. However, only veins contain
one-way valves which prevent back flow of blood. One must understand that the pressure in the vlins is not very
large, and so there is not a tremendous amount of force to propei blood. When ihe blood is propelled (skeletil
muscle contraction is one method of providing the force), we wish the b1oo6 to remain *ou1ng forward. The
presence of these one-way valves carry out this function. The correct choice is C.

6. B is correct, venous pressure does not change significantly over the heart cycle. The MAP is an average figure
because the pressure in the arterial system changes significantly over the heart cycle. It is very high right out oltne
aorta, but becomes smaller and smaller toward the capiiiaries. Therefore, we need to take un aui.ug" value. The
pressure in the veins does not change very dramatically over the entire heart cycle. The total pressure difference in
the peripheral veins is usuaily 5 to 10 mmHg. Therefore, we do not need to take an average value. The fact that the
arterial pressure does change dramatically is true, but it is not proper rationale for trying to explain why we do not
need to take an average for the veins. Furthermore the cross-sectional area and the pressure responses to increases in
volume offer us no explanation as to why no average is taken. The average is not taken because the value remains
relatively constant. The correct choice is B.
7. D is correct, arterial systolic pressure. We are told in the question that we need to use the pressure in the cuff to
collapse the artery totally and prevent blood flow. Therefore, the pressure in the cuff must counter the maximum
pressure offered by the artery. The maximum pressure in the artery will be represented by the arterial systolic
pressure. Recall that systoiic pressure is the maximum arterial blood pressure during cardiac cycle. To coliapse the
artery, the pressure in the cuff must exceed this value. The correct choice is D.

D is correct, glycoiysis. It is important to know which cells have organelles and which do not. It will also be
important to know certain metabolic processes and where they take place. E,ven though we have not formally
discussed metaboiism yet, this question is designed to get you to think about cellular function. Mature red blood
celis are enucleated eukaryotic cells. They do not have any organelles and they have only one membrane, the
plasma membrane. As we will see in future discussions, the Krebs cycle, electron transport, and oxidative
phosphorylation are all associated (in eukaryotic celis) with mitochondria. Even though these three systems are
responsible for the majority of energy generated in metabolism, they are not found in red blood cells. However, red
blood cells do require energy and they obtain that energy from glycolysis. The correct choice is D.

Copyright O by The Berkeley Review L37 The Berkeley Review


Specializing in MCAT Preparation
Biology Ileart & Lungs Section II Answers

9. C is correet, reductase. The vitamin K dependent reaction, as written from glutamate to y-carboxyglutamate, is a
carboxylation and would require a carboxylase enzyme. If we wanted to go from y-carboxyglutamate to glutamate,
it would be a decarboxylation reaction, involving a decarboxyiase enzyme. Proteases are proteolytic enzymes that
catalyze the hydrolysis ofpeptide bonds.

Vitamin K Vitamin K
Hydroquinone Quinone
HO ,ll HO
I tl Iil
_N-C*C-N_ _C_C-N-
ttt 'l tt
HCH2H /H cH, H
I I
cH" coz CH
l'6 /: O
coo coo

Glutamate y-Carboxyglutamate

In this reaction we are not degrading a protein. We are regenerating vitamin K hydroquinone, which is the reduced
form of vitamin K. In order to go from the oxidized form (vitamin K quinone) to the reduced form, a reduction
needs to occur. This reaction will be catalyzed by a reductase. The correct choice is C.

10. B is correct, thromboplastin (Factor III or Tissue factor). This question can be answered by looking directly at
Table 1 and Figure 1 in the passage. In Table I the symbol " f " denotes dependency on vitamin K. All that is
needed is a correlation between the common names and the factor numbers. The correct choice is B.

11. A is correct, consumption of leafy green vegetables. The major source of vitamin K is from one's diet, especially in
leafy green vegetables. There are also intestinal bacteria that can synthesize vitamin K. Since vitamin K is b"ing
made availabie to be a cofactor in the conversion of glutamate to y-carboxyglutamate, we would expect an increase
in clot growth.

Since blood clots can break free from blood vessels and wreak havoc within the circulatory system (by causing heart
attacks and strokes), their growth must be limited. Clot growth can be limited by inhibiting the serine proteases
which act at various steps in the cascade. Since blood is dynamic (constantly moving), the areas near i clot are
always being diluted with fresh blood. This helps to prevent a build up of cascade intermediates involved in the
clotting process. The intermediates that are removed by the flow of blood are transported to the liver where they are
removed and degraded. The correct choice is A.

12. C is correct, 0.50 (or 50Vo). Men have an X and a Y chromosome; women have two X chromosomes. We are toid
in the question that the defect for both types of hemophilia (h) resides on the X chromosome and that it is recessive,
The woman's father has hemophilia, which we can designated as XhY. Since her mother is normal (XX) the woman
must be a carrier (XXh) because she received one good X chromosome from her mother and one defective Xh
chromosome from her father. This is shown in the Punnett square in Figure I below:

X X ah X

yh XXh XXh X XXh XX

Y XY XY Y xhv XY

Figure I Figure 2

The woman (XXh) now marries a man who is normal (XY). The mode of inheritance of their offspring is indicated
in Figure 2 above. There are two sons and two daughters. However, we care about only the sons. One will be
normal, while the other will express the trait. Therefore, the probability that this woman will have an affected son is
0.50 or SOVo. Even though we have not formally discussed simple Mendelian genetics yet, this question is designed
to get you to think along those lines. The correct choice is C.

Copyright @ by The Berkeley Review 138 The Berkeley Review


Specializing in MCAT Preparation
Biology Ileart Er Lungs Section II Answers

13. A is correct, citrate. Calcium is an important cofactor in the cascade mechanism outlined in Figure 1 in the passage.
This ion bears a +2 charge and can be chelated by certain compounds that bear negative charges. As stadd in the
passage the modified amino acid y-carboxyglutamate is an excellent chelator of Ca2@. When this amino acid is tied
up in a protein, it presents two carboxyl groups to the calcium ion. Based on this we could assume that the more
carboxyi groups on a molecule, the better the chelator. That is, the better the molecule is at sequestering calcium.

Ifwe follow this assumption, we can eliminate choice B first. Glutamate has two carboxylate groups. However,
note the positively-charged amino group. This would tend to repel a positively-charged caliium ion. Not only that,
but the positiveiy-charged amino group will be tied up with the cr-carboxyl group through electrostatic interactions.
This will diminish the chelating abiiity of that molecule. We can eliminate choice D based on similar reasoning.
The major difference between the last two choices is that choice C (succinate) has just two full carboxyiate g.ouft
whereas choice A (citrate) has three full carboxylate groups, In this case, the more carboxyl groups, ttie Uetter ttre
chelator, and the better the molecule is at preventing ciotting in stored blood. The correct choiie is A.

14. B is correct, proconvertin (Factor VII). This question is designed to get you to think about a number of items at the
same time. As stated in the passage, warfarin is an antagonist (i.e., a competitive inhibitor) of vitamin K. Now, you
need to ask yourself, "Which coagulation factors in the cascade are affected by inhibition of the use of vitamin k?"
The factors which are affected (from Table 1) are those which are dependent on vitamin K. Those factors turn out to
be choices A'D in the.answers (i.e., prothrombin, factor II; proconvertin, factor VII; Christmas factor, factor IX;
Stuart-Prower factor, factor X). Since there is just one answer (and not four), we need to consider some more
information.

In the question it states that 3 days after oral administration, the PT (prothrombin time) is maintained at 1.2-1.5
times the controi. The PT screens the extrinsic and common pathways. Note that in the cascade shown in Figure 1,
the extrinsic pathway includes factor VII and the common pathway includes factor X and factor II. The exlrinsic
pathway does not screen factor IX as it is part of the intrinsic pathway. This allows us to eliminate choice C
(Christmas factor, factor IX) as a possible solution.

How do we distinguish between the remaining choices? Another way to look at this is to ask yourself, "Which
single factor decreases so much after 3 days as to maintain a PT between 1.2-1.5 times the control?" you should
now be think about the haif-life ofthese factors (see Table I in the passage).

The one factor which has the shortest hatf-life is factor VII (between 2-6 hours). Let's assume that its half-life is
about 4 hours' In 3 days (72 hours) factor VII will have been reduced by 18 half-lives. The half-life offactor X is
between 25-60 hours. Let's assume that its half-tife is about 43 hours. Factor X wiil have been reduced by about 2
half-lives. Finally, factor II has a half-life between 50-120 hours. Let's assume that its halflife is about 85 hours.
Factor II will have been reduced by about t half-tife. [A11 values have been rounded up for simplicity.] Based on
this analysis, we see that factor VII will have been decreased the most, because it has iuch a shbrt fiie span. The
correct choice is B.

B is correct, arterioles. The answer to this question is not found in the passage but rather comes from our
knowiedge of the flow of blood through the circuiatory system.

Roughiy 64Vo of biood in the circulatory system is contained in the veins while about l5Vo ts contained in the
arteries. The veins include the large veins, small veins, and venules. The small veins and venules together contain
abott 25Vo of the total blood volume, leaving about 39Vo of the total blood volume found in the large veins. The
arteries include the large arteries, small arteries, and arterioles. The large arteries and small arteries together contain
about l3Vo of the total blood volume, leaving about 2Vo of the total blood volume found in Ihe arterioles. About 5Vo
of the blood is contained in the capillaries. The remaining 76Vo of the blood is contained in the heat and pulmonary
vessels (associated with the lungs). This allows us to pick the arterioles as having the LEAST blood volume (at any
given time) in the circulatory system. The correct choice is B.

16. C is correct, lower due to the accumulation of CO2. Recall that as blood passes through the tissues it gives up 02
as a nutrient and picks up CO2 as a waste product. The blood that is leaving the pulmonary artery is coming from
the right ventricle, which in turn came from the right atrium, which in turn came from the venous system.
o
CO2+ H2O: H2CO3

- ::r nght @ by The Berkeley Review 139 The Berkeley Review


Specializing in MCAT Preparation
Biology Ileart & Lungs Section II Answers

Blood in the venous system is, for all practical purposes, deoxygenated blood. The oxygen had been given up at the
level of the respiring tissues. The CO2 that was picked up by the venous system is either dissolved in the blood as
free CO2 (about 57o), bound to hemoglobin as the carbamate (about 5Vo), or in the form of the bicarbonate ion
(about 90Vo). When the bicarbonate ion is formed (Bohr reaction), hydrogen ions are produced which tend to lower
the pH of the blood. The correct choice is C.

t7. A is correct, (see the graph below). When there is a constriction of a blood vessel there is more resistance to blood
flow. This ieads to a greater pressure drop (i.e., a greater change in pressure, AP) between the veins and arteries.
The result is a lower venous pressure. Therefore, the curve would be expected to drop along the y-axis.

The curve intersects at the same point on the x-axis because if the heart were to stop and the cardiac output reduced
tozero,thepressuresinthevenousandarterialsystemswouldsoonequilibrate. Inotherwords,thevenouspressure
would not change. This makes choice A a good answer.

l0
8
-1
6
g 4

o 2
o
0
-4 0 4 8 1216
Venous pressure (mmHg)

Choice B indicates a decrease in the blood volume. When the blood volume decreases there is less blood to fill the
ventricles. This leads to less of a contractile force and a lower blood pressure. Choice C indicates decreased
arteriole resistance while choice D indicates and increase in blood volume. The correct choice is A.

18. D is correct, vasopressin and aldosterone. When the blood volume is low, neural impulses form the atria and
arterial baroreceptors are sent to the hypothalamus and vasopressin (antidiuretic hormone, ADH) is released into the
blood. Also, if the blood volume is low, chances are that the blood osmolarity has increased. An increased
osmolarity is also detected at the level of the hypothalamus and this allows for the release of ADH as well. ADH
acts at the levei of the late distal tubules and the collecting ducts in the kidney and directly promotes water
reabsorption (back into the blood). This increases the blood volume.

A low blood volume is usually associated with a low blood pressure. This is detected by the juxtaglomerular
apparatus in the kidney and granular cells in that complex release renin. This enzyme converts angiotensinogen to
angiotensin I, and then angiotensin I is converted to angiotensin II by the angiotensin-converting enzyme. Not only
does angiotensin II cause vasoconstriction of the arterioles (to help increase blood pressure), but it also stimulates
the release aldosterone from the cortex of the adrenal glands. Aldosterone stimulates the reabsorption of sodium at
the level of the kidney. Water follows down its concentration gradient into the blood and leads to an increase in
blood volume. The correct choice is D.

19. C is correct, (see the graph below). The sympathetic nervous system is a component of the autonomic nervous
system. Recall that the sympathetic nervous system is involved in the fight-or-flight response. Activation of the
sympathetic nervous system will stimulate the heart and cause it to be a stronger pump. In other words, the activity
of the heart increases. This will cause the cardiac output (C.O.) curve to shift to a higher level. We see this response
in choices A and C.

Z\
310 l_l c.o. i0
Ll c.o.
58 ,t 8

B6
o a
6

O+
=t o 4
Idr o
E 2

(50 O 0
-4 0 4 8 1216 -4 0 4 8 1216
Venous pressure (mmHg) Venous pressure (mmHg)

Choice A Choice C

Copyright @ by The Berkeley Review 740 The Berkeley Review


Specializing in MCAT Preparation
Biology fleart & Lungs Section II Answers

Activation of the sympathetic nervous system will also cause vasoconstriction of arteries, arterioles, and veins.
Contraction of venous smooth muscle will lead to a decrease in the diameter and increase in the pressure within the
vessel. This will allow more blood to be returned to the right heart (i.e., the right atrium and right ventricle) from
the veins. The result is an increase in venous return and an increase in the mean circulatory pressure.

Since cardiac output must equal venous return (see the first paragraph in the passage), we can equate the y-axis of
our curve in the question not only to cardiac output but also to venous return. Therefore, we would expect the curve
to move upwards along the y-axis for venous return (V.R.) and outwards along the x-axis for an increase in mean
circulatory pressure (see above). The correct choice is C.

[Note: In the third paragraph of the passage it states that if the cardiac output and the venous return is increased, the
pressure in the arterial system will increase while the pressure in the venous system will decrease. Be careful of
what it means when it says "the pressure in the venous system will decrease." If we are at a venous return of 0
L/min, the venous pressure is about 8 mmHg (see Figure 1 in the passage, or below). If we now move to a venous
return of 5 L/min, the venous pressure is about -4 mmHg. It is a decrease form the mean circulatory pressure
(MCP).
l0
8
J
E 6

,i) 4

o 2
o
0

*""3, 0.1,,,1,ii'il

In question 5 we have increased the MCP by constricting vessels. An increase in cardiac output due to sympathetic
stimulation will decrease the venous pressure relative to the new MCP.I

20. A is correct, norepinephrine, because it opens Nao and Ca2o channels in myocardial cells. This question ties in a
concept we examined when we discussed the nervous system. Recall that the preganglionic and postganglionic
nerve fibers of the parasympathetic nervous system release the neurotransmitter acetylcholine (ACh). The
preganglionic fibers of the sympathetic division release ACh while the postganglionic fibers release the
neurotransmitter norepinephrine. We can now eliminate choices B and D.

Next, we need to recall that sympathetic fibers will stimulate the heart (think of the fight-or flight response). if the
heart is to be stimulated, we would expect to see rapid depolarization of the myocardial cells. Depolarization of a
cell's membrane is due to an influx of Na@ into the cell. This allows a negative resting membrane potential to
become more positive. An influx of both Nao and Ca2e into a cell will allow the resting membrane potential to
become depotarized. Choice A looks like a good answer at the moment. What would happen if the Ko channels
were to open up? Potassium would leave the cell, making the resting membrane potential more negative. The cell
would have a higher threshold potential and would not be depolarized as readily. We can eliminate choice C. The
correct choice is A.
t1 D is correct, decreased arterial pressure. The key word here is compensatory. How does the body handle a
hemorrhage? The first thing that must be done is to prevent more loss of blood. Constriction of blood vessels would
help, whilh means that anlncreased sympathetic discharge to the arterioles (choice A) is true. We still want the
heart to pump blood. Therefore, we do not want to inhibit its action. We can remove inhibitory signals to the heart
by decreasing the amount of parasympathetic discharge at the heart (choice B). After a blood loss the body still
nleds to maintain a reasonable cardiaCoutput. Therefore, we would expect to see an increase in the cardiac output
(choice C). What we would not expect to see is a decreased arterial pressure (choice D). Why? A decreased
urt".iul pressure is the RESULT of a hemorrhage. We want the COMPENSATION, which would be to increase the
arterial pressure. We can reason this out as outlined below.

When a hemorrhage takes place there is a blood loss and a decrease in the blood volume. The venous pressure,
venous return, and-atrial prirrur" will all decrease accordingly. The volume of blood ejected by a ventricle in the
heart (i.e., the stroke volume) during one heartbeat is decreased as well. This leads to a decreased cardiac output and
a decreased arterial blood pressure.

141 The BerkeleY Review


Specializing in MCAT PreParation
Biology Heart & Lungs Section II Answers

Located in the upper neck region are small arteries which branch off of the common carotid
arteries. The common
carotid arteries stem from the aortic arch of the aorta, the main vessel that leaves the left ventricie
of the heart. At
the fork where the small arteries leave the carotids is a region called the carotid sinus.
The caroticl sinus contains
nerve endings which are sensitive to the size of those smail arteries. The carotid
sinus acts as a baroreceptor,
sensing the pressure differences within the small arteries. There are also baroreceptors in
the aortic arch.
lf there is a decrease in arterial blood pressure, baroreceptors will sense this and compensate accordingly. As the
blood pressure drops there is less expansion of^thethe arterial walls and a decreased discharge of'th" n".u" endings at
the baroreceptors. The result is an increased.sympathetic discharge ro the arterioles"lvlins
and heart) ancl a
decreased parasympathetic discharge to the heart (becaur" *" do not want to
inhibit its lunction). The
parasympathetic and sympathetic divisions will act to increase heart rate and therefore increase
cardiac output.
Sympathetic discharge to the arterioles and veins will constrict those vessels. Constriction
of the arrerioles will lead
to an increase in the total peripheral resistance of that system, thereby increasing arteriai p.Lrrr.".
This is the end
resuit of the compensatory mechanism. Constriction of the veins will lead to a"n increasi
in ttre venous pressure,
venous return' and cardiac output. An increased cardiac output leads to an increase
in the arterial p.".rrr". e.gain,
this is the end result of the compensatory mechanism. The correct choice is f).

22. C is correct, arterioles. Arterioles have strong muscular walls. When stimulated by the
sympathetic nervous
system, they constrict. Metabolic products at the level of the arterioles, such as CO2,
He, and 02, regulate the
degree of constriction. The correct choice is C.

23. C is correct, III. It is important to pay attention to the labeis on the x-axis and y-axis. If we look
at posilion I on
the graph we note that the total blood volume is quite low, even though the presiure
is qulte high. Where is blood
experiencing the highest pressure? As it is being ejected from the left ientricie. The pressure
is going to decrease as
the blood reaches the arteries and arterioles. once the blood reaches the capillaries
and the venules, the pressure of
the blood will be approaching an even lower value as inrticated by position II.
As the blood begins to enter rhe
veins, the pressure continues to decrease. As blood enters the right atiium, it is coming
frornitre vena cava. This is
represented by position III. Note that once the blood leaves the vena cava and enterithe
right atrium, it will then
flow into the right ventricle. Contraction of the right ventricle increases the blood pressure"and
thus increases the
velocity of the blood leaving the right ventricle as it flows into the pulmonary arteiy. Biood from
the pulmonary
artery will enter the lungs where the pressure drops to a rather low value. The iorreci
choice is C.
24. D is correct, IV. Consider the x-axis (total blood volume) and the y-axis (area). The total area of the arteries will
be small (point I), and it will calry a small amount of blood volume. The total
area of the venules and veins will be
the largest (point II), and it will carry the largest blood volume. As we move from the veins
ro the vena cava and to
the right atrium, right ventricle, and pulmonary artery, the total blooci volume wilt again
u" rn;nrrrrut while the area
wiil also be minimal (point III). Ilowever, as the blood leaves the pulmonary artery and enters the capillaries of the
lungs, the total area of the vessels begins to increase as does the total bloodvolume
lpoint iv1. Not" that the total
blood volume in the lungs will be less than the total blood volume in the veins simpiy because (a)
there are 11ore
veins in the systemic circulation than in the pulmonary circuiation, and (b) the veins are quite distensible,
The
correct choice is D.

1< C is correct, III. If we look at position I on the graph, we note that the total blood volume is quite low even though
the velocity is quite high. Where is blood experiencing the highest velocity? As it is being-ejected from the left
ventricle. The velocity is going to decrease as the blood reaches the arteries and arterioles. dn"" the blood reaches
the capillaries and the venules the velocity of the biood will be at its minimum vaiue as indicated by position II.
Blood then begins to enter the veins and the velocity begins to increase again due to muscular contractio; squeezing
the veins to help return the blood to the right atrium. As blood enters the right atrium it is coming from ih" uenl
cava' This is represented by position III. Note that once the blood leaves the vena cava and enters ihe right atrium,
it will then flow into the right ventricle. Contraction of the right ventricle increases the blood pr"rru.:" and thus
increases the velocity of the blood leaving the right ventricle as it flows into the pulmonary artery. Thus position IV
will be the velocity of the blood as it leaves the pulmonary artery and enrers the capillary system of the iungs. The
correct choice is C.
26" A is correct, viscosity of blood. Laminar blood flow is blood which flows at a steady rate through a blood vessel,
Blood flow is said to be streamlined. Turbuient flow involves blood which is not flowing in a steaiy stream through

Copyright @ by The Berkeley Review 142 The Berkeley Review


Specializing in MCAT Preparation
Biology Ileart Er Lungs Section II Answers

the blood vessel. Instead, turbulent flow creates eddy currents. These currents result from the blood passing
over a
rough surface in the vessel or.encountering obstructions in the vessel such as a bifurcation in the path of blo'od
flow
(i.e., a splitting of the vessel itself). Turbulent flow, measured by the Reynold's number (Rs), tends
to increase as
the velocity of the blood increases and as the diameter of the vessel increases. Turbulent flow is inversely
proportional to the viscosity of blood. By using the equation R" = (v)(d)/(n/p), where v velocity, d diameter
= = of
vessel, n = viscosity, and p = density, we find that as the Reynold's number increases, the turbulence
of blood flow
can increase. Usually turbulence will occur when the Reynold's number increases above 2000. The
correct choice
is A-
)1 C is correct, systemic arteries. Arterial walls are stronger than the walls of veins. Blood vessel diameter increases
as the internal pressure within the blood vessel increases. This is because the vessels are distensible. Because
arterial walls are much stronger than the walls of veins, the arteriai walls are about 6 to l0 times less distensible than
the walls of veins. What this is saying is that for an increase in pressure there will be 6 to 10 times
more blood in a
vein as there will be in a comparabie artery. Even though pulmonary arteries still have stronger walls
than those of
the pulmonary veins, pulmonary arteries are under less pressure (about 1/6 less) than systemi
arteries. It turns out
that pulmonary arteries have distensibilities about 1/2 that of veins. Even though the pulmonary
arteries are still
distensible, they are not as distensible as the pulmonary-veins or systemic veins. The vascular distensibility
would
be least for the systemic arteries. The correct choice is C.

28. C is correct, The capillaries contain between 57o to l\Vo of the total blood volume. Capillaries are
about I mm in
length and, on the average, most celis are about 0.01 cm away from any given capillary. This allows
for a highly
efficient form of diffusion of 02 from a red blood cell within a capillary io th. su.rounding cells.

r00

480 The value between the ttvo


E dashed lines represents the
E60 total blood volurne within
4)

=_ 40
a)
&20

0 20 40 60 80 100
Total Blood Volume (%)

In order for this diffusion to be optimal, capillaries must have not only a very small diameter (about g pm, which is
just large enough for the passage of a red blood cell), but they must aiso have a very thin wall (about
0.5 pm). The
thickness of the capillary wall is just one endothelial cell layer thick. Even thoughihese values were not presented
in the passage, it shouid be realized that gas exchange (for alt practical purposes) occurs ar the level of the capillary.
It would make sense for them to have the thinnest walls and the smallest diameter, to a1low for maximal and
efficient diffusion. We can eliminate choices A and B.

Capillaries in the systemic system contain about 5Vo of the total blood voiume, while capillaries in the pulmonary
system contain about 4Vo of the entire blood volume. This adds up to between 5% aid l07o of the iotal btoob
volume' Even though this value is not mentioned in the text of the passage, it can be obtained from the first graph in
Figure 1 of the passage. In Figure 1 note that the dashed lines border the value of 20Vo. At first you tiight U"
inclined to pick choice D, which states that total blood volume in the capiliaries is about 20Vo. But think aboui what
the x-axis is telling us. If we move out to l00vo total blood volume, does that mean that at that point in the curve a
vessel contains l007o of the total blood volume? No, it does not.

What the area between the borders of the dashed lines is telling us is that the total blood volume in the capiliaries is
represented by what we see between those two dashed iines. In other words, the total blood volume in the bapillaries
is less than lovo of the total blood volume in the system. The correct choice is c.

29- B is correct, pieural pressure will become more negative. The two lungs in the chest cavity are individually
surrounded by a thin layer of cells called pleura. The visceral pleura attaches to the lung while the parietal pleura

Copyright @ by The Berkeley Review r43 The Berkeley Review


Specializing in MCAT Preparation
Biology Ileart 6r Lungs Section tr Answers

attaches to the diaphragm and interior region of the thoracic cage. Belween the two pleural surfaces is a vely thin
layer of fluid called the intrapleural fluid. The volume of the intrapleural fluid is constant. Pressures in the
respiratory system are listed relative to atmospheric pressure, which is 760 mmHg at sea level. Between inspiration
and expiration the alveolar pressure is the same as atmospheric pressure. Since pressures are measured relative to
atmospheric pressure, the alveolar pressure is said to be 0 mmHg. As stated in the passage the intrapleural pressure
is negative relative to atmospheric pressure (about -4 mmHg).

As an individual inspires the thoracic cavity begins to enlarge. The rib cage moves upward and outward. This is
due to contraction of the external intercostal muscles. The diaphragm descends towards the abdomen. Both actions
move the wall of the thoracic cage away from the srirface of the lungs. However. the parietal pleura is in contact
with the intrapleural fluid which is in turn in contact with the visceral pleura. This intrapleural space increases in
volume by a very smal1 amounl. As the volume of the intrapleural space increases (ever so siightly), the pressure
within that space must decrease (Boyle's Law). In other words, the pressure within the intrapleural cavity will
become more negative (relative to atmospheric pressure).

The same reasoning applies to the iungs. As the thoracic cage moves away from the wal1s of the lungs, the volume
of the iungs increases. As the volume of the lungs increases, the pressure within the lungs must decrease. In other
words, the alveolar pressure becomes subatmospheric and air flows from the outside into the lungs. The correct
choice is B.

30. B is correct, parietal pieura. The parietal pleura and/or the visceral pleura are the most important layers of the
thoracic wali for maintaining the pressure component of the thoracic cage. A puncture of the intercostai muscles,
ribs, or heari is not going to lead to a collapsed lung because these structures are outside the pleura, The correct
choice is B.

31. B is correct, lI only (diaphragm). The diaphragm is the primary muscle involved in normal respiration. The
intercostal and abdominal muscies are excessory muscles and aid in respiration during times of forced expiration and
inhalation. The correct choice is B.
32- A is correct, high pCO2 in the capillaries; low 02 in the capillaries. As blood returns to the capillaries of the lungs
liom the systemic circulation via the pulmonary arteries, it will have a low pO2 (deoxygenated blood) and a high
pCO2. The partial pressures of these gases in the aiveoii of the lungs will be reversed. This sets up a concentration
gradient that will allow oxygen to flow liom the alveoli to the capillaries and carbon dioxide to flow from the
capillaries to the alveoli. The correct choice is A.

33. A is coruect, a rise in pH requires a iower partial pressure of 02 to bind a given amount of 02, The graph shows
that as the pH rises, the curve shifts to the ieft. This shift indicates that less of a partial pressure of oxygen is nceded
in order to reach half saturation of hemoglobin with oxygen. As the blood becomes more alkaline (due to
hyperventilation), hemoglobin shows a tendency to retain more oxygen (at lower partial pressures of oxygen). The
correct choice is A.

34. B is correct, a rise in temperature requires a greater partial pressure of 02 to bind a given amount of 02. The graph
shows that as the temperature increases, the curve moves to the right. The shift to the right indicates that a higher
partial pressure of oxygen is needed to reach half saturation of hemoglobin with oxygen. The correct choice is B.

?{ C is correct, arrhythmias. Arrhythmia means an irregular heart beat or having an irregular rhythm. Blood pressure
is measured with a blood pressure cuff. Choice A is incorrect. Cardiac output is a measure of the volume of blood
pumped with each heartbeat. Choice B is incorrect. Hematocrit is a measure of the ToRBCs in whole biood. It is
used for checking on anemia. Choice D is incorect. The correct choice is C.

36. B is correct, QRS. P represents the depolarization of the atria. During depolarization, they are contracting. Choice
A is incorrect. QRS represents spread of depolarization through the ventricles. During this depolarization, they are
contracting. Chbice B is corect. During the T wave, the ventricles are relaxed and repolarizing. Choice C is
incorrect. There is no T-P phase, it is a fake answer. Choice D is incorrect. The correct choice is B.
37. D is correct, death, if a normal rhythm is not introduced. Ventricular fibrillation means the ventricles are not
pumping together. The cells are contracting in their own unique patterns, producing an unproductive twitching'

Copyright @ by The Berkeley Review 144 The Berkeley Review


Specializing in MCAT Preparation
Biology tleart & Lungs Section II Answers

This can be fatal pretty quickly--within minutes. The individual is definitely affected, therefore choice C is
inconect. If there are no heartbeats at all, choices A and B are incorrect. The correct choice is D.
38. ,4.is correct, sinoatrial node. All the choices are involved in conduction of action poientials in the heart, but only
the sinoatrial node is actually in the right atrium, as the question asks. The atrioventricular node is located on th!
septum between the atria and the ventricles. Choice B is incorrect. The bundle of His travels though the
interventricular septum, and breaks into branches, forming the Purkinje fibers. Choices C and D are incorre"i. the
correct choice is A.
39. A is correct, tissue fluids contain ions that participate in conduction. Tissue fluids contain ions, such as sodium,
potassium, bicarbonate, and chloride. Choices B and D are incorrect. Ions are required for conduction. That is,
deionized watet'does not conduct electricity. Choice C is incorrect. The correct choice is A.

40. B is correct, a marathon runner. Exercise conditions the heart, so that greater volumes can be pumped per beat.
This means the heart rate can slow and still maintain the same cardiac output as an untrained p"rion. -Infa;B have
faster heart rates than adults. Choice A is incorrect. Weight lifting does not provide aerobic conditioning to
strengthen the heart. Choice C is incorrect. A sedentary adult is also less conditioned than a trained athlete. Choice
D is incorrect. The correct choice is B.

41. C is correct, plasma folate represents folate intake over the past day, while RBC folate represent folate intake over
the past few months. As stated in the passage, folate is a water-soluble vitamin. It is excreted by the kidney, so it is
required every day. The serum contains folate eaten in the diet on that day. Choices A and D are incorrect. RBCs
live about 120 days, so their supply of folate represents the composite intake of folate over their lifetime of a few
months, not a year. Choice B is incorrect. The correct choice is C.

42. B is correct, centrifugation. Both extraction with an organic solvent and thin-layer chromatography would break
the membrane of the RBC, mixing the plasma and the cellular contents. This would not be helpful. Choices A and
C are incorrect. Dialysis tubing would contain the RBCs and probably the folate, too. This would not lead to a
separation. Choice D is incorrect. Finally, centrifugation is the perfect way to separate the heavy cells from the
plasma. Following a short spin (10 minutes at 3000 rpm) the cells would sink to the bottom of the centrifugation
tube and the plasma would move to the top, allowing separation for independent analysis of the two components.
The correct choice is B.

43. A is correct, The subject's sample is added to a known amount of bacteria, incubated for a time period, and the
turbidity of the medium is measured. In this assay, the growth of the folate-dependent bacteria provide the
indication of the folate level present. The serum or RBC sampie is added to a known amount of the bacteria in a
liquid medium, incubated, and the growth is measured by the turbidity (cloudiness) of the solution. The more
growth, the more turbid the solution. No extra folate is added to confuse the results. This makes choice B incorrect.
No further enzyme activity assay is required, the bacteria provide the enzymatic assay. And they do not produce
folate. Choice C is incorrect on both these points. Choice D is trying to remind us of the Ames's test for mutagens.
It is also incorrect. The correct answer is A.
JJ. A is correct, they were consuming between 300 and 400 pg/day. Compare the initial levels with the final levels lor
each dosage group. Although all the women started with nearly identical piasma and RBC folate levels in each
group, the 200 and 300 groups's levels fell. This means they were consuming less than their usual levels of folate.
This means choices C and D are incorrect. However, the 400 group had a rise in both plasma and RBC folate levels.
This means the experiment provided more folate than their usual levels. Choice B is therefore incorrect. The
correct answer is A.
-15. D is correct, macrocytic anemia. The question tells you that the cells become too large before they are released
from the bone marrow. The prefix "macro" refers to iarge, while "micro" refers to small. Choice A is incorrect.
Choices B and C are made up words. The correct choice is D.

16. B is correct, folate needs increase during pregnancy due to synthesis of many new cel1s. Since the experiment
indicated 2A0 'p"glday was not adequate for nonpregnant women, and pregnant women are synthesizing more
nucleotides for the fetus, it makes sense that folate needs are increased during pregnancy. Choice A is incorrect.
The placenta does not synthesize folate. Choice C is incorrect. Foiate is a water-solubie vitamin, and is not stored
in the liver. Choice D is incorrect. The correct choice is B.

I -:-, right O by The Berkeley Review t45 The Berkeley Review


Specializing in MCAT Preparation
Biology fleart & Lungs Section II Answers

41. D is correct, thymine' First, make a distinction.between purines and pyrimidines. purines
pyrimidines have 1. Choices A islhe purine guanine while choice have2rings, while
B is the purine adenine. This leaves us with the
structures shown in choice c and D' Is there iny information
the passage that will allow us to distinguish between
these two nitrogenous bases? No. This questionis designed
we can. and then to make an educated guess at the correct
; ;"i;;l; eLminare as many of the possible choices as
onr*Jr. The correct choice is D.

of the blood following a hearrbear. when the first sound is heard,


*:
S,::":::t"""::.:1"^:11lt^1:T_pj:-ttr'"
p.",,,." rhe arreriat
lT:::"^i:^ri::.T"::"Tl"r, "j 1;; tJi;;;;;",rr."fi;,"', ,n" JilHJ#'il"',:ff"':the artery reaches.
n:*. o{ brood is interrupied tr;',; ##;;;#;r-
:*:.".^*,:'^t:::'::1 Il:
compressing the arm at all.'"Tli"'
Choice B is incorrect. The heartbeat ' not represented
#i,ii!'J*ii!'""j;
presiure is ireasured. choice by taking blood pressure.
D is incorrect. The correct choice i, c.tt
49. A is correct, the reading will be falsely high. If a cushion of fat absorbs
some of the pressure from the cufT, then the
cuff must be inflated more to co-ptess the artery compietely.
This would mean that the reading of pressure needed
to close the artery is falsely high. This would leid to un influt"o
blood pressur".;i;;.- Ti. correct choice is A.
Discorrect'afluidmovesasaseriesofindividual layers,eachofadifferentvelocity.
physics' try to figure out the answer based on vocabuiary. Ifyouhavenotseenthisin
A laminated countertop *"riril-or a layer of plastic or
other material glued onto a different material. Laminar."r".r
,o iuf"r.. gtirninur" .hoi"" A. since the question asks
about layers (laminar) eliminate choice c as well, although
urorn, ao have their own ranclom activity. To get the
correct answer' think of water moving in a stream. It is siowest
closest to lhe bank, where there is a lot of friction
with dirt and rocks' It is fastest in thi center" Each layer moves
with a differ"rt choice B is incorrect.
The correct choice is D. "Lro.rt).
51. c is correct, 211' Read the tabie ofblood pressures The systolic pressure in a 46-year-old
leveloftheheart. Addinthegravitvfactorli00l0.l:80)ri;t;;ler131+80=zir. woman is 131 at the
men' 210 is your answer and is incorrect. choice B is incorrect. riloureadthecolumnfor
answer. choice A is incorect. choice D is just incorrect. The
ir you read the a;astotic column, 162 is your
correct choice is c.
c is correct' the location of the cuff does not matter. on Earth, gravity changes the
reading fiom site to site on the
body' In space, since there is no gravity, this change will not o.!ui.
crroi.es A and B are incorrect. The cuff will
function.f ust fine, as will the heart, in an anti-gravitf situation
ctroice D is incorrect. The correct choice is c.
B is correct, II only' Normaily, arterial blood flow is laminar and smooth.
Turbulent flow is noisy, just iike in a stream. when the artery is
It makes no sounds or vibrations.
Jgr,tiy by ,ir" ,rrf the flow becomes
turbulent and noisy' Before the systolic sound is heard, no blloa "onstri"tJ
is fio*ing tkough the cuff. choice I is incorrect,
After the diastolic sound, flow is silent and therefore laminar. choice
between the systolic and the diastolic sound can noises be detected,
IIi is inJorrect. only during the interval
so that flow must be turbulent. Choice II is
correct. The correct choice is B.
D is correct, g4' The calcuiation uses data from the table. A 27-year-old,man,
on average, would have a systolic
pressure of 125 and a diastolic pressure of 78. The difference bet*Len
the two readings is 125 - 7g !1. The pulse
pressure is 47. MAP = 78 + (41/3) j8 + 16 94. The correct choice =
= = is D.

B is correct, av/aP' we. are looking for the- best relationship for aortic compliance. Think about what
"compliance" means' compiiance is the act of conforming. In
this case, it is the conforming (stretch) of the aorta in
response to successive injections of liquid. In the body biood is injected
into the aorta from the heart which will
cause a rise in pressure' The volume of the aorta will change, depending
on ttr" p."rr,r...-irr" compliance of the
aorta is the ratio of the change in volume in response to,a ciiange^in pressure"
T[is relationship is denoied on the
graph by the slopes of the lines, as the y-axis represents the voluire uno
th" *-u"ir ."p."r"nir-p."rrur". compliance
is the change in volume in response to a change in pressure. The correct choice
is B.
D is correct, inverseiy proportional to aortic compliance. We can arrive at this
answer by the fbllowing logic: We
are told from the passage that the fractional change (AD/D) is a reflecrion
or itr" ;h;";""i; volume of rhe aorra
during left ventricular contraction. Therefore, one can say that (AD/D) can be repla8ed
by AV. Making this

Copl,right O by The Berkeley Review 716 The Berkeley Review


Specializing in MCAT Preparation
Biology fleart 6r Lungs Section II Answers

substitution in the equation for the elastic modulus, one sees that Ep = AP/AV. This is, of cours,e the inverse of
compliance, so the elastic modulus is inversely related to the compliance. As the compliance increases, the eiastic
modulus decreases. Looking at Figure 2, one sees that with age, the elastic modulus increases. This should clue
one to the idea that with age, compliance decreases. Nevertheless, the relationship is one of an inverse proportion.
The correct choice is D.
5/. D is correct, Curve E. In order to answer this question correctly, one must have made the conclusion that
compliance decreases with age. From Figure 2 and the ideas in the passage, we can come to this conclusion. We
have already established that in Figure 1; the compliance will be represented by the slopes of the lines. Each line
represents a par:ticular age group. Let us look at the normal pressure range of 80-120 mmHg. It is clear that Curve
A has the highest slope, which represents the highest compliance. We are looking for the oldest age group, which
should have the lowest compliance (according to our conciusion). This is represented by the curve with the lowest
slope which is best represented by curve E. The correct choice is D.

58. B is correct, later in systole. This question requires a careful reading of the passage. First, think about when the
peak arterial pressure is going to occur. Is it going to occur during diastole or systole? The heart is contracting
during systole, and that is when we shall see the most force and hence the peak arterial pressure. Therefbre, *" .un
eliminate the answers which claim that the peak arteriai pressure will occur during diastole. Then the question
becomes when will we see the peak arterial pressure. The last sentence in the passage tells us that as compliance
decreases, the heart is unable to eject its stroke volume as rapidly when compared to a more compliant system. If the
heart is slowed at ejecting its stroke volume. it can be logically concluded that it will take progressiveiy longer for
the peak arterial pressure to occur. The correct choice is B.

59. D is correct, least at very high and low pressures and greatest over the usual range of pressure variations. We can
gatl-rer this information from looking at Figure i. We shouid already be aware that Curve A represents the youngest
age group. We simply need to look at siopes. It is ciear that at certain points in the curve, the compiiance of the
youngest individuals is smaller than those of older individuals (this does not invalidate our general conclusion that
compliance decreases with age). On the other hand, it is clear that for the youngest individuals, their compiiance is
higher for most of the range of pressures. Therefore, when compared to the rest of the age groups, one cannot say
that the compliance for the youngest individuals is either greater or less than for the entire range of pressures. Oni
can then eliminate choices A and B. We then look at choices C and D. Curve A is sigmoidal. The ilopes are least
at the extreme pressures, and greatest during the normal range of pressures. This translates into the compliance for
the youngest individuals is least at very high and low pressures, and greatest during the normal range ofpr"ssu."s.
The correct choice is D.

60, C is correct, decreases with age, a manifestation of increased arterial rigidity. One should easily be able to
eliminate choices A and B. These claim that compliance increases with age, and Figure 1 clearly does not support
this clairn. Therefore, we now look for a more likely manifestation of the decreased compliance. If the aorta does
not change its volume as well in response to a given change in pressure (less compliant), the arterial system is more
likely to be rigid. The more rigid the aorta, the less it can conform, or comply with a given change in pressure. The
increased rigidity is a result caused by progressive changes in the contents of collagen and elastin in the arterial
walls. The increased rigidity with age results in a decreased compliance with age. The correct choice is C.

61. C is correct, tricuspid valve. This question draws not on information from the passage, but from our own
knowledge of the anatomy of the heart. There are vaives between the atria and the ventricles known as the
atrioventricular valves. The valve between the right atrium and the right ventricles has three cusps, making it the
tricuspid valve. When the right ventricles begins to contract, the valve between the two chambers closes and
prohibits the back flow of blood into the right atrium. The correct choice is C.

62. B is correct, Ko. We are looking at the permeability of potassium. As the cardiac action potential begins, the
permeability of potassium decreases as potassium channels ciose. This is the key to answering this question.
Decreasing the permeability of potassium while increasing the permeability of sodium will result in depolarization
of the membrane. During the plateau of the action potential, the potassium permeability stays below the resting
value. Ultimately, repolarization does occur when the petmeabilities of calcium, sodium, and potassium all return to
their original state. As a note, Graph A represents the permeability of sodium, while Graph B represents the
permeability of calcium. Because Graph C dips right as the depolarization begins, one can safely conclude that
Graph C represents the permeability of potassium. The correct choice is B.

Jopvright @ by The Berkeley Review 147 The Berkeley Review


Specializing in MCAT Preparation
Biology fleart & Lungs Section II Answers

63. A is correct' Region A' This region of the action potential is unlike
a normar resting potential. Note that the resting
potential of the SA node cell is not steady but instead
manifests u .ro* depolarizat[". rrrlr gradual depolarization
is known as a pacemaker potential. The pacemaker potential
u.i"gr trr" membrane potential to threshold, at which
point an action potential occurs. After the peak of irr" u.tion p"[ntial,
the potentiai repolarizes, and the gradual
repolarization begins over again. Thus, the capacity forupontu'n"ou,
rhythmic setr excitition is best manifested by
Region A of the action potential. The correct ctroice is a.

64. c is correct' the flow of positive ions out of the cell equaling the flow of positive
ions into the cell. In myocardial
cells' the original membrane depolarization causes uottage-gaiea
calcium c^hannels in the plasma membrane to open.
This results in a flow of calcium ions down their eiecfto;rrJ-i"uig.uaient
into the cell. i""uur" there is a delay in
their opening, these are called slow channels- The-flow oiporiii"L
calcium ions into the cell, along with some
sodium also entering through the slow channels just balancesirr"
Ro* ofpositive potassium charge out of the cell.
This keeps the membrane depolarized at the plateau value. In
other words, the flow of positive ions out of the cell
equals the flow of positive ions into the cell. Do not
be fooied uy ttr" other answerr. wt ii" they may
certain aspects of the cardiac-,action potential we are rooting be true for
membrane potential is essentially not changing throughout trr"
io; the best ."u*n ,o the fact that the
piit"* region. The correct "*iiuin
choice is c.
65. c is correct' gap -iunctions between cells. The wave of depolarization spreads
from contractile cell to contractile
cell via gap junctions' Recall that gap junctions are channeli which
allow cells to share cytoplasm. In this way, the
current responsible for depolarization in one cell can spread
to another ceil in a u".y qui"t'*anner. There are no
neurotransmitters involved, which greatly reduces the amount
of time needed to cause contraction over the entire
chamber. The correct choice is C.
66. D is correct' Graph c is the result of vagus nerve stimulation.
we know from the diagram that all three graphs are
pacemaker potentials' The difference between the three
graphs ouuiou.rry lies in the slope at the beginning
the potential' In Graph B, we see a large slope. In part of
other rriords, we.each threshold very quickiy. on the
Graph C reaches threshold after quite-some time. one could other hand,
.onclude that Graph n *ouiJ-^ost likely represent
sympathetic stimulation, while Graph c represents parasympathetic
stimulation.' ttereiore c.upt C most likely
represents stimulation of the vagus nerve (a major paiasympathetic
iiber). The correct choice is D.

67. c is correct, favor movement of fluid from vessel to^interstitial space. one
needs to make the fbllowing conclusion
from the information in,the passage. There are two forces which
favor movement of fluid out of the vessel. These
forces
rvrvuJ 4re
are the intracapillary hydrostatic pressure and the interstitial fluid oncoric nreqsrrrc ,r.hara .*- ?,-,^ r^-^^^
urc lrrlr'luaplllary-nyorostatlc
oncotic pressure. There are two forces
which oppose movement of fluid out of the vessel. These are
fluid hydrostatic pressure. with this understanding, we
,h;l;;;, protein oncotic pressure and the interstitial
,""'rhui un increase in the intracapillary hydrostatic
pressure would most iikely favor movement of flui-d'from"un
the vesselio the interst;t;ot ,piri. such acrion
affect the concentration of osmotically active moleculls, and therefore will not
we can eliminate choice A. Also, such action
will not aff-ect the
venous resistance, and therefore we can eliminate choice D.
From the passage we can euminate
choice B because we are told a decrease in the arterial pressure
r"rutt, in a decrease in th" intru.upillary pressure.
The correct choice is C.
68. A is correct, P" - Pi > P.. According to the figure in the passage, filtration is the movement of
vessel' In addition, *"^
fluid outof the
told from the passage that the foice oifiltration comes from the difference
T
P. and P1 ' The force of filtration is thus P. - P1. According to the diagram, between the
the p" always has a positive value and is
thus larger than the P1 ' If P1 is a negative number, then the force of filtration
will actually become larger than the
capillary hydrostatic pressure. This is indicated by the equation in choice
this conciusion, and all of them can be eliminated. The correct choice is A.
A. None of the other equations follows

69. B is correct, result in a reduction of capillary hydrostatic pressure. we are reducing


the diameter of a precapillary
vessel' This will increase the resistance of that vessel so biood will not
flow througlithat vessel as well. The result
of this increased resistance is that the capillary will not receive as much blood
volume. In turn, this will result in a
decrease in the capillary hydrostatic (blood) pressure. A reduction
in blood volume will decrease the amount of
force/ unit area placed on the walls of the capillary. A reduction in P. will not
favor movement of fluid from the
vessel to the interstitial space, eliminating choice A. The result of this
action will not be equivalent to an increase in
venous resistance, but.is_exactly the opposite. Think about it! Finally,
resistance-cffies will not affect the
concentration of osmotically active particles in the interstitial space, so choice
D can be eliminated. The correct
choice is B.

Copyright @ by The Berkeley Review 148 The Berkelev Review


Specializing in MCAT Preparation
Biology Ileart 6r Lungs Section II Answers

70. A is correct, Fluid movement = k[(Pc + It) - (Pi + np)l. This question requires us to put together knowledge
gathered in the passage into an equation form. It brings us back to our discussion of forces. The forces moving
water out of the vessel are the capiilary hydrostatic pressure and the interstitial oncotic pressure. Therefore, we
should see these two forces added together. This eliminates choices C and D. The two opposing forces, P1 and the
plasma protein oncotic pressure, should be added together because they move fluid in the same direction. This
eliminates choice B and leaves us with choice A as the correct answer. We see the opposing forces subtracted from
each other will result in fluid movement. If the aigebraic sum is positive, filtration will occur, and if the sum is
negative, absorption will occur. The correct choice is A.
-1. C is correct, carries electrical charges at blood pH which attracts various electrolytes. The answer can be arrived at
through a process of elimination. We are told that albumin exerts a greater osmotic force than can be accounted fbr
solely on the basis of the number of molecules dissolved in the plasma. Therefore, there is something special about
albumin. It is not very likely that it could be replaced by an inert molecule and have no effect on the plasma protein
oncotic pressure. Therefore, we can eliminate choice A. Consider choice B. How many proteins do you know of
that dissolve into more than one protein? Proteins are not simple ionic salts. We are talking about a large protein. It
ri i1l not dissolve into more than one molecule, making choice B an unlikely choice (be aware that this would
increase its osmotic activity, if it happened). Choice D provides no explanation for the claim in the question. In
iact. choice D would allow us to predict a smaller plasma protein oncotic force. Therefbre, the best answer is choice
C. Nothing in the passage tells us that it does carry a charge, but that is not the point. One should be able to reason
ihat this is the best answer given our knowledge. This is indeed the case. The protein carries negative charges,
\'' hich attract primarily sodium ions. In addition, chloride ions attach themselves to the protein, which attract even
:nore sodium ions. The additional electrolytes provide the increased osmotic strength not accounted for by the
.',rncentration of plasma albumin. The correct choice is C.

D is correct, returns to the venous circulation via the lymphatic system. Since the remaining fluid is picked up by
::e lvmphatic system, we can eliminate choices A and B. The fluid will not remain in the interstitial space. If it did,
3lema would result. Furthermore, the fluid in the interstitial space is certainly not going to increase the
r:racapillary hydrostatic pressure. Having decided that the lymphatic system is going to pick up the fluid, one has
t: ask where is the fluid being returned to. Nothing in the passage tells us this, so one must draw on their own
o;::ou ledge. The lymphatic system returns fluid into the venous system, not the arterial system. In fact, the lymph
:.:rllaries drain into larger vessels that finally enter the right and left subclavian veins at theirjunctions with the
::spective internal jugular veins. The correct choice is D.

B is correct, increased left atrial pressure. We are looking for a situation which causes an increased volume of
:.: rJ tn the pulmonary capillaries. The increase in volume will cause the distension of the vessels. Increased left
.-:-.1 pressure indicates that blood is filiing the left atrium, but it is not being injected into the left ventricle. This
:'.:-. 'oe due to left heart failure. If blood is not being ejected from the pulmonary atrium, the pressure will increase
: ': llood from the pulmonary capillaries will not be able to move into the atrium. This will cause congestion and
", .-. ir.-rease the volume in the pulmonary capillaries, causing distension. All of the other answers act to keep blood
: '. ::, rlom or out of the puimonary capiilaries. The correct choice is B.
D is corrrect, i.09 L. According to the ideal gas law, which states that PV= nRT, we find that:

PiVr - PzVz and thar: vz=v';


Tr Tz H(|i)
: P- = 1 and T2lT1 (in Kelvin) > 1. The only possible answer choice is D, because we are iooking for an answer
i:.:i:: than one. The answer then becomes (lL)(2981273) = 1.09 L. The correct choice is D.
il- is correct. the venous blood from bronchial venules and heart vessels contaminate the pulmonary venous outflow.
',;,':
02 partial pressure. From the passage, we know that the
"re iooking for an explanation to the decrease in
:: .:i:ra1 \'enous flow along with vessels from the heart join with pulmonary venuies. In effect, by increasing the
-rrr3 eii blood without changing the amount of oxygen, we are decreasing the concentration of oxygen in blood.
ll":r correct choice is C.

t_t'-' 'r r lli-: Berkeiey Review r49 The Berkeley Review


Specializing in MCAT Preparation
Biology Ileart & Lungs Section II Answers

76. C is correct, 150 mmHg. The question asks for the partial pressure of oxygen in dry inspired air at the trachea.
First, we need to know the fiaction of oxygen in the air. We get this from the passage, and it is 21%. Note that in
the alveolus the fraction of oxygen dips to l4.3Vo, but in the trachea it still is 217o. Next, we must get the total
pressure of the dry air. The total pressure of the air is 760 mmHg, but 47 mmHg is due to the vapor pressure of
i{2O. The total dry air presssure is 760 - 4l = 713 mmHg. The partial pressure of oxygen in anhydrous air becomes
0.21 x113 mmHg = 150 mmHg' The correct choice is C.
77. A is correct, 102 mmHg. The total anhydrous pressure is 760 - 47 mmHg. The only difference is the fraction of
oxygen found in the air. In alveolar gas, the value is 0.143 (from the passage). To find the partial pressure of
oxygen'wesimplymultiply0.l43xTl3mmHg=l02mmHg.ThecorrectchoiceisA.
78. Ciscorrect. l97mlO2/L. Wearelookingforthetotal concentrationof oxygeninthesystemicarterial blood. This
includes oxygen found in hemoglobin and oxygen dissolved in the blood itself. Let us start with the latter. We
know this vaiue straight from the passage. The value is 3 ml O2lL. Now, let us determine the oxygen content for
hemoglobin. From the passage, we see that hemoglobin binds 1.34 ml 02lg. The concentration of hemoglobin is
150 gI-. We need to multiply these values. We get 1.34 ml O2lgx 150 g/L = 201 ml O2lL. The question states that
only 91% of hemoglobin is saturated with oxygen, so let us do an approximate calculation. We multiply 0.97 x 200
ml O2lI-. This gives us 194 ml O2/L. Adding the 3.0 ml O2lL that are dissolved in the plasma, we obtain 197 ml
O2lL. The correct choice is C.

79. B is correct, after aspirin treatment, endothelial cells produce new CO, but platelets do not. From the passage, we
learned that the turnover time of platelets is four days. Since the platelets in Figure I have the most active CO after
4 days, this means they cannot produce more when aspirin blocks CO. This eliminates choices A and C. However.
endotheiial cells show a rapid rise in CO after the aspirin dose. These cells simply produce more CO enzyme since
they have the machinery to do so. Choice D is incorrect. The correct choice is B.

80. D is correct, four degrees of unsaturation. Each double bond is counted as one degree of unsaturation, and each ring
is counted as one degree of unsaturation. The correct choice is D.

81. A is correct, acetylated serine residue. The passage tells us that a serine is acetylated to inactivate CO. Choices B
and D indicate a cysteine residue, and are incorrect. Choice C is a phosphorylated serine residue, and is incorrect.
The correct choice is A.
82. B is correct, I and II only. Too much aspirin would lead to poor clotting. This means that minor accidents could
more easily lead to bruising, and bleeding time would be prolonged. Choices I and II are correct. A prolonged
bleeding time means that the wound is slow to clot, not rapid. Choice III is incorrect. The correct choice is B.

83. D is correct, 5, 8 1 1, l4-eicosatetraenoic acid. Start counting from the carboxyl end. There are twenty carbons,
which requires the prefix "eicosa". There are 4 double bonds and the carboxyl group, giving the "tetraenoic acid"
part. Choices A and B are incorrect. The carbons are counted from the carboxyl end, giving 5, 8, I I, and 14 as the
positions for the double bonds. The correct choice is D.

84. A is correct, the dosing regimen keeps platelet aggregation low. The passage said nothing about the prolif'eration
(growth) of endothelial ceils. Eliminate choices C and D right away. To prevent heart attacks in people with
nirrowed arteries fiom cardiovascular disease, it is often desirable to keep the blood from excessive clotting. This
means that platelet aggregation (clumping together) should be low. Choice B is incorrect. The correct choice is A.

85. C is colect, aspirin should not be used in late pregnancy. Since aspirin inhibits the synthesis of prostaglandins by
inactivating the first enzymatic step, it could possibly interfere with the normal prostaglandin cycles involved in
labor and delivery. This would probably not stimulate labor. Choices A and B are incorrect. Birth defects are
initiated in the first few weeks of pregnancy. Choice D is incorrect. The correct choice is C.

86. C is correct, a decrease in HbF and an increase in HbS with age. HbA is normal, nonsickling hemoglobin. An
increase in HbA would not promote sickling of cells. Choice A is incorrect. The maternal hemoglobin is separate
from the fetal hemoglobin, and they do not mix. Choice B is incorrect. There are no antibodies to HbS, or that
would lead to an euen bigger obstruction problem. Choice D is incorrect. The correct choice is C.

Copyright O by The Berkeley Review 150 The Berkeley Review


Specializing in MCAT Preparation
Biology tleart & Lungs Section II Answers

87. A is correct, all cells contain both HbS and HbA. The hemoglobin in mixed in each blood ceil. There are no
exclusive HbS-containing cells or HbA-containing cells. Choice B is incorrect. In a heterozygote, both HbS ancl
HbA are present. Choices C and D are incorrect. The correct choice is A.

88. C is correct, HbF promotes a higher oxygenation state inside the cell. Adult (HbA) and I'etal (HbF) hemoglobins do
not bind to each other. This is seen in question 1, in which sickle cell anemia is diagnosed only after f'etafHb levels
fail. We also know that adults who have only HbA do not sickie. Choices A and D are incorrect. The cells sickle
under lower oxygen conditions. This means that a beneficial action is to raise oxygen levels inside the cell, so that
the HbS does not fall out of solution and cause sickling. Since HbF is beneficial, it rnust promote a higher oxygen
state inside the cell. Also, we know from our reading that HbF binds oxygen more aviclly then HbA. This is how
the oxygen gets transfered from the mother to the fetus. Choice B is incorrect. The correct choice is C.

89. D is correct, sodium metabisulfate would deoxygenate hemoglobin from red blood cells and the sickling pattern
would be noted in atfected individuals. Forget about the rusty color answers. We are interested in the sickling
phenomenon itself. Choices A and C are incorrect. The conversion of sodium metabisuifate to sodium persulfatE
involves the removal of oxygen from the other substrate. Since HbS crystaliizes and causes sickling under lowered
oxygen tension, this removal of oxygen would allow cells to sickle if the individuai has the sickle cell trait.
Hemoglobin would not be oxygenated under these conditions. Choice B is incorrect. The correct choice is D.

A is correct, decreased delivery of oxygen to tl.re tissues compared to other adults. HbF binds oxygen more tightly
and therefore reieases less oxygen to the tissues. This would be a problem in disease states in *t-riil Unp is really
high, such as the thalassemias. Choice B is inconect. This question has nothing to do with HbS, but rather it aski
about HbF. HbF, as is stated in the passage, does not sickle. Eliminate choices C and D. The correct choice is A.

91. C is correct, I and II only. A single base pair is changed to produce the HbS product. Since the DNA diff'ers at
least in this spot, specific restriction enzymes can used to cleave the DNA to produce a unique pattern of fraginents.
When fragments are separated by gel electrophoresis, the pattern of homozygotes (both sickie cell afTected and *itA-
type) and the heterozygotes can be easily distinguished. Choice I is correcr. The hemoglobin proteins will move
differently during gel electrophoresis. The pattern of bands can also be used to distinguish the groups. Choice II is
correct. Centrifugation is used to separated things of different masses or viscosities. The hemoglobin proteins are
too similar to be separated this way for diagnosis. Choice III is incorrect. The correct choice is C.
q7 B is correct, I and Ii only. HbF does not bind 2,3-BPG. This is one feature that increases its oxygen saturation
capacity. Choice I is correct. HbF does have a higher oxygen saturation at a given pO2 than HbA. This is a
mechanism that ensures the fetus gets oxygen preferentially from the mother. Choice II is true. HbF contains two
alpha and two gamma chains. Choice III is incorrect. The correct choice is B.

C is correct, the arteriai pCO2 should be maintained at 40 mmHg. We want to assess the effects of a changing
arterial pO2 on the ventilation rate. To do this, we shouid keep the pCO2 at a constant va1ue, This elin.rinates
choices A and B. The question becomes whether the pCO2 is maintained at 40 or at 46 mmHg. We are interested in
the effects of the arteriaL pO2. where the pCO2 is 40 mmHg. Therefore, to obtain the most accurate results, we
should keep constant the pCO2 at the value which can be found in the arterial circulation. The correct choice is C.

B is correct, total amount of orygen transported is relatively unaffected. The question tells us that a drop of 30
mmHg from the normal resting value of arterial pO2 does not change the rate of ventiiation. Recall the oxygen
dissociation curve, which tells of the percent saturation of hemoglobin with oxygen depending on the partial
pressure of oxygen. Remember that the curve is sigmoidal, with the top reaching an asyn'rptotic value. One can
descend fiom 100 mmHg of oxygen to 60 mmHg of oxygen and the percent of hemoglobin saturated with oxygen
does not change dramatically. In other words, the amount of oxygen delivered to the body is relatively unaffected.
The correct choice is B.
Qi A is correct, an unchanged arterial pO2. Carbon monoxide has a very strong affinity for the iron on the heme of
hemoglobin which is responsible for binding oxygen. For this reason, the number of hemoglobins able to transport
oxygen is decreased. Does this decrease the arterial partial pressure of oxygen? No. The reason is that only the
oxygen dissolved in the plasma contributes to the partial pressure. The oxygen on hemoglobin is bound and is thus
not soluble in the fluid. For that reason, carbon monoxide will not affect the partial pressure of orygen. The
correct choice is A.

Jopl,right @ by The Berkeley Review 151 The Berkeley Review


Specializing in MCAT Preparation
Biology tleart E( Lungs Section II Answers

96' C is correct, resisted by reflexes regulating ventilation to a greater degree than are equivalent changes
in arterial
pO2. This question is asking us to look at the graphs and ask ourselvei the following question:
Is o"ur body more
sensitive to changes in the partial pressure of oxygen or to changes in carbon dioxidl? Looking
at the curves, the
answer is clearly changes in the partial pressure of carbon dioxide. We also know that
the reflexes discussed in the
passage are involved in resisting changes that.occur to the partial pressure, in hopes of
restoring values to normal.
For that reasonr changes in the arterial partial pressure of carbon dioxide are r.esisted by ieflexes regulating
ventilation to a greater degree than are equivalent changes in arterial pO2. The correct choice is
C.
97. C is correct, hyperventilation is caused by increased neural output from the peripheral chemoreceptors. In a
metabolic acidosis' we have an overproduction of protons from some metabolic ioniition, like excessive
exercise.
These extra protons will stimulatc both the central chemoreceptors and the peripheral chemoreceptors
to increase
their output, and cause a rise in the rate of ventilation. Therefore, we can elimlnatl choices A
and B, which call 1or a
hypoventilation' Now, while the rise in protons will stimulate both central and peripheral receptors
over time, ask
yourself which one,will be the primary receptor. Can protons simply .ro5 ou'". tie
biood-brain barrier to have
access to the central receptors? The answer is no, because they are a charged species.
Therefore, we can deduce
that it is the peripheral receptors which are primarily responsible for the risJin the rate
of ventilation. The correct
choice is C.

98' D is correct, arterial pCO2 increases. During exercise, it is clear that the cells will be using more
oxygen and thus
producing more carbon dioxide' For that reason, the levels of venous pCO2 will increase.
Will this lead to a rise in
the arterial partial pressure of carbon dioxide? The answer is no, because the raie of ventilation
will increase. We
know that we breathe "harder" when we are exercising. In other words, the excess carbon dioxide
is released as a
result of the higher rate of ventilation. The increase in the carbon dioxide production is
equivalent to the increase in
breathing rate. The result of this is that.the arterial partial pressure of carbon clioxide does
not change, making D a
false statement. The correct choice is D.

99' A is correct' low pCo2 is permitting one to hold their breath, the exercise may lower the pO2 to levels
which may
induce unconsciousness. The swimmer's hyperventilation will result in a lowered partial
pressure of carbon dioxicle,
because they are eliminating a good deal of gas. During the race, that lowered level
of carbon dioxicie will allow
them to hold their breath. However, the exercise will lowir the ievel of oxygen. Recall the
body is more sensitive to
leveis of carbon dioxide relative to oxygen. For that reason, the body does" not know
to breathe, but oxygen levels
may be reaching dangerous lows. The level may become low enough to induce unconsciousness,
as nol enough
oxygen is reaching the brain. The correct choice is A.

100. B is correct, an increased arteriai pCo2 and a decreased arterial po2. This cluestion is very
straightforward. Durrng
times of sleep, the body is not breathing as often. The end result of this is that we are bringing
in less amount of
oxygen' and eliminating lesser amounts of carbon dioxide. The question tells us that the body;s risage of
oxygen per
production of carbon dioxide is unchanged during this dormant period. Therefore, we should
to"see an
increased pCO2 and a decreased pO2. The correct choice is B. "^p""t

Copyright @ by The Berkeley Review t52 The Berkeley Review


Specializing in MCAT Preparation
ffiffiffiKwffiy
A" The Gastrointestinal T.ract
Sec&Ewm EEE l. Nutrients & Digestion
B. The Kidneys
Gas&rep6m&es&€naaE l. Renal Function
2. Renal Physiology
€ract, 5. Homeostatic Mechanisms
Se
Ln
Practice Passages & Answers
K€dsa€ys
a^\
.-lji,ll
,a_a .,.,,
)F7.1;
* 1

c,--
!t

tfie
f,ffiKf,fuffiY
R.n.v.I"E.kV*
Specializing in &fCAT Preparation
Gastrointestinal Tract & Kidneys
Top lO Section Goals

Be familiar with the anatomy of the gastrointestinal tract.


Know how food passes from themouth, down the esophagus, into the stomach, through the iniestinal
svstem. and how it is eliminated as feces.

Understand the interactions between the various gastrointestinal secretions.


The for.rr gastrointestinal peptide hormones of importance are gastrin, cholecystokinin, secretin, and
glucose-dependent insulinotropic peptide. Be aware of their functions.

Be familiar with the process of digestion and the areas of nutrient absorption.
@w Know what types of nutrients are broken down in the stomach and what types are degraded in the
intestines. Understand how peristalsis is related to the movement of thyme in"the system.

Be familiar with the anatomv of the kidneys.


Understand how molecules are filtered at the glomerulus and which ones are t"uUtorU*a f.o,r.r tt *
different parts of the nephron tubular system. "

Be aware of the effect of the autonomic nervous on the


Know how the kidney responds to sympatJrehic and parasympathetic stimu-lation. Know tire difference
between vasoconstriction and vasodilation at the jevel of the glomerular arierioles.

Understand how the renin-angiotensin-aldosterone system functions.


"w This system stimulates the reabsorption of Nae in.the disial convoluted tubule and in the collccting
duct. iJnderstand rvhy this is imptirtant to the body.

Be aware of the different types of transport systems that line the tubules.
Understand the differences between passive reabsorption (no energy required) and acfive reabsorption
(energy required) and generally where and what ii reabsorbed ilong'the tubules.

Understand how the kidneys respond to a decrease in arterial blood pressure.


Be aware thai there is a short-term adjustment and a long-term ad justrnent for a decrease in arterial
blood pressure. Understand how ihis works in terms of ihe glomerular filtration rate.

,w Be familiar with diabetes mellitus and diabetes insipidus.

key feature of diabetes insipidus is a vasopressin deficiency. Both lead to a^large urine loss,

Be familiar with some of the conditions brought about by renal failure.


,g? The two most life-threatening consequences of renal failure involve the retention of Ke, due to poor
tubular secretion of Ke, ind mefabolic acidosis, due to improper tubular secretion of Ho.
Biology Gastrointestinal Tract & Kidney Nutrients & Digestion

Iiutrients Er Digestion
-:: s examine the gastrointestinal system. Digestion is the process by which food
:.:i is eaten is broken down into progressiveiy smaller particles and uitimately
::-.crbed by the intestinal tract. The gastrointestinai system includes the moutir
:---j associated salivary glands, esophagus, stomach, small intestine, large
-- -=strne, and certain aspects of the iiver and pancreas. In the average adult this
-:-::. running from mouth to anus, is about 30 feet in length and is actually an
. , :=:.sron of the external environment.

-- = ilod that we eat has a diverse composition. A typical meal might include
-,-
-;3n1qlg6ules such as starch in the form of bread or potatoes, cellulose in the
,:'f
=-. a salad or other greens, protein in the form of meat or cheese, and fat in
.--= :,'rm butter or ice cream. Each of these dietary categories needs to be
- = :: a led before they can be absorbed inio the body.

::r::i"r and glycogen are both important polysaccharides in nature. starch,


. ;--rJ:eristic of plant cel1s, and glycogen, characteristic of animal cells, are
- -:--.''zed within the digestive tract by enzymes called amylases.rJpon
' --:- -.'sis both polymers release the monosaccharide glucose. Cellulose is
''
'-- ''iii-rin the cell wa11s of plants and is a polysaccharide consisting of glucose
---
'- : - -r
=-. rinked together. The enzyme cellulqse can hydrolyze cellutse lnto its
-.

. ::.:*:nt glucose residues. See Figure 3-1.

Amylase Cellulase
-._:,jt L-) UluCose Cellulose
An example of
bacteriai symbiosis

' - -..::oivsis of starch and celiulose.

- : -::ant to note that even though starch, glycogen, and cellulose all contajn
'. ==rdues linked together to form iong polymers, we as humans cannot
:' -: ' ;lose. The reason that we cannot digest cellulose is because the glucose
- :: -r. ,e11ulose are linked together in a different configuration than the
-- :=srdues in starch or giycogen. This special linkage can only be
" -:---
'-nlr.by the enzyme cellulase--an enzyme that we do not have. Why,
:. -r;\-s eat grass? The only vertebrates that can utilize celluiose are the
'': :rimais (1ike the cow and goat). The rumen of these vertebrates
. ,': .--.-::oorganisms which manufacture and secrete the cellulase that can
- --ie cellulose in the vegetation being consumed. This is an example of
,:-c ielationship between the host ruminant and the microorganisms
---:- ihe rumen. The microorganisms get a warm place to live and the
.:.> energy in the form of digested celiulose.

::.entioned in previous sections, proteins are composed of amino


proteases can hydrolyze proteins to their constituent
::-.es called
:=sidues. Fat cells or adipocytes store triacylglycerols (also called a

The Berkeley Review 155 The Berkeley Keview


Specializing in MCAT Preparation
Biology Gastrointestinal Tract & Kidney Nutrients & Digestion

triglyceride but better known as just plain fat). Fats are hydrolyzed into fatty
acids and glycerol by the enzyme lipase. See Figure 3-2.

proteins Protease
----)- Amino Acids patsr-:)
Lipase
%l%:::1'
Figure 3-2
Enzymatic hydrolysis of proteins and fats.

Vitamins are needed in only small amounts in the diet yet they are essential in
order to prevent certain nutritional deficiencies. [For example, the disease
beriberi is due to a deficiency of vitamin 81 or thiamine.] As a group, vitamins
participate in many different chemical reactions in the body.

Inorganic minerals such as iron, potassium, calcium, and zinc are also important
for proper development. Iron is the atom in the center of the heme group that
carries oxygen in both hemoglobin and myoglobin, two proteins thal transport
oxygen within the body.

All of the proteins that we are concerned with in the biological sciences are
composed of a basic set of 20 different amino acids joined together in peptide
linkages. Out of these 20 different amino acids we need to obtain 9 of them in our
diet. The rest we can synthesize. In other words, we cannot synthesize certain
amino acids from our metabolic reactions and must therefore obtain them from
the food we eat. The amino acids that we cannot synthesize are referred to as
being essential amino acids. Thus, the five general components to a complete
diet are carbohydrates in the form of sugars like glucose, proteins, fats, vitamins,
and minerals.

The Gastrointestinal Tract


The gastrointestinal tract includes the mouth, esophagus, stomach, small
intestines and large intestines. Let's consider a cross section of the small intestine
and examine the tissue layers as we work outward from the lumen. The first
barrier we encounter is a convoluted layer of epitherial cells (Figure 3-3a).

The convolution of this layer helps to increase the surface area of the
gastrointestinal tract for absorption of nutrients. Scattered throughout this
epithelial layer are ducts from external exocrine glands like the paricreas and
liver (and the salivary glands in the oral cavity). ]uxtaposed to many of the
epithelial cells are endocrine cells which contain hormones that can be released
into the blood. These hormones influence other cells in the gastrointestinal
system. Both the parasympathetic and sympathetic nerves (of the autonomic
nervous system) innervate the gastrointestinal system. The more important
nerves stem from the parasympathetic system. Recall that the parasympathetic
nerves are more active during times of relaxation and digestion while the
sy'rnpathetic nerves are more active during times of ,'fight or flight."

As food is passed from the mouth to the lower portions of the gastrointestinal
tract the smooth musculature surrounding the epithelial cells begin to contract in
peristaltic waves. These waves (peristalsis) are not controlled by conscious
thought but rather by the action of the parasympathetic division and by the
action of hormones. Peristaltic action is rather quick because of the elecirical

Copyright @ by The Berkeley Review r56 The Berkeley Keview


Specializing in MCAT Preparation
Biology Gastrointestinal Tract & Kidney Nutrients & Digestion

- -':r.ltnuity imposed by the gap junctions in the circular and longitudinal muscles
-'Jre gastrointestinal tract.

(a)

Blood Vesseis
and/or Lymphatics

Epithelial
Cells

Parietal Cells
(HC1)

Chief Cells
(Pepsinogen)

:';rgitudinal
\Iuscie
Nerves to and
Gland outside the
tiil from the CNS
gastrointestinal tracl
4r
il.

F':,:ure 5-3
, : -- -r:n through the small intestines
:l

- : : --:
-o\\r a bolus of food as it passes its way
through the gastroiatestinal tract.
-- ': - ::-outh the muscles of mastication move the jiw and the food is ground
': - ::r. ihe teeth. salivary amylase is secreted into ihe oral cavity and be"gins to
' =! , :ie starch and glycogen. This secretion is contiolled u-v tr,e
-
r- : - i : " :'.:athetic nerves.

= :rrd
is swallowed and passed into the pharynx access to the nasal cavity
-=: -\ flap of tissue called the epiglottiJ covers the opening to the larynx
:=''ents food from entering into this passageway (to prote-t the airway).
:re food passes into the esophagus and then down into the stomach.
": : t:ol\ring reflex is controlled by centers in the medulia.
' -:
= food reaches the stomach a sphincter (circular muscle) called the
r-^
: r.-'::: esophageal sphincter contracts and prevents regurgitation of the food
. , ::-'lhe esophagus. If this sphincter were not closed off, stomach acid
r - : =:-:er the esophagus and irritate the nerve endings in the smooth muscle.
- ": --
-::ir1g sensation is referred to as heartburn.

-: enters irrto the stomach a little more digestive action takes place. Two
:f
the stomach is to break the food down into smaller partiiles and to
:: ibv acidic secretions). There are four major types of secretion within
. .:::'.. Mucus is secreted by surface cells and acts to protect the lining of
::-::n and lubricate the food. Gastrin, located in endocrine cells in the

. ti The Berkeley Review r57 The Berkeley Review


Specializing in MCAT preparation
Biology Gastrointestinal Tract & KidneY Nutrients & Digestion

lower portion of the stomach, is secreted in response to protein entering into the
stomach. This hormone stimulates the secretion of HCI and pepsinogen. HCl, at
a pH of about 1, is secreted by parietal cells. Pepsinogen is secreted by the chief
cells. Parietal cells and chief cells are located in the gastric pits the line the
epithelium of the stomach (see Figure 3-3b). Pepsinogen is the inactive form of
the peptidase enzyme pepsin. Peptidases, like pepsin, cleave peptide bonds.
Since protein is composed of amino acids and amino acids are linked via peptide
bonds, pepsin hydrolyzes proteins (at specific places in their amino acid
sequence). Pepsinogen is converted to pepsin by the action of HCl. However,
once pepsin is formed it can autocatalytically act on pepsinogen to form more
pepsin.

Besides secreting HCI the parietal cells also secrete a glycoprotein called intrinsic
factor. This glycoprotein is important because it complexes with vitamin B12 and
is then absorbed by the intestinai epithelial cells and transported by the
bloodstream. if there is a defect in the synthesis of intrinsic factor, vitamin 812
cannot be bound and therefore cannot be absorbed by the epithelial cells in the
intestine. Vitamin B12 is important in erythrocyte (red blood cell) formation.

If too much acid is secreted into the stomach, ulcers can occur in the stomach and
in the small intestine (which is directly connected to the stomach). Ulcers are
simpiy erosions of the walls of these two organs, and if they are extensive
enough can cause bleeding. One of the most powerful stimulants that causes HCI
to be released into the iumen of the stomach is histamine. It turns out that a
compound called cimetidine (trade name is Tagamet) inhibits the binding of
histamine to its receptor on the parietal cells. This reduces the amount of HCI
secreted in the lumen of the stomach. Cimetidine and its analogs are quite
commonly prescribed by doctors as a non-surgical treatment for ulcers.

As the dissolved food (refered to as chyme) passes from the lower stomach and
into the small intestine more enzymatic activity takes place. This is partly due to
distension of the stomach and smail intestine and ihe generation of nerve
impulses that stimulate enzymatic secretions. Roughly 90"/o of the digestion and
absorption that takes place in the gastrointestinal tract occurs in the small
intestine. Not only do fiuids enter the small intestine from the stomach but the
pancreas and liver also add secretions as well. The smali intestine acts to
neutralize the acid which has been secreted by the stomach and to further digest
and absorb food particles.

Distension of the small intestine also causes the hormone cholecystokinin


(abbreviated as CCK) to be released from the intestinal mucosa. CCK diffuses by
the way of the bloodstream to the pancreas where it causes the pancreas to
release digestive enzymes. The hormone secretin is released from the small
intestine in response to the entering chyme from the stomach. Secretin is also
absorbed by the blood and is transported to the pancreas where it causes the
release of bicarbonate ion and other fluids.

The pancreas not only has endocrine cells which secrete insulin and glucagon but
it also contains secreting structures called acini that secretes a fluid into the small
intestine which has a high bicarbonate content that is rather alkaline. The
bicarbonate ions combine with the protons from HCI to produce carbonic acid
which is then converted into carbon dioxide and water. Carbon dioxide is
absorbed into the blood and transported to the lungs where it is expired' This

Copyright @ by The BerkeleY Review l5a The BerkeleY Review


Specializing in MCAT PreParation
Biology Gastrointestinal Tract 6r Kidney Nutrients & Digestion

:-echanism increases the pH in the small intestine to a more alkaline value.


See
Figure 3-4.

\a*+HCO3-+H++Cl- HCO3 (bicarbonare ion)


H2CO3 (carbonic acid)

H2Co3 '-) Co2 + Hro


F-igure 3-4
I .:: :'..1 of the bicarbonate ion.

-: - lir-er has a diverse set of functions, one of which is to synthesize a compound


:.--=: bile which it concentrates and stores in the galibladder. The'ma1or
: i gment in bile is a compound called bilirubin, which ii a breakdown produci
of
- .:.::Iobin. Bile also contains bile salts which are important in the digestion
r ' ::;orption of fats. when bile is rereased from the guilbludde, it passeJdown
r ri:i that joins with the pancreatic duct, througlia constriction called the
*':-:.incter of oddi, and empties into the small intestine. Bile acts to emulsifv
fats
: -1e.rease their surface tension in order to break them up into smaller sizes)
r: - -: :lso helps the epithelial cells of the small intestine absorb those fats. The
::::.:-:e of fats in the small intestine releases CCK which then acts on the
:**:,::;er and causes contraction, and on the sphincter of oddi and causes
-:-r":.:,-n, so the bile can pass into the lumen of the small intestine. [The
-::::: --i:- :o1or of the gallbladder is due to the various breakdown products found
r- :-:: :,-...entrated biie.]

Basolateral

', ',Na+
1: lL li.t COS€
: \ **ou^o
Glucose

tr-umen Blood

ii*rruumne 3.5

' \ : j:'. . ir.entioned, the small intestine is where the majority of absorption of
-".* r- r --;:tients takes place. Located on the apical and basolateral regions of
r , ::"r.--i="". cel1s are specialized transport proteins ("carriers")
r"spor,ribl" fo,
r"u,' irrt:",
-:l':1 of sodium, chloride, sugars, amino acids, vitamils, and other such
Tr .r -i '.. For exampie, consider the absorption of
glucose into the epithelial
,r-" - - :: s. .-i oniv able to enter the cell if sodium is coiransported along with it.
- . r - -: .:.-:ers the cell down its concentration gradient, it "drags"
glucose into
'''r -- -.:-:::-". of the cell. Because the intracellular concentration of glucose
I r''' r:'::i -- - able to diffuse out the basolateral side and eventually it,to th"
, ,':r --.: --:-= .oncentration of sodium increases in the cell it is aiso pumped out

' " ;L: The Berkeiey Review 159 The Berkeley Review
Specializing in MCAT preparation
Biology Gastrointestinal Tract & Kidney Nutrients & Digestion

on the basolateral side in exchange for potassium via the sodium-potassium


pump. See Figure 3-5.

As the ions and nutrients are being absorbed by the epithelial celis of the
small
intestine, water is diffusing_ through the membrine. witer is trying
to equilibrate
on both sides of the membrane by osmosis. Recall that choleri toxiln causes
massive loss of fluid by diarrhea. In the majority of cases this can
be treated by
giving the individual large quantities of glucose and saiine solutions.

How much fluid is being absorbed by the small intestine? Roughly 1.2 iiters
of
water per day are taken in from the foods we eat. Another 7.0 literi
are secreted
by the stomach, pancreas, iiver, small intestines and so on. Roughly g.1 liters
of
the total 8.2 liters is reabsorbed in the small intestine. similirly,
out of the
rgughly 800 grams of solid food consumed, roughly 570 grams are absorbed
in
the small intestine. The remaining fluid and solid"material i"s
excreted as waste.

Recall that fats are degraded to fatty acids and glycerol by the
action of the
enzyme lipase. These compounds can diffuse intJtire intesiinal
epithelial cells
where they are resynthesized into triglycerides (fats) and aggregate
into
structures called chylomicrons. These aggregates are released at
the basolateral
membrane and into the extraceluiar spice (Figure 3-6). The chylomicrons
enter
into the lyt ph and are transported to the veins and eventually the tissues.

Fatty
acids
)r @
Glycerol IFATS o
To the lymph
and tissues

Figure 3-6
Release of chyiomicrons.

The large intestine absorbs most of the water and ions that are left in the chyme
as it passes from the small intestine. As the sodium and chloride ions are
absorbed an osmotic gradient is established that allows for the absorption of
water. \44rat is not absorbed is passed out the body in the feces.

Copyright @ by The Berkeley Review t60 The Berkeley Revien'


Specializing in MCAT preparation
Biology Gastrointestinal Tract & Kidneys Renal Function

Renal Function
r.:ush1v 60% (a littie less than 2/3) of an,averageT}-kg (154-1b) man's
body
' ::ght is water- Most of this fluid is located insidJ
the ceiis (called intraceilular
:-*:;,and about 113 of it is located in the interstitial spaces outside the cells
::-'ec the extracellular fluid). If you think about the organ systems which
-=..: pre'iously discussed, you will reaiize we
that watei"u."b" from the
lost rrullt
vs lwDL lltt
.:".y:i"_"|,!*," r\t"lby evaporation, the gastrointestinal tract, and the lungs.
:::: can also be tost fiom;;;;;U;;il;.
ne body by these four-avenues exictry
il;;;;ffi#riilliSl,
matches the water gained by the
::rrough consumption of fluids and regulation at the level of tte
kidneys.
r: .:.-- ihat we have mentioned that osmoiarity refers to the totai solute
- --:.:.fation of a solution. Remember, the highbr the osmolarity of a given
' -:-rr, the lower will be the concentration Jf water in that solution. If an
: r::-.r1 can change the internai
ionic concentration of its body fluids to meet
: :-'--:.e surrounding environment, they are referred to as osmoc."f";;;;.
:..: --.arine invertebrates are osmoconformers. However, most vertebrates do
- : ::.=.qe the internal ionic concentration of
their body fluids to meet that of a
': -"-: - j:.:::ig
environment. organisms like this are referied to as osmoregulators.
- :
" :--:::fence between osmoconformers
and osmoregulators can be seen on the
;: ri:: -: Figure 3-7.

Osmoreguiator

Environmental osmolarity

lllL{rnrrc 5-;
'' iLr ..:t'' :rx-. - : :,i:'; een osmoconformers and osmoregulators.

,t-:'rr.ii rid.ev presumably arose from freshwater teleosts (fishes). The


'r . :r - -::i1- of a fieshwater tlleost is about 300 milliosmols/liter,
which is
,.,i: ,:r :-':-- ::.a. of the surrounding
titl" ir
aqueous environment. This means that the
,i -:-'1----:- :: rfater is less
in the freshwater fish than it is in the water in
" rrr
' -": 1i:. :s sh-rmming. water from the environment will diffuse d.own its
r ':::*r--::- ::adlent and into the fish. The excess water that is contrnually
: . : jr - -:-- =€ fish is filtered and removed by the kidneys and excreted in tlle

i' -..-t :-" The Berkeley Review r6l The Berkeley Review
Specializing in MCAT preparation
Biology Gastrointestinal Tfact & Kidneys Renal Function

form of a dilute urine. Since copious amounts of urine are excreted. on an hourly
basis the freshwater fish also loses important ions like sodium and chloride ioni.
These ions are in rather low concentration in freshwater habitats and sd the
freshwater teleost has developed (via evolution) gills {or active transport of
sodium and chloride into its circulatory system. See Figure 3-g.

Water enters
by osmosis
Sallt
though

Active transport
Copious of salt through gills
urine
(salt and water loss)

Figure 5-8
Salt regulation.

Generalized Kidney Function


The kidneys in vertebrates like the freshwater teleost have three main functions.
The first function is that of filtration. The functional unit in the kidney is the
nephron which consists of a glomerulus (Latin, meaning "little ball"), Bowman's
capsule, and a tubular system. The glomerulus is a collection of capillaries that
receives blood from an artery terminating in the renal system. Blood is pumped
into the glomerulus by the hydrostatic pressure of the heart and that pressure
-The
forces the blood through the capillary walls and into Bowman's capsule. cell-
free ultrafiltrate found in Bowman's capsule lacks many of the plus*a proteins
found in the blood. Blood plasma is simply a solution which is about 906/o water
that is composed of organic (e.g., proteins, sugars, amino acids, etc.) and
inorganic substances (e.g., various ions like sodium and chloride). within the
blood plasma one also finds red blood ce1ls, white blood cells, and platelets. The
filtrate in Bowman's capsule is essentially the plasma minus the (large molecular
weight) proteins.

The second function of the kidney is reabsorption of important organic and


inorganic compounds from the filtrate in Bowman's capsule. Reabsorption occurs
through many of the epithelial cells which line the tubular lumen of the nephron.
The cilia of these epithelial cells move in such a way as to help propel the filtrate
through these renal tubes. Glucose, small proteins, amino acids, salts, bicarbonate
ions, and water are reabsorbed along this tubular system and transported back
into the blood. The epithelial cells also have the ability to secrete protons,
potassium, urea, uric acid, and ammonia. The third function of the kidney is
excretion of waste, salts, and excess water.

Copyright @ by The Berkeley Review r62 The Berkeley Review


Specializing in MCAT Preparation
Biology Gastrointestinal Tfact & Kidney Renal Physiology

R $Ey.lffiir;rrt r. , .otl rrffi-


Both human kidneys sit on the dorsal side of the abdominal cavity and are bean-
shaped organs about the size of a fist. Blood from the descending aorta enters
into the renal artery and eventualiy into the glomeruli of the r,uph.onr. Bioocl
ieaves the kidney by way of the renal vein which itself empties into the inferior
','ena cava. Leaving each kidney is a ureter which transports the urine to the
bladder. urine exits the bladder by way of the urethra. These anatomical
:onsiderations are shown in Figure 3-9.

Inferior Vena Cava---

,- - ::- position of the kidneys.

:: . --- ..jnev contains more than a million nephrons. The glomeruius and
' : - s capsule of each nephron is located in the cortex and gives this portion
r-- = .-:lnel' a granular appearance. In contrast, the striated appearance of the
* :': ;--a '- primarily due to a portion of the tubular system of the nephron called
: , I r ! of Henle and the collecting duct. The collecting ducts, whiih collect the
'-- .
=:,:tv into the renal pelvis of the kidney and then into the ureter. This is
. -: Figure 3-10.

= br The Berkeley Review The Berkeley Review


Specializing in MCAT Preparation
Biology Gastrointestinal Tract & Kidney Renal Physiology

Renal
Pelvis

Figure 5-lO
Kidney anatomy.

The Nephron
We mentioned that the functional unit of the kidney is the nephron. As the renal
artery branches into smaller divisions and enters the kidney it becomes the
afferent arteriole. It is the afferent arteriole which enters into Bowman's capsule
and forms the capillary bed called the glomerulus. An efferent arteriole leaves
the giomerulus and forms a capillary network which surrounds the renal tubules.
The blood that leaves this capillary network does so by a venule which later
empties into the renal vein that leaves the kidney. Extending from Bowman's
capsule is a long tubular structure which is divided into the proximal convoluted
tubule (PCT), loop of Henle, distal convoluted tubule (DCT), and the collecting
duct. Many different nephrons can attach to a single collecting duct. These
structures are shorlrn in Figure 3-1.1.

The cardiac output of the heart is roughly 5 liters of blood every minute. Out of
this 5 liters of blood roughly 20% of it (about 1/5) passes through the kidneys
each minute. Thus, the volume of filtrate passing into Bowman's capsule will be
rather large. Roughly 180 liters (47 gallons) of filtrate per day passes into all
these capsules and of that about 1.0 t6 1.S liters enters into the ,rtit Depending
".
on how much fluid you drank, the upper end of urine production might be about
3 liters per day.

The osmolarity of the filtrate in Bowman's capsule and in the proximal


convoluted tubule is about 300 milliosmols per liter. Recall that this is essentially
the same osmolarity we found in the plasma. The concentration of the urine that
is collected varies depending on the circumstance. If you wanted to excrete a
dilute urine, then the concentration of the fluid leaving the collecting ducts
should be lower than 300 milliosmols per liter. For example, a urine
concentration which is low might have an osmolarity which is 0.7 times that of

Copyright O by The Berkeley Review 164 The Berkeley Review


Specializing in MCAT Preparation
Biology Gastrointestinal Tract & Kidney
Renal Physiology

osmolarity of the prasma. In other word.s, the


::'re
osmolarity of this ,,d1ute,, urine
-tight be 200 milliosmols per liter. h contrast, a concentrated urine would have a
:.::h osmolTity,
:9Tuth11s in the range of 4.2 times thar oiirr" osmolarity of
:-tu<ma (or about 1200 milliosmols per li*ter).

Bowman's
Glomerulus
Efferent
crp;;;" Proximal
Convoluted
Arteriole

Afferent
Arteriole

Cortex

----ule tr) l

Loop of
Henie
Medulla

Capillary
)'ietwork =)

qliquunc }t I
'' ll :ruo![tr-]-:r iis related capillary system.
=:
,n F ;rir: proximal convoluted tubure. The pCT rs the obligatory
=e-:ecause
'n*s;;r:'11 section of
lrrt# roughly 6s./" of arl reabsorptio, ;;,;;r"iio" occurs
"'llrLu':
*'rr*r*al cells rn the pCT are quite metatohcally active here.
and support a
'iiir""rrrlrrri :,r ::ti-e transport processes. Glucose, small moiecular weight
rrrrru*iun:r ;l..
proteins,
a'd vitamins^are completery (100%) ,"uurortua in the pcr.
ril*,;11111L*9111r:;
lJ', of the Nae, Cle, and *ui"r are also reabsorbed in the pCT.
Many
rrrt' lg4s
''s'-' '- 'ms for reabsorption involve sodium. ro, grrr"o"" ur,i
,lir Lir'ulrfT r:rrs are reabsorbed
together as are the amino acids"ru*pt",
and sod.ium ions.
ii'lr flrr,rrur]r r:r5 a:e also
reabsorbed with chl0ride ions. Little ,"grrlurion
lhlT
ulllltrrrlri
occurs in

,11llrll"ti.gl: I The Berkelev Review 165 The Berkeley Review


Specializing in MCAT pre;raration
Biology Gastrointestinal Tract & Kidney Renal Physiology

As the PCT begins to descend from the cortex into the medulla it forms the loop
of Henle. The loop of Henle has a thin portion and a thick portion (see Figure 3-
Jt) rn9 descending thin portion of the loop of Henle is very permeable to water
but only relatively permeable to ions like sodium and molecules like urea. The
ascending thin portion of the loop of Henle is much more permeable to urea but
much less permeable to water than the thin descending portion of the loop.

As the loop of Henle ascends it becomes thicker. The epithelial cells in this region
of the loop actively transport ions like sodium and potassium from the lumen of
the loop into the interstitial fluid. However, this region is impermeable to urea
and water. This means that as the filtrate is passing up the thick portion of the
loop of Henle it is becoming more dilute beciuse of the fact that ions are being
transported out into the interstitial fluid. Remember, the lower the osmolarityl
the higher will be the concentration of water (i.e., a more dilute fluid).

The filtrate from the thick portion of the loop of Henle passes into the distal
convoluted tubule. The segment of the DCT closer to the loop of Henle is also
quite impermeable to urea and water but rather permeable toions like sodium.
Therefore, as the filtrate passes through this segmlnt of the DCT it will become
a
bit more dilute.

The epithelial cells of the segment of the DCT closest to the collecting duct and
that portion of the col]ecting duct located in the cortical region of the fidney are
still rather impermeable to urea. However, these two areis of the nephron are
quite sensitive to the hormone aldosterone (secreted by the cortex of the adrenal
glands which sit on top of the kidneys) which regulates sodium absorption. An
increase in the concentration of aldosterone causes sodium to be reabsorbed by
the epithelial cells. When sodium ions are pumped out into the interstitial space,
potassium ions are simultaneously transported into the lumen of this .eglol of
the nephron. Epithelial
:]lr in this tegiot of the nephron also resp6nd to
antidiuretic hormone (ADH, produced by hypothalamls and releasea uy tt-re
posterior pituitary) which regulates zuater absorption. If. the concentration of ADH
increases, then water will be reabsorbed from the epithelial cells in the coilecting
duct and passed to the iaterstitial space. Removing water from the lumen of thE
collecting duct acts to concentrate the urine. The celis of the collecting duct can
also secrete hydrogen ions (as can the cells of pCT and DCT).

Copyright @ by The Berkeley Review 166 The Berkeley Review


Specializing in MCAT preparation
Biology Gastrointestinal Tract & Kidney Homeostatic Mechanisms

tlomeostatic Mechanisms :

Suppose we want to maintain the levels of sodium in the body. Certain ce1ls in
:re cortex of the adrenal gland are sensiiive to the leveis of soiium in the blood.
',"-lten the concentration of sodium decreases in the blood these cells release the
:.'Jrmone aldosterone. Aldosterone acts at the level of the DCT and the collecting
:-rcts and stimuiates the epithelial cells in those regions to reabsorb sodium. As i
::sult the blood concentration of sodium begins to rise. This acts as a signal and
=eds back on the cortical cells of the adrenal gland telling it to reduce the
s-cretion of aidosterone.

::ppose we want to maintain the proper levels of water in the body. If there has
':en a decrease in the plasma volume of the bodv, there will be a tendency to
- r\-e a lower than normal blood pressure (detected by baroreceptors) but a
- -gher than normal osmotic pressure (detected by osmoreceptors). Receptors
=ceiving this information stimulate specific cells in the hypothalamus to
.', nthesize and transmit ADH to the posterior pituitary where it is released into
:.e blood. ADH stimulates the epithelial cells in the latter portion of the DCT
'":rd the collecting ducts to reabsorb water. This leads to less water excretion and
: righer plasma volume. If you were to drink too much riquid (e.g., pitchers of
:=er), then the opposite process would happen and you would excrete copious
::-Lount of urine. It turns out that the release of ADH is inhibited bv alcohol.

Excretion of Nitrogenous Waste


.:oteins and nucleic acids are the two primary metabolic sources of nitrogenous
,','astes. There are essentially three ways to get rid of the nitrogen ihut i,
::oduced during the metabolism of these compounds. Nitrogen can be removed
.. ammonia/ as urea, or as uric acid. The structures of these compounds are
--- 'rr n in Figure 3-12.

-C
I
H- N'

o,c'ry
NH: H2N- C - NHz H
Ammonia Urea Uric acid
r toxic, soluble) (iess-toxic, soluble) (toxic, insoluble)

Figure 3-12
"' :.iogenous waste compounds.

.--' glutamine is metabolized near the gills of fish, the nitrogen in this amino acid,
:':r be converted to ammonia which can then combine with a proton and, in
=''.:hange for a sodium ion, be carried away with the passing water. The
::lrnonium ion can also be excreted by mammals. one of the by-products of the
:::irno acid glutamine is urea which is carried by the biood to the kidney where it
--. excreted in the urine. Urea is excreted by most mammals, amphibians and

' --rre
reptiles and birds. The ammonia that is produced from the metabolism
-:rm the amino acids glycine, aspartate, and glutamate can be converted to uric
acid which is the excretory product of birds and reptiles.

- rpvright @ by The Berkeley Review 167 The Berkeley Review


Specializing in MCAT Preparation
Biology Gastrointestinal Tract & Kidney Homeostatic Mechanisms

Control of pII and tI@


The hydrogen ion concentration of the extraceliuiar fluid is
closely monitored.
The normal pH of the body is 7.4. This varue can fructuate
between a pH of 7.35
and.7.45.. If the pH of the body decreases to values beiow
7.3s for an extended
period of time, the individual becomes lethargic and death
can result. If the pH is
above 7.45 for extended periods of time, the individual
becomes hyper and ieath
can res-ult. The regulation of brood pH is by means of a buffeiing
system in
which hydrogen ions are reversibly Lound to a buffer like the
bicarbonate ion
(HCo3e), the hemogrobin morecule, or even certain plasma
proteins. For
example, if there are hydrogen ions in the plasma, they can
be taken up by the
bicarbonate ion as shown in equation (3_1) to form carbonic
acid. This is a weak
acid.and decarboxylates to form carbon dioxide and water.
Carbon dioxide is
eliminated by respiration. In fact, the pCo2 levels in the alveoli
of the lungs are
quite important in regurating the prasma pH. This can be seen if
you examine the
(Henderson-Hasselbalch) equation shown in (3-2). The
levels of bicarbonate and
hydrogen ion are controlled by the kidney whereas the levels
of carbon dioxide
levels are controlled by respiration.

H* + HCO3- H2CO3 ---= CO, + HrO


(3-1)
-
pH = pKu + tog tlqqd
- [Coz] (3-2)

Cholera Toxin
S-uppose you had a severe case of diarrhea caused by
the bacterial cholera toxin
(vibrio cholera) that we have previously discussed. This
would result in a
decrea.se in the pH and ur in"r"ur" in the pCo2.
why? Recall that the
gastrointestinal tract secretes a large amount of sodium
bicarbonate. lvhat
happ-ens if you lose a lot of sodium bicarbonate by way
of the gastrolntestinal
tract? Metabolic acidosis results. Look at equation (3_1). If you decrease
the
concentration of HCore, thel by LeChdtelier's principll (from
general
chemistry) Co2 will combine with H2o to make H2co3. This wiil
dissociate into
HCo3, and H@. In other words, the hydrogen ion concentration has increased
or,
if you like, the pH has decreased. This same conclusion can be derived from
equation (3-2).

A decre_ase in pH will stimulate the DCT to secrete hydrogen


ions and reabsorb
as much bicarbonate_ as possible. By a negative reeabalk loop this
tends to
counter the initial decrease in pH. simultaneousry, an increase in pCo2
stimulates the central chemoreceptors in the medulla'and pons
to increase the
contraction of the diaphragm and chest muscles in order to'increase
respiration.
An increase in the respiratory rate will eliminate more Co2 from the lungs.

Copyright @ by The Berkeley Review l6a The Berkeley Review


Specializing in MCAT preparation
GastrointestinaE Tract
& Kfidneys
To Go
15 Passages

100 Questions

Time for All Passages Taken Together as a Practice Exam


125 Minutes

Passage Titles Questions


I. G as tro int e s tinal S mo o th Mus cle 1-5
il. Protein Tuntover 6-12
III. Livef Pancreas, & Intestines 13 -20
IV. Digestion Safeguards 1l
LI
1'7
- L I

v. Digestion o.f Fats 28-33


VI. D i aberi c D i ct Expe rim enr 31-40
vII. The Kidney 4l-48
VIII. Kidnelt & pH 49-53
IX. Dialysis & Ultrcfiltration 54-60
X. CaJfeine 6t-66
XI. Kidney & Calcium 6l -12
xu. Lymph 73 -19
XIII. Renal Clearance 80-86
XIv. G as t ro inte stinal J unc tion 87 -93
XV. Cholera Toxin 94 - 100

Specialtzrng in MCAT Preparation


Suggestions
The passages that follow are designed to get you to think in a conceptual manner about the processes
of physiology at the organismal level. If you have a solid foundation in physiology, many of these
answers will be straightforward. If you have not had a pleasant experience with the topic, some of these
answers might appear to come from the void past the Oort field of the solar system.

Pick a few passage topics at random. For these initial few passages, do not worry about the time. Just
focus on what is expected of you. First, read the passage. Second, look at any diagrams, charts, or
graphs. Third, read each question and the accompanying answers carefully. Fourth, answer the
questions the best you can. Check the solutions and see how you did. \Mhether you got the answers right
or wrong, it is important to read the explanations and see if you understand (and agree with) what is
being explained. Keep a record of your results.

After you feel comfortable with the format of those initial few passages, pick another block of
passages and try them. Be aware that time is going to become important. Generally, you will have about
1 minute and 15 seconds to complete a question. Be a little more creative in how you approach this next
group. If you feel comfortable with the outline presented above, fine. If not, then try different
approaches to a passage. For example, you might feel well versed enough to read the questions first and
then try to answer some of them, without ever having read the passage. Maybe you can answer some of
the questions by just looking at the diagrams, charts, or graphs that are presented in a particular passage.
Remember, we are not clones of one another. You need to begin to develop a format that works best for
you. Keeping a record of your results may be helpful.

The last block of passages might contain topics that are unfamiliar to you. Find a place where the
level of distraction is at a minimum. Get out your watch and time yourself on these passages, either
individually or as a group. It is important to have a feel for time, and how much is passing as you try to
answer each question. Never let a question get you flustered. If you cannot figure out what the answer is
from information given to you in the passage, or from your own knowledge-base, dump it and move on
to the next question. As you do this, make a note of that pesky question and come back to it at the end,
when you have more time. When you are finished, check your answers and make sure you understand
the solutions. Be inquisitive. If you do not know the answer to something, look it up. The solution tends
to stay with you longer. (For example, what is the Oort field?)

The estimated score conversions for 100 questions are shown below. At best, these are rough
approximations and should be used only to give one a feel for which ballpark they are sitting in.

Section III
Estimated Score Conversions
Scaled Score Raw Score

>12 86- 100


10- 11 79-85
8-9 65 -',78
7 59-64
6 54-58
5 48-53
<4 0-47
Biology Gastrointestinal Smooth Muscle Passage I

Fassage I (Questions 1-5) 3. According to the diagram shown below, the force of
smooth muscle contraction is most likely dependent
The smooth muscle ceiis of the gastrointestinal (GI) upon the:
:r:i are typically of the single unit type, roughly 50 to
- -,, microns long and 2-5 microns wide. The action
:,::-ntlals found in smooth muscle are smaller in
L:-:-jtude, but longer lasting than those in neurons. The
i:-s:le cells can be arranged in sheets and can be excited
:, .::ion potentials from neighboring cells. These sheets
:, ::1is also exhibit an intrinsic pattern of periodic
:e:'--larization known as pacemaker activity. Slow waves
: :::'olarization followed by repolarization make up the
rr.es:: :iectrical rhythm. In the presence of excitatory
i:-:; or hormonal input, a slow wave may exceed the
Ilr-::-rld for action potential generation and initiate
:it:-: .- :-r contractions.
),f :,r:1it1, in the GI tract falls into three major
-ri::.::ies. The first is peristalsis, which involves
.:n:r'::ated waves of contraction and relaxation. The
;:-:-: is segmentation, which involves alternating A. size of the action potential.
^ rq--:;::xon and reiaxation of the circuiar muscle layer. B. interval between successive sets of action
-:e -:--d is the mixing motions of villi and microvilli potentials.
r ilrrr::, r3Juce the effect of the unstirred layer of fluid that C. number of action potentials within a set.
i,s, !"._':-'ent to the apical surface of cells. D. time duration of individual action potentials.

4. Segmentation, in contrast to peristalsis, is primarily


responsible for:
l t : t)'pes of smooth muscle, single-unit and multi-
*:--. are distinguishable based on their electrical A. absorption of nutrients from the lumen of the
.u:"'in'. Which of the following statements about GI tract.
::-: :istinction is most likely true? B. movement of intestinal contents through the GI
tract.
d Single-unit fibers are coupled, with many C. mixing of intestinal contents, breaking larger
individual cells containing gap junctions. particles into smaller ones.
ffi* Single-unit fibers are uncoupled, with few D. stimulating the secretion of digestive enzymes
-ndir idual cells containing gapjunctions. from the intestinal exocrine glands.
{--. \Iulti-unit fibers are coupled, with few
individual cells containing gap junctions.
D \Iulti-unit fibers are uncoupled, with many
:idividual cells containing gap junctions.
The mixing of intestinal contents most aids in
digestion because:

A. smaller particles move faster through the GI


tract.
"li lt* ::nduction veiocity of action potentials along B. smaller particles are able to cross the epithelial
, r:':--: rnuscle fiber is low because activation of: layer with increased efficiency.
C. smaller particles carry less charge, enabling
L -:omssium channels is slow.
more efficient passage of particles over
fl. =rdium channels is slow. biological membranes.
r;- :ai:ium channels is slow. D. breaking larger particles into smaller particles
[ :asnesium channeis is slow. increases the surface area available to digestive
enzymes.

,,,.,,rllltfi't'lr'"nn:r ; bt The Berkeley Review t7l The Berkeley Review


Specializing in MCAT Preparation
Biology Protein Turnover Passage II

Passage II (Questions 6-12) 8. The urine carries 80 gm of protein equivalents per


day. The term protein equivalent is used because
The diagram shown in Figure I is a summary of daily protein is degraded and another molecule is usejd as
protein intake and output-turnover-in a 70 kg human. the excretory route for nitrogen. Which of the
fNote: The gastrointestinal (GI) system is equivalent to following is the chemical formula for this excretory
the gut.l product?

Protein Intake
(100 gm)
Body Protein

A"\ht"
(10,000 gm)

Turnover of
White Cells
Body Protein B. NH:
(20 gm)
(250 gm)

o
il
C. H2N- C- NH2

Ahsorbed -i--;--)
lnmino Rctos
----)
(16ogmt
l_ilggd
I

D. H2N- NHz
.skin

Urinary
Fecal
(80 gm protein 9. Rank in increasing order the following organs or
(10 gm protein)
equivalent) tissues according to their amount of daily protein
turnover-
Figure I
A. White cells, gut, liver, muscle.
6. You can see from the diagram that the gut secretes B. Gut, muscle, liver, white cells.
more protein than the person ingests. Which of the C. Muscle, liver, gut, white cells.
following gut secretions contain recyclable protein? D. White cells, liver, muscle, gut.

I. Digestive enzymes
il. Mucus
UI. Bicarbonate
10. Albumin is the most abundant plasma protein. What
A. I only type of experiment could be used to quantify the
B. I and II only albumin turnover rate?
C. II and trI only
D. I, II, and III A. Give a known dose of albumin and sample the
blood periodically to measure albumin
concentration.
7. Given the following equation, what is the efficiency B. Give isotopically labeled albumin, sample the
ofprotein digestion and absorption in the gut? blood periodically, and calculate the decay rate
of labeled albumin.
Protein Absorbed c. Give a known dose of albumin and sample the
Efficiencv
' - Total Protein Available blood periodically to measure albumin decay
rate.
A. 63Vo D. Combine a blood sample with isotopically
B. 77Vo labeled albumin in a test tube and calculate a
C. 9OVo decay rate.
D. 94Vo

Copyright @ by The Berkeley Review 172 The Berkeley Review


Specializing in MCAT Preparation
Biotogy Protein Ttrrnover Passage tr

11. During a fast, which tissue is the first to break down


its protein to provide amino acids for other tissues?

A. Smooth muscle.
B. Cardiac muscle.
C. Enzymes of energy metabolism.
D. Skeletal muscle.

12. The following experiment, a nitrogen balance study,


was performed to determine nitrogen needs in
human subjects. To be in nutrient balance means
that intake - output = zero for that nutrient. For
nitrogen, this means that the nitrogen contained in
protein in food eaten and secreted by the gut equals
the amount of nitrogen excreted in urine, feces, and
through skin losses.

Subjects were placed on 3 different diets


sequentially. The results of their nitrogen balance
are noted in Table 1.

Protein Intake Nitrogen Balance


(gm/kg) (mg/kg)

0.1 -42
0.6 a1

1.5 +12

Table I

What is the amount of protein needed by these


subjects to maintain a nitrogen balance?

A. Less than 0.1 gm/kg.


B. Between 0.1 and 0.6 gm/kg.
C. Between 0.6 and 1.5 gmikg.
D. Greater than 1.5 grn/kg.

Copyright @ by The Berkeley Review 173 The Berkeley Review


Specializing in MCAT Preparation
Biology Liver, Pancreas, and Smalt Intestines Passage Itr

Passage III (Questions 13- 20) 13. Which secretion from the small intestine would be
most appropriate for processing a meal containing a
In controiling the processes of digestion, the pancreas high fat content?
and liver work in tandem to prepare food that exits the
stomach for absorption in the small intestine. Secretion of A. CCK
bile salts from the liver (sodium and potassium salts B. Bile Salts
conjugated to glycine and taurine) is important in the C. Secretin
formation of micelles, water soluble complexes from D. Pancreatic Fluids
which lipids can be more easily absorbed by the intestinal
lining. Secretions from the pancreas contain a highly
alkaline bicarbonate solution that is important in the
neutralization of the hydrochloric acid (HCl) secreted by
the parietal cells of the stomach. Secretion of HCI can be L4, Which of the pancreatic enzymes activated in the
inhibited by gastrin inhibirory pepride (GIp), an small intestine is important for the initiation of the
enterogastrone released by the duodenum. HCI can leave cascade shown in the diagram below?
the stomach and enter into the duodenum through the
pyloric sphincter.
Proenzyme Enzyme
The pancreas also secretes enzymes needed to
breakdown proteins entering the duodenum. All of these
/\
_/\---------.-._

r_l
. Enterooentidase
pancreatic enzymes are secreted as inactive proenzymes. lrlpsrnogen TrYpsin
Trypsin inhibitors are secreted by the same pancreatic
cells to prevent activation of these enzymes. These -+-+
proenzymes travel through the ductal system to the Chymotrypsinog.n, SI5r- Chymotrypsins
duodenum were the enzyme enteropeptidase Trvnsin
(enterokinase) is secreted from the brush border of the ProelastaseJ} Elastase
small intestine. Enteropeptidase initiates the conversion Procarboxypepri6ur", Trypsin-- Carboxypeptidases
of the pancreatic enzymes to their active state.

Bile salts and exocrine secretions of the pancreas are


controlled by secretin and cholecystokinin (CCK), two
A. Bile salts
enterogastrones released from endocrine cells in the upper
B. Secretin
portion ol the small intesrine. C. CCK
D. Enterokinase
Secretin is released from the S cells in the intestinal
mucosa when the pH of the duodenum falls below 4.5.
Secretin release is increased by gastric acid bathing the
lining of the small intestine and products of protein
digestion. Secretin stimulates the ductal cells of the 15. Deficiency of the enzyme enteropeptidase results in
pancreas to release a fluid containing a high concentration
a disorder displaying which of rhe symproms listed
below?
ofbicarbonate ion but a low concentration ofchloride ion.
Secretin, also increases the production of pancreatic
enzymes, decreases acid secretion in the stomach, and
A. Fat deficiency
may cause contraction of the pyloric sphincter.
B. Increases in fat absorption
C. Protein deficiency
CCK secretion is stimulated by peptide, amino acid,
D. Vitamin B12 deficiency
and fatty acid products coming in contact with the
intestinal lining. CCK stimulates the contraction of the
gall bladder, a storage organ that concentrates bile salts
produced in the liver. Furthermore, CCK increases the
secretion of pancreatic fluids rich in enzymes and alkaline 16. Which hormone is important in initiating the
phosphates, inhibits gastric emptying, increases neutralization of gastric secretions in the small
enteropeptidase production, and increases motility of the intestine?
small intestine and colon.
A. Secretin
B. Enterokinase
i. Proenzymes
D. CCK

Copyright @ by The Berkeley Review 174 The Berkeley Keview


Specializing in MCAT Preparation
Biology Liver, Pancreas, and Small Intestines Passage III

17. The enzyme phospholipase A can remove a fatty 20. Antacid tablets, which are mixtures of magnesium
acid residue from lecithin (a component of bile) to hydroxide and aluminum hydroxide, have been
form lysolecithin, a compound that damages cell shown to heal duodenal ulcerations.
membranes. Activation of phospholipase A in the
pancreatic ducts causes damage to the surrounding Mg(OH)2 + 2 HCI + MgCt2 + 2H2O
tissue and can lead to acute pancreatitis, a severe and
often fatal disease. Which of the following catalytic AI(OH)3 + 3 HCI -+ AlCt3 +3H2O
enzyme is most likely involved with this disorder?
Based on the diagram shown below, antacids should
A. Trypsin be taken 1 and 3 hours after a meal because they:
B. Endopeptidase
C. Bile salts
D. Secretin

19. -\s acidic chyme enters the small intestine, secretin o


stimulates the release of the bicarbonate ion from the E

pancreas. The following reaction will result in the a


duodenum:

HCI + NaHCO3 -+ NaCl + H2CO3 J-)


Hours After Eating a Meal
Carbonic acid (H2CO3) will immediately:
A = No Antacid
begin to increase the acidity of the duodenum B = Antacid taken t hour after meal
and activate CCK in order to decrease motilitv C = Antacid taken 1 and 3 hours after meal
of the small intestine and increase absorption.
B. decrease the pH of the intestinal lumen so the
entering chyme can easily be hydrolyzed by A. will allow for additional release of acid to aid
the acidic medium. in food digestion.
C. B. delay the appearance of acid in the stomach.
dissociate into CO2 and H2O in order to allow
C. combine with either magnesium hydroxide or
the NaCl solution in the small intestine to
aluminum hydroxide to form an acid which is
remain neutral.
more concentrated.
D. dissociate into Ho and HCO3o in order to D. produce water as a by-product of the reaction
increase the acidity of the intestinal lumen and and reduce the acidity of the stomach.
prepare it for nutrient absorption.

S:,.ere duodenal ulcers, leading to the removal of


::-: duodenum, will cause an increase in all of the
::;iou'ing EXCEPT:

{- gallbladder relaxation.
B. parietal cell activity.
C. pancreatic ductal cell activity.
D. passage of chyme into the small intestine.

3 by The Berkeley Review 175 The Berkeley Review


Specializing in MCAT Preparation
Biology Digestion Safeguards Passage IV

Passage IV (Questions 21-27) 23. Which parts of the digestive tract are lined with
mucus?
The body is abour 95-98Vo efficient in digesting
protein of animal origin. SinceZ}Vo of the human bodv ii
protein oJ animal origin, we require some sophisticated
I. Mouth
il. Colon
safeguards to avoid digesting our own tissue. UI. Small intestine
First, mucopolysaccharides line the epithelial tissue of A. I only
the digestive tract. Mucus is indigestiblJ and protects the B. I and II
tissues from the actions of acids and enzymes of C. II and III
digestion. Second, the pancreas contains prbteolytic D. I, II, and III
enzymes in an inactive form (zymogens) to avoid
digestion of the pancreatic cells. These zymogens are
released unchanged through the pancreatic duct into the
duodenum. 24, What is the approximate pH of the stomach?

Trypsin, a pancreatic protease, is activated in the A. 6.8


duodenum by the enzyme enterokinase. Trypsin, in turn, B. 0.2
activates the other zymogens by cleaving a portion of the c. 9.0
molecule so that the active site is exposed. A further D. 2.0
mechanism for protection of the GI iract is the rapid
turnover of cells. The epithelial lining of the GI traci is
replaced every 5-7 days. Finally, although the stomach
acidifies the bolus of food considerably-, the pancreas 25. What could happen if enterokinase entered the
secretes bicarbonate to neutralize the pH of the duodenal pancreatic duct and went to the pancreas?
contents.
I. The zymogens would be activated.
II. Pancreatic enzymes would appear in the
bloodstream.
ilI. Insulin secretion would be impaired.

21. Raw soybeans contain a protein that acts as a trypsin A. I only


inhibitor. What would be the digestive effect of B. I and II only
eating a large quantity ofraw soybeans? C. II and III onty
D. I, II, and Itr
A. Trypsin would be more active, but the other
enzymes would remain as zymogens.
B. Trypsin would be inactive, and the other
pancreatic enzymes would function normally. 26. What type of acid(s) do rhe parietal cells of the
C. Trypsin would be inactive, and the otirer stomach secrete?
pancreatic enzymes would remain as
zymogens. L Hydrochloric acid
D. Trypsin and other pancreatic enzymes would u. Sulfuric acid
function normally. IU. Carbonic acid

A. I only
B. I and II only
C. II and III only
22. Chemotherapeutic agents target rapidly D. I, II, and III
growing and
dividing cells. This is beneficial for attacking tumor
cells, but may be detrimental to other body cells.
Which cells would be most adversely affected?
27. To what major class of molecules do
mucopolysaccharides belong?
A. neurons
B. cardiac muscle cells A. Protein
C. gastric epithelium
B. Lipid
D. skeletal muscle ceils C. Carbohydrate
D. None of the above

Copyright @ by The Berkeley Review t76 The Berkeley Review


Speciatizing in MCAT preparation
Biology Digestion of Fats Passage V

Passage V (Questions 28-33) 29. Which of the components of fat digestion is mosl
ciosely associated with principles utilized by
The digestion of fats and cholesterol by the body is household detergents to remove grease?
:rimarily carried out in the duodenum of the small
:rtestine. Triglycerides are the most common fat products A. Emulsification of fats.
:n the diet. The initial process in the digestion of fats is B. Micelleendocytosis.
.le emulsification of fat droplets by bile salts released C. Exocytosis of fatty acids.
-iom the iiver and gall bladder. Biie salt release is D. Packaging of fat products in the endoplasmic
.irmulated by increased leveis of long-chained fatty acids reticulum.
--ontaining more than 10 carbon atoms) within the
:uodenum. Bile salt emulsification of fat is not essential
, rr digestion and absorption since as much as 60Vo of all 30. Sprue, a disorder of malabsorption in the small
.:iel1'cerides can be absorbed without the aid of bile salts. intestine, is characterized initially by the increased
,:olesterol processing, however, is dependent on the soap-like appearance of fat in the stools. Which of
:::lons ofbile salt. the following reasons couid lead to this symptom?

Cel1s within the duodenum sensitive to fatty acids will A. Decrease in chylomicron production.
:iease cholecystokinin (CCK), a hormone which will B. Decrease in bile salt secretion.
.--:nulate contraction of the gall bladder and increase fluid C. Decrease in intestinal cell microvilli.
.:cretion from the pancreas. Once the fats have been D. Increase in pancreatic lipase production.
-rnulsiiled, pancreatic lipase, an enzyme produced from
.:: exocrine pancreas and activated by the enzyme
:-. psin, will hydrolyze the fats into monoglycerides, free 31. What will occur if bile salt secretion is decreased?
...:r'acids, and glycerol. The hydrolyzed products offat
-:rllets are then packaged by bile salts into water soluble A. Increases in cholesterol absorption in the small
--:elles and then transported to the cellular membrane of intestine.
.: intestinal microvilli. The micelles cross the cell B. Complete blockage of fatty acid digestion in
--:mbrane and are incorporated into vesicular packages the small intestine.
--:."lomicrons) in the endoplasmic reticulum. The C. Inhibition of cholesterol absorption in the
-:.,. lomicrons are then exocytosed into the intercellular small intestine.
,:":e and taken up by the lacteals ofthe villi. D. Decreased levels of CCK secretion from the
small intestine.
Chylomicrons also contain triglycerides, cholesterol,
-:-: phosphoiipids. They are transported by way of the
.::rphatics and the capiliary system to the liver. In route 32. Chylomicrons are degraded in the liver. Which of
, tne liver, lipoprotein lipase, an enzyme located in the the following compounds is a component of a
-,:rllaries, strips triglycerides from the chylomicrons for chylomicron as it enters into the liver to be
:.rra-ee in adipose tissue. The remaining components of degraded?
-,: chylomicrons are degraded in the liver and stored for
:l:: production of steroid hormones, VLDLs, or excreted A. Fatty acids.
:.:r bile salts into the intestine. B. Biie salts.
C. Lipoprotein iipase.
D. Cholesterol.

:8. A trypsin antagonist is present during fat


metabolism. Which of the following processes will 33. Disorders characterized by increases in fat
NOT occur? deposition in the liver may be produced by one of
the abnormalities below.
A. Emulsification of fats.
B. Increase in fat content in the small intestine. A. Decreased leveis of pancreatic lipase.
C. Hydrolyzation of fat droplets in the small B. Decreased leveis of CCK.
intestine. C. Decreased levels of lipoprotein lipase.
D. Decrease in CCK production. D. Decreased levels of chylomicrons.

- rpvright @ by The Berkeley Review t77 The Berkeley Review


Specializing in MCAT Preparation
Biology Diabetic Diet Experiment Passage VI

Passage VI (Questions 34-40)


Table 2: Plasma lipid and lipoprorein levels
during the study.
Diet control is important in non_insulin-dependent
diabetes mellitus (NIDDM), not only to improve control Measurement Base. ADA HF
of hyperglycemia but also to reduce the risk of coronary (mg/dl,) line Diet Diet
heart disease by improving plasma lipid levels. Total Cholesterol )) \+ 20s r96
Triglycerides 285 218 631
The American Diabetes Association (ADA) currently 1

recommends the following diet for patients wittr XmnU: VLDL Cholesterol 58 43 28$
LDL Cholesterol 135 131 134
. 30Vo of energy as fat (with l\Vo as saturated fat) HDL Cholesterol 32 30
. 40-50Va complex carbohydrates (starches) t P < 0.02 for baseline versus
34$
ADA and HF values
. less than l}Vo simple carbohydrates (sugars) t P < 0.01 for ADA versus HF values.
$ P < 0.005 for ADA versus HF values.
Lastly, protein makes up l0-20Va of the diet's energy.

Some studies indicate that diets like the ADA diet, that
are high in carbohydrates, Iead to increased triglyceride
levels, increased very low density lipoprotein tevels,
and
reduced high density lipoprotein i"u"tr. These would
not
be beneficial in a diabeiic population concerned with 34. Which of the following statements are supported by
maintaining optimal plasma lipid levels. the data in Table 1?

following experiment was designed to test the The HF diet lowered plasma glucose in these
^^Theof the ADA
effect diet versus a higher rionounsaturated subjects.
fat diet (HF) on plasma glucose and-tipid levels. II. Patients required more insulin on the HF diet
than at the baseline measurement.
IIr. I-ess glucose was excreted in the urine
during
the HF dier compared to the baselinE
Experiment I measurements.
After a baseline measurement of 1 week, l0 NIDDM
patients ate the ADA diet and the HF diet for 5 weeks A. I and III only
in
random order during 2 hospital admissions. Metabolic B. I and II only
mea-surements regarding glucose (Glc) are outlined
in C. II and III only
Table i. lNote: Hbrefers iohemogiobin.l D. I, II, and III

Table 1: Merabotic variables during the study.


35. In Table 1, glycosylared hemoglobin (G_Hb) is
B ase. ADA HF reported as an indicator of glucose metabolism in the
Variable line Diet Diet body- Glycosylated meanJthat glucose residues are
Plasma Glc (mg/dl-) 129 117 101* attached. G-Hb decreases on both the HF and the
Urinary Gtc (mg/day) 550 142 0t ADA diets during the study. Is this decrease
Insulin Requirements beneficial or detrimental?
84 81 70t.
(unirs/day)
Glycosylated Hb (Vo)
A. This decrease is detrimental, because
11.3$ 7.8 8.1 glycosylated hemoglobin indicates persistent
i P < 0.02 for HF versusADA and baseline. levels of low plasma glucose.
t P<0.0i for baseline versus HF values. B. This decrease is detrimental . because
$ P < 0.005 for baseline versus HF and ADA values. glycosylated hemoglobin indicates persistent
levels of high plasma glucose.
C. This decrease is beneficial. because
glycosylated hemoglobin indicates persistent
Metabolic measurements on plasma lipids and levels of low plasma glucose.
lipoproteins are outlined in Table 2. D. This decrease is beneficial, because
glycosylated hemoglobin indicates persistent
levels of high plasma glucose.

Copyright O by The Berkeley Review 178 The Berkeley Review


Specializing in MCAT preparation
Biotogy Diabetic Diet Experiment Passage VI

-16" The table shown below indicates the fatty acid 39. Some patients with NIDDM maintain good glucose
composition of both ADA diet and HF diet in Vo control using drugs that increase insulin secretion.
energy intake/day. What is the target of these drugs that affect insulin
secretion?
Fatty ADA HF
Acid Diet Diet A. The alpha cells of the pancreas.
2:0 0.2 0.2
B. The exocrine cells of the pancreas.
C. The beta cells of the pancreas.
4:0 0.2
0.3 D. Both the exocrine and beta cells of the
6:0 6^8 7.6 pancreas.
6: I u.l 0.4
8:0 2.4 2.2
8:i t.4 29.4
8:2 '7.2 8.0 40. Insulin is secreted from the pancreas into the hepatic
8:3 0.0 0.2 portal vein. Which is the first major organ insulin
204 0.2 0.2 encounters after entering this portal system?
22:0 0.3 1.5
49.9
A. Small intestines
Total: 25.0
B. Stomach
Table 3
C. Brain
D. Liver

Which of the fatty acids was used to supplement the


monounsaturated portion of the HF diet ?

A. Palmitoleic acid
B. Stearic acid
C. Oleic acid
D. Palmitic acid

37. Which of the following oils or fats could be used to


increase the amount of monounsaturated fat in a
diet?

A. Olive oil
B. Lard
C. Butter
D. Beef ta1low

38. Which of the following statements is/are supported


by the data in Table 2?

L Both VLDL and LDL were lower in the HF


diet compared to the ADA diet.
IL Triglycerides were significantly lowered by
the HF diet compared to baseline.
III. HDL cholesterol was increased by the HF diet
compared to the ADA diet.

A. I and III only


B. I and II only
C. II and III only
D. I, II, and trI

Copyright @ by The Berkeley Review t79 The Berkeley Review


Specializing in MCAT Preparation
Biotogy The Kidney Passage Vtr

Passage VII (Questions 41-48)


Antidiuretic hormone (ADH) and aldosterone are both
regulators of urine composition. Both hormones
The functional unit of the human kidney is the nephron promote
NaCl reabsorption in the ascending limb of the
(Figure 1), and each kidney contains approximately t loop of
million of these tubular structures.
Henle and in the DCT and cofecting ducts. ADH
promotes water reabsorption in the DCT-
and collecting
ducts while aldosterone promotes Ko secretion
in the
DCT.
Efferent
afteriole Glomerulus

41. Constriction of the efferent arterial will lead to:

A. a. decrease in glomeruiar pressure, followed


first by a decrease and then by an increase in
glomerular filtration rate.
B. al increase in glomerular pressure, followed
first by an increase and then by a jecrease in
glomerular fi ltration rate.
o
c C. a. decrease in glomerular pressure, followed
@
o
first by an increase and then by a decrease in
glomerular fil tration rate.
0c
D. an increase in glomerular pressure, followed
first by a decrease and then by an increase in
glomerular fil tration rate.

Figure I

42. The re-absorption of sodium across the epithelial


Blood is supplied to each kidney by a renal artery. The cells of the proximal convoluted tubule and into the
smallest branch of this artery is the afferent arteriole, a peritubular capillary would be significantly reduced
ves.sel-influenced by sympathetic stimulation. A capillary
by'
bed (glomerulus) is fbrmed from the afferent arteriole and
is contained within Bowman's capsule. The efferent A. reduced secretion of ADH from the posterior
arteriole that leaves the glomerulus iJ further divided into pituitary.
a capillary system that surrounds different portions of the
B. an increased rate of secretion of aldosterone
nephron.
from the adrenal cortex.
C. reduced plasma glucose concentrations.
_ The plasma ultrafiltrate that passes through the D. an increase in the tubular colloid
glomerulus and into Bowman's capsule is essentially osmotic
pressure.
protein-free and devoid of cellular structure. This filtrate
flows inro the proximal convoluted tubule (pCT), through
the loop of Henle and into the distal convoluted tubule
(DCT), through rhe collecting ducts and inro rhe renal
p:lytt before making its way to the urerer and urinary
bladder. 43. Sympathetic stimuiation of the kidneys primarily
affects the:
The PCT reabsorbs essentially all of the glucose and
amino acids fiitered by the glomerulus and r6ughly 70Vo A. afferent arterioles and decreases the volume of
of the liltered Ko, Nao, Cle, and water. Approximately urine flow.
2_OVo of the Ke, Nao, Clo, and water ttrat passes
from the B. afferent arterioles and increases the volume of
PCT into the loop of Henle is reabsorbed it the loop. The urine flow.
remaining 10Vo of the Ke, Nao, and Cle is reabsorbed at C. efferent arterioles and decreases the volume of
the DCT. Variable amounts of water also reabsorbed at urine flow.
the DCT D. efferent arterioles and increases the volume of
urine flow.

Copyright @ by The Berkeley Review lao The Berketey Review


Specializing in MCAT preparation
Biology The Kidney Passage Vtr

44. Which of the following graphs BEST represenrs the


46, All of the following
plasma osmolarity (X-axii) of the blood and the statements about aldosterone are
true EXCEpT it:
plasma concentration of antidiuretic hormone (y_
axis)?
A. promotes sodium reabsorption in the distal
convoluted tubule.
A. B. B. promotes the secretion of potassium into the
lumen of the distal convoluted tubule.
C. is a steroid hormone secreted by the adrenal
q a medulla.
X x D. is the major mineralocorticoid in humans.
I I

X-Axis X-Axis 47. Excessive aldosterone secretion will result in all


of
the following EXCEpT:
C. D.
A. hyperpolarization of nerve and muscle
membranes.
B. increased muscle contraction.
a
X
q C. an excessive loss of potassium ions from the
X extracellular fluid.
I
I
D. hypokalemia.

X-Axis X-Axis
48, Which of the following subsrances is filtered by the
glomerulus and into Bowman's capsule?
Which of the following graphs best represents the
plasma concentration of glucose A. Platelets
1X_aiis) and the B. Proteins
rate ofglucose excretion in the urine (y-axis)?
C. Electrolytes
D. Erythrocytes
A. B.

o
x X

X-Axis

C. D.

x X
I
I

X-Axis X-Axis

Copyright @ by.The Berkeley Review l8l The Berkeley Keview


Specializing in MCAT preparation
Biology Kidney & ptl Passage VItr

Passage VIII (Questions 49-53)


Tubular Interstitial
Most metabolic reactions are highly sensitive to the pH Lumen Fluid
of the fluid in which they occur. Due to this sensitivity,
the hydrogen ion concentration of body fluids is closely Filtered
regulated. J
II

Buffering systems, both intracellular and extracellular,


act to minimize changes in the hydrogen ion
concentration. However, these buffering systems do not
have the ability to eliminate or retain hydrogen ions from
the body. This role is left for the kidneys, which
,HCO:'

-t"
HrCOr
'\l H,COT
H9o-r

accomplish this regulation in two ways. The first is by 1.^


II'O + CO2
altering the secretion of hydrogen ions and the second is Ir.20 + Q,O,
by altering the reabsorption of bicarbonate. Hydrogen ion
excretion and bicarbonate reabsorption are both achieved
by the secretion of hydrogen ions.

CA = Carbonic Anhvdrase

Tutrular lnterstitial
Lumen Fluid Figure 2

In the body, the events shown in Figure I and Figure 2


NHr HrO + COo occur together, producing a situation where a high rate of
hydrogen ion secretion achieves complete reabsorption of
t.o- filtered bicarbonate.

=(l
49. Which of the following situations will NOT produce
Urine CA = Carbonic Anhydrase a gain of hydrogen ions?

A. An increase in the concentration of CO2.


Figure I
B. Production of lactic acid during severe
exerclse.

According to Figure i, the hydrogen ion to be secreted


C. Excessive vomiting.

into the lumen is generated in the tubular cell rather than


D. Loss of bicarbonate due to diarrhea.
coming from the blood. Once in the lumen, the hydrogen
combines with a buffer and is secreted in that form. In
Figure 1, the bicarbonate is transported to the interstitial
fluid.
50. The events shown in Figure 1 and Figure 2 MOST
The hydrogen ion in the lumen can also bind with likely illustrate renal compensation for:
filtered bicarbonate to form CO2 and H2O according to
Figure 2. This is in essence bicarbonate reabsorption. A. a decreased blood hydrogen ion concentration.
The net result is that filtered bicarbonate disappears while B. an increased blood hydrogen ion concentration.
bicarbonate (the result of intracellular events) appears in C. a decreased sodium ion concentration.
the blood. This situation is equivalent to one where D. an increased sodium ion concentration.
filtered bicarbonate is reabsorbed back into the blood.

Copyright @ by The Berkeley Review ta2 The BerkeleY Review


Specializing in MCAT Preparation
Biology Kidney & pll Passage VItr

51. The transport of a hydrogen ion from a tubular cell


to the tubular lumen shown in Figure I represents:

A. simple diffusion.
B. facilitateddiffusion.
C. primary active transport.
D. secondary active transport.

52. Respiratory alkalosis is best characterized by :

A. an increase in the hydrogen ion concentration,


resulting from an increase in the level of CO2.
B. a decrease in the hydrogen ion concentration,
resulting from a decrease in the level of CO2.
C. an increase in the hydrogen ion concentration,
resulting from a decrease in the level of CO2.
D. a decrease in the hydrogen ion concentration,
resulting from an increase in the level of CO2.

53. During metabolic acidosis, the partial pressure of


CO2 can be expected to:

A. increase, due to increased ventilation.


B. increase, due to decreased ventilation.
C. decrease, due to increased ventilation.
D. decrease, due to decreased ventilation.

Copyright @ by The Berkeley Review ra5 The Berkeley Review


Specializing in MCAT Preparation
Biology Dialysis and UltrafiItration Passage IX

Passage IX (Questions 54-60) 55. Which structure of the kidney performs


ultrafiltration?
Dialysis is a process used to separate molecules in
solution based on their size. Semipermeable membranes A. Peritubuiarcapillaries
that have pores smaller than the macromolecule of interest B. Loop of Henle
are used to contain one or more large molecules. Dialysis C. Afferent arterioles
tubing is filled with the solution of interest, and the ends D. Glomerularcapiliaries
of tubing are tied. The package is immersed in a large
quantity of solvent. Diffusion of molecules smaller than
the pores of the tubing allows exchange of solvents, salts,
and small metabolites between the solution in the tubing
and the surrounding solution. This can be used to remove
excess salts, for example, from a protein to be purified.

Ultrafiltration is a related technique that is used to 56. A student is purifying an enzyme in the laboratory
concentrate macromolecules. Pressure is used to force the by overnight dialysis. The student tests for
solution through the semipermeable membrane. The successful dialysis using an enzyme activity assay.
solvent and small molecules pass through the membrane, What would rhe srudenr find if the dialysis tubing
while a more concentrated solution of macromolecules is was punctured with a pin before the experiment?
left behind. Normal kidneys perform ultrafiltration
constantly in the removal of waste products from the A. No enzyme activity in the bag.
blood. The entire blood supply of 5-6 liters is filtered B. Increased enzyme activity in the bag compared
approximately every 45 minutes. to the activity measured before the experiment.
C. There is no effect caused by the pinhole, and
Diseased kidneys require exterior dialysis support to the enzyme activity in the bag is the same as
prevent dangerous increases in the plasma solute load, before.
including, salts, electrolytes, and urea. Uremia is the D. Slightly decreased enzyme acrivity in the bag
condition of having an increased urea plasma compared to the activity measured before the
concentration. experiment.

Dialysis support is conventionally performed in one of


two ways. The first involves passing the blood through a
machine containing semipermeable membranes that allow
waste products to diffuse out. An in vivo technique uses
the patient's peritoneal membranes (in the abdominal
cavity) to form an intraabdominal dialysis system. A
sterile fluid is instilled through an abdominal catheter, 57. Which of the following hormones would most likely
allowed to remain from 15 minutes to t hour, and drained be supplernented during chronic kidney disease?
through a second abdominal catheter. This process is
repeated many times. Both methods are time-consuming, A. Erythropoetin
but they allow the necessary removal of waste products B. Antidiuretic hormone (ADH)
from the blood. C. Epinephrine and norepinephrine
D. Insulin

54. What complications could result from the peritoneal


dialysis procedure?

I. Increased risk ofabdominal cavity infections.


II. Changes in electrolyte balance.
nI. Kidney damage by ultrafiltration. 58. What would be a physiological effect of uremia?

A. I only A. Increased blood pH


B. I and II only B. Edema
C. II and III only C. Increased urination
D. I, II, and III D" Alkalosis

Copyright @ by The Berkeley Review ta4 The Berkeley Review


Specializing in MCAT Preparation
Biology Dialysis and Ultrafiltration Passage IX

59. When a dialysis machine is used to clean blood,


what precautions are taken to avoid loss of glucose
and salt?

A. Use of a dialysis membrane that retains


glucose and salts.
B. Use a dialysis solution that is isotonic to the
blood in glucose and salts.
C. The patients drinks an electrolyte and glucose
drink during dialysis.
D. Nothing can be done to avoid loss, so the
patient has an IV of saline and glucose at the
same time.

60. If a person has 5 liters of blood and it is passed


completely through the kidneys every 45 minutes,
what is the glomerular filtration rate?

A. 225 mUmln
B. 111ml/min
C. 66 ml/min
D. 9 ml/min

Copyright @ by The Berkeley Review la5 The Berkeley Review


Specializing in MCAT Preparation
Biology Caffeine Passage X

Passage X (Questions 61-66) 62. Which of the following hormones would most
increase plasma FFA concentration ?
Caffeine is a commonly used beverage in the U.S.
The average individual consumes 2-3 cups per day, with A. CCK
some people drinking 10-15 cups. Coffee contains about B. Thyroid hormone
150 mg caffeine/cup for drip-style makers. Caffeine is the C. Epinephrine
physiologically active agent in coffee. The following D. Insulin
experiments were carried out to examine the effects of
caffeine in coffee on free fatty acid (FFA) metabolism:
63. In Experiment 4, how did the sucrose lead to lower
All
subjects followed on overnight fast. At 8:00 AM, FFA concentrations?
each subject drank one of the following beverages:
A. Sucrose was digested to glucose and fructose;
Experiment l: fructose mediated the effects on FFAs.
500 mL hot water containing 5 g instant coffee (250 B. Sucrose stimulated glucagon, which decreased
mg caffeine) and 3 tablets of sodium saccharin (15 release offatty acids from adipocytes.
mg). C. Sucrose acted as a competitive inhibitor of
caffeine.
Experiment 2: D. Sucrose stimulated insulin, which decreased
500 mL hot water containing 5 g decaffeinated coffee release offatty acids from adipocytes.
(20 mg caffeine) and 3 tablets of sodium saccharin
(15 mg).
64. The action of caffeine is mediated through increased
Experiment 3:
cAMP levels in target cells. What is the enzyme that
500 mL hot water containing 3 tablets of sodium is activated by second messengers to make cAMP?
saccharin (15 mg)

Experiment 4:
A. Epinephrine
B. G protein
500 mL hot water containing 5 g instant coffee (250 C. Adenylate cyclase
mg caffeine) and 25 g sucrose D. Proteinphosphorylase

Blood samples for all experiments were taken before


the start of the experiment and at 1, 2, and 3 hours 65. From Experiment 2, what would you conclude about
following ingestion of the test beverage. The following the effects of the caffeine present in decaffeinated
table follows the FFA concentrations throughout the coffee on plasma FFA concentration?
study:
A. There are slight increases in FFA con-
Expt. 0 hours t hour 2 hours 3 hours centration due to the 20 mg of caffeine present
I 0 +1 82t +245{ +384+
B. The effects are the same as drinking coffee
with 25 g sucrose.
2 0 16x +18* +93* C. There are slight decreases in FFA con-
centration due to the 20 mg of caffeine present.
--) 0 19 -59 +11 D. The effects are the same as drinking water.
4 0 -e6I -e0t +1201
* Not signihcant compared to Experiment 3 66. What was the point of Experiment 3?
{ Signihcant compared to Experiment 2 and Experiment 3

f Signihcant compared to Experiment 1 I. To determine the effects of hot water


Table 1: Mean Changes in Plasma FFAs (ttEQ/L)
il. To determine the effects of sodium saccharin
III. To establish control data

61. Which of the following mechanisms is the most A. I only


important in raising plasma FFA concentrations? B. II only
C. I and II only
A. Lipolysis I, II, and Itr
B. Lipogenesis P.
C. Increased fat oxidation
D. lncreased dietary fat

Copyright @ by The Berkeley Review ta6 The Berkeley Keview


Specializing in MCAT Preparation
Biology Kidney & Calcium Passage XI

Passage XI (Questions 67-72) 68. According to the diagrams in Figure l, all of the
following statements concerning substances X, y.
Kidney function can be charactefized by three andZ are false EXCEpT:
:rLrcesses. The first involves movement of an essentialiy
: liein free plasma from renai glomerular capillaries into
! r'.iman's capsule. This is referred to as giomerular
. -i::tion. Tubular reabsorption refers to the movement of Afferent Efferent
: -::lances from the renal tubular lumen to the peritubular
Arteriole Arteriole
-.;:-iary' plasma. Movement in the opposite direction is // ./
'.-:r:d tubular secretion.
The kidney, among its many
-, -',6fl with reguiating levels ofother functions, is
calcium ion. The
, :,.,-e11u1ar calcium concentration is normally held Peritubuiar
' : :.r'. 3ly
constant. However, a low calcium ion Capillary
- .::lrration increases the excitability of nerve and
- ---..' .'ell membranes.

.-.1;rum homeostasis is achieved through the Urine


nins of three major effector sites. These sites are
- -. : -- :
: l:e\'. bone, and gastrointestinal (GI) tract. Almost
^ :..
" ' : - i total-body calcium is contained in
bone, which is
. :.::: * ork of organic molecules upon which calcium
- ' ):.-rie crvstals are deposited. Bone is a dvnamic
tissue
" -. ran either remove or deposit calcium into
the
, ..-:--ular fluid. In the Gi tract, control of active
--.-:: :t svstems that move calcium into the blood can
: - .: larqe fluctuations in the amount of calcium
"" : .:::J. The kidney modulates calcium levels by
" :, :::'on of both calcium and phosphate ions.
:- '--t:3e effector sites are under control of parathyroid
-- -:. uhose plasma concentration fluctuates directly
1 ,: '_sr ro changes in plasma calcium
levels.
Figure i

A. Z is filtered and partially reabsorbed.


B. Y is totally reabsorbed.
C. X is filtered and secreted, but not reabsorbed.
D. X, Y, and Z are all secreted.

- :he following statements about glomerular


::::t t,l are true EXCEPT:

-rlid pressure in Bowman's capsule


69. In response to a decrease in plasma calcium leveis,
opposes one would most iikely expect to see:
-.-rration.
rsnotic fbrce due to protein in plasma favors
A. increased plasma parathyroid levels, with
__-rat10n.
increased urinary calcium excretion.
_:-omerular capillary blood pressure favors increased plasma parathyroid levels, with
:-."ration.
increased removal of bone.
- -,:ces favoring filtration are larger than those
decreased plasma parathyroid levels, with
::osing filtration.
decreased urinary calcium excretion.
decreased plasma parathyroid levels, with
decreased removai of bone.

ta7 The Berkeley Keview


Specializing in MCAT preparation
Biology Kidney & Calcium Passage XI

70, According to information in the passage, it can be


concluded that:

A. the concentration of extracellular calcium in no


way effects neuromuscular excitability.
B. persons with low extracellular calcium
concentrations may suffer total skeletal muscle
spasms.
C. persons with increased extracellular calcium
concentrations may suffer from frequent
muscle spasms.
D. the effect of extracellular calcium
concentration on membranes is distinct from
its role as a mediator of muscle contraction.

71. A decrease in calcium will most likely cause:

A. an increase in the extracellular concentration


of phosphate.
B. no change in the extracellular concentration of
phosphate.
C. a decrease in the urinary excretion of
phosphate.
D. a reduction in the tubular reabsorption of
phosphate.

72. Parathyroid hormone is secreted by the:

A. anterior pituitary.
B. posteriorpituitary.
C. parathyroid gland.
D. thyroid gland.

Copyright @ by The Berkeley Review ta8 The Berkeley Review


Specializing in MCAT Preparation
Biology Lymph Passage XII

Passage XII (Questions 73-79) 75. Fat-soluble vitamins are carried from the cells of the
small intestine into the blood system via the lymph.
The lymphatic system has 3 functions: (1) its They are packaged into chylomicrons and with fatty
.''mphocytes provide immunological defenses, (2) it acids. Which of the following vitamins are fat-
'iansports absorbed fat from the small intestine to the soiuble vitamins?
:.ood, and (3) it transports interstitial fluid back to the
:1ood. I. Vitamin A
il. Vitamin K
Both lymph nodes and lymphocytes of the immune nI. Vitamin E
-... stem play important roles in immunological defense.
j -' mph nodes contain phagocytic cells through which A. I only
-'.mph is filtered before returning to the blood supply. B. II only
l'. mph nodes also contains germinal centers that produce C. II and III only
-'.mphocytes, which are present in the lymph and in the D. I, II, and III
:-:sma.
76. What is the function of the lymph node?
The lymphatic system also carries dietary fat from the
:rall intestine to the blood supply. Absorbed fat, A. To destroy microorganisms.
r:luding long chain fatty acids and cholesterol, along B. To filter chylomicrons out of the iymph.
.'" ith fat-soluble vitamins, are packaged into
chylomicrons C. To produce chylomicrons.
:t, the cells of the small intestinal mucosa. These D. To add intestinal nutrients to the lymph.
::r'lomicrons transport fat from the intestine to the
blood circulation. Other nutrients from digestive What force moves the lymph inside the lymphatic
=:neral
:iocesses in the small intestine travel first to the liver via vessels?
:,e hepatic portal vein.
I. Squeezing by the muscles.
The liquid part of lymph is an ultrafiltrate of plasma, II. Contraction of fhe muscular layer of the lymph
: -'led interstitial fluid. This is the fluid that is squeezed vessels.
:,, of capillaries under pressure. Interstitiai fluid enters III. Blood pressure.
q
:: ivmph capillaries through its endotheliai cells which
:;\'e porous junctions. This fluid, along with A. I only
::', iomicrons, microorganisms, and lymphocytes, is then B. I and II only
,z.led lymph. From the lymph capillaries, the lymph C. II and III only
r-rves into the lymphatic vesseis, which have a 3-layer D. I, II, and III
. -iucture simiiar to veins. The lymphatic system
-,:imately returns the fluid to the cardiovascular system The term ultrafiltrate means fluid formed by the
. r" the right and left subclavian veins. hydrostatic pressure of the blood acting against a
semi-permeable membrane. Interstitial fluid is one
example of an ultrafiltrate. Which of the foliowing
organs also produces another ultrafiltrate?
-3. What is the consequence of the chylomicron system
of fatty acid packaging? A. Liver
B. Kidney
A. Muscle and adipose tissue have first access to C. Pancreas
dietary fatty acids from chylomicrons. D. Salivary gland
B. Liver tissue has first access to dietary fatty
acids from chylomicrons. 79. Which of the following statements are TRUE of
C. Microorganisms are packaged into both lymph and plasma?
chylomicrons.
D. Interstitial fluid carries chylomicrons. I. Both contain chylomicrons after a meal.
[I. Both contain lymphocytes.
-{. [II. Both contain erythrocytes.
What is the major component of lymph?
A. I only
A. Fat-soluble vitamins B. I and II only
B. Chylomicrons C. II and III only
C. Water D. I, and III
II,
D. Triglycerides

-rrpyright O by The Berkeley Review la9 The Berkeley Review


Specializing in MCAT Preparation
Biology Kenal Clearance Passage XIII

Passage XIII (Questions 80-86) 81. What is the volume of urine that could be collected
in 30 seconds if the [ ]u of substance Y is 50 mg/ml,
The concept of renal clearance, based on the principle the [ ]O of substance Y is 1 mg/ml, and the clearance
of mass balance, is a means to quantify the excretory for substance Y is 50 ml/min?
function of the kidney. The relationship of clearance (C)
to the concentration of a substance in the urine ([ ]r), the A. 0.00025 L
urine flow rate (Fu), and the plasma concentration of a B. 0.0005 L
substance ([ Jp) is C. 0.001 L
D. 0.0015 L
c=([]u)x (Fu)/ []p

The glomerular filtration rate (GFR) is the volume per


unit time that enters the kidney from the plasma. Under
certain conditions, the amount of a substance filtered is
equal to the amount excreted in the urine. To calculate
the amount filtered, one can multiply the GFR by the
Isubstance]0. 82. Which of the following equations represents the
glomerular filtration rate (GFR)?
Similar to the plasma, not all of a substance traveling
through the renal artery is filtered at the glomerulus. The A. GFR = [substance]p x []u x Fu
fraction of plasma filtered can be calculated as the ratio of B. (GFR) x (Fu) = ([ substance Jp) x ([ Ju)
GFR to renal plasma flow (RPF). The fraction normally C. GFR = (l Ju x Fu) / ([ subsrance ]o)
takes on values of 75Vo to 20Vo. D. (GFR) x ([ ]u) x ([ Jp) x (Fu) = 1

The renal handling of metabolites may be instrumental


in diagnosing certain clinical disorders such as diabetes.
When determining the amount of glucose reabsorbed in
the nephron, one must subtract the amount excreted from
the amount filtered at the glomerulus. The amount of
glucose reabsorbed by epithelial cells of the tubule can be
thought of as having a maximum transport rate, known as 83. According to the passage, the amount of a substance
the tubular transport maximum. The [plasma glucose] filtered is equal to the amount excreted in the urine
will ultimately determine whether glucose is excreted or is only under certain conditions. Which of the
1007o reabsorbed. The level of plasma glucose which following conditions CANNOT exist for this
first gives rise to glucose in the urine is termed the plasma statement to be TRUE?
threshold.
A. The substance has no barrier to filtration at the
glomerulus.
80. Mass balance describes the principle that the mass of B. The substance must not be reabsorbed and
a substance entering the kidneys must equal the must be secreted by the nephron.
mass of that substance leaving the kidneys. Using C. The substance must not be metabolized or
the following key, and information from the passage, produced by the kidney.
which relationship BEST describes this principle? D. The rate of glomerular filtration cannot be
altered by the substance.
Key

[ ]A = concentration of substance in renal artery

[ ]V = concentration of substance in renal vein

PFo = plasma flow rate in renal artery

PFy = plasma flow rate in renal vein 84. Glucose is reabsorbed through the epithelial cells of
the:

A. []e x PFy = ([ ]u x Fu) + ([ ]y x PF4) A. proximal convoluted tubule.


B. ([]n x PFu) + ([]u x Fo) = []v x PFa B. loop of Henle.
c. ([]e x PFa) + ([]ux Fu) - []y xPFy C. distal convoluted tubule.
D. []r x PF4 = ([ ]u x F") + ([ ]u x PFv) D. collecting duct.

Copyright @ by The Berkeley Review 19() The Berkeley Review


Specializing in MCAT Preparation
Biology Kenal Clearance Passage XIII

$5. According to the graph beiow, the plasma threshold


ls:

a^
;E
FE"

Ho
vro
E<
o
J
0246810
Plasma Glucose (mg/ml)

-{. 3.5 mg / ml.


B. 7.6 mg / ml.
C. 380 mg / ml.
D. 400 mg / ml.

The following graph illustrates the influence of size


and charge on a substance's filterability at the
glomerulus. Which of the following statements is
most likely true based on this graph?

.:. 0.8
-o
b 0.6
ii
9 0.4
6
O Polyanionic
& 0.2
dextran

18 22 26 30 34 38 42
Effective Molecular Radius (A)

The cationic species has the highest


filterability, because cationic atoms are smaller
than anionic atoms.
The cationic species has the highest
filterability, because of the presence of anionic
glycoproteins on the surface of all glomerular
filtration components.
C. The anionic species has the highest
filterability, because cationic atoms are larger
than anionic atoms.
The anionic species has the highest
filterability, because of the presence of
cationic glycoproteins on the surface of all
glomerular filtration components.

Copyright @ by The Berkeley Review 191 The Berkeley Review


Specializing in MCAT Preparation
Biology Gastrointestinal Junction Passage XIV

Passage XIV (Questions 87-93) 89. It is known that a hypertonic solution in the
duodenum decreases the rate of gastric emptying.
The pyloric sphincter separates the terminal portion of This mechanism prevents a:
the stomach (gastric antrum) fiom the beginning segment
of the small intestine (duodenum). This sphincter, which A. rise in the blood pH.
consists of a ring of connective tissue preceded by two B. decrease in the blood pH.
distinct rings of circular smooth muscle, carries out two C. rise in the biood volume.
important functions. First, it heips to regulate the rate of D. decrease in the blood volume.
gastric emptying into the duodenum. The transfer of the
stomach's contents to the duodenum is dependent on the
duodenum's ability to process chyme. Second, it prevents
the reflux of small intestinal contents into the antrum. 90. 100 mM HCI was injected into the duodenum of a
These two functions are controlled by both hormones and dog while the contractiie activities of both the
neurai input. Norepinephrine and cholecystokinin (CCK), antrum (AC) and duodenum (DC) were measured.
for example, both cause a constriction of this sphincter. Which of the following graphs BESTS represents
the expected results of the experiment?
It has been discovered that several distinct mechanisms
can lead to an alteration in the rate of gastric emptying. A A.
lowering of the pH in the duodenum has been shown to
slow gastric emptying. This decrease in pH has been
shown to not only affect motility, but also increase the
release of bicarbonate ion into the small intestine via the
DC
hormone secretin. In addition to pH, the presence of fat
results in a retarded gastric emptying. This response is
believed to be carried out mainly by the hormone CCK. t t
The hormone gastric inhibitory peptide (GIP) is also HCL Infusion HCL Infusion
Starts Ends
believed to be a component of this mechanism, as GIP has
been shown to decrease the rate of gastric emptying. B.
Finally, the secretion of gastrin lowers the rate of gastric
emptying. Gastrin is secreted in response to the presence
of amino acids and peptides in the stomach. lllllililtililtililtI
|ilrililtlliltIlII
87. Based on information in the passage, which of the
t
HCL Infusion
i
HCL Infusion
following statements is TRUE?
Starts Ends

A. A low rate of gastric emptying may lead to C.


duodenal ulcers.
B. A high rate of gastric emptying may lead to IIIII ililililililil1ililil IIrI I
AC
stomach ulcers.
C. Regurgitation of duodenal contents may lead DC
to duodenal uicers. ilililililt I I I I I I I I I ilillllllll
D. Regurgitation of duodenal contents may lead
to stomach ulcers.
t
HCL Infusion
t
HCL Infusion
Starts Ends
D.
88. Norepinephrine is released in response to
sympathetic stimuiation and acts to increase pyloric
sphincter contraction. Norepinephrine is an example
ilil1|iltilllrililrI
of a:

A.
B.
mineralocorticoid.
catecholamine.
t i
HCL Infusion HCL Infusion
C. steroid. Starts Ends
D. peptide.

Copyright @ by The Berkeley Review 192 The Berkeley Revier'r'


Specializing in MCAT Preparation
Biology Gastrointestinal Junction Passage XIV

91. As stated in the passage, GIp is involved in


decreasing the rate of gastric emptying in response
to fats in the small intestine. This hormone is mosr
likely to increase the secretion of which of the
following hormones?

A. Glucagon
B. Bile
C. Insulin
D. Cortisol

qrL Cholecystokinin is known to stimulate contraction of


the antrum and promote constriction of the pyloric
sphincter. The overall effect of CCK is to:

-{. increase the rate of gastric emptying, and


contribute to the mixing of stomach contents.
B. increase the rate of gastric emptying and
inhibit the mixing of stomach contents.
C. decrease the rate of gastric emptying and
contribute to the mixing of stomach contents.
D. decrease the rate of gastric emptying and
inhibit the mixing of stomach contenis.

\\-hich of the following statements is FALSE


regarding parietal cell secretion of protons and
-nancreatic release of bicarbonate ion?

-\. The pH of the stomach decreases.


B. The pH ofthe blood increases.
C. The acid entering the smali intestine is
neutralized.
D. The bicarbonate ion is derived from carbon
dioxide.

- ,r:;ight @ by The Berkeley Review 193 The Berkeley Review


Specializing in MCAT preparation
Biology Cholera Toxin Passage XV

Passage XV (Questions 94-100) 96. Cholera toxin affects a second-messenger signal


cascade. The toxin would NOT affect the actions of:
Cholera is a potentially lethal disease caused by the
bacterium Vibrio cholerae. Individuals afflicted with A. epinephrine.
cholera suffer from severe dehydration due to diarrhea, B. gastrin.
which left untreated can be fatal. C. norepinephrine.
D. testosterone.
V. choLerae bacteria replicate in the small intestine of
the host after infection (usually from contaminated 97. Why is rehydration therapy alone not sufficient
drinking water), producing a protein known as cholera cure severe cholera?
toxin (CT). CT then binds to the surface of intestinal
epithelial cells, and through a complex series of events A. The body cannot establish normal blood
irreversibly alters membrane-associated G-proteins such pressure after severe dehydration.
that they are constitutively active, allowing for the B. Pores in the plasma membrane will allow
constant overproduction of the second messenger cyclic- water to eventually leak back into the intestinal
AMP. This triggers a sodium and chloride efflux that is Iumen.
responsible for the water loss associated with diarrhea. C. Chloride ion must be ingested to replace that
Rehydration therapy for cholera sufferers takes advantage which is secreted.
of an inwardly-pumping sodium/glucose symport present D. The underlying bacterial infection must be
in the apical membranes of intestinal cells. treated so that dehydrating diarrhea ceases.

CT is frequently used in research to help elucidate the 98. Humans are one of the few mammals affected by
functions of G-protein-related signal transductional cholera. Unaffected mammals :
mechanisms. Recently, lines of mice have been
engineered which have a transgene encoding the CT A. Iack G-protein mediated signaling.
protein linked to a promoter which allows protein B. always produce large amounts of cyclic-AMP.
expression only in cells of the pituitary gland which C. iack membrane receptors for cholera toxin.
produce growth hormone. These mice exhibit gigantism, D. normally have low cyclic-AMP leveis.
an effect of excessive growth hormone.
99. Assuming they could be applied specifically to the
pituitary, which of the lbllowing would be an
effective inhibitor of the gigantism seen in the
transgenic mice described above?

94. Cholera toxin causes intestinal cells to secrete large


I. An enzyme which degrades cyclic-AMP
amounts of Nao/Cle into the intestinal lumen. How
il. Excessive normai G-proteins
does this cause dehydration?
III. Excessivecyclic-AMP

A. Salt loss triggers the opening of water-


A. I only
permeable pores in the plasma membrane.
B. II only
B. C. III only
Salt loss creates an osmotic gradient, which D. II and III only
water follows out of the cell.
C. Salt loss triggers the active transport of water 100. A recent hypothesis suggests that individuals who
out of the cel1. are heterozygous for the recessive gene that causes
D. The distal convoluted tubules of the kidney cystic fibrosis (CF) may exhibit some resistance to
reabsorb more water. cholera (i.e., the CF gene confers some selective
advantage). Which of the following statements
provides evidence in support ofthis theory?

95. What should be given to help rehydrate a cholera A. The frequency of the CF gene is 1ow
patient? populations chronically exposed to choiera.
B. The frequency of the CF gene is high
A. Distilled water. populations chronically exposed to cholera.
B. Water and Na@ ions. C. The frequency of the CF gene is high
C. Water and glucose. popuiations rarely exposed to cholera.
D. Water, glucose, and Nae ions. D. Individuals with cholera are often
heterozygous for the CF gene.

Copyright @ by The Berkeley Review 194 The BerkeleY Review


Specializing in MCAT Preparation
Biology Gastrointestinal & Renal Section III Answers

1. A is correct, single-unit fibers are coupled, with many individual celis containing gap junctions. This questioir
draws on our knowledge of smooth muscle types. Even if we did not know the ansivJr ;grrt rro11 the question,
the
answer can be arrived at through some common sense. If something is acting as a singlJunit, then it is titety'tnat
the muscle fibers which make up the unit will be coupled to each other. T=hereforelr" eliminate choice B.
Furthermore, in order for the muscle fibers to be electrically coupled to each other, many"un
individual cells should
have gap junctions. Renember, gap junctions offer connections between two different c"llulu. cytoplasms, which
will allow current to move from cell to cell in a rapid fashion. The correct choice is A.
C is correct, calcium channels is slow. As in cardiac muscle, an inward calcium current is an important component
of the action potential in smooth muscle. We are told that the conduction velocity is low aiong smooth muscle fiber.
It is logical to assume that the conduction velocity of action potentials aiong Ct smooth-muscle fibers is slow
because activation of the calcium channels is slow. This question was just designed to make one think
about the roie
:
of calcium in the conduction velocity of an action potential of smooth muscle. the correct choice is C.
i
l
C is correct' number of action potentials within a set. We are looking at the diagrams to get an understanding
i of the
relationship between force and action potentials. In the graphs *e r"e three acion poten;ials giving rise
to i larger
',1.

force of contraction when compared to two action potentials within the set. Therefore, we can say that
the sizJof
the force depends on the_ number of action potentials within a burst. There is simply no evidence or support
from the
graph to consider the other_answers as viable possibilities. We cannot compare^amplitudes as the aciitn potentials
are all of the same size. Thus, the force of contraction depends on the number of action potentials. The correct
choice is C.

C is correct, mixing of intestinal contents, breaking larger particles into smaller ones. As stated in the passage,
segmentation allows alternating contraction and relaxation within the circuiar muscle. This type of movement-is
similar to opening and closing your hand. If you had a tomato in your hand, it would soon turn to mush. The same
happens to food within the intestines. It is continually being broken down into smaller and smaller pieces. There is
no indication that absorption of nutrients occurs because of segmentation, nor is there any direit evidence that
segmentation causes movement of the intestinal contents through the GI tract. Even tirough peristalsis and
segmentation may stimulate (indirectly) the secretion of digestive enzymes, this is not the best ansr"er choice. The
conect choice is C.
D is correct, breaking larger particles into smaller particles increases the surface area available to digestive
enzymes. We are told from the question that miring aids in digestion. We know that mixing will break-larger
particles into smalier ones. This aids in digestion because the surface area exposed to digestive inzymes will hive
increased. These enzymes wili break down complex molecules into those molecules that ian be absorbed across the
layer and eventually be used by the body. While the other answers may seem attractive, one has to ask themselves,
rvhat would this do to aid in digestion. Moving through the gut faster does not seem to aid in digestion, thereby
eliminating choice A. Furthermore, there is simply no evidence from the passage that smaller moleiules will cany
less charge or cross over the epithelial layer of the lumen any easier. Absorption will most likely be occurring
through channels in the cell layer. Therefore, we can say that increasing the surface area availabie to digestivE
enzymes will aid in digestion. The correct choice is D.

A is correct, I only. Enzymes are proteins, and their amino acids, can themseives be digested into amino acids and
taken back up to feed the body. Choice I is correct. Mucus, a protective coating on the GI tract, is made of
polysaccharides. Mucus is not digestible. How could it protect tissue if it were digested by our enzymes? Choice II
is incorrect. Bicarbonate is not protein. Choice III is incorrect. The correct choice is A.
D is correct,947o. If we merely looked at food intake and fecal output, we would have calculated 90/100
=90Vo.
This did not consider secretion of proteins from the gut that were recycled. Choice C is incorrect. If we did not
include food intake, we would have calculated 63Va. Choice A is incorrect. Choice B is also incorrect. The total
protein available was 100 gm from the diet and 70 gm from gut secretions to make a total of 170 gm ofprotein
available. 10 gm was excreted in feces, so 160 gm was digested and absorbed. 160/l-10 =94Vo. Notice that the skin
is not included in this calculation The correct choice is D.

C is conect, urea. Choice A is the chemical structure for uric acid. Choice B is ammonia. Choice C is urea.
Choice D is hydrazine. The correct choice is C.

: .:ight O by The Berkeley Review 195 The Berkeley Review


Specializing in MCAT Preparation
Biology Gastrointestinal & Kenal Section III Answers

9. D is correct, white cells, liver, muscle, gut. From Figure 1, look at the amount of protein each tissue turns over each
day: The correct choice is D.
10. B is correct, give isotopically labeled albumin, sample the blood periodically, and calculate the decay rate oflabeled
albumin. The concentration of albumin will not give information about the turnover rate. Choice A is incorrect. An
isotopic label will allow quantification of the decay rate of the label is the blood as the albumin is recycled. A
nonlabeled albumin will not help. Choice C is incorrect. Since a turnover rate in the body involves 111uny diff"r"nt
interactions, studying this is a test-tube would not work. Choice D is incorrect. The correct choice is B.
11. D is correct, skeletai muscle. The first tissue to break down would be the most expendable one. Smooth muscle is
the muscle of the diaphragm and many other internal organs. This is not expendable. Choice A is incorrect.
Cardiac muscle is heart tissue. It is not the first to be broken down either. Choice B is incorrect. Enzymes of
energy metabolism are required even during a fast, especially the enzymes controlling gluconeogenesis. ihey are
not used first, either. Choice C is incorrect. Skeletal muscle serves as a reserve of amino acids foi use during a fast.
It is the first tissue to break down. D is correct.
12. C is correct, between 0.6 and 1.5 gn/kg. The nitrogen balance witl be zero mg/kg at the appropriate protein intake.
The chart moves from negative to positive between 0.6 and 1.5 gm/kg. This means zero iJcontainediomewhere in
that intake range. The correct choice is C.

13. A is correct, CCK. CCK from the small intestine is stimulated by free fatty acids entering the duodenum. CCK
secretion will stimulate bile salt and pancreatic juice secretion. Secretin will be stimulated by gastric acids and high
protein content in the small intestine. The correct choice is A.

14. D is correct, enterokinase. Enterokinase (enteropeptidase) is the enzyme located within the mucosal layer of the
small intestine. It catalyzes the conversion of trypsinogen to trypsin. Trypsin will catalyze the remaining
conversions. Secretin and CCK are both hormones that will stimulate release of these proenzymes from the
pancreas, while bile salts, which are not enzymes, are produced in the liver. The correct choice is D.
15. C is correct, protein deficiency. The lack of enteropeptidase will result in decreases in the breakdown of protein
products entering the small intestine. Breakdown of these proteins is essential for absorption by the small iniestine.
Enteropeptidase does not affect fat or B12 metabolism. The correct choice is c.

1,6. A is correct, secretin. Secretin is stimulated by gastric acids entering the small intestine. It will stimulate secretion
of bicarbonate from the pancreas to neutralize tf,e acidity of the gaitric juices. Enterokinase and proenzymes are
enzymes that do not affect gastric acids. CCK will stimulate the secretion of bile salts needed for the emuliification
of fats in the small intestine. The correct choice is A.

17. A is correct, trypsin. Acute pancreatitis is produced by abnormal levels of trypsin, which is produced as a
proenzyme (trypsinogen) in the pancreas. Trypsin is a powerful enzyme that is not normally active in the pancreas.
Activation of this enzyme within the pancreas will cause the breakdown/digestion of the pancreas itself. One type of
damage to the pancreas involves lysolecithin (as outlined in the question). Secretin is a hormone in the small
intestine. Bile salts are not found in the pancreas. Endopeptidase is a fictitious enzyme. It sounds like
enteropeptidase (also known as enterokinase), which is the enzyme responsible for converting trypsinogen to
trypsin. The correct choice is A.
18. C is correct, dissociate into CO2 and H2O in order to allow the NaCl solution in the small intestine to remain
neutral. The whole point behind the function of the small intestine is to neutralize the acidic chyme that enters from
the stomach. The bicarbonate ion (HCO:e) that is released from the pancreas neutralizes the HCI from the stomach
by forming the neutral salt, NaCl. If we want to form more of this neutral salt, then we must pull the reaction given
in the question to the right (via LeChAtelier's principle). We can do that by removing carbonic acid (H2CO3). How?
By allowing it to dissociate to CO2 and H2O. The CO2 is absorbed into the blood and carried back to the lungs
where it is blown off as a gas. The H2O becomes part of the intestinal fluid. One reason that we do not want to
increase the acidity of the intestinal lumen is because peptic ulcers can result. The correct choice is C.

19. C is correct, pancreatic ductal cells. Severe duodenal ulcers, leading to the removal of the duodenum, will result in
the removal of cell-types associated with the upper small intestine. Two important cell types are those that secrete
CCK and secretin. CCK stimulates gallbladder contraction. The gallbladder stores and concentrates bile. If the

Copyright @ by The Berkeley Review t96 The Berkeley Review


Specializing in MCAT Preparation
Biotogy Gastrointestinal & Renal Section III Answers

cells that secrete CCK are removed, then contraction of the gallbladder will not increase. The gallbladder tends to
stay in the relaxed state for longer periods of time. Secretion of HCl from the parietal cells in the stomach is
inhibited by GIP, an enterogasterone released by cells in the duodenum. Removal of GlP-secreting cells will
remove the inhibition on the parietal cells and lead to an increase in parietal cell secretion (of HCl). Both CCK and
secretin act to inhibit gastric emptying into the smail intestine. Removal of the cells that secrete these two hormones
will result in an increase in the passage of chyme into the small intestine. Secretin also acts to increase the
production of bicarbonate form the pancreatic ductal cells. Removal of the cells which secrete secretin will lead to a
DECREASE in the production of pancreatic bicarbonate and a DECREASE in pancreatic ductal ceil activity. The
correct choice is C.
20. B is correct, delay the appearance of acid in the stomach. The name antacid tells us what it does. The antacid
tablets act to neutralize the increase of gastrointestinal acid. This is exactly what the two equations in the question
are addressing. We do not see acid being formed in either equation. Instead, we see the neutralization of HCl. We
can eliminate choices A and C. Even though it is true that water is a by-product of the reaction with the antacid and
HCl, it is not addressing the question as to why antacids should be taken 1 to 3 hours after a meal. The deiay in
taking antacids after a meal is simply to delay the appearance of acid in the stomach. This is seen from the graph in
the question. If the antacid is taken t hours after a meal, the appearance of acid in the stomach is delayed for about
2 hours. We can eliminate choice D. The correct choice is B.

2t. C is correct, trypsin would be inactive, and the other pancreatic enzymes would remain as zymogens. This is to test
our understanding of the passage. Trypsin is required to activate all other secreted pancreatic digestive enzymes.
Raw soybeans contain a trypsin inhibitor, so trypsin would be inactivated. Choices A and D are incorrect. Since the
other enzymes require cleaving by trypsin to achieve activity, they will remain inactive as zymogens. Choice B is
incorrect. The correct choice is C.
)) C is correct, gastric epithelium. The cells that are most rapidly turning over of those listed are in the gastric
epithelium. Neurons and muscle cells turnover minimally, if at all, in adults. The correct choice is C.
23. D is correct, I, II, and III.
Since the passage tells you mucus lines the digestive tract, we could just pick choice D
and be done. However, mucus is present in the mouth to protect the tissue and to lubricate food. Choice I is conect.
Mucus is present in the colon. Choice II is correct. Mucus is very important in protecting the small intestine.
particularly the duodenum, from auto-digestion. Choice III is correct. The correct choice is D.
24. D is correct, 2.0. The stomach is acidic. choice C, a basic pH, is incorrect. A pH of 6.8 is almost neutral, so choice
A is incorrect. A pH of 0.2 is 10 times more acidic than a pH of 2.0, and is much too acidic. A pH of 2.0 is corect.
The correct choice is D.
)< D is correct, I, II, and III. Enterokinase activates trypsin. If enterokinase entered the pancreas via the pancreatic
duct, then trypsin would be activated, and it would activate the other zymogens. They would all go to work on the
pancreatic tissue, causing damage so that pancreatic enzymes leak into the blood. Extensive damage could damage
the islet cells that secrete insulin, leading to impaired insulin secretion. Actually, a person would probably be very
sick or have some sort of surgery before the damage reached this level. It is important that we think of both the
exocrine and the endocrine parts ofthe pancreas. The correct choice is D.
26. A is correct, I only. The stomach's parietal cells are the acid-secreting cells. HCl, hydrochloric acid, is the acid in
stomach contents. The correct choice is A.
'r1 C is correct, carbohydrate. The give-away is in the name polysaccharide. This term refers to sugar units, so it is a

carbohydrate derivative. The correct choice is C.

28. C is correct, hydrolyzation of fat droplets in the small intestine. The hydrolyzation of fat droplets is dependent on
the enzyme pancreatic lipase. This enzyme is activated by the pancreatic enzyme trypsin. Emulsification of fats is
dependlnt on bile salts iecreted by the gall bladder. Stimulation of bile secretion is induced by increase in fat
content in the small intestine and CCK stimulation. The correct choice is C.
29. A is correct, emulsification of fats. The removal of household grease does not involve micelle endocytosis,
exocytosis of fatty acids, or packing of fat products in the endopiasmic reticulum. The correet choice is A.

Copyright O by The Berkeley Review t97 The Berkeley Keview


Specializing in MCAT Preparation
Biotogy Gastrointestinal 6t Kenal Section III Answers

30. C is correct, decrease in intestinal cell microvilli. Because of the soap-like appearance of the stools, bile salt
secretion is probably normal. Therefore, the problem most likely lies in the absorption of the fat droplets.
Decreases in the surface area of the small intestine due to decreases in the microvilli will initiate this respbnse.
Increases in pancreatic lipase alone will not cause this problem because any increase in free fatty acids and micelles
will usually be absorbed by the intestines. Chylomicron production is not involved with this process. The correct
choice is C.

31. C is correct, inhibition of cholesterol absorption in the small intestine. Cholesterol is dependent on bile salts for
absorption in the small intestine. Fatty acids, even though aided by bile salts, are still able to be absorbed. With the
increase in fatty acid and cholesterol content in the small intestine, CCK secretion will be elevated. The correct
choice is C.
32. D is correct, cholesterol. By the time chylomicrons reach the liver, lipoprotein lipase wiil have cleaved all the fatty
acids from the molecule. Lipoprotein iipase is found in the arterial system. The bile salts remaining in the
circulatory system are not attached to the chylomicrons. The correct choice is D.
33. C is correct, decreased levels of lipoprotein lipase. If levels of lipoprotein lipase are decreased, chylomicron
deposition of fats in the liver will increase. Lipoprotein lipase, while in the peripheral circulation, will cleave
triglycerides from the chylomicrons for deposition in fat stores in the periphery. Without lipase the liver will cleave
and store these same fats. Decreased levels of CCK and pancreatic lipase will only allow fatty acids to be lost in the
stoois due to malabsorption. The correct choice is C.

34. A is correct, I and III only. Table I indicates a drop in the plasma glucose levels from the baseline to the HF group.
Choice I is correct. Insuiin requirements were lower in the HF diet phase compared to baseline. Choice Ii is
incorrect. Less glucose was excreted in the urine in the HF group compared to baseline. Choice III is correct. The
correct choice is A.
35. D is correct, this decrease is beneficial
because glycosylated hemoglobin indicates persistent leveis of high plasma
glucose. When blood glucose is elevated persistently, then plasma proteins become glycosylated (glucosi residues
are attached to the proteins). A decrease in glycosylated hemoglobin means that, on the averagg blood glucose
levels were lower during the past few months. (The life span for a RBC is about 120 days). This would be a
beneficial change. Choices A and B are incorrect. Glycosylation occurs when plasma glucose is high. Choice C is
incorrect. The correct choice is D.
36. C is correct, oleic acid. The notation for the fatty acids is (# of carbons: # of double bonds). In the HF diet, the
amount of 18:1 is dramatically increased. In the passage, you are told that the HF diet is specifically higher in
monounsaturated fatty acids. Monounsaturated means having one double bond. Palmitic acid (16:0) nor stearic acid
(18:0) have double bonds. Choices B and D are incorrect. Palmitoleic acid is 16:1. and oleic acid is 18:1. Choice A
is incorrect. The correct choice is C.

37. A is correct, olive oil. Even if you have no clue about nutrition, use your test-taking skills to eliminate wrong
answers or see patterns. One of these things is not like the others. Olive oil is from piants and the others are from
animal fats. Anyway, oiive oil is a good source of monounsaturated fatty acids (oleic acid to be specific). The other
fats are more saturated (they are soiid at room temperature). The correct choice is A.

38. C is correct, II and III only. Although VLDL was lower in Table 2, LDL was nor. Choice I is incorrect.
Triglycerides are lower on the HF diet. (Use the significance symbols to help interpret the data. Researchers must
apply statistical methods to see if a finding is real or not.) Choice II is correct. HDL did increase on the HF diet.
Choice III is correct. The correct choice is C.
39. C is correct, the beta cells ofthe pancreas. The alpha cells ofthe pancreas secrete glucagon. Choice A is incorrect.
The exocrine portion of the pancreas secretes digestive enzymes, while the endocrine portion secretes hormones.
Choice B is incorrect, as is choice D. The correct choice is C.

40. D is correct, the liver. The word hepatic refers to the liver. The hepatic portal vein drains the blood supply of most
of the organs of digestion and passes it to the liver first for processing. Insulin released from the pancreas first
reaches the liver. Choices A, B, and C are incorrect. The correct choice is D.

Copyright O by The Berkeley Review l9a The Berkeley Review


Specializing in MCAT Preparation
Biology Gastrointestinal & Renat Section III Answers

41. B is correct, an increase in glomerular pressure, fbllowed first by an increase and then by a decrease in glomerular
fiitration rate. Ifthe efferent arteriole is constricted, blood cannot flow pass the point ofconstriction. This ieads to an
increase in the glomeruiar pressure, and allows us to immediately pick choice B or D. Constriction of the efferent
arteriole also means that the blood flow in the glomerulus will decrease. and more plasma will begin to filter out into
Bowman's capsule. With this information we can pick choice B. The more the plasma filtered out into Bowman's
capsule, the more the solute concentration begins to increase in the glomerulus. Eventually this will decrease the
glomerular filtration rate. The filtrate simply wili not be able to flow out of the glomeruius and into the capsule
because of the change in the concentration gradient. The correct choice is B.

42. C is correct, reduced plasma glucose concentrations. Recall that in the proximal convoluted tubule the transport of
sodium across the apical membrane (from the lumen of the proximal convoluted tubule to the cytoplasm of the
lumenal epithelial cell) is mediated by proteins called symports (a cotransporter). For example, thesesymports allow
for the passage of both sodium and glucose together into the cell. Another example is the symport that aliows for the
passage of both sodium and various amino acids into the ceil. If the concentration of glucose is decreased in the
filtrate, the amount of sodium that can be reabsorbed at the level of the proximal convoluted tubule is reduced. The
correct choice is C.
43. A is correct, afferent arterioles and decreases the volume of urine flow. Sympathetic innervation of the kidneys
primarily affects the afferent arterioles. This innervation constricts the afferent aiterioles and reduces the glomeruiar
filtration rate. The urine output will be dramatically reduced. The correct choice is A.
14. A is correct, (see the diagram below). Notice that along the X-axis the plasma osmolarity is steadiiy increasing. At
some particular point the solute concentration in the plasma will be so great that osmorecepto.s, located ii the
supraoptic nuclei of the anterior hypothalamus quickly respond to a change in the osmolarity of the extracellular
fluid (especially to the sodium ion).

X-Axis

Since the osmolarity of the plasma is becoming higher (hyper-osmotic), water wiil flow down its concentration
gradient (osmosis) and out of the osmoreceptors and cause them to shrink. As the receptors shrink their rate of
discharge increases. If the rate of discharge increases, the posterior pituitary gland releases more ADH into the
plasma. This is exactly what we see as the plasma ADH concentration along the Y-axis begins to increase. If there is
more ADH in the system, then there is more ADH to act at the baso-lateral membrane of the late distal tubules,
collecting tubules, and collecting ducts. In other words, the cells in those iocations are more permeabie to water.
Water is reabsorhed into the blood and the urine becomes more concentrated. Notice that as ADH is being released,
the osmolarity does not change that much. The correct choice is A.

-15- D is correct, (see the diagram below). Again, along the X-axis the plasma concentration of glucose begins to
increase. As soon as the concentration of glucose reaches a particular point where it can no longer be reabsorbed
from the lumen of the proximal convoluted tubules, it is passed through the lumen of the kidney tubules in an ever
increasing rate.

't

X-Axis

Copyright @ by The Berkeley Review r99 The Berkeley Review


Specializing in MCAT Preparation
Biology Gastrointestinal E( Renal Section III Answers

This is shown along the Y-axis where we observe a continual


increase in the rate of excretion of glucose in
of the kidney. we do not see this type of curve in the other the urine
tt r"e unrr"rs. The correct choice is D.
46. c is correct' is a steroid hormone secreted by the adrenal medulla. Aidosterone
which is synthesized in the zona glomerulosa (ihe outermot, is a cholesterol-derived steroid
."gi"rj.r rhe adrenal aioorterone is referred to
as a mineralo-corticoid because of its affect on electrolyt", "o.t"*.
about 30 minutes' Aldosterone promotes Nae
,u-"h as Na@ K*.-i;i;'rro*ron" has a half-life of
""d at the late
to u g."u, extent
,reabsorption distal tubular region, the
cortical collecting tubules, and the collecting ducts. Aidosteroneilso controls the secretion of K@ ions into
lumen of the distal convoluted tubule. The coirect choice is the
C.
47. B is correct' increased muscle contraction. Excessive aldosterone
secretion results in an excessive loss of Ke from
the extracellular fluid and into the urine. This lowers rhe prasma
hypokalemia' Loss of K@ ions from a cell tends to decrease
K* .o";;;;;;ii"",'"'.*oirion referred ro as
the K@ concentration in that cell, making the resting
membrane potential for K@ more negative that it would have
been had aldosterone
Ko rushes out of the nerve cell, that cell tends to. hyperpolarize. Hyperpolarization leveis been normal. Recali that as
contraction, not an increase. The correct choice is B. leads to a decrease in the muscle

48. C is correct,electrolytes. If we were to centrifuge down a sample


of blood, we would find ,,formed elements,,such
as red blood cells (erythrocytes), white biood cjts^6e*ocy,.rl,
pi.r"r"o, and proreins (high molecular weight) at
the bottom of the testtube' In the plasma we would find
such ruurtunr", as sugars, amino aclds, and the
a few' Electrolytes are rather small molecules rhat electrolytes,
can u" filtered by the glomerulus. The correct
:i:rxT"rrt3: "utlry

49. c is correct' excessive vomiting' we are looking for a situation


which.wiil not result in the gain of hydrogen ions.
3L:,*'-T"'?1:T *:jj,ryfl$ :"."1.,"
or net 6odilv c"t"
leaving the bodv. Recall tha_t vomiting has irs origins'in"the
;;-i";;i hydrogen ions is gasrroinresrinal secrerions
j:tiil:;;ffi::il1i::t::t".1:
r.-"rrr,
vomitus leaving the body will result in a net loss, not a net gain
,'ri";:#r"*,
of hydrogen ions. The correct choice is c.
50. B is correct, a increased blood hydrogen ion concentration.
It is clear from figures one and two that two events are
occurring' one event is the secretion of hydrogen ions from
the iunutu, celli of the tiJ*y in,o the renal tubule.
These hydrogen ions, after reacting with a buffe'ring
system,-will r"uu" trr" body in irr" *i""] The body
hydrogen ions' This tells us thaithe hydrogen ion-concentration is removing
must havl been elevated, and the kidney is
compensating for this elevation. second, Figure 2 is indicating
th. ."ubrorption of bicarbonate. one must understand
that reabsorption of a bicarbonate ion is eqriivalent ro rhe
acting as a compensatory mechanism to reiieve an elevated
l"r;;i; il;rogin ion. Therefore, rhis reabsorprion is also
hyd-;; Ln concentration. The correct choice is B.
51. D is correct' secondary active transport. This question-draws
on your previous knowledge of transport systems. As
one can see' a hydrogen ion is transported out of the tubular
cell white a sodium ion is nioved tiom the renai tubule
lumen to the cell' Sodium moves inlo the cell because it is falling
down its concentration gradient. Even though this
is not stated in the passage, one should know that the sodium
ion"concentration is always higher extracellularly when
compared to intracellularly, The reason sodium can continue to
fall down its concentratlon"!.uoi"ntis because of a
sodium/potassium ATPase pump which uses the. energy of ATF
to p"-p sodium out of the cell. One can ref'er to
sodium moving down its gradient as primary active trinsport.
To trispo.t the hydrogen ion across the membrane,
one can harness the energy found in the sodium-concentration gradient
(really a form ofpotential energy). since the
is cotransported along with the sodium ion, it is
:il:i:ff;:" &;; ;d rerm iecondary aitive transport. The correct
<) B is correct, a decrease in the hydrogen ion concentration, with a decrease
in the level of Co2. rtshould be clear
that an alkalosis is a situation where the pH is above that of neutral,
and there i
this case, we have u" u#;#i, ffi'; ffi,iffJ:l?";:":l?:;il"I.jli,:l'.':ffi;:**:i::?;,t l{f:"j:ij"Jl:,ll
the equation that is shown in Fj
l:I ":g::
,: produce a siruation *h;,;;fi;i,la-"irii"".i##il;;;;',H?;H:,'ff:i'":l?:il,i:1ff;
I:::":'iT^':^1i:':?.?1.1ll":g7
i i;,;;r;;.u"il';'^#;tii;;,i'i#""'ol,'*""lffi#JJu5;',:'fffi:',,J,:,'r:l
v L .v vv'LLPwt
The result of thatshift is-that there is a decrease in the amount
of hydrogen ions. A reduction in the level of
lll"llt_t: ions clearly results
hydrogen in alkalosis. The correct choice is B.
53. c is correct, decrease, due to increased ventilation. we have a situation here where there is simply
hydrogen ion' we want to restore the original levei of hydrogen ion- too much
Again, refer to the equation shown in Figure 1l

Copyright @ by The Berkeley Review 200 The Berkeley Review


Specializing in MCAT preparation
Biology Gastrointestinal & Renal Section III Answers

One way we can reduce the hydrogen ion concentration is to lower the ievel of carbon dioxide. This should sound
very familiar, like the previous question. How do we lower the partial pressure of carbon dioxi<te? We breathe air
out with an increased frequency. In other words, we increase the ventilation rate. This will cause a decrease in the
partial pressure of carbon dioxide, and shift the equation in such a manner so we experience a decrease in the level
of hydrogen ion. This will help restore the normal level of hydrogen ion from its currint elevated level. The correct
choice is C.

B is correct, I and II only. Since the solution is introduced into the abdominal cavity through catheters, there is
always a possibility of infection. Choice I is correct. Depending on the person's state of hydration, dialysis can leacl
to electrolyte imbalance. Choice II is correct. Since there is no pressure involved in this technique, there is no
ultrafiltration occurring. Choice III is incorrect. The correct choice is B.
D is correct, glomeruiar capiliaries. The glomerular capillaries are fenestrated, that is, they contain large pores.
Although large proteins cannot pass, small molecules and water are free to pass. The pressuie of the bloo? acts to
force the molecules through the semipermeable capillaries. Choice D is correct. Thi ultrafiltrate coliects in the
glomerular capsule. The afferent arterioles are not permeable to these molecules. Choice C is incorrect. The
peritubular capillaries are involved in resorption. Choice A is incorrect. The loop of Henle is involved in
concentrating the urine. Choice B is incorect. The correct choice is D.
:!6. A is correct, no enzyme activity in the bag. Although enzymes are large compared to atoms, they arc small
compared to a pinhole. This is why you test your dialysis tubing for holes before you begin enzyme purification,
Since there is no way the amount of enzyme can increase, choice B is incorrect. Since theie is an efl'ect caused by
the pinhole, choice C is incorrect. Choice D is incouect since the enzyme would all (or almost all) move out during
an overnight dialysis. The correct choice is A.

A is correct, erythropoetin. The kidney produces erythropoetin in response to low blood volume, This hormone
signals the bone marrow to make more red blood cells. If a kidney were diseased, this hormone may not be
produced. Choice A is correct. Choices B, C and D are incorrect because the kidney does not make these hormones.
ADH is released from the posterior pituitary where it is stored after its manufaiture in the hypothalamus. The
adrenal giand produces epinephrine and norepinephrine. Insulin is produced by the beta cells oft-he pancrcas. The
correct choice is A.
58. B is correct, edema. You are given 2 choices that give the same answer, choices A and D, Elirrinate them. If
uremia is present, the kidney is probably diseased, so urination is probably decreased. Choice C is incorrect. Edema
means an accumulation of fluid in the intracellular spaces, leading to tissue sweiling. Uremia indicates kidney
function is declining. This would lead to an increased solute load in the blood and retention of fluid. The fluid
increase would mainiy move to the extraceilular spaces, since cells have only a little room for expansion. The
correct choice is B.
59. B is correct' use of a dialysis solution that is isotonic to the blood in glucose and salts. The pores of dialysis
membrane are not small enough to retain the small molecules of glucose and salts. Also, if glucose an<l salts were
retained in the dialysis membrane, urea would be, as well. Choice A is incorrect. Choices C an<i D are incorrecr,
because even through glucose and salts will enter the blood, they would be lost to a hyporonic dialysis solution.
Finally, to prevent movement of glucose and salts, the dialysis solution should be isotonic in irs concenrrarion of
these molecules. This will prevent loss. The correct choice is B.

60. B is correct, I 11 ml/min. 5 liters = 5000 mi. 5000 m1/45 min = I I I ml/min. The correct choice is B.

6X. A is correct' iipolysis. Lipolysis is the mechanism of breakdown of stored triglyceride to release FFA and glycerol
into the blood. This would raise FFA concentrations. Lipogenesis is the synthesis of fat. This wouid not raise FFA
levels. Choice B is incorrect. Increased fat oxidation would decrease plasma FFA levels. Choice C is incorrect,
Increasing dietary fat would increase chylomicrons, but not FFAs. Choice D is incorrect. The correct choice is A.

62. C is correct, epinephrine. CCK is a hormone secreted from the duodenum in response to dietary fat. It causes gall
bladder contraction and release of pancreatic enzymes. Choice A is incorrect. Thyroid hormone does not increasc

Copyright O by The Berkeley Review 20t The Berkeley Review


Specializing in MCAT Preparation
Biology Gastrointestinal & Renal Section III Answers

plasma FFA concentration. Choice B is incorrect. Insulin decreases lipoiysis, and increases fat storage. Choice D is
incorrect. Finally, epinephrine promotes lipolysis to increase FFA concentration in the blood in response to stresses.
Think of the fight or flight mechanism. The muscles will need fuel and the elevated FFAs will provide that. The
correct choice is C.
63. D is correct, sucrose stimulated insulin, which decreased release of FFAs from adipocytes. Insulin turns down
lipolysis. If you have just eaten, and insulin is secreted, your body does not need the extra fuel that circuiating FFAs
provide. You have switched from withdrawal from stores to depositing into your stores. Sucrose does not stimulate
glucagon, so choice B is incorrect. Fructose does not mediate the effect on FFAs, so choice A is incorrect. Sucrose
is not an inhibitor ofthe action ofcaffeine, so choice C is incorrect. The correct choice is D.

64. C is correct, adenylate cyclase. The wrong answers in this question are all part of the cAMP activation pathway.
Be careful ! A hormone such as epinephrine stimulates a receptor on the cell. A second messenger, the G protein,
communicates the message internally to the enzyme adenylate cyclase. Adenylate cyclase produces cAMP. cAMP
activates protein phosphorylases. The question is for the enzyme, so adenylate cyclase is your answer. The correct
choice is C.

65. D is correct, the effects are the same as drinking water. The answer to this question in the table. The ,,, f, and t
indicate if differences between groups are significant. This is a very common way for data to be summarized.
Experiment 2 has the symbol "*", which indicates that the results were not significantly different from Experiment
3, water. Although the numbers look like a slight increase, the x indicates there is actually no difference. The
correct choice is D.
66. D is correct, I, II, and III only. This is an experiment to determine the control data for plasma FFA concentrations
over time. To simulate the real experiment as much as possible, hot water and sodium saccharin were both included.
This means I, II, and III are all valid choices. The correct choice is D.

67. B is correct, osmotic force due to protein in plasma favors filtration. This is a false statement, Filtration is the
movement of substances from the glomerular capillaries into Bowman's capsule. The protein that is unable to be
filtered creates an osmotic force which pulls the movement of fluid away from Bowman's capsule and into the
glomerular capillaries. Therefore, the protein in the plasma creates an osmotic force which opposes, not favors,
filtration. We are looking for a false statement, and choice B certainly fits the description. The correct choice is B.
68. C is correct, X is filtered and secreted, but not reabsorbed. This question is requiring one to interpret Figure 1. It is
clear that substance X leaves the glomerular capillaries and enters the renal tubule at Bowman's capsule. This
classifies as filtration. Also, it is apparent that substance X leaves the peritubular capillaries and enters the renal
tubule after Bowman's capsule. This classifies as secretion. The only event that does not occur is movement of
substance X back into the peritubular capillaries. In other words, there is no reabsorption. Therefore, X is filtered
and secreted, but not reabsorbed is a true statement. The correct choice is C.

69. B is correct, increased levels of parathyroid hormone, with increased removai of bone, This question is re quiring
one to think about the role of parathyroid hormone. We have a decrease in the level of calcium, and so we will
therefore want to act to increase the levels of calcium. This results in an increase in the levels of parathyroid
hormone. Why? One of the functions of the parathyroid hormone is to remove bone. Removal of bone releases
calcium and phosphate from the organic molecules. The release of calcium acts to counteract the original decrease.
Therefore, we will see an increase in the level of parathyroid hormone, and an increase in the removal of bone. All
of the other choices are either fa1se, or lead to a flrther decrease in calcium levels. The correct choice is B.
70. D is correct, the effect of the extracellular calcium concentration on membrane is distinct from its role as a mediator
of muscle contraction. We are told from the passage that a low extracellular calcium concentration leads to an
increase in nerve and muscle cell excitability. In fact, a person with a iow calcium concentration may suffer from
muscle spasms. We often think of calcium as being involved with the troponin-tropomyosin complex, where
calcium is needed to initiate contraction. In that situation, a low calcium concentration would lead to decreased
excitability or contraction. We are told that in this case, a low extracellular concentration leads to increased nerve
and muscle excitability. Based on this apparent contradiction, we can conclude that the effect of calcium
concentration on the membrane is distinct from its role as a mediator of contraction. In fact, these effects reflect
calcium's abiiity to bind to piasma membrane proteins that function as ion channels. The binding alters the state of
the these channels. The correct choice is D.

Copyright @ by The Berkeley Review 202 The Berkel€y Kevi€w


Specializing in MCAT Preparation
Biology Gastrointestinal & Renal Section III Answers

D is correct, a reduction in the tubuiar reabsorption of phosphate. This question is not easy. The following logic
can be used to arrive at the answer: We have a low calcium concentration. We will want to iounteract that decreise
t'ith an increase. We will see an increase in the removal of bone due to increased levels of parathyroid hormone.
T1-ris not only reieases calcium, but from the passage, one should be able to conclucle that it releases phosphate
ion as
n'e11. Now, if the extracellular concentration of phosphate were to increase as a result of this^release, further
novement of calcium from bone would be hampered because a high extracellular concentration of phosphate would
rind with calcium and cause the salt to be re-deposited on bone and other tissues. The point is that we want to get
rid of the phosphate ion. One way to do this is to decrease the amount reabsorbed in the kidney. In that wiy,
phosphate ion is released with the urine and calcium levels can return to normal. Again, we will most likely see'a
:e duction in the tubular reabsorption of phosphate ions. The conect choice is D.

C is correct, the parathyroid gland. This question is simply asking us to recall the function ofour endocrine glands.
Parathyroid hormone, a protein hormone, is reieased by the parathyroid gland. Whiie the parathyroid gland is
:mbedded in the thyroid gland, the giands are distinct. The correct choice is c.

A is correct' muscie and adipose tissue have first access to dietary fatty acids from chylomicrons. The chylomicron
packaging system avoids direct delivery of fatty acids to the iiver. This is stated in the text of the passage. Choice B
i: incorrect. Although microorganisms may be present in the lymph, they are not packaged intoihe chylomicrons.
Choice C is incorrect. Lymph, not interstitial fluid, canies chylomicrons. Choice D-is inconect. The coirect choice
is A.

C is correct, water. 'Water is the main component of our body fluids. Since lymph is a derivative of interstitial fluid,
-: is mostly water. Fat soluble vitamins are usually present in very small amounir (pg), triglycericles may be p."r"ni
i larger amounts (grams), but water is the most abundant. There must be more wateithanihylomicrons so that they
::main soluble. Although chylomicrons carry fat, neither they nor the fat they carry is thl major component oi
. mph. Choices A , B, and D are incorrect. The correct choice is C.
D is correct, I, II, and III. AIl three vitamins are fat-soluble vitamins. The correct choice is D.

-\ is correct, to destroy microorganisms. Chylomicrons are procluced and adde<l by the intestinal cells. Choices B
.rd C are incorrect. Intestinal nutrients are either packaged in chylomicrons and put into the lyinph at the small
-ltestine, or nutrients from the smaii intestine directly enter the hepatic portal vein and travel to thl tiver. The lymph
-'rde does not add intestinal nutrients. Choice D is incorrect. Lymph nodes filter microorganisms througi tle
:raeocytic cells, as mentioned in the passage. The correct choice is A.
-\ is correct, I only. In the passage, the structure of the lymph vessel is described as similar to the vein. The vein
l --es not have a muscular layer like the artery. Choice II is incorrect. Blood in the vein is moved by squeezing by the
.,;eletal muscles. Choice I is correct. Blood pressure leads to the creation of interstitial fluid. it does not moue
. nph. Choice III is incorrect. The correct choice is A.

B is correct, kidney. Urine is formed as an ultrafiltrate of blood in the glomeruiar capsule of the kidney. Choice B is
- rffect. Neither the liver, the pancreas, nor the salivary gland produces an ultrafiltrate. Choices A. C, and D are
.:.Jonect. The correct choice is B.

B is correct, I and II only. After a meal, chylomicrons first enter the lymphatic vessels, then the blood vessels.
- roice I is correct. Lymph does not contain erythrocytes (red blood cells). Choice III is incorrect. Lymphocytes are
.:: second most abundant type of white blood cells in the plasma. They are aiso he lymph. Choice II is
, -.irect. The correct choice is B.

D is correct, []l X PFa = (l lu X FU) + (t lV X PFy). The principle of mass balance is the idea of whar enrers the
jney must leave the kidney. That which enters the kidney wili be from only the plasma of the renal artery. This
).-

::n be quantified as [ ]e X PF4. There are two ways substances can leave the kidney. One is through the urine,
---:ntifiedasFuX[]u. Theotheristhroughtherenalvein,quantifieciasPFuX[]u.Therefore,massbalanceis
.:st represented as []a X PFa = ( []u X Fu) + (l lv X PFv). The correct choice is D.

by The Berkeley Review The Berkeley Review


Specializing in MCAT Preparation
Biology Gastrointestinal & Kenal Section III Answers

81. B is correct, 0.0005 L. We are interested in knowing the volume of urine collected in 0.5 minutes. We need to
know the rate of urine flow to figure out the volume. The rate of urine flow can be calculated using the relationship
established in the passage. In other words, Fu = (C) X (t lp)/t lu. Putting in the values, we get Fu = 50 ml/min X 1
mg/ml divided by 50 mg/ml, which turns out to be 1 ml/min. We are asked for 0.5 minutes, so we have 0.5 ml as
our volume. 0.5 ml X I L/1000 ml = 0.0005 L. The correct choice is B.

82. C is correct, GFR = [ ]u X Fu/[ substance]p. From the passage, we know that under certain conditions, the amount
of a substance filtered is equal to the amount excreted in the urine. The amount of a substance excreted in the urine
will be [ ]u X Fu. To calculate the amount filtered, we can multipiy the GtrR by the concentration of the substance
in theplasma. Equating thetwo, wehave GFRX IJp = 1]u XFu. Therefore, theGFR = [ ]u XFu = [1p. The
correct choice is C.
83. B is correct, the substance must not be reabsorbed, and must be secreted by the nephron. From the passage, we
know that only under certain conditions can the amount filtered at the glomerulus equal the amount excreted in the
urine. It should make logical sense that one of those conditions should be that nothing is added or taken away from
the filtrate as it passes through the nephron. Statement B claims that the substance must be secreted into the
nephron. If this was the case, the amount excreted will be more than likely greater than the amount filtered.
Therefore, condition B cannot exist if our claim is to be valid. The correct choice is B.

84. A is correct, Proximal Convoluted Tubule. This question is simply calling on our previous knowledge of kidney
function. Recall that most reabsorption occurs in the proximal convoluted tubule. This is the first part of the
nephron. The substances reabsorbed are returned to the venous system through the peritubular capillaries. The
reabsorption of glucose takes place via a secondary active transport, and under normal conditions, l00Vo of glucose
is reabsorbed. The correct choice is A.

85. A is correct, 3.5 mg/ml. The plasma threshold is defined as the level of plasma glucose which first gives rise to
glucose in the urine. Looking at the curve, we are interested in the line labeled excretion. When this line first
becomes positive, we need to look at the value of the blood glucose which gave rise to this first appearance of
glucose in the urine. This line becomes positive between the values of 2 and 4. Looking carefully, the value of the
blood glucose is around 3.5 mg/ml. The correct choice is A.
86. B is correct, the cationic species has the highest filterability because of the presence of anionic glycoproteins on the
surface of all glomerular filtration components. Let us read the curve given in the question. Draw an imaginary
vertical line in the graph. With this line, we can say that if a molecule has the same radius, the cationic species will
have the higher filterability. Therefore, any choice that uses the argument that cationic or anionic species have
different radii is invalid. In other words, we can compare a positive and negative molecule that has the same radii,
and know from the graph that the positive cationic species will have the greater filterability. That means there must
be some other reason for the enhanced filterability. The most likely reason is the presence of anionic glycoproteins
on the surface of all filtration components. The opposite signs will create an electrical attraction that can lead to an
enhanced filterability. The correct choice is B.

87. D is correct, regurgitation of duodenal contents may lead to stomach ulcers. We know from the passage that the
pyloric sphincter carries out two essential functions. One of these functions is to prevent the reflux of small intestinal
contents into the antrum. This is the case because the lining of the stomach is not protected from the bile that is
secreted into the small intestine. The reflux of this bile containing content may contribute to the development of
stomach ulcers. So again, while the stomach is protected from the acid it secretes, it is vulnerable to the bile secreted
by the small intestine. The opposite holds true for the small intestine. The small intestine is protected from the bile,
but is vulnerable to the acid from the stomach. Therefore, regurgitation of duodenal contents may lead to stomach
ulcers. The correct choice is D.

88. B is correct, catecholamine. This question cannot be answered from any of the information in the passage, but must
be answered solely on our own knowledge of hormones and hormone classification. Norepinephrine, along with
epinephrine and dopamine, are catecholamines which fall under the major category of amine hormones. The correct
choice is B.

89. D is correct, a decrease in blood volume. If a hypertonic solution is introduced into the lumen of the small intestine,
this creates a situation where water will enter into the lumen from the vascular system. In other words, the water

Copyright @ by The Berkeley Review 204 The Berkeley Keview


Specializing in MCAT Preparation
Biology Gastrointestinal & Renal Section III Answers

from the vascular system will cross the intestinal wail and diffuse to where there is a higher concentration of
solute.
This will certainly impair the vascular system. In particular, it will cause a decrease in btood volume which
could
lead to a dangerous drop in arterial blood pressure. The correct choice is D.

90. A is correct. We are told from the passage that a decrease in the pH of the duodenum will cause a decrease in the
rate of gastric emptying. One way to achieve a decrease in gastric emptying would be to decrease the contractility
of
the antrum and to increase the contractiiity of the duodenum. In this way, we could achieve our goal of a decreised
flow of material from the antrum into the duodenum. Looking at Graph A, the injection of HCI into the duodenum
certainly does cause a decrease in the contractility rate of the antrum. In addition, the injection of the HCl certainly
does cause a significant increase in the rate of contraction for the duodenum. Therefbre, Graph A BEST representl
our expected results. The correct choice is A.
91. C is correct, insulin' We are iooking for a hormone that is most likely to be released in response to a rise in GIp.
Since we are looking for a hormone, bile can automaticaily be eliminated. We know that GIi, is itself released
as a
result of fat in the small intestine. The fat will soon find its way into the vascular system by first going
through the
lymphatic system. The fat will want to enter into the adipose tissue. Insulin is the hormon" i"rponrlbl"*;or
taking fat
from the vascuiar system and facilitating its uptake into adipose tissue. Think in very general terms. We are in
an
absorbing state for the body, as food is being absorbed from the small intestine. Foi this reason, insulin
is the
hormone that is necessary' and GIP is stimuiating the release of insulin in anticipation of the arrivai
of f'at into the
vascuiar systen. The conect choice is C.
92. C is correct, decrease the rate of gastric emptying and contribute to the mixing of stomach contents. We
know from
the passage that CCK does retard the rate of gastric erhptying. For that reason ilone, r" can eliminate
choices A and
B. We now must concentrate our efforts into understanding whether the actions of CCK contribute or inhibit the
mixing of stomach contents. The question tells us that CCK causes a constriction of the pyloric sphincter. However,
it also causes an increase in antrum contractility. Think about this. If the antrum has an in"r"ur"d contractility and
the contents of the stomach cannot enter into the small intestine, they will simply be pushed back into
the bohy of
the stomach. The contents of the stomach have no where to go but baikwards. Therefore, there will
be an increase in
the mixing of stomach contents and an overall decrease in the rate of gastric (remember, the pyloric
sphincter is closed). The correct choice is C. "-ptying
93. B is correct, the pH of the blood increases. We are looking for afalse statement surrounding the notion
of parietal
and pancreatic cell secretion. We know that a parietal cell secretes protons into the lumen of tlhe
stornach. Firit, how
is this proton obtained. The parietal cell converts a carbon dioxide molecule into a bicarbonate ion and proton.
a The
cell secretes the proton into the lumen. What does it do with the bicarbonate ion? It transports the ion into the
b1oo4.
In that way, the parietai cell itself does not become basic. However, that does present i problem for the pH of the
blood, right? Well, not exactly. A pancreatic cell does just the opposite. The ceil secretes a bicarbonate ion into the
lumen of the small intestine and is left with a proton. The protonls released into the blood. If the secretions
of the
parietal cell and pancreatic cells are equivalent, the pH of the blood does not change throughout the digestive
process. The correct choice is B.

94. B is coffect, salt loss creates an osmotic gradient, which water follows out of the cell. The epithelial cells lining the
intestine form a selectively permeable boundary between the blood and the lumen, o. .uuiiy, of the intestin"i. In
cholera, these epithelial cells secrete sodium and chlorine ions into the lumen. This changes the relative
concentrations of solutes, meaning that there is a lower concentration of solutes inside the celt in comparison to
outside the cell. Water diffuses across membranes by osmosis, foliowing a gradient from iow dissolved solutes to
high dissolved solutes (i.e., high water concentrations to low water concentraiions; the concentration of water is iess
in a solution with a lot of ions in it.). Therefore, water leaves the epithelial cells and enters the lumen of the intestine,
following the sodium and chloride ions. This causes diarrhea. Choice A is wrong because the membrane is already
highiy permeable to water, and extra pores won't make a difference. Choice C is wrong because water cannot be
actively transported. Choice D involves reabsorption of water and is completely incorrect. The correct choice is B.
qi D is correct, Water, glucose, and Na@ ions. From the passage, we learn that rehydration therapy involves an
inwardly-pumping glucose/sodium symport. A symport is a protein which transports two ions at a time across a
membrane, in this case, into the cell. If a cholera patient is given both glucose and Nao with water, the
glucose/sodium symport will transport both of these ions into the intestinai cells, increasing the solute concentration
inside. This will cause water to follow osmotically and reenter the blood (via rhe epithelial cells). A11 of the other

-opyright @ by The Berkeley Review 205 The Berkeley Review


Specializing in MCAT Preparation
Biology Gastrointestinal & Kenal Section III Answers

answer choices are wrong because none of them allows solute to enter the cell, favorably changing the osmotic
balance. The correct choice is D.

96. D is correct, testosterone. This probiem requires that we know that mechanisms
of the hormones that are mentioned.
Epinephrine' gastrin, and norepinephrine are peptide hormones which operat"
uy ul"oing tJ receptors on the outer
surface of ceil membranes'-This then triggerJ a signai cascade involving
,".oni-..r"rr"n-g"r, ut" cyclic-AMp. As
such' the cholera toxin would affect their actions by. increasing cyclic-AMp
levels. Testosi".on", on the other hand,
is a steroid hormone. Steroid hormones can pass through t# piur*u membranes
of cells, directly (or through a
bound protein complex)
{fectr-ng DNA tranicription. Steroid'hormones therefore do not use second-messenger
systems. The correct choice is D.

97. D is correct' the underlying bacterial infection must be treated so that dehydrating
diarrhea ceases. Rehydration
therapy is just that, a therapy, not a cure. It does not address the underlying
piolr"*,"*hi.h i, ,t . uu.r".ial infection
that produces the cholera.toxin in the first place. In severe infections,
anfmicrobial'agents must be administered to
destroy the bacteria. otherwise, dehydrition witl simpiy ,"o""ri
after each rehydration artempt due to the
persistence of the cholera toxin which causes salt secretion
and water loss. choice a it rvronl because the body can
reestablish a normal blood volume and pressure. choice B is inco'ecl
osmotic gradient (see Question 1). choicJ C is not right because more
ti6;" il;;", af1'ect the exisring
chloride wiliju;i m;ie the osmotic gradienr
steeper and more water will leave the body. The correct choice is
D.
98. c is correct, lack membrane receptors for cholera toxin. The question^is
basically asking why
class of animal closely related to humans, relatively speaking) some mammals (a
u."nit uff".t"d adversely by cholera in the way that
humans are' Choices A and.B are wrong because mammals, like
humans, use G-protein
most studies are done in.mice).and generally only_have high cellular -"diut"o signaling (in fact,
cyclic-AMp levels when cells are stimulated
(i'e'' by hormones)' choice D is wrong beciuse if cyctic-itrlp tevets are
normally low, then cholera toxin would
increase them' choice c is correct beciuse the cholera toxin
binds to specific ..""pto., on tn" ,r.tu.e of intestinal
cells' If these receptors are absent, as in many mammal species, the
toxin can,t attach and exert its effects. The
correct choice is C.
99. A is correct, I only' The transgenic mice described in the passage are engineered
to have the gene which encodes
the cholera toxin' This gene is attached to a promoter (l
?NA ,"q"u"n"* thit allows DNA to be transcribed) which is
only active in the growth-hormone producing cells of the, pituiiary gland.
Therefore, these mice produce cholera
toxin specifically in their pituitary cells' causing a sustained in"r"ur"
subsequently exhibit gigantism, which results-from,excesr g-*1tr-i'o.*one.
ii cyclic-AMp levels in these cells. These mice
Therefore, we can assume that the
increased cyclic-AMP levels are somehow signal-ing these.cell"s
to proouce more growth hormone. The question asks
what would inhibit this effect. Excessive noimal d-proteins wouldn'r
cholera toxin and would become constantly active. bxcessive F"-gr" tr,'ry *oriJri-ply be alrered by the
would add tothe gigantism because it
seems to be high levels of cyclic-AMP which is.causing it in
the"y"ii"-evtr
iirst place. An enzyme which degrades cyclic-AMp
would reduce the effect. The correct choice is A.
100. populationschronically exposed ro cholera. rhe theory
3":,,'"'"||e|:,,i1"^T::,X:1"."^l:?:.91c^:i:,1'-llil.in
'r Lrrv YuvJLrv' DLcLeD rudl LIE cysuc ltDrosls gene' wnen present ln one copy (i.e., heterozygously) may
confer a selective advantage because it provides a degree-ofresistanci to
cholera. This is a similar case ro sickte-cell
anemia hererozvgore., ;;:;;;':;:fi;TJ'J;?nHH:'fi4H"-"Jif;i1l,llJ.:Tiili'jjffi:HL::: a selective advantage on
*::l:lrry::rl!:: ;;;;;f ht;il;l*" il#::l["r:r"i.
y" woutd expecr ro see a.higi frequencv of tr'i" g"*
because people with the gene would be more hkJly to survive epidemlcs
and would evoiutionarily be more ,,fit,, than
people without the gene. All of the other answers are wrong because
they do not provide .uiJ.n.. that supports this
theory, even though they may not contradict it. The correcichoice is B.

Copyright @ by The Berkeley Review 206 The Berkeley Review


Specializing in MCAT preparation
ffig#HCIgry
A. Keproduction
SeeEioxr EV l. Maie Reproduction
2. Female Reproduction

Reproduction B. Development
l. DevelopmentalStaees
& 2. Dei'elopmental Mechanisms
5. Human Embryo Development
Eleve[oprment
Practice Passages & Ansrt'ers

'Tfte
NKKETEY
R.n.v.I.E.l[It
Speciahztng in MCAT Preparation
Top lO Section Goals

Be familiar w'ith lhe male reproductive anatomy.


O- hv
ff?i: iii:J,:l*,ji:,'n:'ff;tff.il :lm.l:"

aB Know how s uced


Be familiar with the actions of LH and FSH on developirn
*.*. $now the different cell tvpes
involved in sperm production. M;k; ;;;;;;'".ffi?.r",1"a both mitosis and meioiis.

sv Be familiar u'ith the female re


KnowthegeneralstepsofhowanoVumdevelops'wi+*
uctive ana
of cells wit-hin the ovdry and their relarionship toin* J*rli"p*g -gg

Know how

in egg production. Make sure you understand botlifritSri" ir",Jrr-,"ioris.

Be able to and contrast male and female ction.


"w Understand whv the maie Leydig cells are analoqous to the female theca
5ertoli cells are analogous to the female follicle ceils.
cells and why the maie

Be familiar with thq different hormonal feedback mechanisms at the brain.

Understand urhat ha once fertilization of the takes place.


Be familiar wjth the process of fertilization, where ii is most likely to occur, and where
the fertilized
egg is most likely to impiant in the uterus.

Understand the hormonal relationships associated nith fetal development.

r'o*ih";;;p;;r";;;il;1,il;;il,ffi ;;ild;;iy:;ffi[",]#l1,il:S;]tr$;

Be familiar with the different


fi,V of onic deve
Developmentbeginswil.hfertilizationand'proceeds..th'o.'gh.Go,,u_g",gu@
neural crest formation, organogenesis, and'eventuatly puit"urltio". ft f;;ili* ,ri#in"r;-il;;;:
Biology Reproduction & Development Male Reproduction

MaI,€i,lffi HIU.A.U otidfi


General Reproductive Endocrinotogy
During the endocrinology of reproduction a hormone called gonadotropin
releasing hormone (GnRH) is released from the hypothalamus. GnRH signals
the anteriol pituitary to release the gonadotropins luteinizing hormonelLH)
ar-rd follicle stimulating hormone (FSH). Both LH and FSH travel to the gonads.
In the maie the gonads are the testes while in the female they are the ovaries.
Each of these gonadotropins elicits a number of different responses at their target
iissues. There are two general types of responses that we will be considering. The
:irst is the production of steroid hormones. The second is the production of germ
:eils. In the male the germ cells are cailed spermatozoa while in the femalelhey
are the ova.

Male Keproductive Anatomy


The organs which carry out the reproductive functions in the male are the testes,
:pididymis, vas deferens, ejaculatory ducts, seminal vesicles, prostate,
iulbourethrai glands, and penis. These structures are shown in Figure 4-1.

Ureter
--*
I

Prostate )
\
gland

Urethra
Seminal
vesicle

Ejaculatory
Corpus duct
sponglosum Vas deferens
Bulbourethral
Scrotum gland

Testis Epididymis

Figure 4- I
: lale reproductive anatomy.

.-11 of these structures are important in the synthesis and delivery of sperm to the
:emale vagina. The actual production o{ sperm is referred to as spermatogenesis.
lecause the formation of sperm requires a temperature lower than that of body
:emperature, they are produced in the testes which iie in the scrotum outside of
-ile body cavity. Within each testis are a series of convoluted tubules called the
seminiferous tubules. It is within these tubules that we find the spermatogenic

di
Jopyright O by The Berkeley Review 209 The Berkeley Review
Specializing in MCAT Preparation
Biotogy Reproduction & Development Male Reproduction

cells. Not oniy are sperm produced within the testes but the hormone
testosterone is synthesized within specialized interstitial cells called Leydig celts
that lie outside the seminiferous tubules. A third cell type found within the area
of the seminiferous tubule is the Sertoli cell. As we will see, these cells act to
promote spermatogenesis and they also produce the protein hormone inhibin.

Sperm Production
The target tissue for hormones like FSH and LH are the gonads. In the male the
gonads are the testes. Within the testes are a series of convoluted tubules
referred to as the seminiferous tubules. There are severai different cell types that
interact within the seminiferous tubules to allow for the development of
spermatozoa. Between adjacent seminiferous tubules are the Leydig cells (also
called the interstitial cells). If we were to take a cross-section of a seminiferous
tubule, we would note a lumenal space in the center. Adjacent to the lumenal
space are fairly large cells called Sertoli cells. In direct contact with the Sertoli
cells are a series of spermatogenic cells. As you proceed from the basement
membrane of the seminiferous tubules toward the lumen, the cells become more
and more differentiated. In other words, those cells are becoming more and more
developed. An important aspect of this development is that the Sertoli cells are in
constant contact with the spermatogenic cells. You could view this as the Sertoli
cells regulating the development of the spermatogenic cells.

Let's consider the development of the spermatogenic cells. If we consider a


portion of the seminiferous tubule, we will see a basement membrane separating
the Sertoli cells from the interstitial cells. The Sertoli cells are in contact with one
another and essentially form a ring around the inside of the basement
membrane. Different developmental events take place on different sides of the
Sertoli cells. For example, spermatogenic cells closer to the basement membrane
turn out to divide in a completely different way than they do further away from
the basement membrane. The spermatogenic cells that are closer to the basement
membrane are called spermatogonia. They have 46 chromosomes.

The way that spermatogonia divide is by mitosis. The resulting "daughter" cells
are exactly the same as the parental cell. In the area ciosest to the basement
membrane the spermatogonia are undergoing constant division. Recall that the
first thing that happens in mitosis is duplication of the chromosomes. These
chromosomes eventually migrate towards the cell's equator and the cell
undergoes cytokinesis to produce two daughter cells which are exactly like the
parental cells. The spermatogonia that undergo mitosis by the basement
membrane are referred to as primary spermatocytes. As the primarv
spermatocytes form they begin to squeeze by the juxtaposed Sertoli cells and
move towards the lumen of the seminiferous tubule. In the area near the lumen a
completely different set of events takes place. The primary spermatocytes
undergo meiosis and will reduce their genetic complement to 23 chromosomes.

Meiosis is simply one duplication with two divisions. Recall that during the
meiosis the first thing that happens is duplication of the chromosomes. The
chromosomes line up at the cell's equator and the cell undergoes cytokinesis. The
cells that are formed after the first meiotic division are called secondary
spermatocytes. At this point a second meiotic division takes place. Each
secondary spermatocyte will again divide to give two daughter cells. The
daughter cells of the second meiotic division are referred to as spermatids and
they have half the chromosomal complement (i.e., 23 chromosomes) as the

Copyright @ by The Berkeley Review 2to The Berkeley Review


Specializing in MCAT Preparation
Biology Keproduction & Development Male Reproduction

original spermatogonia. Therefore, every primary spermatocyte that undergoes


mitosis and meiosis ends up producing four spermatids.

The very next event that takes piace is a transformation event. Each of the
spermatids is transformed rnto a spermatozoa. The transformation process has
an interesting characteristic to it. When the spermatids are initially formed, they
are all connected to each other by cytoplasmic bridges. During the
transformation process the spermatozoa essentially bud out from the spermatids.

At the very tip of a spermatozoan there is a structure called the acrosome which
contains a lot of digestive enzymes. These digestive enzymes help the
spermatozoa gain access to the interior of the egg once fertilization has taken
place. Inside the head of the spermatozoa is a nucleus which contains DNA. in
the midsection are mitochondria, providing energy for the whipping movement
of the tail. This enables the sperm to swim towards their destination (the egg).

Hormonal Control of Sperm Production


It is important to remember that the spermatogenic ce1ls are in constant contact
with the Sertoli cells. The Leydig ceils have a rather special function. They
convert cholesterol to testosterone (which we can abbreviate as T). Testosterone
has several roles. Testosterone can diffuse out of the Leydig ceils and move to
other target tissues of the body. It can aiso leave the Leydig celis and diffuse into
the Sertoli cells. Within the Sertoli cell testosterone binds to a specific receptor
and is converted to a compound called dihydrotestosterone (which we can
abbreviate as dHT). This complex diffuses into the nucleus of the Sertoli celi and
instructs the DNA to synthesize RNA. It is the products of the RNA synthesis
that affect the spermatogenic cells. See Figure 4-2.

Recall that we mentioned that the hypothalamus releases GnRH which then acts
on the anterior pituitary and causes it to release FSH and LH. These two
gonadotropins irave a direct effect on the Sertoli cells and Leydig cells. LH binds
to a specific receptor on the membrane of the Leydig cell. The secondary
messenger produced from this binding interaction increases the conversion of
cholesterol into testosterone. FSH has a different function. FSH will bind to a
surface receptor on the Sertoli cell and induce a secondary response within the
Sertoli cell. This secondary response helps convert testosterone into
dihydrotestosterone. It also induces the synthesis of the receptors. [A thought
question. If you had to make a male contraceptive, what events would you
disrupt in this pathway? You would disrupt the production of
dihydrotestosterone or FSH. One of the problems with producing a male
contraceptive is that it is rather difficult to stop the synthesis of FSH.I

As with any hormonal system there is always the consideration of a feedback


mechanism. Lr males there is a constant production of LH and FSH. The reason is
due to a feedback mechanism. One of the consequences of testosterone diffusing
out of the Leydig celis is that it has a negative feedback at the anterior pituitary
and at the hypothalamus. Testosterone is preventing the synthesis of GnRH and
LH. The Sertoli cells are involved rn a different feedback mechanism. The Sertoli
cells also secrete a compound called inhibin. Inhibin acts as a negative
modulator of the anterior pituitary. The reason that the gonadotropin levels are
relatively constant is that you have a constant synthesis of the spermatogenic
cells. If testosterone levels are too high, the feedback mechanism will decrease
the levels of LH. If the levels of dihydrotestosterone are too high, there is an
increase in inhibin svnthesis which results in a decrease in the levels of FSH.

Copyright @ by The Berkeley Review 2ll The Berkeley Review


Specializing in MCAT Preparation
Biology Reproduction & Development Male Reproduction

Hypothalamus
I
GnRH

(-)
<:.::-.-_ I
I
I
I
I
I
I
I
Testosterone

U)
o
t
dHT
Cholesterol O
oc
\l,
\
CD

Testosterone o
@

loffi]
I T*Q:?
RecePtor I A
a

I I
Leydig Cell membrane Sertoli Cell
I

Figure 4.2
Hormonal regulation in the testes.

Secondarlr Sex Characteristics in the Male


Almost all of the secondary sex characteristics in the male are due to testosterone.
some of those characteristics are development of a beard, pubic hair, deepening
of the voice, texture of the skin, muscle distribution, bone development, sexual
drive, and, if you are genetically predispositioned to it, baldness.

Sperm Delivery
Sperm leave the seminiferous tubules and enter into the epididymis and then the
vas deferens. Sperm are stored in the epididymis and vas deferens for about L4
days before ejaculation. Contraction of smooth muscle tining the wal1s of these
structures ejects the sperm down the vas deferens and into the ejaculatory duct.
The seminal vesicles, prostate gland, and bulbourethral gland secrete
components that make of the remainder of the fluid that is ejaculated (now called
semen). Some of the components secreted by these glands are fructose vitamins,
bicarbonate zinc, prostaglandins, and mucus. The total volume of the fluid
ejaculated with the sperm is about 3 to 5 milliliters.

Copyright @ by The Berkeley Review 212 The Berkeley Review


Specializing in MCAT Preparation
Biology Reproduction E( Development Female Reproduction

Feffial€,]REp'fddub i
Female Reproductive Anatomy
The essential anatomical features of the female reproductive system includes the
r-agina, uterus, Fallopian tubes, and ovaries. Once sperm have been ejaculated
irto the vagina they can iive for about 48 hours. However, about a half an hour
after ejaculation the leading sperm arrive at the site of fertilization which is
usually the oviduct (also called the Faliopian tubes). The fertilized egg continues
to move down the oviduct towards the uterus where it will implant in the uterine
Lrning. About a week after ovulation the fertilized egg, no\4/ called a blastocyst,
rmplants in the iining of the uterus where it will continue to grow and deveiop
r-urtil parturition (delivery). A few of the anatomical structures mentioned here
are shown in Figure 4-3.

Fallopian OvarY
Fimbriae

:,".:.

Urethra
/,t
Vagina

Figure 4'3
emale reproductive anatomy.
'-

Ova Production
}e production of the female germ cells occurs via a process called oogenesis. hr
:ertain aspects this process is similar io that of spermatogenesis in males. The
-::rportant sirnilarity in the female is that when cell division is complete the ovum
.',-iii have half the DNA as the original parent cell. The regulation of oogenesis,
:iowever, is quite different. First, there are a series of events in which the oogonia
ritotically divide to form primary oocytes. The primary oocytes, which have 46
:hromosomes, are analogous to the primary spermatocytes in the male. kr the
:emale ail of these divisions occur ruithin the first two to three months of fetal
:.":oelopment. About the third month of gestation all the mitotic divisions cease. In
rther words, al1 the oogonia that a female will produce will be produced within
-;re first three months of fetai development. In fact, at birth a female will have
,'.'bout 400,000 primary oocytes in both of her ovaries. F{owevet, by the time she
:.as finished her reproductive life only about 400 will have ever matured. The
:emaining primary oocytes will have degenerated at various stages during the
'evelopmental process. This process of degeneration is referred to as atresia.

Copyright @ by The Berkeley Review 213 The Berkeley Review


Specializing in MCAT Preparation
Biology Reproduction & Development Female Reproduction

The primary oocytes will eventually undergo the first meiotic division, caused
by a surge in the gonadotropin LH. The first meiotic division happens in
monthly cycles. Some of these primary oocytes could take as long as 50 years to
have their first meiotic division. Once the first meiotic division is complete the
secondary oocyte, which has 23 chromosomes, is formed. During this division
one of the two daughter cells obtains all the cytoplasm while the other cell is just
a small sphere of DNA called the first polar body. The secondary oocyte
undergoes the second meiotic division only after fertilization hss taken place. Tine
product of that division is the ovum and a second polar body. [The first polar
body also divides to give two second polar bodies as well.] The ovum now has 23
chromosomes. v\4ren sperm and ovum unite in fertilization, the complement of
46 chromosomes will be restored (23 from the male and 23 from the female).

If we take a cross-section of the ovary, we will see a series of specific cell types
called primary follicles which are in different developmental states. The primary
follicle is actually a primary oocyte surrounded by a layer of foilicle celli. These
follicle cells are in constant contact with the primary oocyte. Eventually one of
these primary follicles will start to develop.[An exception arises with fraternal
twins in which case two follicles will form and develop at the same time.]

Theca Growing
Cells Follicle
Primary
Follicle

Zona
Pellucida

Granulosa
Cells

Regressing Corpus
Corpus Luteum
Luteum

Figure 4-4
The ovarian cycle.

In order to mature the primary oocyte estrogen, LH, and FSH are needed. This
developing time period is referred to as the follicular phase and lasts up to about
the 14th day of the woman's monthly cycle. During this period ihe primary
follicle gradually develops. Surrounding the primary oocyte will be a membrane
called the zona pellucida. The zona pellucida is surrounded by more follicle cells
called granulosa cells and finally by theca cells. See Figure 4-4. These cell types
are a direct result of the estrogen, LH, and FSH that are present. The theca cells
are analogous to the Leydig cells in the male while the granulosa cells are
analogous to the Sertoli cells in the male.

Copyright @ by The Berkeley Review 214 The Berkeley Review


Specializing in MCAT Preparation
Biology Reproduction & Development Female Reproduction

\ F
o
t
\P.os"rt"ron.
Estrogen
II \
I o
q

ffi

p
lu$
notucutar Phase
+lC=:= Luteal phase t-+l
28 Days

Fi,gure 4-5
-:: :.nrmones of the ovarian cycle.

'---mn
the primary follicie a fluid starts to build up forming the antrum. The
: :l€rir is now primed for ovulation. The next event that takes place is the LH
ir:ge. This sr,rrge causes the primary oocyte to undergo the first meiotic
;irision, forming a polar body and the secondary oocyte. The LH surge also
.;--<es the production of a series of enz)..rnes which break down the membrane in
--= orimary follicle. Once the secondary oocyte is released, ovulation has
'-:rrred. Ovulation usually comes at about the 14th day in the woman's monthly
"- le. As we have mentioned, the process leading up to ovulation is referred to as
r--= ioliicular phase, The left over follicle (after ovulation) is transformed into a
;;rd-like structure called the corpus luteum. One of the main functions of the
. -:!us luteum is to produce estrogen and progesterone. If fertilization does not
. ---;r and there is no pregnancy, the corpus luteum will degenerate and the
:.:1e cycle wiil start again. From the point of ovuiation, at about the 14th day,
*:r-l the beginning of the menstrual fiow is the luteal phase. See Figure 4-5.

fflormonal Control of Ova Production


-"":ring a woman's monthly cycle her gonadotropic hormones fluctuate rather
:--arLatical1y. LH remains relatively constant until just before ovulation at which
--:e there is a sudden surge. FSH will also have a surge, but not as great as the
LH surge. There will be a surge of estrogen before ovulation and then a smaller
,:le after ovulation. The increase in estrogen before ovulation actually leads to
':"e increase in L H. Another hormone called progesterone will surge after
-
-.-ulation as well. F{ow are these hormones regulated?

- rpyright @ by The Berkeley Review 215 The Berkeley Review


Specializing in MCAT Preparation
Biology Reproduction & Development Female Reproduction

(+)
(-)
!r r r r
Hypothalamus (-)
<... .;'.
I
GnRTI

(+) on LH
i. . . (-l Anterior Pituitary I

_ (-) on FSH :
I
I
I
I

,r//\ FSH
\"nFSH I
f
I
Inhibin I
I

O Testosterone
Cholesterol
()tr
b0 gl rl' '1,
!()
o C\0 Testosterone
n9
Ar High
level
Theca cell membrane Follicle cell

Corpus Ovaries
Luteum
Figure 4-6
The hormones of the ovarian cvcle.

In the female the theca cells convert cholesterol into testosterone. The
testosterone diffuses into the follicle cells where it is converted into estrogen. One
of the effects of estrogen is to help in the development of the primary foftcle (i.e.,
the primary oocyte). We had mentioned that ihe primary oocyte reaches maturit.v
by undergoing reactions with estrogen, LH, and FSH. Recall that the anterior
pituitary secetes LH and FSH. LH affects the theca cells while FSH affects the
follicle cells. As estrogen is being synthesized, the primary oocyte is developing.
At the same time we have a proliferation of the follicie cells. If we have more
follicle cells, then we will be able to slmthesize more estrogen. This is where the
first estrogen surge comes from. It turns out that at low concentrations of
estrogen there is a negative feedback on FSH production. See Figure 4-6.

Flowever, the follicle cells are growing and at a certain point in time there is a
threshold level in estrogen. This ineans that relative to the system we no longer
have a low concentration of estrogen but instead have a high concentration of
estrogen. At high concentrations we see that estrogen has a positive feedback on

Copyright @ by The Berkeley Review 216 The Berkeley Review


Specializing in MCAT Preparation
Biology Keproduction Er Development Female Reproduction

the anterior pituitary and LH production. Thus, with a high level


of estrogen
production we now have the LH surge. This is import"ant. The increased
concentration of estrogen is having a positive feedback and therefot"
sees these"u"ri"g
more LH production--giving tfe LH r,-rige. The reason that one
is because of the rapid growth of the rouicte cerls during deveropment
,rrg"!
or ine
primary follicle.

Immediately following LH surge the levels of LH clrop down to very low


-the
levels. \Ahat causes the drop in LH utrd th" increase in estrogen und progesterone
curing.the iuteal phas.el If we no longer have a high concentration
of lstrogen,
*'e will lose the positive feedback on the anterlor pituitary. Recall
thai ai
ovulation all the other follicle cells become transformed into the
corpus luteum.
ftuY, no longer follicle cells. During that transformation they transientiy
1T lose
lhe ability to produce estrogen. f-hls abihty to produce estrogen
is lost lusi tong
enough so the LH ievels decrease. However, the revels ofioth
:rogesterone begin to increase. \M-rere is this coming from? "strog"r,
u'i

The corpus luteum becomes an endocrine gland and begins


to synthesize
estrogen and progesterone. Estrogen and progesterone,
in combination, have a
negative feedback on LH- and FSH ptod,rciion ir-, the anterior
pituitary as we1 as
:- negative feedback on the hypothilamus
and the synthesis or cnnH. what is
:ne result of having a negative feedback on LH and FSH?
Essentially what is
rappenrng is that the primary follicle is prevented from developing.
As long as
'ne corpus luteum is pr9{uginq estrogen Ld progestetrone, the primary foilicie
is
;nable to develop. But this is zuhat rue want to inppZn. At this poiit
we do not need
'rother primary foliicle being made. lwith this information, what is the basis
for
:-rnale oral contraception? Birth control pills are essentiaily
a combination of
:strogen and progesterone in concentrations that represent
the needs of the
: ''oman.] Remember, estrogen
arone has a positive feedback. Estrogen and
::ogesterone in combination have a negative feedback.

'r different set of errents takes place during pregnancy. what happens
when
'..e 'er)r
spermatozoan comes in coniact with the orium? ih"
of the
:'-^ermatozoan allows for the digestion of the membrane of the ".rorome
secondary oocyte.
the membrane of the ovum (zona pellucida) are receptor sites for
""-ithin the
::erm' These receptor sites prevent cross-species fertilizatiorr. Thu only thing
that
- .ters the secondary oocyte is the nucleus of the sperm. After the nucleus of the
j:erm enters the secondary oocyte the zona pellucida
changes and prevents any
. ::'Ler spermatozoa from entering. This
process js referred to as fertilization. At
-- rs point the secondary oocyte undergoes the second meiotic division
--.-: ovum and to form
the poiar body. The nuileus of the sperm and the nucleus of the
::g fuse together to form the zygote, which now has a complemeni of 46
:,-.:omosomes.

- -e zygote rapidiy begins, to deveiop and in about 7 d,ays


attaches itself (as a
:-astocyst) to the uterine lining. A blastocyst is essentiaily a smali
baII of cells
-} a central fluid-filled cavity. \zVhen the implantation takes place the placenta,
::de up of maternal and fetai cell types, begins to form.
lr
ii
'1at one does not want to happen during pregnancy is the development of
pt"gn*iy there aie high levels oiestrogen
'-- other prirnary follicle. Throughout
.---i progesterone. The reasons for maintain'ing high levJls of estrogen ind
:: rgesterone are different during the first three months of pregnancy than they

.ry'right O by The Berkeley Review 217 The Berkeley Review


Specializing in MCAT preparation
Biology Reproduction & Development Female Reproduction

are during the last six months of pregnancy. shown in Figure 4-7 is adiagram
of
the hormonal considerations during the iirst three moriths of pregnaniy. The
placenta itself is an endocrine gland. During the first three monthJ thu
luteum is still a viable gland and it secretes estrogen and progesterone. "orpr6
The
placenta synthesizes chorionic gonadotropin (CG) *ni.n stlmulites the corpus
luteum to make estrogen and progest"totr". orre of the best pregnancy tests is to
detect cG in the blood-stream. The only time CG is made is aurLg thl first three
months of pregnancy.

Corpus Anterior
Luteum Pituitary

cclf r.,

Placenta Mammary
Glands

Figure 4.7
The placenta and its relationships.

what are the functions of estrogen and progesterone? Estrogen and progesterone
have a negative feed-back on the anterior pit.titury, tnus titttUitinglH"and rSH
production. This prevents the formation of the primary foihcles during
pregnancy. It,also means that ovulation and menstrual cycles will be eliminateJ
Chorionic for the duration of the pregnancy. Another important gland during pregnancy is
Gonadotropin the mammary gland- There are a series of hormones 6at positivell.feeiback on
the mammary glands. one hormone, prolactin, from the anterior
o
d
"o*". chorionic
pituitary. The placenta forms another hormone called somato-
mammotropin (cs) which also acts on the mammary glands. They help the
o marunary glands to grow. Estrogen and progesterone have positive feedback on
the mammary glands as well.
x
IJ.]
During the last six months of pregnancy there in only a slight variation on this
theme. CG is no longer made. This results in a loss of feedback to the corpns
D luteum. After three months the corpus luteum breaks down and the supply of
o 7 2 3 4 5 6 7 8 9 l0
estrogen and progesterone from the corpus luteum comes to a halt. Howevei, at
Months after start of iast
that point in time the placenta itself starts to synthesize estrogen and
progesterone. In fact, the placenta makes much more estrogen and progesterone
menstrual period
that the corpus luteum did. This can be seen in Figure 4-g.
Figure 4-8
Placental synihesis of Secondary Sex Qharacteristics in the Female
estrogen and progesterone. Some of the effects of the female sex steroid estrogen involve development of
female body configuration, growth of breasts, growth of external genitaLia,
pattem of pubic hair, fluid retention, influence on the cardiovascular syslem, and
it may be involved in inhibiting atherosclerosis.

Copyright @ by The Berkeley Review 2ta The Berkeley Reviev


Specializing in MCAT Preparation
Biology Reproduction & Development Developmental Stages

DE,VEl;ogffiilffilstegC$
Recall that when we discussed meiosis we mentioned that the haploid sperm and
the haploid egg can unite to form a zygote. The zygote will undergo a series of
mitotic ceil divisions which will form a clone of cells. A11 of those mitotically
dividing cells were derived from a singie precursor cell and eventually they wiil
give rise to an embryo. The embryo wiii continue to develop and lafer foims a
sexually immature juvenile. The juvenile undergoes a maturation process and
becomes the adult.

Thoracic Cavity

)l
Spinal Cor
t
,t:- Abdominal Caviry
- /!

Figure 4.9
-..e body outline.

-:- our discussion on development we will focus on the z:ertebrate.In particular,


' e wiil want to examine the arrangement of the body plan in vertebrates. As we
.-li see, human beings, as all other vertebrates, have essentiaily the same body
:-an. For example, the skeletal system in vertebrates heips to support the body.
}e aspect of the skeletal system is a dorsally located vertebral iol.tm.,. Within
-:.:s vertebral column is the spinal cord. Anterior to the spinal cord is the brain.
- ie brain and the spinal cord comprise the central nervous system. Nervous
:-ssue outside the central nervous system is referred to as being part of the
;eripheral neryous system. ventral to the vertebral column is the body cavity, or
:telom. The coelom is divided into the upper thoracic caaity ur,d thu lower
'-.bdominql caoity. The thoracic cavitv contains the heart and lungs while the
-:dominai cavity contains a variety of organs like the stomach and the iatestinal
--.-stem. These basic iandmarks are shown in Figure 4-9.

he body is composed of a variety of highly differentiated cell types. For


=''.ample, there are stratified epithelial cells found in the epidermis. Cuboidal

-:,pyright O by The Berkeley Review 219 The Berkeley Review


It Specializing in MCAT Preparation
rn
Biology Reproduction & Development Developmental Stages

epithelium can be found in the kidney. Smooth muscle cells can be found in the
walls of the intestine. Skeletal muscle cells can be found in the voluntary muscles
and cardiac muscle cells can be found in the heart. This is just a sample of some
of the cell types fond in the body. Cells of similar type form tissues and tissues
can come together to form organs like the heart and liver. During development
the cells, which initiaily all start out alike, begin to grow and differentiate. This
happens even though those cells all contain the same genetic constitution. Th"y
all have the same genes. Not only is there differentiationbut some of these cells
undergo morphogenesls (i.e., they begin to take on different shapes).

There are a number of stages that one can use to characterize development (see
Table 4-1). The first deaelopmental stage involves the union of the male and
female gametes. This process is called fertilization. The gamete producing
organs are the gonads. In the male these are the testes and in the female they are
the oaaries. V\4ren the gametes unite a zygote is formed. The next developmental
stage involves cleaaage of the zygote. During cleavage the zygote rapidly
divided into many smaller cells without an overall increase in size. Gqstrulstiotr
is the third developmental stage. Many embryologists consider this to be the
most important part of an organism's development. During this stage the cells of
the zygote move to form the three primary germ layers (ectoderm, mesoderm,
and endoderm) of the organism. The next developmental stage is neuyulation.
During neurulation we begin to see the formation of the nervous system. This is
the first organ system to begin differentiation. Neural crest f.ormation is the fifth
developmental stage. The neural crest cells help to form parts of the nervous
system, skull, and sensory organs. The last stage of development is
olganogenesls. It is during this stage that the different organs of the body are
formed.

l. Fertilization
2. Cleavage
3. Gastrulation
4. Neurulation
5. Neural Crest Formation
6. Organogenesis

Table 4-l

Development In The Ffog


We can uie the frog as an example of vertebrate development because of ihe
large size of its eggs (about 1.5 millimeters in diameter). The life cycle of the frog
is shown in Figure 4'10. As we have mentioned, males give rise to haploid sPelm
(1N) while females eventually give rise to haploid eggs (1N). Initially the female
frog will give rise to a primary oocyte which then undergoes the first meiotic
divlsion. The secondary oocyte is formed but it is arrested at metaphase of the
second meiotic division. This second meiotic division is not completed until the
sperm from the male frog fertilize the secondary oocyte. Fertilization initiates a
pio."rr called egg activation. This is a signal for development to begin. The
lecond meiotic division in the secondary oocyte is completed and the zygote
nucleus is formed. This nucleus is now diploid or 2N. The zygote undergoes a
series of cleavages and an embryo is formed. Further cell division leads to the

Copyright O by The BerkeleY Review 22o The BerkeleY Keview


Specializing in MCAT PreParation
Biology Keproduction & Development Developmental Stages

formation of the tadpore. The tadpole, being an aquatic organism,


eventualy
loses its tail and forms legs so
it can begin its ierrestriat life aJan adult frog. This
is a process of metamorhosis.

Male 2n

M eta morph osis

6
o
Errly
t.rdpole Zr
Secondary spermJtoc ytes

,'{-h\
AJ.lr\\
l{. /.. \ ,l ) Enrbryo 2z

\i,/
ffff
5permatids
Cleavage
,/---\
//'-\\
({b ))
f Ito la r
body

€lsqP - \;/-----/./
Lgg's second 1 ,'
meiotic division

\ \ \\\'
\\
\\-
\ Spern

Figure 4-lO
The life cycle of the frog.

o egg there is a large amount of yolk that resides in the zsegetal


-eg."tf",ttilized
gole (i.e., the lower hemisphere). This yotk will act as food for the devetolping
,animsl p.ole (i.e., the upperhemisphere) contains mainly cytoplasml
n a fertilized egg an interesting phenomenon iakes place. on the side of
=blyo:Jhe tir" ugg
:pposite to where the sperm penetrates the egg's membrane, a structure called
ie gray crescent forms. The gray crescent islocated on the dorsal aspect (the
:uture back) of the animal. This is where we will eventually find the spinal cord
:nd brain. The side opposite the dorsal aspect is tine oentral aspect. This is where

lopyright @ by The Berkeley Review 221 The Berkeley Review


Specializing in MCAT preparation
Reproduction Er Development Developmental Stages

the belly of the organism will form. An imaginary line which connects the two
poles is called the meridian. Imaginary lines which run around the circumference
of the cell are equatoriai in nature. The middle of the gray crescent defines the
body midline of the future organism. By the time the gray crescent has appeared
the main axes of the body of the organism has been established.

Cleavage
Once the zygote is formed it undergoes a special cell division to increase its mass
but not its overall size. The first ceilular division occur along the body midline
which bisects the gray crescent. This division gives two cells. The next division
also occurs along the meridian, but this time at right angles to the last division.
We now have four cells. The third celluiar division occurs equatorially and gives
four cells in the animal pole and four cells in the vegetal pole. The individual
cells involved in this growth are called blastomeres. Eventually a small, solid ball
of cells will be formed called a morula. Further celluiar division results in the
formation of a hollow ball of cells called the blastula. Within the blastuls is a
fluid-filled cavity called the blqstocoel. These structures are outlined in Figure &
11.

dlflD#Tffiffi
Animal Pole

Vegetal Pole
\y \.1.7 V
Grey
/ \17
Cleavage Down Meridian Equatorial Meridian
Crescent Body Midline Cleavage Cleavage Cleavage
Unfertilized (2 cells) (4 cells) (8 cells) (16 cells)
Foo
"bb

Blastocoel

Morula Blastula Cross Section


(32 cells) of Blastula

Figure 4- I I
Different stages in embryonic development.

Gastrulation
During gasf rulation rearrangement of cells occurs. Not far from the gray crescent
an open develops in the blastula called theblastopore. Cells from the animal pole
begin to migrate inwards through the dorsal lip of the biastopore. As this outer
layer of cells migrates inward they form a second layer of cells immediatelr-
below that outer iayer. The blastocoel is reduced in size and is eventuallr-
eliminated. in its place a new cavity is formed called tlte archenteron. At thts
stage the embryo is referred to as a gastrula (because gastrulation has taken
place). Invagination of this outer cell layer produces two cell layers. The outei
cell layer is called the ectoderm. The inner cell layer is called the endoderm. A
layer of mesoderm will later form between these two cell layers. These structures
are shown from different viewpoints in Figure 4-12.

Copyright @ by The Berkeley Review 222 The Berkeley Revie*


Specializing in MCAT Preparation
Biology Reproduction & Development Developmental Stages

The events that lead up to gastrulation differ widely in the animal world.
Flowever, gastrulation is common to all of them. It seems that the generation of
the three cell layers (ectoderm, mesoderm, and endoderm) has been conserved in
throughout the evolutionary process. An animal cannot be made if gastrulation
does not occur.

Blastocoel

Dorsal Lip
of Blastopore Dorsal Lip
of Blastopore
Blastopore

Archenteron

Blastocoe
Dorsal Li
of Blastopore

Ectoderm
Lateral Margin
ofEndoderm Mesoderm Dorsal Lip
of Blasto
Dorsal Lip
Archenteron of Blastopore Yolk Plug
Yolk Plug

Remains of
Blastocoel

Figure 4-12
Tire process of gastrulation.

lle ectoderm, mesoderm, and endoderm cell layers play different roles during
ire developmental process. They have different deuelopmental fates. For
:rample, the ectoderm will eventually differentiate into structures like the skin,
--ne lens of the eye, and the brain and nervous system. Mesoderm will
tiiferentiate into structures like the notochord, heart, skeleton, muscle, the outer
:overings of internal organs, and the reproductive organs. Endoderm will
iiJferentiate into the inner lining of the digestive tract and the respiratory tract,
and major glands of the body like the liver and pancreas. These are just a few of
--he clevelopmental fates of these cell layers.

Xeurulation
During this stage of development the ectoderm, mesodetm, and endoderm begin
:o form the structures that will eventually define the embryo and later the adult.

Copyright @ by The Berkeley Review 223 The Berkeley Review


l Specializing in MCAT Preparation
I
Biology Reproduction E( Development Developmental Stages

During neurulation the formation of the notochord takes place along the body
midline. This structure is derived from mesoderm. Superior to the notochord is a
mass of ectodermal cells called the neural plate. The neural plate will begin to
fold in on itself and form the neural grooae. As the edges of this folding fuse with
one another the neural tube is formed. Within the neural tube wili form the
spinal cord (encased in the spinal column) and anterior to the spinal cord will
form the brain. In other words, the neural p1ate, which is composed of
ectodermal cell, gives rise to the neraous system. This tissue is referred to as
primordium (from the Latin primus, first, + ordior, to begin) and it is the earliest
stage of development of a structure. During neurulation the embryo is sometimes
called aneurula.

As the neural plate begins to fold in on itself it will form the neural groove. Two
views are shown in Figure 4-13. Belolv the neural groove is the notochord and on
either side of the notochord is mesoderm. As the neural groove begins to form
the mesoderm is split and forms a coelom (i.e., a body cavity). The lungs will
eventually develop within this coelom. When the edges of the neural groove fuse
together the neural tube will be formed. Within the neural tube forms the spinal
cord and anterior to the spinal cord forms the brain.

Neural Crest

Neural Crest
Cells

Neural
Plate I Neural Tube
Notochord

Neural Neural
Neural Tube
Groove
Plate
Notocho
Mesoderm
Ectoderm Coe

Figure 4- l3
Neural tube formation.

Neural Crest Formation


As the edges of the neural groove fused together and became the neural tube,
specialized ectodermal cells were left in a more dorsal position on the neural
tube. These specialized ce1ls are called the neural crest cells (Figure 4-1-3)' As
these cells begin to move to the sides of the developing embryo, they begin to
functionalize. For example, ectodermal cells in the anterior portion of the
developing embryo associate with the neural crest cells and forms placodes.
These structures will eventually form the sense organs located in the head. Some
neural crest cells will help lorm sensory cells (e.g., olfaction and touch). Other
neural crest cells will form the adrenal medulla. During times of stress the animal

Copyright @ by The Berkeley Review 224 The BerkeleY Revien-


Specializing in MCAT PreParation
Biology Reproduction & Development Developmental Stages

will experience the "fight or flight" syndrome. In order to respond to this stress
the adrenal medulia releases adrenaline. This hormon" prepurur the animal to
respond by increasing biood sugar leveis, heart rate, and blood pressure.

Organogenesis
During the initial stages of organogenesis there is an interaction between
ectoderm and mesoderm. The neural fube becomes longer and thinner and has
an anterior to posterior developmental gradient. Neural crest cells begin to
migrate and take up positions in the vicinity of the neural tube. Mesodermal cells
migrate towards the neural tube. Eventually they will form the vertebral column.
The brain begins to form at the anterior portion of the neural tube. Optic vesicles
begin to form. As the neural tube continues to form segments of mesodermal
tissue called somites begin to appear. The somites will eventually give rise to the
vertebrae, connective tissue, and the muscles of the body.

In the frog embryogenesis is complete with the appearance of the sexualiy


irnmature tadpole. These cteatures have gills and live in an aquatic environment.
They have no limbs. Instead, they have a tail which is used to propel them
through the water. The last critical phase in the life of a tadpole L one of
morphogenesis. The tadpole will change from a sexually immafure organism into
a sexually active organism, which is the frog. During this process of
morphogenesis the tadpole grows limbs, develops lungs, loses its tail and its
gi-Ils. This change is initiated by a hormone which is reieased during
a specific
lirrre of development. The hormone which is released is thyroxin and it is
released from the thyroid gland.

\Iany organisms pass through a larval stage (sexuarly immature) like the
:adpole. \rvhen the frog eggs hatch the tadpoles are left to fend for themselves. If
Jre post-embryonic organism is left on its own, then the development is termed
ndirect. IrL contrast, -direct development involves care being given to the post-
embryonic organism by the mother. For example, a human fetui develops within
--:e female of the species. Even after birth the female (and
male) care for the
'.-oung until they are able
to take care of themselves.

- ,r1'right @ by The Berkeley Review 225 The Berkeley Review


Specializing in MCAT Preparation
Biology Reproduction & Development Developmental Mechanisms

Developmental Mechanisms
If the cells of the body all contain the same genetic information, how can they
become so differentiated? There are two geneial classes of interaction associated
with the differentiation of cells that we need to consider. There are intrscellular
interactions which involve interactions between the components within the
cells
themselves. There are also intercellulqr interactions in which the interactions
are
between cells. The intracellular interactions usually result in the setting up of a
prepattern. The intercellular interactions usually undergo deaeloimintal
induction.

Intracellular Interactions
we have mentioned that the unfertilized egg has an animal pole, which contains
the cytoplasm, and a vegetal pole, which contains the ytk. The egg is not
homogeneous. However, at this point in the egg's existence a longitudiria"l
axis is
already present. In the frog the animal pote witt give rise to the head while
the
vegetal pole will give rise to the tail. This is true for the frog but it
is not a truth
set in stone. Remember, there are always variations on a give"n theme.

Recall that after fertilization and induction of development the gray crescent
f9r1s at a point opposite to where the sperm penetrate-d the egg. T;he formation
of the gray crescent is due to an intracellular interaction. The"iormation of the
gray crescent is not a prepatterned phenomenon that took place before
the sperm
interacted with the egg. This is clearly shown by the fact that the gray crescent
developed after the penetration of the egg by the sperm. The entry*of ihe sperm
in the animal half of the cell is random. However, one entry is made and the grarv
crescent forms, the dorsal midline is established. This allows us to define
directions. In other words, prior to the first cellular division the axes of the
organism is established. In this case a prepattern is laid down which is adhered
to during the rest of development.

llans Spemann
During the 1920's Hans Spemann was able to demonstrate that the gray crescent
is an important landmark in the future development of the embryo. In his
experiment, spemann took a newly fertilized frog egg and tied a string around it
such that the string bisected the gray crescent (see Figur" 4-l4a). He Jlowly tied
the string tighter and separated the egg into two hal-ves, each with half a grav
crescent. Two separate blastomeres developed and two separate embryos riere
formed. since these embryos were twins, ihir proc"dure was called iwinning.
This experiment demonstrated that each blaitomere was equivalent in G
potential to form a complete embryo. Even though two complete embryos were
formed, they were smaller than the normal frog embryot. wnyz Because when
the fertilized ugg was cut in half, each blastomere receivea nil1 of the original
amount of yolk.

There is one other important point to make note of in this experiment. During ttle
last lecture we said that the body midline bisected the gray crescent and thaithis
was what eventually formed the neural tube. In the spemann experiment the
body midline was itself bisected. In other words, instead of the midhne being in
the middle it was to the side of the bisected gray crescent. However, the embrlm
developed normally. what this means is that the midline had to migrate to a
position which was in the middle of the bisected gray crescent. Hence, Ih"r" *o

Copyright @ by The Berkeley Review 226 The Berkeley Revier


Specializing in MCAT preparation
Biotogy Reproduction & Development Developmental Mechanisms

a deaelopmental regulation inside the blastomere which allowed for the


movement of the body midline.

Spemann did the same experiment on another fertilized frog egg but this time
he
cleaved the egg at right angles to the cleavage in the pt"r1o.ir""^periment (see
Figure 4-14b). Two blastomeres were formed. one blaitomere had the compiete
gray crescent while the other blastomere did not contain any piece of the gray
crescent. As development proceeded a complete embryo was from the
blastomere with the complete gray crescent. The other blastomere formed a mass
of cells that did not differentiate into an embryo. It did not gastrulate. Again, the
embryo that was formed was smaller due to the reduction in the amount of volk
received. These two experiments proved for the first time that there *u, u
Jl"u,
crepattern during the development of an egg.

(a) Bisection Bisection


.+.- I
/'"T-,\
^Gray i_
Crescent)>\itry r ) (i\
t-"1$' {-
\\J-,/
Grav
Crescent
I
I

fr
/1\
+ I
zr-f|f\\
/t'\ /'l\
[ \iti,.,+." ) \\.rll1-,t
J4.l

ng
f^> /^>.(
gs
\
/

lr
11
f /
("\
/,{
ti I 1r
\9 9
l

Complete Embryos Undifferentiated Complete


will Deveiop Mass of Cells Embryo

Figure 4-14
:i:ns Spemann's gray crescent experiment.

Hilde Mangold and [Ians Spemann


- 7924Hilde Mangoldperformed an experiment in which she removed a section
: the dorsal lip from one species of salamander embryo (before gastrulation
.:arted) and transplanted it into the belly of another species of salamander
=nbryo. These two species differed in their pigmentation 1tn" donor being of
,-:hter pigmentation that the recipient). Recall that the dorsal tip is that irea
'""-here cells from
the animal pole begin to invaginate into the blastula. This
:rocess leads to the formation of the blastopore and the development of the
:ichenteron. The blastopore, and hence the dorsal lip of the blastopore, form
:-cse_ to theboundary of the vegetal hemisphere and the gray crescent.
[n Figure
1-14b one blastula did not receive the gray crescent. Therefore, it could not
::r'elop the dorsal lip and ectodermal cells could not invaginate. This resulted in
:,e blastula never developing into an embryo.]

- rpyright @ by The Berkeley Review 227 The Berkeley Review


Specializing in MCAT preparation
Biology Keproduction & Development Developmental Mechanisms

The recipient salamander embryo now has two dorsal lips. \Alhen it developed it
formed a second embryo (of a lighter pigmentation) at the location of the
transplant (Figure 4-15).

The transplanted dorsal lip deveioped into a notochord. In turn the notochord
induced the formation of a neural plate and eventually a neural tube. What this
experiment is saying is that the ectoderm of the recipient salamander was
induced by the dorsal lip of the donor to take on a fate that it was not normallr'
designed to fulfill. In other words, the dorsal lip acts as an organizer and
organizes the cells of the recipient such that a second embryo is formed. This
process later became known as embr)'onic (or developmental) induction.

The mesoderm is removed


opposite the dorsal lip of Mesoderm is removed from a donor embryo near the
recepient embryo. dorsal lip and transplanted into the recepient embryo.

'*> @ a
^ f"'4*r
'T f: \2$

;ry Recepient Embryo Recepient Embryo Donor Embryo

Primary
Primary Neural Secondary Neural
Neural Fold
Development

)
+
Secondary Development of a double embryo.
Neural Fold

Figure 4- 15
Transplant of the dorsal lip.

Hilde Mangold was a student of Hans Spemann when they did this experimm:
She later died while cooking in her kitchen when her gas stove blew up. -{
number of years later Spemann got the Nobel Prize for his participation in ti':s
work.

In another experiment Spemann examined a chain of successiae inductions th,n


leads to the formation of the eye. In the anterior most portion of the neural tuire
(the forebrain) there are two symmetrical protrusions from the developing brab*
These protrusions extend to the ectoderm. Upon contact the ectoderm begins trn
invaginate and pushes into the growing optic stalk. This creates lhe optic ry
The optic stalk will eventually become the optic nerae. The lens (placode)of rm
eye develops from ectoderm which is in contact with the edges of the optic on*
As the developing lens pulls away, the remaining ectoderm develops into =u

Copyright @ by The Berkeley Review 224 The BerkeleY Reriem


Specializing in MCAT Preparatic
Biology Reproduction & Development Developmental Mechanisms

cotrre&. The remainder of the optic cup develops into


the retina,which contains
the photoreceptors necessaly foi vision (Figure 4-16).

Ectoderm
Optic Cup

Optic Stalk
#y Invagination

Figure 4- I6
)evelopment of the eye.

Erperrments like these indicate that there is a hierarchy


of developmentar stages.
)uring the 1930's this gave rise to the field of chemicai
Researchers
ai the time thought that induction was due to some kind"*ury"logy.
:rrms out that this line of thought was a waste of time
of biochimi"ut ii
ug"rrt.
and effort."whyi
\{olecular biology had not yet been invented. The selfish
g"r" ,,""a"d to be
=rplored.

- rpyright @ by The Berkeley Review 229 The Berkeley Review


Specializing in MCAT preparation
Biology Keproduction & Development Human Embryo Development

,ruffiH. t*ti;+,...'1,,, ;' , ,1' .,,.

Recall that fertilization of the egg by the sperm usually takes place in the oviduct
(Fallopian tube). Within the first 35 hours following fertilization the zygote will
undergo its first mitotic division. It will continue to divide as it travels down the
oviduct toward the uterus. By the fifth or sixth day the embryo will reach the
uterus. At this point the embryo is called a blastocyst. The blastocyst is a hollow
ball of cells with a mass of cells on one side.

The surrounding cells which form the ball are called trophoblast; the others are
called the inner cell mass. If the lining of the uterus is prepared to receive the
embryo, then the embryo will implant. fOtherwise, the embryo is rejected and
sloughed off during the next menstrual period.] After implantation the
trophoblastic cells grow into the lining of the uterus. In fact, the cells of the
trophoblast grow little finger-like projections into the uterus.

As the fetus gets bigger it will need to get nourishment from the mother's blood.
The blood vessels from the umbilical cord will grow into the projections of
trophoblast. There is a layer of cells between the trophoblast and the fetal blood
vessels called the chorion. The chorionic villi are the projections of
fetal/umbilical blood vessels and the chorion that covers them. The trophoblast
abuts the uterine cells of the mother, but allows arteries in the uterine lining to
drain into sinuses around the chorionic villi. The chorion preserves the barrier
between the mother's and fetus's blood (but diffusion of nutrients and waste
products may still occur). This whole exchange apparatus is called the placenta.

The inner cell mass will undergo changes similar to those in the frog such that
the three basic cell layers are formed. First, the inner cell mass forms a cavity
within itself. This is called the amniotic cavity and the cells which line it are the
ectoderm. Below the amniotic cavity and its ectoderm, on the hollow side of the
blastocyst, a layer of cells will form along the ectoderm and will evenfually cover
the entire hollow blastocyst. This layer is the endoderm. So what we have at this
stage is a hollow ball of cells, the blastocyst, which is surrounded by trophoblast.

Inside this ball there are two compartments; an amniotic cavity defined by
ectoderm and yolk cavity defined by endoderm. It is the two-layered sheet of
cells formed by the ectoderm and endoderm that will develop into the fetus. So
unlike the frog which developed from the whole ball of cells, the human only
develops from the sheet of cells suspended in middle of the blastocyst. The rest
of the cells of the blastocyst will be important in forming the placenta and
chorion which protect the fetus inside the womb.

The primitive streak, equivalent to the neural plate in frogs, forms in the
ectoderm above the endoderm. Cells from the primitive streak migrate down
between the ectoderm and endoderm to become mesoderm. Further folding of
the primitive streak gives rise to a neural groove and then a neural tube, etc. So
the formation of the primitive streak in mammals marks the beginning of
gastrulation. It is quickly followed by neurulation. The fate of cells from the three
basic cell layers will be the same as in frogs: endoderm will become the GI lining;
mesoderm will become connective tissue, bones, muscle, blood; ectoderm will
become skin and nervous tissue (from the neural grove and neural crest cells),
etc.

Copyright @ by The Berkeley Review 230 The Berkeley Keview


Specializing in MCAT Preparation
Biology Reproduction & Development Human Embryo Development

tlormones and Pregnancy


The embryo cannot implant if the uterus is not receptive, that is, if it is not
quiescent. Furthermore, pregnancy will not be maintained by the uterus even
after implantation unless it remains quiescent. So to maintain the uterine lining
through the first part of pregnancy the trophoblastic cells secrete chorionic
gonadotropin. Chorionic gonadotropin (CG) is a hormone which causes the
.orpus luteum to continue to produce estrogen and progesterone. As long as the
,evels of estrogen and progesterone, particularly progesterone, are sufficient,
irev keep the uterine lining quiescent and pregnancy continues. Eventually, the
:iacenta is able to take over the production of estrogen and progesterone itself
::rd the corpus luteum is not needed.

rhe piacenta secretes increasing amounts of estrogen and progesterone.


iorvever, the levels of estrogen increase faster than the levels in progesterone.
-'-ld at some point near the end of pregnancy the ievels of progesterone plateau.
ri a crucial ratio of [progesterone]:[estrogen] the uterus is no longer quiescent; it
::gins to have contractions. The smooth muscle of the uterus contracts, putting it
'-:rder tension. Also the size of the fetus at this stage can put the smooth
muscle
-:L the walis of the uterus under tension. when the walls of the uterus are put
'::rder tension thev send nervous impulses to the hypothalamus. The
-,"-pothalamus sends signals to the posterior pituitary (via nerves) to release
-'rr-tocin. oxytocin is released into the blood and is a strong inducer of more
:rntractions of the smooth muscle of the uterus. Oxytocin also stimulates the
::oduction/secretion of prostaglandins which further induce contractions.

.i11of these hormones and nerves form a positive feedback system. Once started
:.e., contraction) each step stimulates the next and eventually stimulates the first
::ocess more (stronger contractions). Eventually the contractions force the fetus
rrough the vagina and into the hard, cold world. we call the birth process
parturition.

Lactation
-here are two different processes that go on in lactation. First, there must be milk
:roduction, and second, there must be milk ejection. Breast milk is a fairly
:rmplex fluid containing proteins, fats, vitamins, and other goodies (antibodies,
=ic.). There are epithelial cells within the breast that make up the glands that
::oduce the milk. Around these epithelial cells are myoepithelial cells which can
:r-)ntract around the milk glands, ejecting the milk that has been produced.

lroduction of milk is stimulated by the suckling of the infant on the mother's


:ipple. The nipple sends a nervous impulse to the hypothalamus, The
:..-pothalamus releases PRH, prolactin releasing hormone, which acts on the
:nterior pituitary. PRH stimulates the anterior pituitary to release prolactin.
lrolactin goes into the bloodstream and stimulates the epithelial cells in the
::easts which comprise the milk glands to produce more milk.

l"tilk ejection uses a similar process, but different hormones. The suckling on the
:reast sends a nervous signal to the hypothalamus. The hypothalamus send a
:.ervous signal to the posterior pituitary. The posterior pituitary responds by
:eleasing oxytocin. Oxytocin causes contraction of the myoepithelial cells in the
::east. Milk gets squeezed out.

Llopyright @ by The Berkeley Review 23t The Berkeley Review


Specializing in MCAT Preparation
Reproduction
& Development
To Go
15 Passages

102 Questions

Time for All Passages Thken Together as a Practice Exam


125 Minutes

Passage Titles Questions


I. Spermatogenesis, Oogenesis, & RU486 1-8
U. Male Contraception 9-15
ilI. Fertilization Of The Ovum t6-2t
IV. Pregnancy Fuel Utiliz.atiort 22-29
V. Female Reproductive Hormones 30-35
VI. Endocrine Control Of Ovarian Cycle 36-43
VII. Gestation 44-49
VIII. Female Birth Control Vaccine 50-56
IX. Estrogen 57 -63
X. Isoflavone Experiment 64-69
XI. Ovulation l0 -76
XU. Vertebrate Gastrulation 11 -83
XIIII. Oxytocin & Labor 84-90
XIV. Puberry 9r -96
XV. Testicular Cancer 97 - r02

ffi
EKT{ruNY
V. f .,E' E'.
t
t
trli
il
Speciabzing in MCAT Preparation
Suggestions
- The passages that follow are designed to get you to think in a conceptual manner about the processes
of physiology at the organismal level. If you have a solid foundation in physiology, *u1y of these
answers will be straightforward. If you have not had a pleasant experience with the tJpic, some
of these
answers might aPpear to come from the void past the Oort field of the solar system.

- Pick a few passage topics at random. For these initial few passages, do not worry about the time. Just
focus on what is expected of you. First, read the passage. - Seco-nd, look at u.,y diugru-s, charts, or
graphs. Third, read each question and the accompanying answers carefully. Fourth, answer the
questions the best you can. Check the solutions and see how you did. \A/heth"t ybu got the answers
right
or.wrong/ it is important to read the explanations and see if you understand with) whaiis
1urdugr""
being explained. Keep a record of your results.

After you feel comfortable with the format of those initial few passages, pick another block of
Passages and try them. Be aware that time is going to become importani. Gerierally, you will have about
1 minute and 15 seconds to complete a question. Be a little more creative in how yo,, upprouch
this next
group. If you feel comfortable with the outline presented above, fine. If not, then try different
approaches to a passage. For example, you might feel well versed enough to read the
questions first and
then try to answer some of them, without ever having read the passage. Maybe you can answer
some of
the questions by just looking at the diagrams, charts, or graphs that ai pt"r"r,t"a in a particular
passage.
Remember, we are not clones of one another. You need to-begin to devllop a format that
works best for
you. Keeping a record of your results may be helpful.

The last block of passages might contain topics that are unfamiliar to you. Find a place
where the
level of distraction is at a minimum. Get out your watch and time yo,rrr"lf on these passages,
either
individually or as a group. It is important to have a feel for time, and how much is passing as"you try
to
answer each question. Never let a question get you flustered. If you cannot figure o.rt 1"hut
the answer is
from information given to you in the passage, or from your own knowledge-tase, dump it and
move on
to_ the next question' As you do this, make a note of that pesky question
Jt d .ornu bacl to it at the end,
when you have more time' \Mhen you areJinished, check yow urr*"rs and make sure you understand
the solutions. Be inquisitive' If you do not know the answer to something, look it up. The solution
tends
to stay with you longer. (For example, what is the Oort field?)

The estimated score conversions for 100 questions are shown below. At best, these are rough
approximations and should be used only to give one a feel for which ballpark they are sitting in.

Section IV
Estimated Score Conversions
Scaled Score Raw Score

>12 86- 100


10- 11 79-85
8-9 6s-78
7 59-64
6 54-58
5 48-53
<4 0-47
Biology Spermatogenesis, Oogenesis. & RU 4g6 Passage I

Passage I (Questions 1-8) 1. Cellular division does NOT occur in which of the
following cell types?
Human development begins with the union of the
sperm (male gamete) and the ovum (female gamete) A. Spermatogonia
during fertilization. The unicellular organism which is B. Oogonia
tormed is called the zygote. C. Spermatids
D. Primary oocytes
The formation of the male and female gametes occurs
during gametogenesis, a process in which there are two
successive meiotic divisions to reduce the chromosome .,
Before fertilization the secondary oocyte is arrested
number from a diploid state to a haploid state. in meiosis II at:

During spermatogenesis the male germ cells (called A. interphase.


sf.ermatogonia) grow and are transformed into primary B. prophase.
spermatocytes. Each primary spermatocyte forms a C. metaphase.
:aploid secondary spermatocyte after the first meiotic D. anaphase.
jivision. After the second meiotic division, secondary
.rermatocytes form spermatids which are then gradually
:ransformed by differentiation (spermiogenesis) into 3. Roughly 3 days after fertilization in the ampulla of
:rature spenn. Spermatogenesis takes about 64 days. the human fallopian tube, the morula:

During oogenesis the femaie germ cells (called I. develops a blastocoel.


- r_eonia) grow and are transformed into primary oocytes. II. contains about 16 blastomeres.
These oocytes begin their first meiotic division before IfI. passes into the uterus.
:.:.rturition but are arrested in prophase until after puberty. IV. forms an outer cell layer called the tropoblast.
Tris reductive division is not completed until shortiy
::lore ovulatiion. As the primary oocyte grows duri
unng A. I and II only
: *berty, follicular epithelial cells begin to surround the B. II and III only
.,rcvte, forming a primary follicle. As soon as a second C. III and IV only
.','e1 s; epithelial cells is formed the primary follicle D. I and IV only
:-romes a secondary (or mature) follicle. The second
:-:iotic division begins at ovulation but is arrested
::dway through the cycle unless the secondary oocyte is 4. As the mature ovarian fotlicle begins to swell, a
::rilized by a sperm. Once fertilization takei place the small oval protrusion called a stigma forms on the
:rrure oocyte is referred to as the ovum. side of the follicle juxtaposed to the peritoneal
cavity. Rupture of the stigma:
The monthly reproductive cycles of the female is
-;_lulated by the gonadotropins, follicle-stimulating
-:,!mone (FSH) and luteinizing hormone (LH). FSH
I. releasesprogesterone.
[. is due to increased foilicular pressure.
' luces the development of an ovarian follicle. However, rrl. releases a secondary oocyte.
, ih FSH and LH are required for follicular maturation. IV. results from stimulation by prostaglandins.
--s the mature follicle grows, it begins to produce
::'-rogen. High estrogen levels produce a surge in the A. I oniy
,r:thesis ofLH and this surge triggers ovulation ofthe B. II and III only
:-:ondary oocyte. The mature foliicle develops into a C. IV only
:.:ndular structure called the corpus luteum and begins to D. I, II, III, and IV
":rrete both progesterone and estrogen. These hormones
::Ein to prepare the endometrium of the uterus for
.' :lantation of the blastocyst. Dispermy is an abnormal fertilization process in
which two sperm fertilize an ovum. The resulting
If fertiiization occurs, the corpus luteum becomes the embryo will:
- rpus luteum of pregnancy and is maintained for about
-

- - $'eeks until the placenta can assume production of A. show trisomy 21.
::rsesterone and estrogen. If fertilization does not occur, B. be triploid and therefore exhibit aneuploidy.
,--,: corpus luteum begins to degenerate about 12 days C. show monosomy.
.-:er ovulation and becomes the corpus luteum of D. will not exhibit polyploidy.
:,:nstruation.

- :,pvright @ by The Berkeley Review 235 The Berkeley Review


Specializing in MCAT Preparation
Biology Spermatogenesis, Oogenesis, & KU 48G Passage I

6. The molecular structure that BEST represents 8. The molecular structure of the synthetic steroid RU
oxytocin is: 486 is shown in Figure 1.

A.
OH
I f",
C_ CH" NHz CH l
t' - o I
H N.

o
HrC
HO
H:,

B. (", c= c- cH.,
RU 486
HOHO
otiltlt
n.N*F c-f-?-c-o-cHl
: -/z ,
i3""*'0 o

Figure 1

C.

The molecular structures of four naturally


occurring steroids derived from cholesterol are
shown below.

D.
o s-s------l
H,N- Cys- Tyr- Ile- Gln- Asn- Cys
HrN- C- Gly - lru- Pro
il
o

Progesterone Testosterone

7. RU 486, used widely in France to terminate


unwanted pregnancies, was originally tested as an OH
antagonist for glucocorticoid receptors. During o I

testing this compound was also shown to have a C - cH2

high affinity for other steroid receptors as well.


Administered alone, RU 486 has about an SOVo
success rate in expulsion of the embryo from the
endometrial lining of the uterus. However, when a
drug which causes uterine contractions is
administered about 36 to 48 hours after receiving
RU 486, the success rate increases by about 26V0. Cortisol Estradiol
RU 486 has a high success rate because it blocks
steroid receptors from binding:

A. glucocorticoids and allows administered Based on the structures of the four steroids showu
progesterone to cause uterine contractions. above, which receptors will RU 486 bind to with
B. testosterone and allows administered oxytocin the strongest affinity?
to cause uterine contractions.
C. progesterone and allows administered prosta- A. Progesterone and testosterone receptors.
glandins to cause uterine contractions. B. Estradiol and cortisol receptors.
D. estradiol and allows administered glucocor- C. Testosterone and estradiol receptors.
ticoids to cause uterine contraction. D. Cortisol and progesterone receptors.

Copyright @ by The Berkeley Review 236 The Berkeley Revieu


Specializing in MCAT Preparation
Biology Male Contraception Passage tr

Passage II (Questions 9-15) 12. What changes does a vasectomy cause in the
hypothalamus, particuiarly invoiving the secretion of
Sterilization is a contraceptive option for people who FSH and LH?
are positive they wish to have no children. About 500,000
men in the U.S. choose to have vasectomies per year. A. Both FSH and LH are increased.
Vasectomy is the surgical ligation of the vas defeiens, B. FSH is increased, and LH is decreased.
resulting in sterilization. Although sperm are produced C. FSH is decreased, and LH is increased.
normally in men with vasectomies, the sperm cells are not D. Both FSH and LH are unchanged.
able to exit the testis. In a local phenomenon, sperm cells
are phagocytosed in the testis by macrophages and other
rmmune system cells in men with vasectomies. As a result
rf this immune attack, antibodies to sperm are elevated in
nen with vasectomies.
13. What is the anatomical location of sperm
production?

A. Vas deferens
B. Seminiferous tubules
9. What is the effect of vasectomy on spenn production C. Prostate gland
and secretion after several months? D. Testicle interstitium
A. Both sperm production and sperm secretion
lncrease.
B. Both sperm production and sperm secretion
decrease.
C. Neither sperm production nor sperm secretion L4, Which of the following statements are TRUE?
change.
D. Sperm production is unaffected and sperm The sperm count ultimately drops to zero
secretion stops. following a vasectomy.
II. The female counterpart of vasectomy 1S
hysterectomy, the removal of the uterus.
ilr. Vasectomy decreases testosterone levels.

n{i. What is a potential consequence of reversing a A. I only


vasectomy? B. I and II only
C. II and III only
A. Antibodies to sperm may reduce fertility. D. I, II, and III
B. Antibodies ro sperrn may increase fertility.
C. Birth defects are more likely in subsequent
children.
D. Birth defects are less likely in subsequenr
children.
15. In an experiment on dogs, researchers used an
injection of a stable polymer to block the vas
deferens. What results could be expected from this
experiment?
11. If a man were diagnosed with oligospermia, what
implications would this have for his fertility status? A. The dogs undergoing this treatment would
remain fertile.
A. He could not ever father a child. B. The dogs undergoing this treatment would
B. He has reduced fertility status compared to the develop enlarged testicles.
population average. C. The dogs undergoing this treatment would
C. He has increased fertility status compared to become sterile.
the population average. D. The dogs undergoing this treatment would
D. He has the same fertility status as the develop autoimmune disease.
population average.

,:p1'right O by The Berkeley Review 237 The Berkeley Review


Specializing in MCAT Preparation
Biotogy Fertilization Of The Ovum Passage Itr

Passage III (Questions 16-21) 18. During pregnancy the levels of progesterone and
estrogen:
In fertile women, ovulation occurs approximately 12-
16 days after the onsetof the previous menses. The ovum A. increase during the first trimester, but decrease
must be fertilized within 24-48 hours if conception is to during the remaining six months.
result. B. increase during the first six months, but
decrease during the last trimester.
Fertilization of the ovum by the sperm usually occurs C. increase steadily until parturition.
in the mid portion (ampulla) of the urerine rube. The D. mimic the levels found during the menstrual
fertilized ovum, now called a blastocyst, moves down the cycle.
uterine tube into the uterus. Once in contact with the
endometrium, the blastocyst becomes surrounded by an
outer layer of syncytiotrophoblast and an inner layer of
cytotrophoblast. The syncytiotrophoblast erodes the
endometrium, and the blastocyst burrows into it, a process
called implantation. After this process, the development
of the placenta then proceeds. 19. According to the passage, it can be inferred that
during pregnancy leutenizing hormone and follicle
Human chorionic gonadotropin (hCG) is secreted by stimulating hormone levels would:
the placenta, and during the early stages of pregnancy,
acts to maintain the corpus luteum, an endocrine organ A. remain relatively low, thereby eliminating
that was maintained by luteinizing hormone (LH) during further follicle development and ovulation.
the luteal phase of the menstrual cycle. The corpus B. be undetectable in either fetal or maternal
luteum, now termed the corpus luteum of pregnancy, blood.
primarily secretes estrogens and progesterone. In addition C. increase in the maternal blood but decrease in
to secreting hCG, the placenta, after about the 6th week of the fetal blood.
pregnany, takes over the function of the corpus luteum D. progressively increase until ovulation has
and produces sufficient estrogen and progesterone to taken place.
continue suppression of gonadotropin releasing hormone
(GnRH), follicle stimulating hormone (FSH), and LH.

20. Ovariectomy (removal of the ovaries) or damage to


the ovaries during the last six months of fetal
16. Removal of hCG during the first 6 weeks of development would:
pregnancy would result in:
A. have no effect at all on the prcgnancy.
L continuation of the pregnancy. B. lead to termination of the pregnancy.
tr. regression of the corpus luteum. C. result in a decrease in the levels of hCG.
uI. termination of the pregnancy. D. result in an increase in follicular development.

A. I only
B. II only
c. I and II only
D. II and trI only
21. If implantation of the blastocyst does not occur in the
uterine lining, then the proper hormonal levels
needed to maintain the lining are removed and
menses normally follows. Which of the following
17. The protein hormone hCG is very similar but NOT structures is sloughed off during menses?
identical in structure to:
A. Endometrium
A. LH B. Myometrium
B. FSH c. Primary oocyte
C. GnRH D. Primary fbllicle
D. Estrogen

Copyright @ by The Berkeley Review 23a The Berkeley Review


Specializing in MCAT Preparation
Biology Pregnancy Fuel Utilization Passage IV

Passage IV (Questions 22-29) 23. Insulin is important for energy storage during early
pregnancy. Which of the following statements is
Human gestation is 40 weeks. Along with the changes FALSE regarding the effects of insulin on adipose
apparent to the eye, fuel metabolism changes considerably tissue?
irom early pregnancy (0-25 weeks) to late pregnancy (25-
-10 weeks). In early pregnancy, the main goal is to build A. Insulinincreases synthesisoftriglyceride.
up maternal stores of adipose tissue. The fetus is quite B. Insulin increases production of o-glycerol
small and has low energy needs during this time. The phosphate.
nother experiences increased insulin sensitivity which C. Insuiin increases uptake of lipid from
increases glucose and fatty acid uptake by adipose tissue, circulating lipoproteins.
especially in the lower body depots. D. Insulin increases synthesis of glucose.

However, in late pregnancy, the fuel needs of the fetus


.rre increased. The mother becomes insulin resistant, so
ihat glucose is diverted to the fetus. The growing fetus
-eeds continuously. Glucose, received through the 24. Some women experience gestational diabetes during
:lacenta via facilitated diffusion, is its prefened fuel. The late pregnancy. This is partially due to insulin
:nother mobilizes fatty acids from her adipose tissue to resistance. Therefore, many obstetricians screen all
:leet her energy needs. Maternai gluconeogenic enzymes their patients for abnormal glucose tolerance using
.re increased, as well, during late pregnancy. an oral glucose tolerance test. An oral load of 50
grams (200 kcals) of glucose is given to a fasting
person. A blood sample is taken t hour later. What
22. When triglycerides are mobilized from adipose changes in glucose and insulin compared to pretest
tissue, which of the following compounds are values would be seen in a woman with sestational
released into the blood? diabetes?

I. A. Increased glucose, increased insulin.


HO B. Increased glucose, decreased insulin.
til
H-C-O-C-(CH,),,-CH, C. Decreased glucose, decreased insulin.
I' D.
tfl
H- C- O-C- (CH2)14- CH3
Decreased glucose, increased insulin.

H
Il?
I

- C-O-C- (CH2)ri- CH3


25. Which organ produces insulin?
H
A. Pancreas
B. Thyroid gland
II. C. Liver
H
I
D. Spleen
H-C_OH
I

H- C_ OH
I
H- C_ OH
I 26. Which of the following statements are TRUE for
H late pregnancy?

I. Gluconeogenic enzymes provide glucose for


III. the fetus when the mother is not eating.

cH3- (cH2)r4- coo


et I Fatty acids are a required fetal energy source.
IfI. Maternal fat stores are rapidly filled during
late pregnancy.

A. I only A. I only
B. II only B. I and II only
C. II and trI only C. II and III only
D. I and III only D. I, II, and III

-:p1,right @ by The Berkeley Review 239 The Berkeley Review


Specializing in MCAT Preparation
Biology Pregnancy Fuel Utilization Passage IV

27. How would untreated gestational diabetes most


likely affect the size of the fetus?

A. Lower birthweight.
B. Higher birthweight.
C. No change in birthweight.
D. During the first trimester fetal weight is
slightly above normal, but at parturition the
birthweight is much lower.

28. Which of the following is TRUE regarding glucose


transport across the placenta?

I. It is an energy requiring process.


il. It occurs by facilitated difussion.
IU. It is higher during early pregnancy.

A. I only
B. II only
C. I and lll only
D. I and IV only.

29. What type of cells contain gluconeogenic enzymes?

A. Placental cells
B. Neurons
C. Hepatocytes
D. Intestinal mucosal cells

Copyright @ by The Berkeley Review 240 The Berkeley Review


Specializing in MCAT Preparation
Biology Female Reproductive tlormones Passage V

Passage V (Questions 30-35)


32. According to the passage, the preovulatory decline
of FSH is due to:
The female reproductive system undergoes a series of
regular cyclic changes termed the menstiual cycle. The
A. positive feedback of estradiol.
most obvious of these changes is periodii
vaginal B. negative feedback of FSH.
bleeding resulting from shedding of the endometrial lining
of the uterus. It results primarily from the interaction of
C. negative feedback of estradiol.
hormones derived from the hypothalamus, pituitary gland,
D. positive feedback of FSH secretion.
and ovaries. In most women in the middle reproductive
vears, menstrual bleeding recurs 25_35 days, with a
median cycle length of 28 days. The interval from the
onset of menses to ovulation is termed the follicular or
proliferative phase. The time proceeding ovuiation to the
onset of menstrual bleeding is termed the luteal or 33. In the absence of
secretory phase. Ovulation normally occurs at about the
pregnancy, menses normally
occurs. This may be due to the decline of thl
1.lth day of the cycle.
hormonal requirement needed by the endometrium.
In normal menstrual cycle, serum concentrations of
Th." primary hormones required by the
endometrium, so that menses does not occur. are:
both leutenizing hormone (LH) and follicle stimulating
hormone (FSH) begin to increase prior to menses. FSH
I. progesterone.
;oncentrations attain maximum levels during the first half
u. estradiol.
of the foilicular phase and, with the exceptlon of a brief
peak at midcyle, continue to fall uniil th* lowest Itr. hCG.
IV. FSH and LH,
;on^centration in the cycle are reached during the second
nalf of the luteal phase. The preovulatory deiline of FSH
.s due to the increasing concentration of estradiol. LH
A. I and II only
levels increase gradually throughout the follicular phase
B. I and III only
.nd at midcyle, there is a large peak in serum C. II and IV only
:oncentration of LH. Subsequentiy, LH levels gradually
D. III and IV only
jecline reaching their iowest concentration
late in the
.rteal phase.

Women treated over a long period of time with 34. All of the following structures will either secrere
relatively large doses of progesterone and estrogen estrogen alone, progesterone alone, or estrogen and
do not ovulate. This is most likely due to: progesterone together EXCEpT the:

I. inhibition of FSH. A. anterior pituitary.


II. direct inhibition of progesterone by estrogen. B. placenta.
rII. over stimulation of FSH by estrogen. C. granulosa cells.
D. adrenal cortex.
A. I only
B. II only
C. I and II only
D. II and III only

35. The stimulus for FSH and LH production and


secretion is governed by the pulsatile release of
Ovulation is marked by a maximal peak of LH gonadotropin releasing hormone (GnRH). Which
concentration. This is primarily due to: one of the following structures produces GnRH?

A. positive feedback by progesterone. A. Anterior pituitary


B. positive feedback by estradiol. B. Posterior pituitary
C. negative feedback by progesterone. C. Hypothalamus
D. negative feedback by estradiol. D. Pineal gland

- -rp1'right @ by The Berkeley Review 241 The Berkeley Review


Specializing in MCAT preparation
Biology Endocrine Control Of Ovarian Cycle Passage VI

Passage VI (Questions 36-43) 36. What is meant by negative feedback?


Endocrine control of the ovarian cycle is complex, A. The rate-limiting enzyme in a pathway is
with cases of both positive and negative ieedback control.
activared by the producr(s) of the pathway.
The following diagram illustrates interactions between the --
B. The rate-limiting enzyme in a pathway can
hypothalamus, the anterior pituitary, and the ovaries. The
only be inhibired.
following abbreviations ar" used in Figure 1: GnRH The rate-limiting enzyme
(gonadotropin releasing hormone), nSH
C. in a pathway is
ltotticte- inhibited by rhe producr(s) of the pith.ay.-
stimulating hormone), and LH 0uteinizing hormone).
D. The rate-limiting enzyme in a pathway. can
only be activated.

Hypothalamus 37. Where is the ovum released upon ovulation?

A. Uterus
? B. Uterine tube
o C. Cervix
D. Abdominal cavity
Anterior E
Decreased TD

Pituitary
negative feedback 'U
FO
inhibition o 38. The corp'rs luteum produces which of the following
FSH & LH ^4, hormones?
IJ
s
Ovaries
(Day 1) J Estradioi
(Day 28) J Progesterone
A.
B.
GnRH
Progesterone

t
s
Estradiol Corpus luteum
C.
D.
LH
FSH

regresses

s
Increased sensitivity
39. Oral contraceptives contain both estradiol
progesterone. Usually, the pills are taken for 3
and
Ovaries
of foilicles to FSH weeks and stopped for I week to allow menstrual
n I-H onlrfr flow. What happens to the ovarian cycle when oral
\,2 contraceptives are taken?
J r-n
a-)
Growth of
J psu I. inhibition of ovulation
p. Follicies
()
n fr a II. Decreased FSH and LH
ts
\.2 Anterior
cD
c) III. Increased growth of follicles
o o
t Estradiol pituitary tf
.: A. II only
H l-l
\z
(+) Feedback
fr B.
C.
II and III only
I and II only
Anterior J cnRtt (D

Pituitary D. I, II, and III only

s
LH Surge
Hypothalamus
(-) Feedback'f 40. Menopause is the cessation of the menstrual cycie.
New follicles cease to develop in the ovary, although
s
Ovaries
t
t Estradiot
Progesterone
the anterior pituitary continues to function normalh..
What levels of FSH and LH would be seen in the

rtU
\.2
Empty follicle will
blood of a menopausal women who takes no
exogenous hormones?
Ovuiation r----\
(Dav 14i
become the A. Increased FSH, decreased LH.
' corpus luteum B. Decreased FSH, increased LH.
C. Decreased FSH, decreased LH.
Figure 1 D. Increased FSH. increased LH.

Copyright @ by The Berkeley Review 242 The Berkeley Review


Specializing in MCAT preparation
Biology Endocrine Control Of Ovarian Cycle Passage VI

41. To what class of hormones do estradiol and


progesterone belong?

A. Steroid hormones.
B. Growth hormones.
C. Peptide hormones.
D. Glucoregulatoryhormones.

42. A hysterectomy is the surgical removal of the uterus.


What effect would this have on the cycle depicted in
Figure 1?

A. No changes in ovarian cycle, but no


menstruation.
B. Anovulation due to lack of negative feedback
from estradiol and progesterone.
C. Decreased development of corpus luteum.
D. Prolonged menstrual flow.

43. What is the trigger for ovulation?

A. Regression of corpus luteum.


B. Follicle sensitization to FSH.
C. LH surge.
D. Increasedestradiol.

Copyright @ by The Berkeley Review 243 The Berkeley Keview


Specializing in MCAT Preparation
Biology Gestation Passage Vtr

Passage VII (Questions 44-49) 45. One of the factors involved in the differentiation of
the Wolffian ducts is testosterone. The production
By the 42nd day of gestation, the embryonic gonads and secretion of testosterone, during this stage of
are distinguishable. Under the influence of the genes that development, is under the influence of:
code for male sex determination, the gonad will begin
testicular differentiation by 43-50 days of gestation. In ihe A. follicle stimulating hormone (FSH).
gonad destined to be an ovary, the lack of differentiation B. human placental lactogen (hpl).
persists. AtlT-84 days, a significant number of germ cells C. human chorionic gonadotropin (hCG).
enter meiotic prophase to characterize the transition of D. gonadotropin releasing hormone (GnRH).
oogonia into oocytes, which marks the onset of ovarian
differentiation from undifferentiated gonads.
46. Testosterone is produced by which of the following
In the 7th week of gestation, the embryo has both male structures?
and female primordial genital ducts. In a normal female
fetus, the Mtillerian duct system develops into the uterine I. Corpus luteum.
tubes, uterus, cervix, and upper one-third ofthe vagina. In il. Interstitial cells of Leydig.
normal male fetus, the Wolffian duct system on each side III. Sertoli cells.
develops into the epididymis, vas deferens, seminal
vesicles, and ejaculatory ducts. A. I only
B. II only
In the presence of a functional testes. the Mtillerian C. I and II only
ducts involute under the influence of "Mtillerian inhibiting D. II and III only
factor" secreted by Sertoli cells. The differentiation of the
Wolffian duct is stimulated by testosterone secretion from 47. Treatment of an XY embryo at gestational day 77
the testes. In the presence of an ovary or in the absence of with anit-Mrillerian inhibiting factor antibody wouid
a functional fetal testis, Mtllerian duct differentiation result in development of:
occurs and the Wolffian ducts involute.
A. Wolffian duct structures
Experimental evidence has accumulated demonstrating B. Miillerian duct structures and male external
that a small portion of the distal short arm of the i genitalia.
chromosome, termed H-Y antigen, is critical for testicular C. Wolffian duct structures and male externai
organogenesis of the bipotential gonads. The following genitalia.
experiments are a summary that supports this hypothesis. D. Miillerian duct structbres and female external
genitalia.

Experiment I 48. Administration of anti-H-Y antigen antibody ar


Using the "moscona" technique, dissociated cells derived gestational day 80 to an XY embryo would result in:
from either mouse or rat newborn testes in culture
reorganized to form seminiferous tubules. Another group A. undifferentiated testes.
treated with anti-H-Y antibody resulted in the B. formation of "follicular-like" structures.
reorganization of cells into "follicular-like" structures C. no change.
rather than seminiferous tubules. D. undiff'erentiated testes and formation of
"follicular-like " structures.
Experiment 2
In a similar experiment, bovine or human fetal XX
49. Treatmentof an XX embryo with H-y antigen
undifferentiated gonads formed testicular-like structures during gestational day 90 would result in
development of:
when incubated with H-Y antigen.
I. Wolffian duct structures.
44. The Sertoli cells, which produce "Mrillerian
II. Miillerian duct'structures.
inhibiting factor" are iocated in the:
Itr. female external genitalia.
IV. male external genitalia.
A. seminiferoustubules.
A. I only
B. epididymis.
B. II only
C. vas deferens.
C. I and III only
D. corpus luteum.
D. II and III only

Copyright @ by The Berkeley Review 244 The Berkeley Review


Specializing in MCAT Preparation
Biology Female Birth Control Vaccine Passage VItr

Passage VIII (Questions 50-56) 52. If the study were continued by giving a repear
injection of the same concentration of vaccine at
A birth control vaccine for women has been week 6, what would happen to the anti-hCG
researched by the World Health Organization since 1974. antibody titer?
This vaccine promotes antibodies against the hormone
human chorionic gonadotropin (hCG) which is produced A. Increase and stay constant
by the embryo. hCG maintains a functional corpus B. Decrease and stay constant
luteum, allowing it to continue to produce estrogen and C. No change in titer
progesterone during pregnancy, and promotes normal D. Increase and then decrease
implantation of the embryo into the uterine lining.
Normal, nonpregnant levels of hCG are 0.000 nmol/L in
plasma, and levels increase during the first trimester of 53. Why is diphtheria toxin used?
pregnancy.
A. To make a larger complex for antibodies to
The vaccine incorporates a synthetic peptide recognize, allowing more effective antibodv
representing the amino acid sequence 109-145 of the C- lormation
terminal region of the B subunit of hCG. This particular B. To protect LH against antibody formation r.ia
region was chosen to avoid homology between hCG and cross-reactivity
luteinizing hormone (LH). The hCG peptide is conjugated C. The toxin prevents implantation of the embrvo
to diphtheria toxin to fbrm a hapten-protein carrier
D. To provide simultaneous diphtheria immunln
complex. Antibodies to hCG are raised with this complex and birth control.
in female baboons and in women, following one injection.
The vaccine was tested in 30 sterilized, human subjects, 54. At-home pregnancy tests contain monoclonal
rvith the foilowing results: antibodies in a test kit to which a small volume of
urine is added. Which of these compounds are
Anti-hCG antibody concentration present in urine only during pregnancy?
Vaccine Dal 1* Day 7 Week 5
Group
dose (pg) (nmol/L) (nmol/L) (nmol/L)
A. Estrogen
B. Human chorionic villi
50 0.000 0.033 0.226 C. Human chorionic gonadotropin
D. Progesterone
2 100 0.000 0.025 0.268
J 200 0.000 0.022 0.229 55. How would this anti-hCG vaccine affect a woman's
normal menstrual cycles?
4 500 0.000 0.044 0.950
5 1000 0.000 0.058 0.183
A. The presence of hCG would mimic pregnanc) .

halting menstruation.
* Injection
B. The anti-hCG antibody would also bind LH.
preventing ovulation, and causing irregular
menstruation-
50. How could a researcher determine the concentration C. Although the vaccine would raise hCG leveis
of an antibody in a blood sample? to pregnancy levels, it would allow normal
menstruation.
A. Affinity chromatography using LH. D. The vaccine would not change normal
B. Ion exchange chromatography. menstrual cycies.
C. Affinity chromatography using hCG.
D. Bradford protein assay. 56- Which of these compounds can act as an antigen in
the vaccine?

51. Researchers estimated that a concentration


of 0.52 I. The hCG synthetic peptide
nmol/L were required to provide a contraceptive II. Diphtheria toxin
effect. Which group achieved this effect by Week 5? IIr. LH
A. Group I A. I only
B. Group 2 B. II only
C. Group 3 C. I and II only
D. Group 4 D. II and III only

Copyright @ by The Berkeley Review 245 The Berkeley Beview


Specializing in MCAT Preparation

,d
_-s-
Biology Estrogen Passage IX

Passage IX (Questions 57-63) 59. Throughout a women's life, estrogen is produced


primarily by rhe:
Estrogen, a steroid hormone, is not limited in its role
as being one of the major hormones present during a A. uterus.
woman's reproductive cycle. Estrogen receptors can be B. ovaries.
found in liver cells, melanocytes, neurons, vascular C. anrerior pituirary.
endothelial and vascular smooth muscle ceils. After D. posteriorpituitary.
menopause, a prolonged estrogen deficiency results. This
prolonged deficiency has been shown to promote
osteoporosis and atherosclerotic disease. Studies indicate
that women using estrogens after menopause have half the 60. Increasing the diameter of a blood vessel bv a factor
risk of dying of a myocardial infarction when compared to of 2 results in a resistance which is:
matched women not using estrogens. Furthermore,
women with established coronary artery disease show a
J}Vo decrease in mortality over a l0 year period when
A. 1/2 of the original resistance.
compared to similar women who do not receive estrogen.
B. 1
/4 of the original resistance.
C. 1/g of the original resistance.
In the liver, estrogen stimulates enzyme production D. 1/16 ofthe original resistance.
that affects cholesterol catabolism. The overalllffect is to
impede the development of a lipid profile consistent with
atherosclerosis. In addition, estrogens have direct effects
on arterial walls. Recent cell culture studies suggest that 61. Estrogen has been shown to induce arteriolar
estrogen may inhibit plarelet aggregation and adhesion relaxation in arteries lacking endothelial cells. The
seen in early atherosclerosis. BEST explanarion for this is that:

Endothelial derived relaxing factor (EDRF) is the most A. EDRF is nor a vasodilator.
important molecule produced by the endothelium. EDRF B. EDRF is not produced by the endothelial celis.
inhibits myofibril contraction in smooth muscle, leading C. estrogen directly acts on vascular smooth
to vasodilation. Estrogen appears to stimulate EDRF. muscle to cause relaxation.
EDRF is opposite in function to endothelin, a potent D. estrogen stimulates the production of EDRF.
vasoconstrictor released by the vascular endothelium.
Estrogens have been shown to induce arteriolar relaxation
in which the endothelium has been removed. Subsequent
studies have shown that estrogens block voltage-gated 62. Which of the following effects of estrogen is not
calcium channels. consistent with the idea that estrogens benefit the
circulatory system?

57. An estrogen receptor complex within a cell will I. Introduction ofestrogen leads to a rapid
most likely: decrease in vascular resistance.
Ir. Introduction of estrogen leads to decrease;
A. be carried through the blood in search of a catabolism of LDl-cholesterol.
target organ. III. Estrogens inhibit endothelial celi expression ..,-
B. result in the production of an intracellular adhesive molecules.
secondary messenger.
C. create mutations within estrogen regulated A. I only
elements of the genome. B. II only
D. bind to DNA, resulting in changes in C. I and II only
transcriptional rates. D. I and [I I only

58. EDRF most probably inhibits myofibril contraction


by, 63. Based on information in the passage, it couid :re
concluded that estrogen:
A. permitting calcium entry into the muscle cell.
B. inhibiting calcium entry into the muscle cell. A. moderately stimulates release of endothelin.
C. permitting potassium entry into the muscle B. significantly stimulates release of endothelin
cell. C. inhibits release of endothelin.
D. inhibiting sodium entry into the muscle cell. D. plays no role in regulating levels of endothel:l

Copyright @ by The Berkeley Review 246. The Berkeley Revierr


Specializing in MCAT Preparatia
Biology Isoflavone Experiment Passage X

Passage X (Questions 64-69) 64. Isoflavones are antagonists to estrogen. What is the
role of an antagonist?
The rate of breast cancer is about JSVo lower in Far
Eastern countries compared to Western countries. A. An antagonist mimics a hormone, interacts
E,pidemiological studies from migrant populations with the hormone's receptor, and leads to very
suggests that an environmental rather than a genetic similar intracellular effects.
:rpianation is plausible. When people immigrate to other B. An agonist mimics a hormone, interacts with
,-ountries, their rates of breast cancer come to resemble the hormone's receptor, and leads to very
-he country where they are living rather than their home similar intracellular eff'ects.
:ountry. C. An antagonist mimjcs a hormone, interacts
with the hormone's receptor, and leads to a
In the Far East, a significant quantity of soybean biock of the intracelluiar effects of the
nrotein is consumed in the diet in many different forms, hormone.
:rcluding beans, miso, tofu, and soy milk. Soy provides a D. An agonist mimics a hormone, interacts with
:rch supply of isoflavones, nonsteroidal compounds with the hormone's receptor. and leads to a block of
:strogenic properties. Isoflavones are structurally similar the intracellular effects of the hormone.
,o estrogens. They bind to the estrogen receptor and act to
:artially antagonize estrogen.

In a clinical trial, 6 women were studied during several


renstrual cycles while living in a research center. After
65. What effect does the soy-protein diet have on the
concentration of FSH and LH at ovulation?
:ecording usual changes during severai menstrual cycles,
"rey began the ciinical trial. First they followed a control A. The soy diet group has suppressed levels of
:iet for 1 menstrual cycle, then they followed another 1- FSH and LH compared to the controls.
:onth diet with similar composition of macronutrients, B. The control diet group has suppressed levels of
:\cept the protein was largely from soy isolate (60 g of a
FSH and LH compared to the soy group.
'3 g protein diet). C. FSH is suppressed by rhe soy diet, bur LH is
unchanged.
fhe effects of a soy diet on the concentrations of LH and
:SH as compared to the control diet is shown in Figure
D. LH is suppressed by rhe soy dier, but FSH is
1.
unchanged.

1^
t1 J
66. If the amount of soy protein could be increased to
ii -
> completely suppress LH and FSH, what would be
16 3ro
,- a
the theoretical effect on the subject?
I!
5 A. The subject could become pregnant at any
phase of her menstrual cycle.
-3 -2-t 0 I 234 B. The normal menstrual cycle wouid continue
Day of Ovulation Day of Ovulation without any change.
C. Menstruation would begin sooner than
Figure 1 predicted by the past menstrual cycie.
D. The subject would not ovulate.
-
'b,le 1 indicates the results of several blood parameters
--.lt were followed during the study.

Plasma Hormone] Control Diet Soy Diet 67. Which of the following statements could you infer
from the passage?
Testosterone (nmoVl) i.25 + 0.38 \.46 + 052
Cholesteroi (mmol/L) 4.21 + 1.08 3.86 + 1.01 A. The sole risk factor for breast cancer is genetic
background.
B. Japanese women and European women have
Table I comparable rates of breast cancer.
C. Exposure to estrogen is a probable determinant
The soy protein for this dietary intervention provided of developing breast cancer.
-5 mg of isoflavones. For comparison, a Japanese diet D. Breast cancer is not related to diet.
: rntains about 150-200 mg of isoflavones/day.

lopyright @ by The Berkeley Review 247 The Berkeley Review


Specializing in MCAT Preparation
Biology Isoflavone Experiment Passage X

68. What percentage of the protein in the soy diet was


provided by the soy isolate?

A. 98Vo
B. 25Vo
C. 33Vo
D. 617o

69. What was the effect of the soy protein diet on


cholesterol levels?

A. The soy diet was hypercholesterolemic.


B. The soy diet was hypocholesterolemic.
C. The soy diet was isocholesterolemic.
D. The soy diet was microcholesterolemic.

Copyright @ by The Berkeley Review The BerkeleY Revieu


Specializing in MCAT Preparation
Biotogy Ovulation Passage XI

Passage XI (Questions 70-76) 70, What structure is the source of the rise in
progesterone after ovulation?
By studying physiological changes during the
nenstrual cycle, a woman may plan a pregnancy based on A. Corpus luteum
:he time interval during which she is fertile. One change
lhat is frequentiy tracked by hopeful parents is the basal
B. Egg

lody temperature, that is the temperature taken just after


C. Corpus albicans

arr akening in the morning. The hormone progesterone


D. Cervix

"eads to a rise in basal body temperature of about 0.5oF.


This rise signals ovulation has occurred. The temperature
:emains high until menstruation. By developing a
:emperature log over several months, a woman may use 71. Suppose a woman is planning a pregnancy and is
--iris information
to predict times when she is most iikely plotting her basal body temperature daily. What
:!a Conceive. conditions should she observe to make an accurate
measurement?

A. Eating or drinking nothing before arising.


ir.
B. Lying still during the measurement.
=9 C. Taking measurement at same time daily.
D. All of the above.
L

-:l 72. Which hormonal changes precede ovulation?

I. Rise in luteinizing hormone.


il. Rise in progesterone.
IIr. Rise in estrogen.

A. I only
I B. I and III only
{ C. II and III only
D. I, II, and III
j
o

73. Contraceptive methods either inhibit ovulation, act


as barriers to prevent conception, or prevent
implantation of a fertilized egg. Which of the follow
methods inhibits ovulation?
23 I 5'l 14 2t 28 5
Days of the Ovarian Cycle
A. Condoms
Figure 1
B. RU-486
C. Birth control pills
D. Diaphragm

-\nother predictor for ovulation is a changes in the


:ucus produced by the cervix. Cervical mucus, under the
,.:'luence of estrogen, becomes more clear and watery
*rinediately prior to ovulation, to allow penetration by 74. If a clinician tells a woman planning a pregnancy
i;€rrn. The mucus may be placed on a microscope slide, that she is exhibiting ferning, what does this mean?
:::ed. and examined for a pattern the resembles a fern
:-r.nt, a phenomenon called "ferning". This phenomenon A. She is pregnant.
.j :resent at ovulation and signals fertility. B. She is anovulatory.
C. She is infertile.
D. She is ovulating.

Jrpyright @ by The Berkeley Review 249 The Berkeley Review


Specializing in MCAT Preparation
Biology Ovulation Passage K

75. Which of the following hormone(s) directly triggers


ovulation?

I. Luteinizing hormone
II. Follicle stimulating hormone
III. Estrogen

A. I only
B. I and II only
C. II and III only
D. I,II, and III

76, What molecule is the precursor for the hormones


estrogen and progesterone?

A. Sphingomyelin
B. Cholesterol
C. Arachidonic acid
D. Phosphatidylcholine

Copyright @ by The BerkeleY Review 250 The BerkeleY Review


Specializing in MCAT PreParation
Biology Vertebrate Gastrulation Passage XII

Passage XII (Questions 77-83) 79. During invagination, sheets of cells around the
blastopore increase in length and extend themselves
Gastrulation is one of the most dramatic and crucial into the inrerior of the embryo. Which of the
stages of vertebrate embryogenesis. The embryo, at first following could be responsible for this process of
simply a hollow bail of cells, undergoes a transformation cell sheet extension?
into a multilayered structure with a central gut tube and
bilateral symmetry. The outer layer of cells formed during I. Cell shape changes within the cell sheet.
gastruiation is considered ectoderm, while the middle and II. Mitotic division within the cell sheet.
inner layers are considered mesoderm and endoderm, III. Unidirectional migration of every cell in the
respectively. Since interactions between these three ceil sheet.
layers will determine the further deveiopmental fate of the
embryo, events occuring during gastruiation are vital for A. I only
ihe proper development of the organism. B. II only
C. I and II only
Gastrulation begins when cells around the embryonic D. I, II. and III
:lastopore begin to invaginate, or move towards the inside
rf the embryo. The site where this invagination initiates is
:eferred to as the dorsal lip of the blastopore. In the first
raif of this century, it was learned that the dorsal lip acts 80. In an embryo that is about to undergo gastrulation,
ts an organizer, inducing the tissue directly around it to the outermost celis protect more interior cells from
be-uin invaginating while triggering tissue further away to the external environment. This is partially
:dopt other specific cell fates. accomplished by tight junctions which form near rhe
outer surface of these cells. This outer cell layer is
It was later found that the cells ofthe dorsal lip secrete most similar in structure and function to:
' diffusible signalling substance that slowly degrades after
::s secretion. The resuit is a concentration gradient of the A. epithelial cells.
.ubstance, with higher concentrations existing closer to B. neuronal cells.
,re source of the secretion. Cells at different distances C. smooth muscle cells.
::om the dorsal lip are exposed to different concentrations D. connective tissue cells.
i the substance, leading to the signaliing of different
::haviors. Due to its effect on morphogenesis (the
:haping" of the embryo), this substance was referred to
': a mctrphogen. 81. The early stages of human embryogenesis look very
similar to comparable stages in the formation of
other vertebrate embryos. For example, human
embryos have gill slits and a tail at a certain stage in
development. This observation does NOT support
which of the following statements?
'i. If the mesodermal cell layer is improperly formed
during gastrulation, which of the following later A. Humans evoived from lower vertebrates.
structures would most likely be affected? B. Primate embryos should not have gill slits.
C. Vertebrates may share a common ancestor.
A, Nervous system D. There is little selective advantage in altering
B. Heart these stages of embryogenesis.
C. Stomach lininc
D. Liver

82. If the morphogen mentioned in the passage were nol


slowly degraded after being secreted, which of the
-8. Which of the following could NOT act as a following would result?
morphogen?
A. A very steep concentration gradient of
A. A small, inorganic compound morphogen would form.
B. A steroid hormone B. The concentration gradient of morphogen
C. A secreted protein would persist indefinitely.
D. A transmembrane protein C. A concentration gradient of morphogen couid
never be formed.
D. There would be no effect on gastruiation.

- -pyright O by The Berkeley Review 25t The Berkeley Review


Specializing in MCAT Preparation
Biology V ertebrate Gastrulation Passage )ilI

83. The dorsal lip from a frog embryo is removed and


grafted opposite the dorsal lip of a second frog
embryo. Assuming this was done prior to
gastrulation, which of the following would be like1y
to result?

A. Gastrulation would never occur and the


embryo would die.
B. The grafted cells would be rejected by the
immune system of the host embrYo.
C. The embryo would develop into Siamese twin
tadpoles, the result of two independent
invaginations during gastrulation.
D. The embryo would gastrulate and develop
normally.

Copyright @ by The BerkeleY Review


The BerkeleY Keviw
Specializing in MCAT PreParatiu
Biology Oxytocin and Labor Passaqe XIII

Passage XtrI (Questions 84-90) 86. Which of the following would NOT increase
contractions in a female in labor?
_ Human pregnancy lasts on average 270 d,ays. During
the last month of pregnancy, irregular uterine contractioni A. Intravenous injection of oxytocin.
increase in frequency. At the time of labor and delivery, B-. Intravaginal administration ofprostaglandins.
the cervix dilates and softens, and the muscular body of C.
the uterus contracts to push out the fetus.
Intravenous injection of human chorionic
gonadotropin.
D. Oral administration of prostaglandins.
Oxytocin plays both a direct and an indirect role in
labor. It acts directly on uterine smooth muscle cells to
make them contract. It aisqstimulates the production of
prostaglandins in the endometrium of the uterus. which
enhance the contractions produced by oxytocin.
87, What strategy could be used to prevent a pre-tefin
birth?
During early labor, the uterus contracts without
significantly higher ievels of oxytocin compared to A. Administration of an oxytocin secretagogue.
prepregnancy. Once labor is initiated, a positive feedback B. Administration of prostaglandins such as RU_
,oop is set up between the cervix and secretion of 486.
oxytocin. Signals of dilation from the cervix lead to C. Administration of inhibitors of prostaglandin
sisnals in afferent nerves that promote increased secretion production.
of oxytocin. In late labor, the plasma oxytocin level rises, D. Administration of an oxytocin agonist.
leading to higher concentrations of oxytocin presented to
the uterus.

88. A prostaglandin is a twenty-carbon long fatty acid


that contains a five-membered carbon rins. pictured
below is the prostaglandin thromboxane n"2.

Oxytocin

&{. Where is oxytocin produced?

A. Posterior pituitary
B. Hypothalamus Which fatty acid is the precursor for prostaglandins?
C. Anrerior pituirary
D. Placenta A. Stearic acid
B. Myristic acid
C. Linolenic acid
D. Arachidonic acid
95. During early labor, plasma oxytocin levels are not
higher than prepregnancy levels of 25 pgiml. What
mechanism(s) could explain an increase in uterine
contractions without an increased plasma level of 89. Oxytocin is a polypeptide hormone. What does this
oxytocin? imply about its method of interaction with uterine
cells?
I. An increase in the number of uterine receptors
for oxytocin during late pregnancy. I. It works through a second messenger system.
il. Paracrine release of oxytocin that does not il. It stimulates mRNA synthesis directly.
communicate with the bloodstream. Itr. It is binds to a cellular membrane receptor.
III. Increased conjugation ofoxytocin in the liver.
A. I only
A. I only B. I and III only
B. I and II only C. II and trI only
C. II and III only D. I,II. and Itr
D. I, II, and trI

Jopyright @ by The Berkeley Review 253 The Berkeley Review


Specializing in MCAT preparation
Biology Oxytocin and Labor Passage )iltr

90. Studies indicate that paraplegic women can go into


labor and deliver with few difficulties. What would
this finding suggest?

A. Delivery can occur without stretch signals


from cervix.
B. Delivery requires voluntary contractions of the
abdominal muscles.
C. Prostaglandins play only a minor role in the
delivery of children to paraplegic mothers.
D. Smooth muscle contraction does not occur in
these women.

Copyright @ by The Berkeley Review 254 The Berkeley


Specializing in MCAT
Biology Puberty Passage XIV

Passage XIV (Questions 91-96) 93. The development of the seminiferous tubules is
under the influence of:
After an interval of childhood quiescence,
hypothalamic-pituitary-gonadal ac ri vity in te niifies in the I. leutenizing hormone (LH).
peripubertal period leading to increased secretion of II. follicle stimularing hormone (FSH).
gonadal sex steroids that cause secondary sexual IrI. gonadotropin releasing hormone (GnRH).
development, the pubertai growth spurt, and fertility. This
stage of development is often termed puberty or A. I only
adolescence. B. II only
C. I and II only
The first sign of puberty in the female is an increase in D. I,II, and III
growth. This is accompanied by breast development, a
process stimulated by increase in estrogen levels. Other
deveiopmental changes influence by estrogen include;
enlargement of the labia minora and majora, dulling of the
vaginal mucosa, production of a clear whitish vaginal
secretion prior to menarche, and changes in uterinqsize 94. The stimulation of the inrerstitial cells of Leydig is
and shape. The development of pubic hair is primarily under the direct influence of:
influenced by adrenal and ovarian androgen secretion.
A. follicle srimulating hormone (FSH).
In males, the first sign of normal puberty is an increase B. leutenizing hormone (LH).
in the size of the testes, primarily due to seminiferous C. gonadotropin releasing hormone (GnRH).
tubuiar development. The stimulation of the interstitial D. Miillerian inhibiting hormone (MIH).
cells of Leydig also plays a small component in the
increase in testicular size. Pubic hair development is
under the influence of the adrenal gland, as well as the
lestes, through secretion androgens.

The dramatic increase in overall body growth during 95. In addition to growth hormone's direct effects on
the growth spurt at the time of puberty is under the muscle and bone tissue growth, it also acts indirectly
influence of complex endocrine control. In addition to sex on these tissues by stimulating the production and
steroids, growth hormone (GH) plays an important role in secretion of another hormone or factor which then
ilris phenomenon. acts on these tissues. Which of the following
hormones mediate tbe indirect effects of growt[
hormone?

91. Breast development in females is stimulated by an A. Insuiin


increase in estrogen secretion. Which of the B. Thyroxine
following hormones must increase in order for this C. Epidermal growth factor
phenomenon to occur? D. Insuiin-like growrh factor I (IGF-I)
A. Thyroid stimulating hormone (TSH)
B. Gonadotropin releasing hormone (GnRH)
C. Prolactin
D. Progesterone
96. The hormone that stimulates the production and
of growth hormone is derived from which
release
92. Which of the following events does NOT occur one of the following organs?
within the seminiferous tubules of the male
reproductive system? A. Anterior pituitary.
B. Posteriorpituitary.
A. Spermatogenesis. C. Bone and liver.
B. Testosteronesynthesis. D. Hypothalamus.
C. Secretion of androgen-binding protein by the
Sertoli cells.
D. Enhancement of sperm production by follicle
stimulating hormone (FSH).

Copyright @ by The Berkeley Review 255, The Berkeley Review


Specializing in MCAT Preparation
Biology Testicular Cancer Passage XV

Passage XV (Questions 97 -102) 99. Based on information in the passage, one could
argue that germ cell tumors that come from fetal
Cancer of the testes if found in only l7o of malignant germ cells have a retarded developmental fate.
tumors of male internal organs. This cancer, which shows Which of the following DOES NOT support rhis
little basis in genetics, is ten times more likely to be found hypothesis?
in males afflicted with cryptorchidism, a condition where
the testes have failed to descend down into the scrotum A. Cryptorchidism is a risk factor.
during development. Only 5Vo of testicular tumors do not B. Evidence shows that tumor development
originate from germ cells. Because germ cells undergo occurs in gestation.
spermatogenesis, it is difficult to determine the exact C. In mouse studies, cancer genes act in fetal
origin of tumor cells. There is some evidence that germ cells.
spermatogonia are the likely candidates giving rise to D. Infertile men run a greater risk for carcinoma
testicular cancer. However. researches have found that in situ.
early carcinoma cells bear more of a similarity to fetal
germ cells than spermatogonia.

It has been discovered, through studying teste tissue


samples from infertile men, that certain morphologically
distinct cells were precursors to invasive cancer. These
malignant cells were termed carcinoma in situ (CIS). 100. How many chromosomes are found in a carcinoma
Most CIS cells will evolve into invasive cancer. Looking in situ cell?
for differences between invasive and CIS cells, researches
noted that CIS cells have a tetraploid complement of A. 23
chromosomes. B. 46
c. 69
The initial events transforming germ cells into tumor D. 92
cells is not well understood, but has been likened to
parthenogenesis in the female. Tumors arising from
parthenogenesis, known as teratomas, contain multiple
tissue types with little organization. It is postulated that
testicular cancers may have similar origins.

101. Which statement is TRUE regarding the genetic


97, Which of the following statements supports the basis of testicular cancer?
theory that spermatogonia give rise to testicular
cancer? A. Testicular cancer is an autosomal recessive
trait.
A. Sertoli cel1s give rise to tumors which differ B. Testicular cancer is found commonly withir-
significantly from tumors originating from families prone to testicular cancer.
germ cells. C. Testicular cancer is rare among identical twins
B. Cryptorchidism is a risk factor for testicular D. Testicular cancer is largely solved by surgicd
cancer. resection.
C. Testicular cancer is a rare cancer.
D. Tumor cells do not have a haploid
chromosome number.

98. Which of the following statements is TRUE


regarding testicular cancer? t02. Parthenogenesis is BEST described as th:
development of a:
A. At least 90Vo of testicular cancers are found in
the vas deferens. A. tetraploid adult from a fertilized egg.
B. At least 90Vo of testicular cancers are found in B. tetraploid adult from an unfertilized egg.
the seminiferous tubules. C. haploid adult from a fertilized egg.
C. At least 907o of testicular cancers are found in D. haploid adult from an unfertilized egg.
the epididymis.
D. At least 90Vo of testicular cancers are found in
the rete testes.

Copyright @ by The Berkeley Review 256, The Berkeley Revieu


Specializing in MCAT Preparation
Biology Keproduction & Development Section IV Answers

1' C is correct, spermatids. Cellular division does not occur in spermatids. During spermatogenesis
male germ cells
called spermatogonia are converted into mature spermatozoa. A single diploid
izlif ;;;.t""ogonia is rransformed
into a 1" spermatocyte which then undergo meiosis I to form two traptola
spermatocytes undergo meiosis II to form four haploid spermatids. Each
1x; z"'rp"r.atocytes. The two 2"
spermaiid n"*t undergoes a transformation
process called spermiogenesis to form the mature spermatozoa. The
spermatids do not undergo a cellular
differentiation to form a spermatozoa. They simply diffeientiate. The whole
64 days.
;."";;;;l Jp.r-urog.n"sis takes about
During oogenesis female germ cells called oogonia are transformed into mature ova.
This process involves the
cellular divisions of meiosis I and meiosis II,F{Y in the development of the fetus
oogonia undergo mitosis and are
transformed into 1" oocytes' The diploid (2N) 1" oocyte enters into meiosis
puberty. A short time before ovulation meiosis I is completed and a haploid
I and"is arrested at prophase until
first polar body (which may divide into two more polar todies). At ovuiation the
1p z; oocyte is formed atong witfr iire
is arrested at metaphase. At fertilization the 2" oocyte completes meiosis II
z' oocyteundergoes meiosis II but
and forms the mature ovum and a second
polar body' union of spermatozoa and ovum gives rise to tie zygote which
is diploid. The coryect choice is c.
C is correct' metaphase' See the explanation for oogenesis given above. The
correct choice is C.
3. B is correct, II and III only' Fertilization leads to the formation of a zygote.The
zygote undergoes numerous mitotic
divisions to increase the number of cells but not the cytoplasmic miss. wrren
(about 30 hours after fertilization) those two cells u." r"i".."d
ilie zygote divides into two cells
to as blastomeres. Aboui 5 auy, after fertilization the
dividing zygote has formed about 16 blastomeres (a specialized cell-rype). At rhis
stage the?ll il;;;;;;j';;
as a morula and it begins to enter into the uterus.

Roughly four days after fertilization a fluid-filled cavity, called the blastocyst cavity
or a blastocoei, begins to form
in the central portion of the morula. As this cavity foims the cells within the *o.Llu
parts' one group of cells forms the outer layer of cells called the tropoblast.
;;"r" rhemselves into two
The othei gr""pir cells forms an inner
cell mass called the embryoblast. The tropoblast gives rise-to the piacenta while
the Jmbryoutast gives rise to the
embr.yo' By this stage in development ihe morula is referred io u, u
blastocyst. Around the sixth day after
fertilization the biastocyst (normally) attaches to the endometrium of the posterioi
wall of the uterus. The correct
choice is B.
4. D is correct, I, II' ru, Iy. As the primary follicle.begins to develop, an antrum is formed and the primary oocyte
is moved to one side. Td The formation bf the antrum signals the development of the secondary follicle.
Towards the
the foliicular phase the granulosa cells (i.e., rotticte cells) begin to secrete large
919 "l ariounts of progesterone
( 17u-hydroxyprogesterone) into the antrum. As
the fluid in the antrum-increases tn" piir*. uuilds up ani places a
stress on the stigma. Prostaglandins, which are synthesized and released from
almost all cells in the body, can act to
stimulate the smooth muscle of the uterus and the smooth muscle surrounding the outer
layer of theca cells (called
the theca externa). contraction of this muscle can cause an increase in pressurJ
in theantrum and rupture the siigma.
The contents of the antrum, including the secondary oocyte and the progesterone, are
released into the peritoneal
cavity (the space outside the ovary). The correct choice is D.

B is correct, be triploid and therefore exhibit aneuploidy. If two sperm fertilize an ovum, then
each sperm will
contribute a chromosomal complement of 23. Since the ovum also has a chromosomal
complement of 23, this
results in an embryo that is triploid and contains 69 chromosomes. If there is a deviation
from the normal
chromosomal complement of 46, then that deviation is referred to as aneuploidy.

Trisomy 21 involves the presence of three chromosomes instead of the usual pair of chromosomes.
This type of
chromosomal abnormality is referred to as Down's syndrome and usually results in g".-
u with 24 instead of 23
chromosomes. Monosomy involves embryos that are missing a chromosome. Celli ""iiare polyploid contain
which
multiples of the haploid number of chromosomes. The haploid-number of chromosomes
is 46. If a cell contained 69
chromosomes, it would be triploid. A triploid cell is a cell which exhibits polyploidy.
Itre correct choice is B.
D is correct. oxytocin is considered tobe apeptide hormone, as are the other hormones
of the hypothalamus and
pituitary gland. in general, peptide hormones contain from 3 to over 200 amino acid residues.

Choice A represents adrenaline (epinephrine), a water-soluble amine hormone.It stems


from a class of hormones
called catecholamines. In the brain catecholamines act as neurotransmitters. Choice B is
the methvl ester of a

-.pyright @ by The Berkeley Review 257 The Berkeley Review


Specializing in MCAT preparation
Biotogy Reproduction & Development Section IV Answers

dipeptide called aspartame (trade name is NutraSweet@), an artificial sweetener which is commercially synthesized
in huge quantities. This is not a hormone. Choice C is estradiol, a lipid soluble steroid hormone that is synthesized in
the ovaries. The correct choice is D.

7. C is correct, progesterone and allows administered prosta-glandins to cause uterine contractions. RU 486 resembles
both progesterone (a progestin) and cortisol (a glucocorticoid) and will bind to both of their receptors with high
affinity. RU 486 does not resemble testosterone or estradiol as weil as it resembles progesterone and cortisol.
Therefore, binding to the receptors of these two steroids is minimal (if at all). The reasoning and structures
associated with question #7 would help you with this answer, but it is not necessary.

RU 486 is a progesterone antagonist and binds to progesterone receptors. Recall that progesterone aids in the
conversion of the growing endometrial lining of the uterus into a tissue that can accept a developing embryo (i.e., the
blastocyst). Progesterone allows the cells of the endometrium to synthesize and store glycogen, increases the
secretory activity of the endometrial lining, and causes an increase in the vascularization of the tissue. Progesterone
also acts to inhibit expulsion of the implanted embryo by reducing uterine contractions and constricting the opening
of the cervix. If RU 486 binds to progesterone receptors, then the developing embryo will be facing a hostile
environment. Furthermore, 36 to 48 hours after RU 486 has been taken by the pregnant woman, prostaglandins are
administered. Recall that prostaglandins (released by most cells of the body) cause contraction of smooth muscle.
especially the smooth muscle of the uterus.

Glucocorticoids, such as cortisol, have a variety of effects on metabolism. In the liver cortisol increases
gluconeogenesis and glycogen synthesis. In skeletal muscle it decreases glucose uptake, increases protein
degradation, and decreases protein synthesis. In adipose tissue it decreases glucose utilization and increases lipid
metabolism. Glucocorticoids tend to impair wound healing. An antagonist to glucocorticoid receptors might actuall]
increase wound healing. Glucocorticoids do not cause uterine contractions. The correct choice is C.

8. D is correct, cortisol and progesterone receptors. Glucocorticoid and progestin receptors bind RU 486 with the
greatest affinity. Look for similarities in the molecules. First, note the three substituents at the C-3, C-11, and C-1-
iarbons of RU 486 (see below). Estradiol has a hydroxyl function at the C-3 position; RU 486 does not. Estradio,
does not have a substituent at the C-11 position; RU 486 has a bulky dimethylaminophenyl group. Estradiol only ha*'
a hydroxyl group at the C-17 position. RU 486 has both a hydroxyl and a bulky alkyne residue. A similar analysi.
can be done for testosterone. We will find that the substituents on RU 486 do not match well with those on both
estradioi and testosterone. If we now compare RU 486 to both progesterone and cortisol, we will find more
similarities. For example, both progesterone and cortisol have a carbonyl function at the C-3 carbon. They also have
a (somewhat) bulky substituent aatheC-11 position. The correct choice is D.

9H,
N Z\
H:c' H-o
HrC
.,.4
'/ c= c- cH:
RU 486

o
3n *

D is correct, spefin production is unaffected, and sperm secretion stops. This would be a pretty poor form of birtl
control if sperm secretion was unchanged. Immediately eliminate choices A and C as incorrect answers. Since tir
vas deferens is cut and closed, no rper.n will be present in the semen several months following a vasectomy. Usuall
the patient is checked for aspermia-before the procedure is considered a success. Sperm production is not affected
b1

a vaiectomy. Choice B is incorrect. The correct choice is D.

10. A is correct, antibodies to sperm may reduce fertility. Antibodies to sperm are produced.when macrophages entr
the testis to catabolize the lxtra spenn. These would hamper fertility if they immobilized sperm. Choice B
rm

the sperm turnover cycle is about 10 weeks. Thc


incorrect. There is no effect of vasectomy on birth defects, since
man simply makes new spefin. Choices C and D are incorrect. The correct choice is A.

Copyright @ by The BerkeleY Review 258 The BerkeleY Revieu


Specializing in MCAT PreParatia
Biology Reproduction & Development Section lV Answers

11. B is correct, he has reduced fertility status compared t9 the population average. The prefix ,'oligo,,
means fe.o,.
oligospermia is the condition of having few sperm. (This is compared to asplrmia, the condition
of having no
sperm)' Although this man could impregnate a woman, his chances are less than that of
the population aveiage.
However, his cl.rances are not zero. Choice A is inconect. Choices C anci D are incorrect. The
correct choice is B.
12. D is correct, both FSH and LH are unchansed. Since sperm production continues along its
normal course, really the
only interruption is in transport of speim outside of the bocly. This has no effect-on the hormones controlling
.the
sperm production. Choices A, B, and c are incorrect. The correct choice is D.

13. B is correct, seminiferous tubules. The testis is the site of sperm production. It is compartmentalized into the
seminiferous tubules and the interstitium. The interstitium c-ontains androgen-se.reting Leydig
cells.
Spermatogenesis occurs in the seminiferous tubules, not the interstitium. Choice D ls incorrect.
The vas deferens
drains sperm from the testis. Choice A is inconect. The prostate gland is not invoived
in spermatogenesis. Choice C
is incorrect. The correct choice is B.

14. A is correct, I oniy. Foliowing a vasectomy, the sperm count rvill eventually drop to zero, since
the sperm cannot
exit the testis' Choice T is correct. The female counie.pa.t of vasectomy is the ligarion
of the fallopian rubes (a tubal
ligation)' choice II is incorre_ct. Since sperm production continues normally, the"re is
no."uron to suspect decreased
testosterone leveis. Choice III is incorrect. The correct choice is A.

15. c is correct, the dogs undergoing this treatment would become sterile. This treatment achieves the same etf-ect
as a
vasectomy. The vas def'erens is blocked, so that sperm cannot exit the testis.
The effects would be the same as
expected for vasectomy. The dogs would eventually have a sperm count of zero
and be sterile. Choice A is incorrect.
Normal sperm production continues, but the cells are destioyed by phagocytes in the testis.
There would be no
enlargement of the testicles. Choice B is incorrect. There is no evidenc" ttrit an autoimmune
disease would develop,
since the phagocytic cells are localized in their attack on the sperm cells. not the testis
itseif. Choice D is incorect.
The correct choice is C.

16. D is correct, II and III only (regression ofthe corpus luteum and termination ofpregnancy).
The corpus luteum is
quite dependent on hCG during the first 8 weeks of pregnancy. During the luteal p'rr.r.'"t
the uterine cycle the
corpus luteum was maintained by LH, even though the LH ievels were relatively
low. 'rhe corpus luteum is
responsible fbr the secretion of large amounts of estrogen and progesterone. These
two steroids. in concert. inhibir
release of GnRH. The end result is that FSH ard LH levels are ref,t tow and follicular
development and ovulation
are suppressed. Increased levels of estrogen and progesterone lead to further endometrial
development.
After i'ertilization the outer layer of cells (trophoblast) of the blastocyst begin ro secrete hcG. This
pepride
stimulates and the corpus luteum to continue its synthesis of estrogen and progesterone.
why? Because the
deveioping placenta will not be able to secrete adequate amounts of estrogen or progesterone
until about the 3rd
month of pregnancy. High levels of these two steroids allow the endometriaiiining io piolif'erate
and implantation of
the blsatocyst is maintained. In other words, pregnancy will continue. What happeis if nCC is removed?

Removal of hCG during the first 6 weeks of pregnancy means that the corpus lureum is not being
stimulated. The
corpus luteum begins to regress and the output of estrogen and progesterone decreases. Uterine
bl6od vessels begin
to constrict, depriving cells of nutrients and oxygen. The uterine lining begins to siough. pregnancy
is terminated."ln
turn' GnRH, LH, and FSH levels begin to.is. and the cycle begins agiin. ttre correct choice is D.
1l A is correct, LH. We can reason out this answer based on information in the first sentence of the second paragraph
of the passage. We are told that during the luteal phase of the urerine cycle LH maintains the corpus luteum.
After
fertilization we are told that hCG maintains the corpus luteum. We could assume that if both hormones
maintain the
corpus luteum' then they must have similar structures if not similar receptors. It turns out that
hCG is a glycoprotein
that-consists of an aipha and a beta chain. The alpha chain is aimost identical in sequence
to the alpfia chains of
TSH' FSH' and LH. The beta chain' which determines biologic effect, has a 6i vo hoiology with
the beta chain of
LH. Consequently, hCG and LH have very similar biologic u.tions. The correct choice is A.-
L\. C is correct, increase steadily until parturition. Although FSH and LH levels are low during pregnancy,
human
chorionic gonadotropin takes over the functions ofLH to stimulate estrogen and progesterone secretion.
In addition
to the placenta, the fetal and maternal adrenal cortices may also produce estrogen ind progesterone. The
correct
choice is C.

: . right @ by The Berkeley Review 259 The Berkeley R.eview


Specializing in MCAT Preparation
Biology Reproduction & Development Section IV Answers

19. A is comect' remain relativ-ely low, thereby eliminating further follicle development and
ovulation. LH and FSH are
inhibited by high levels of estrogen and progesterone during pregnancy. Therefore,
they remain relatively lou,
throughout the duratlon ofpregnancy. The correct choice is AI

20. A-is correct, have no effect at all on the pregnancy. The ovaries produce estrogen and progesterone
during the first 6
weeks ofpregnancy' after which, other tissues such as the plaCenta take ovJr tnis rote.inerefore,
pr"lnunry
occur in the second term without the ovaries. The correct choice is A. "un
21. A is correct, endometrium. Unless implantation.of the blastocyst occurs, the continuing secretion
of progesterone
and estrogen by the corpus luteum is stops. The result is that the glandular epithefal
thut muiie up tt e
endometrium begin to slough. Immediately beiow the endometrium is tf,e myometilu-, *rrl.fr ""tt,
is the smooth muscle
tissue of the uterus. A primary oocyte will become surrounded by a layer of granulosa
cells to become a primarl.
follicle. The primary foilicle will be-converted to a secondary oo"yt", und it i, ti'e secondaif
oo"yt" that will become
the ovum. If an ovum is not fertilized, it will be removed in the monthly menstrual
flow. Thus, the only structure
mentioned that is sloughed during menses is the endometrium. The correct choice is
A.

)t c is correct, II and III only. Triglycerides in the adipose tissue are hydrolyzed by the enzyme triglyceride lipase.
This yields 3 fatty acids and 1 glycerol molecule per iriglyceride. All iire gtycerot er"a"r
ti-trr" blood. Some fatn,
acids may remain in the adipocyte for re-esterification, ind the remaindJr"will escapi
choice is C.
t" trr" ur""o. il;^;i,.J";;
.,?,
D is correct, insulin increases synthesis of glucose. We are looking for the FALSE choiice.
We have learned frorn
r'c Pabs'1Btr
the passage that
Llat rhe of early pregnancy ls
goar or
ure goal is storage ot ii adipose
of energy tn adi tissue. If a person eats, insulin levels
t",T: f.o1 the food will.be stored asTat. Insulin does increase iv",rt"rir of triglyceride in
ill"lt^":111 1l lL" :l"..cJ
adipose tissue. It activates triglyceride synthase. Choice A is correct. Glucose that is taken up by the udipo"yt. l'
used-to make a glycerol phosphate, an activated glycerol backbone for triglyceride
synthesis. Choice B is correcr
Food lipids are packed into chylomicrons and are taken up by cells, inciualng
Insulin increases rhe
activity of lipoprotein lipase. Choice C is correct. Finally, tlhe false choice ls"di;";i;.
O. rn" synthesis of glucose is
gluconeogenesis, and that occurs in the hepatocytes. Insuiin does not increase gtu"on"og"*sis
after a meal. The
correct choice is D.
24. A is correct, increased glucose, decreased insulin. Insulin resistance means that the insulin in less effective ir
producing its effects compared to normal insulin sensitivity. The effects of insulin are to lower
blood giucose bi
activating glucose transpofiers and moving glucose into cells for processing. A f'eed-back loop
is set"up so the:
insuiin will decrease as glucose decreases. Since the woman with gestatiorial diabetes is quite insulin
resistanl,
glucose will remain higher longer after the test load. Insulin will also iemain high because
the feed-back loop to rur:
it down is not completed. The correct choice is A.
?( A is correct, pancreas. Insulin is produced by the beta celis in the pancreas. The correct choice is A.
26. A is correct' I only' Glucose is the required fetal fuel source. The mother builds fat stores during early pregnancl
This makes choices B, C, and D incorrect. The correct choice is A.
)1 B is correcto higher birthweight. Since the fetal preferred source of energy is glucose and glucose is higher jr
women with gestational diabetes, the fetus will be able to "overeat". fnis conAition is called macrosomy.
Choice E
is correct. This makes choices A and C incorrect. As for choice D, we have no way of inferring from the passage (or
from our own knowledge) that during the first trimester fetal weight will be slightly above no#at. Not only tha*t, trr
the second part of choice D states that the birthweight is lower when we knbw it should be higher. The correcr
choice is B.
28. B is correcto II only. Glucose transport is by facilitated diffusion, which is passive transport. It occurs throughou:
pregnancy, not just following a meal. It is lower in early pregnancy, when tfie fetal needs are lower,
and motf,er:-i
insulin sensitive. The correct choice is B.
29. C is correct, hepatocytes. The liver's cells are called hepatocytes. The hepatocyte is the only location oi
gluconeogenic enzymes listed here. There is some evidence that the kidney uiro do gluconeogeneiis, but thar ls
not an issue in this question.The correct choice is C. "un

Copyright @ by The Berkeley Review 26o The Berkeley Revieu


Specializing in MCAT Preparation
Biology Reproduction & Development Section IV Answers

30. Aiscorrect,Ion1y(inhibitionofFSH)'Together,::|.-oc"n.an_d.progesteroneactatthelevelffi
$:":H?n,Slli*lll,l:,nr",f*:l:ii':ry jiT"?-",
prevents follicular growth and suppression of LH prevents
*"ffi rhe_anrerior pituitary. Suppression or FSH
estrogen inhibits progesterone or """1"ii""*ri";;:"J
thif estrogen stimulaies *,r tyntrr"Jr
#'il#il;iilii!'i'"X1#"iil|
oi psH. The correct choice is A.
31. B is correct' positive feedback by estradiol..Estradiol-is_normally
an inhibitor of LH secretion, but at increasing

;?::i'",x,:.T;ffi.:;i'flTnf:,:f,Jijftion
rhe LH p;;[;due ro rhe rise ;;;;;g""revels during thE
1) c is correct' negative feedback ofestradiol. Estradiol
inhibits FSH secretion. The correct choice
is c.
33. A is correct' I and II only (progesterone and estradiol).
Progesterone and estradiol are the main
the endometrium' FSH, LH, ano hcG primarily hormones that act on
act on the ovaries. The correct choice is A,
34. A is comect, anterior pituitary' The anterior pituitary is
(GH)' thyroid stimulating hormone (TSH), prolactin, responsible for the release of FSH, LH, growth hormone
una'ua."no"orricorropic hormone (AC'H). All
hotmones are protein hormones' This is information of these
that is ,mforiu,iiro ."-"-ber. Notice that
and progesterone are not produced at the level the steroids estrosen
or tn" u.uit 'Rig;;'uruy this allows us to select
correct answer' The placenta produces both estrogen choice A as ihe
and progestlrone. dranuloru ."rrr?rlell
produce estrogen' Recail thaithe granulosa as thecal cells) can
cells-are ttrose cJtts wt i.r, m-"aiately surround
thecal cells are those ceils that suriound the granulosa."itr. the oocyte while the
follicle celis' The adrenal cortex helps to define the
iog"t;"i, trr" g.unulosa and thecal cells are called the
outer r"gion"oiirr" adre-nal glands. irril,irru"
including androgens (male sex hor,non'"9 uni secetes a variety
(femate sex hormones). The correct
ff::"rttj:rmones, "rr.og"n,
c is correct' hypothalamus' we have learned that both FSH and
pituitary' The release of these two hormones (and LH are synthesized and released from the anterior
the other hormones oi.the
hypothalamic hypophysiotropic hormones, such -"r1". pii"iiary) are regulated by
as GnRH. G"RH ;;;"T-"9_fr:orn the hypothalamus
to three hours in a pulsatile fashion. In turn, the once every rwo
release of borh FSH and LH from rh" ;;;;i;
pulsatile fashion as well'-The poterior pituitary pihirary is seen in a
releases uuropr"rrrn (ADH) and oxytocin.
hormones are svnthesized within the hypothaia.us,
posterior pituitary for release. The pineai gland,
pu.rug-J^*.;"';;;;;yH;i,,":Xd These two peptide
rransported ro rhe
derivid froir the roo^f of the diencephalon and posterior
and amino acii derivative. rr," run"iion or,r'i, to the
:1Ti1:hT::".'::#il,Ti;tl; hormone r,!, nolu""n entirery worked

C is correct, the rate-limiting enzyme in a pathway is inhibited by the product(s) of the pathway.
Firsr of all,
eliminate choices B and-D. Enzymes may be both activated or inhibited, turned up
o.io*n. Nlgative controi means
th'at the product of a pathway acts to decrease its own synthesis. This means
the pathway is inhibited, not activated.
Choice A is wrong. The correct choice is C.

D is correct, abdominai cavity. The ovum is reieased from the mature follicle directly into the
abdominal cavity. It
is picked up and directed into the uterine tube by the fimbriated ends of the tube. Finaliy passes
it into the uterus and
leaves by the cervix if it is not fertilized. The correct choice is D.

-18. B is correct, progesterone. From the diagram, we can see that the hypothalamus produces GnRH, so choice A is
incorrect. We can also see that the anterior pituitary produces LH and FSH, ro cholces C and D are incorrect.
This
ieaves B, progesterone, which is indeed made by the Corpus luteum. The correct choice is B.
j ,t. C is correct, I and II only. Look at the diagram. At around Day 1, the ievels of estradiol and progesterone are
falling. This decreases the negative inhibition on the hypothalamus. Then the normai cycle of follicle?evelopment
and ovulation follows- Therefore, if estradiol and progesierone are kept artificially high with oral contraceptives,
the
negative feedback control is not removed. The brief time between cyiles of pills doei not allow the normal
time for
all the ovulatory steps to get in sync. So there is no ovulation. Choice I is correct. There is also decreased FSH and
LH because the fall in progesterone and estradiol does not occur in pill users. Choice II is correct. Choice IiI is
incorrect, because we have already established that normal follicle deveiopment and ovulation does not occur.
The
correct choice is C.

-lht O by The Belkeley Review 26t The Berkeley Review


Specializing in MCAT preparation
Biology Reproduction & Development Section IV Answers

40. D is correct, increased FSH, increased LH. This is anothgr questiol regarding feedback control.
During the
secretory phase, high levels-o_f e_stradiol and progesterone feed back on the hfpothuiu*u,
unj d""."ur" GnRH. This.
in turn, decreases FSH- and LH. After menopause, levels of estradiol and"irogesteron" ui" low
because normal
follicular development has ceased. This means FSH and LH would remain high-aue to lack
of feedback inhibition.
This means choices A, B, and c are incorrect. The correct choice is D.
4'1. A is correct, steroid hormones' The precursor of estradiol and progesterone is cholesterol, which is
a steroid. This
means choice A is correct. Growth hormone is a_ specific peptloe hormone. So choice
B is incorrect. peptide
hormones are made of chains of amino acids, so choice C is lnionect. Finally, glucoregulatory
hormones control
plasma glucose levels. Estradiol and progesterone do not do this. The correct choice
is A.
42. A is correct, no changes in ovarian cycle, but no menstruation. Removal of the uterus alone will
not affect the cycle
involving the ovaries, hypothalamus, and anterior pituitary. Therefore, choices B and C are incorrect.
There should
be no changes in the ovarian cycle. Choice D is inconecf because there can be no
menstrual flow without the uterus-
which is the source. The correct choice is A.
43. C is correct, LH surge. Although the follicle is prepared by FSH and LH, the final
stimulus for ovulation is a surge
of LH about 16 hours before ovulation. Choices B and b precede ovulation, but are preparatory
steps, not t53
trigger' choice A is about 10 days following ovulation, so it c;uld not be a trigger.
ttre correct choice is C.

44' A is correct, seminiferous tubules. Sertoli cells are located within the seminiferous tubules.
Their cell body extend"
from the base of the-tub.ules to the cytoplasm. In addition to producing Miillerian intrioliing
factor, they alsl provide
nutrients to the developing sperm. The correct choice is A.

45. C is correct, human chorionic gonadotropin


Qcc;. The production of steroid during the early part of gestation is
dependent on hcG derived from the placenta. The correct choiceis c.

46. B is correct, II only (interstitial cells ofLeydig). The interstitial cells ofLeydig are
the testosterone producing celis
located in the testes, outside of the seminiferous tubules. The correct choice is B.

47. C is correct, wolffian duct structures and male__external genitalia. According to the passage, testicular
differentiation occurs at 43-50 days of gestation and MIF has alre-ady taken effect. ThJrefore,
the ferus will develog,
male genitalia. The correct choice is C.

48. C correct, no change. to experiment l, treatment of anti-H-Y antigen to Xy, newborn testes would nm.
-is ^According
differentiate into seminiferous tubules. In the passage, testicular differentiatio-n occurs at 43-50 days of gestation-
Theoretically, anti-H-Y antigen should not have an effect at gestational day 80. The correct choice
is C.
49. D is correct, II and III only (Mrillerian duct structures and female external genitalia). Similarly,
there should not te
an effect since treatmenf occurs beyond the critical period of ovarian diffeientiation (day
ll-g;4 of gestation), Th
correct choice is D.

50. C is correct, affinity chromatography using hCG. Affinity chromatography consists of an inert column whicfu
supports compounds that bind your molecule of interest- Since you want anti-hCG, you attach hCG to the supporL
This will allow binding of your antibody. LH is not the hormone of interest, so choiie A is incorrect. Ion exchalge
chromatography is for separating charged molecules, so choice B is incorrect. The Bradford protein assay only gir--es
total protein, so choice D is incorrect. The correct choice is C.

51. D is correct, Group 4. We can read this answer off the table. Only group 4 had antibody concentrations above the
0.52 level. Only group 4 could be considered protected against pregnancy by the anti-hCG antibody. The correct
choice is D.
{t D is correct, increase and then decrease. Compare this to what we know about being immunized. We get a tetanus
series when we are young, and then we get booster shots to elevate our antibody level as it decreases over time. With
a booster, as in this question, we seen an increase and then a decrease over time. The correct choice is D.

Copyright @ by The Berkeley Review 262 The Berkeley Review


Specializing in MCAT Preparation
Biology Reproduction & Development, Section IV Answers

53. A is correct' to make a larger compiex for antibodies to recognize, allowing more effective antibody formation.
This
is called a hapten-carrier complex. Small molecuies that in immunolo-gically inert alone are -bound to larger,
immunologically robust carriers. This elicits the strongest antibody ."rponr" and ailows your small
molecule of
interest to have antibodies_produced against it. Choice B is incorrect, b"cuur" the toxin does not protect
LH. Choice
C is incorrect, the toxin has no effect on reproductive status. Choice D is incorect, the goui i, not to provide
simultaneous immunity, but contraception. The correct choice is A.
54. C is correct, human chorionic gonadotropin. hCG is present only during pregnancy. Estrogen and progesterone
are
present throughout pregnancy and also during the normal menstrual cycte. f.nis eliminates
Jhoi"", A unA n. Human
chorionic villi are part of the developing tissues accompanying the fetus. The correct choice is C.
5-l)- D is correct, the vaccine would not change normal menstrual cycles. The vaccine causes the body to produce
antibodies to hCG, so that any hCG produced by an embryo is trapped by the antibodies. hcG
is critical for the
progression of pregnancy, so the pregnancy is terminated at this stag-e. Choice A is inconect,
the vaccine does not
contain the hCG molecule, merely a piece of it, made synthetically. Choi." B is incorrect, the peptide
is chosen to
avoid cross-reactivity with LH. Choice C is incorrect, the hCG levels will be reduced to nonpregnant
levels, 0.000
nmol/L. The correct choice is D.
56. C is correct, I and II only. The intended antigen is the hCG synthetic peptide. Choice r is correct.
However, since
the diphtheria toxin is present, a small antibody response o..ur. to it is weil, so choice
hormone and is not an antigen. Choice IrI is incorrect. The correct choice is c.
II is correct. LH is a

57' D is correct' bind-to DNA, resulting in changes in transcriptional rates. As stated in the passage, est1ogen is
a
steroid hormone. Therefore, we must think about how steroid hormones bring about changis on"a
cellulir level.
Since steroids are very hydrophobic in nature, they can cross the cell membra"ne and bind Io receptors
within the
cell's cytoplasm. These. steroid/receptor complexes have the ability to bind to DNA and alter transcriptional
levels of
certain proteins. This is very different from protein hormones which bind to receptors found on
the cell surface.
These types of interactions lead to the formation of secondary messengers that bring
about change within the cell.
Eliminate choice B as a possible answer. Answer choices A ind C ur"l bit non-r"niibl" and do
scheme of how hormones bring about change. The correct choice is D.
iot fit the general
58' B is correct, inhibiting calcium entry into the muscle cell. Entrance of calcium into a muscle cell is what
triggers
contraction. We have seen this in skeletal muscle cell. The question asks what inhibits myofibril (there
are many
myofibrils in a muscle cell, and each myofibril contains many sarcomeres) contraction. Weli, the only real
possibility should involved the calcium ion. As for sodium and potassium, r" huu" no evidence they
are involved
with the contraction of a myofibril. They will be involved in the nerve transmission to stimulate contiaction,
but we
cannot assume they take part in the actual contraction of the muscle. If we want to inhibit the myofibril contraction
which will lead to vasodilation, we will want to inhibit calcium entry into the cell. The correct choice is B.
59. B is correct, ovaries. This is based entirely on our knowledge of hormones. Estrogen is produced primarily by the
ovaries, and its target tissue is the general female reproductive structures. Its principal actions stimulate deveioplent
of secondary sex characteristics in females and growth of sex organs at pubeity. It also primes the uterus for
pregnancy on a monthly basis. Hormone questions are not going to be tough in you know your hormones, where
they come from, and what they accomplish. The correct choice is B.

60. D is correct ,1 I 6 of the original resistance. If we recali from physics or discussions of the cardiovascular system,
the resistance of a tube is inversely proportional to the radius of the tube to the fourth power. If we increase the
diameter by a factor of two, we are increasing the radius by a factor of two. We are decreaiing the resistance by 2 to
the fourth power. Thus, the resistance is one sixteenth of what it was previously.. The correct choice is D.
61. C is correct, estrogen directly acts on vascular smooth muscle to cause relaxation. We know from the passage that
the endotheliai celis produce this EDRF which causes relaxation of the smooth muscle, and ultimately le-*ads to
vessel diiation. Since this is stated in the passage, we have to take it as the truth and thus we can eiiminate choices A
and B. If one removes the endothelial layer, one is removing the ability to produce EDRF. If estrogen stili acts as a
dilator in this condition (no EDRF because no endothelial iayer), then estrogen must have the abilily to directiy acr
on the smooth muscle to cause its relaxation and ultimately vessel dilation. Considering choice D, we run into a
familiar problem. Estrogen cannot stimulate the production of something that cannot be produced without the
presence of endothelial cells. Since these have been removed, estrogen must act directly on smooth muscle. The
correct choice is C.

- :,oyright @ by The Berkeley Review 263 The Berkeley Review


Specializing in MCAT Preparation
Biology Reproduction & Development Section IV Answers

62. B is correct, II only (introduction of estrogen leads to decreased catabolism of LDl-cholesterol.).


Statement I is
consistent with the idea the estrogen benefits cardiac tissue because less resistance
to blood flow allows blogd to
flow-easier, allowing your heart to work easier (assuming our heart was working
harder than it shouid). In other
words, more resistance to blood flow would force our heart to work harder, piacing
stress on the vital organ.
Statement III is consistent with this idea because inhibiting the endotheiiat cett
exprer"ti"" adhesion molecules
inhibits platelet aggregation and other adhesion that is normally seen in the early "r
stages of atherosclerosis. This idea
is that if less "stuff" can stick to the walls of the vessel, the less chance exists
oflto"llng Utooa no*
Statement II isinconsistent because the statement claims a decreased breakdown, or catabolism
of LDL cholesterol
is beneficial to the vascular system. An LDL particle is a low density lipoprorein *ilh
;;;;;orrs
the liver to the body cells via the blood. The cells_tak_e up rhese puil"i"t'ttrrough;Je;;;;i";;;6;;.
cholesrerol from
However, high levels of cholesterol will inhibit LDL rLceptor'synthesis, anithe pirti"f"r
circulatory system. The build up of cholesterol in the blood is thought to ie a p.l-"
will remain in the
ruur" of atherosclerosis, a
condition where the arteries are no longer compliant and blood *ou", though with
difficulty. This condition is
clearly not beneficial to the vascular system, making statement II inconsistent. ihe correct
choice is B.
63. c is correct, inhibits release of endothelin. Throughout the entire passage, we have seen how estrogen
benefits the
cardiovascular system. It is stated in the passage that endothelin is a potent vasoconstrictor.
theme of the passage' it is most likely that estrogen will act to block the
In keeping with the
activity of endothelin. Answer choices A
and.B call for estrogen to.stimulate this potent vasoconstrictor. This is no
evidence for such an interaction, an,l
choices A and B can be eliminated. Choice D wants us to believe that
estrogen will not affect endothelin. Again.
this is not keeping with the theme that estrogen benefits cardiac tissue. Therefore,
estrogen will most likely inhibir
the release of endothelin. This has been demonstrated in rabbit coronary
arteries. The comect choice is C.

64. C is correct, an antagonist mimics a hotmone, interacts with the hormone's receptor,
and leads to a block of the
intracellular effects of the hormone. Choices B and D are incorrect, because they refer
to agonists, not antagonis..
choice B is the colrect definition of an agonist. An antagonist acts against the action
of a"hormone and blocks irs
effects. Choice A is incorrect. The correct choice is C.
65. A. is correct, the soy diet group has suppressed levels of FSH and LH
compared to the controls. Examination ot
Figure 1 indicates that both FSH and LH levels are decreased (suppressed) in the
soy diet trial compared to the
control trial. The correct choice is A.
66. D is the subject would not ovulate. ovulation is triggered by a precise sequence
-correct, of hormonal events. LH ii
the direct stimulus for ovulation. The subject would not oiitut" if-LH were,o-"to*
suppression of LH by the birth control pill). This.is a theoretical question only. The ffiressed (similar to the
amount of soy protein may ne'er
completely halt LH production. The focus of this question is more on ovulation than on soy prbGin.
If the iubjecl
did not ovulate, she couldnotbecome pregnant. Choice A is incorrect. Since she did not ovuiate,
there is a change ir
the normal menstrual cycle. Choice B is incorrect. One cannot predict when menstruation
would occur due to the
lack of influence of the corpus luteum. Choice C is incorrect. The correct choice is D.
67. C is correct' exposure to estrogen is a probable determinant of developing breast cancer. From the passage. \r3
learned that women who moved to countries other than their native countries
lot breast cancer at rates n
natives of the adopted country. This implies an environmental cause rath-er than a genetic cause."ompu.uble
Choice A i;
incorrect. The fact of importance in this passage is women in different countries do not iontract breast cancer at the
same rate. Choice B is inconect. Diet can modify estrogen levels, as seen in the experiment. Choice
D is incorreci
The correct choice is C.
68. D is correct , 6I%. This is taken directly from a,tiny-facet in the passage. The diet provided 98 grams of protein. ol
which 60 grams was the soy protein. 60198 = 6lvo. The correct choice is D.
69. B is correct, the soy diet was hypocholesterolemic. This question tests our knowledge of terminolog y. Hypermean;
increased. Hypo means decreased. 1so means the same or unchanged . Cholesterol"ii, level of cholesterol ll
^"un"by the soy-protein die.,-
the blood. From the data table, we can see that cholesterol levels in the blood were lowered
The correct choice is hypochoiesteremic. Choices A and C are inconect. Choice D is a nonsense answer and u
incorrect. The correct choice is B"

Copyright O by The Berkeley Review 26,4 The Berkeley Revieu


Specializing in MCAT Preparation
tsiotogy Reproduction & Development Section IV Answers

-0. A is correct, corpus luteum. The corpus luteum secretes estrogen and progesterone after the egg is released
from the
follicle. The egg itself does not secrete hormones. Choice B iJinconeit. ih" .o.pu, is a dyin! corpus luteum
which
is no longer secreting hormones. Choice C is incorrect. The cervix secretes mucus, but nbt lio.-on"s.
Choice D is
incorrect. The correct choice is A.
-1. D is correct, all of the above. This answer goes back to our understanding of basal metabolic rate. A basal
measurement is made at rest, before getting out of bed after a night's sleep, after all your food is digested,
while
lying quietly. The correct choice is D.
B is correct, I and IIi only. Look at Figure 1. We can see that LH and estrogen rise dramatically right before
ovulation. Choice I and III are correct. However, progesterone is iow before ovulation and rises drlmatically
afterwards. Choice II is incorrect. The correct choice is B.

C is correct, birth controi pills. Both condoms and diaphragms are barrier methods of contraception. RU-4g6
prevents continued implantation of a fertilized egg. The ievels of estrogen and progesterone
in birih control pills
inhibit ovulation, so no egg is produced. The correct choice is C.
t. D is correct, she is ovulating. Ferning is an indicator of ovulation. The woman is therefore not pregnant.
Choice A
is incorrect. She is not anovulatory (not ovulating). Choice B is incorrect. She is ovulating and
therefore fertile.
Choice C is incorrgct. The correct choice is D.

A is correct, I only. Althoug! FIH and estrogen play critical roles in preparing the follicle and positive
feedback on
the hypothalamus, respectively, LH is the actual irigger for release of tf,e egg-from the follicle.
The correct choice
is A.

h. B is correct, cholesterol. The ring structure of cholesterol is the base molecule for the synthesis
of sex hormones in
men and women. Arachidonic acid is a long-chain fatty acid, a precursor of the class of molecules
called
prostagiandins. Phosphatidyl choline is a phospholipid, used in membrines. Sphingomyelin
is a complex lipid found
in the brain. The correct choice is B.

B is correct, the heart'.The question is asking what structure would be affected if the embryonic mesodenn
were
defective. This is really a matter of memorizing which structures are derived from each of the germ
layers
(ectoderm' mesodetm, endoderm). Choice, A, the nervous system, is derived
from ectodermal precursors, as is the
skin and the lens of the eye. Choice C, the stomach lining, is derived from the endodermal germ layer, which
also
gives rise to other linings of the digestive and respiratory tracts, as well as differentiating into itre majbr glands
of the
body such as the pancreas and the liver (choice D). This leaves choice B, the he#, as the structure which is
mesodermal in origin (along with the notochord, skeleton, muscle, outer coverings of internal organs, and
reproductive organs). The correct choice is B.
''!" D is correct, a transmembrane protein. From the passage, we learn that a morphogen is defined as a diffusible
"substance." Answer choices A, B, and C could all fit this broad
definition. Answei choice D, however, is far from a
diffusible substance. Rather, a transmembrane protein is anchored into the plasma membrane and cannot either be
secreted or diffuse anywhere except within the confines of the membrane. Such a confined protein would not be very
effective at signalling distant cells. The correct choice is D.

C is correct, I and II only. The question inquires about the processes which could bring about the extension, or the
increase in length, of a sheet of cells. Statement I, cell shape changes, is a correct possibility. If individual cells
r'vithin the cell sheet were to change their shape such that they were narrower and longer, the entire sheet would get
nalrower and longer, resulting in extension. Statement II is also a valid possibility. Mitotic division of cells within
the sheet would cause an increase in the total number of cells in the sheet. If these cell divisions were in strategic
locations, the entire cell sheet would become longer and extension would occur. Statement III, on the other hand]is
not a valid explanation for sheet elongation. The unidirectional migration of every cell in the sheet would simply
result in the entire sheet of cells moving somewhere synchronously. This would not result in the elongation ind
extension described in the question. The correct choice is C.

tr, A is correct, epithelial cells. The question gives us some clues about the type of cells on the surface of the embryo.
We learn that they have tight junctions, which are cell-cell adhesions which prevent molecules from slipping in

:,ght @ by The Berkeley Review 265 The Berkeley Review


Specializing in MCAT Preparation
Biology Keproduction & Development Section IV Answers

between these cells' as well helping to establish an apical/basolateral polarity. We also learn
that they serve a role of
protecting inner embryonic celis from the external environment. From these clues alone,
the most iikely candidate
would be answer choice A, epithelial cetls. Epithelial cells can have tight junctions, often to aid them
iniheir major
role of protecting underlying tissues from arbitrary inward diffusion oi lumenaL, or gut, contents.
We can also
approach this problem via a process of elimination. Neuronal, smooth muscle, and coinective
tissue ceils are all
highly differentiated cell types. This question inquires about an embryo which has yet to undergo gastrulation.
It is
very unlikely that these advanced cell types would have differentiated at this stage. The correct
choice is A.
81. B is correct, primate embryos should not have gill slits. The question asks which statement is NOT supported
by the
observation that human embryos have giil slits and a tail at some time during their development. This
observation
does suggest that humans and the other vertebrates share a common ancestri (choice C) as well
as supporting the
theory that humans evolved from lower vertebrates (choice A). The fact that human pass through sLges
which resemble the embryonic stages of lower vertebrates (fish and tailed mammals"rnbryo, in partiiular; sugi"sts tiat
humans might have evolved from these species. Since these stages of development haue not
been drasticiliy altered
from fish to human, it could be said that there is little selective uduuntug" in altering these stages
of embryogenesis.
This supports choice D.. Since we are looking for the unsupported statement, by a piocess of
elimination we are Ieft
with choice B. Since primates are evolutionarily close to humans, we would that they would display similar
embryonic stages. Therefore, they should have gill slits at some embryonic stage."^p*.tIncidentaily, the conservation of
embryonic stages is often referred to as "ontogeny recapitulating phylogeny,""ttat ir, -developmental
stages
reiterate the developmental stages of other species in the iame phylogenetic tiee. The "u.iy choice is B.
"orr""i
82. C is coffect' a concentration gradient of morphogen could never be formed. First, let's think
about what normallv
occurs. If the morphogen is produced at a point source (i.e., the dorsal lip cells), ii is degraded
ur it oifi"r", u*"i.
The result is a concentration gradient of morphogen with a maximum neaithe ,our"", beciuse;";h"g";;;""i;
that get further away are more likely to be degraded because they have been present tong"..
If the mo:rphogen were
not slowly degraded, as the question postulates, what would happen? The gradient would"never
form lmatirig choice
B incorrect). This is because the morphogen would diffuJe uruy ft6- the dorsal lip cells ana equ"alir" in
concentration throughout the embryo. This wouid affect gastrulation because proper signalling
depends on the
establishment of a gradient of morphogen to which cells at different distances ,"rponJ to Oirl"r"nily; therefore
choice A is incorrect. Choice D is likewise invalid because there would be no"ouldgradieni, not a very steep
one. The correct choice is C. "rp""iutty
83. C is correct, the embryo would develop into two S]g.T_"tg twin tadpoles, the result of two independent
invaginations
during gastrulation. This question is referring to Hilde Mangold'i famous dorsal lip transplantation
expJrimens-
First of all, we must keep in mind that the function of the dorsal lip cells is to induce the'invagination
event thar
begins gastrulation. This is accomplished via the secretion of a morphogen gradient. From this,
it follows ther
introducing an additional dorsal lip would induce a separate invagination iu"ni in essence causing
the embryo o
develop into two attached organisms (Siamese twins). Answerlhoice A is incorrect because not
only would
gastrulation occur, it would occur in two separate places. Answer choice D is incorrect because
two dorsal lips in
one embryo would lead to two invaginations. Answer choice B is a tempting one, but remember that an r
this early stage doesn't have a differentiated immune system yel The coriecfchoice is C. "*bryo

84. B is correct, hypothalamus. Oxytocin is produced within the hypothalamus by neurons and transported to the
posterior pituitary, where it is secreted. Choice A is incorrect. Choice C is incorrect. The placenta iiself does nd
produce oxytocin, but the fetus does. Choice D is incorrect. The correct choice is B.

85. B is correct, I and II only. If the uterus is responding more strongly to normal, nonpregnant levels of oxytocin, it ir
reasonable to expect that the number of oxytocin receptors has increased. This w-ould mean that u s-ull dose sf
oxytocin would be very effective at stimulating uterine contractions. Choice I is correct. If oxytocin levels did not
increase in the blood, there is the possibility that some sort of paracrine transfer is occurring. This means th*
hotmone is secreted from one cell to another without actually moving through the bloodstream. Choice II is correct
Usually when things are conjugated by the liver, they are being processed for excretion. If oxytocin wexc
increasingly conjugated, then there would be less present, leading to fewer contractions. Choice III ls incorrec
Choice B is the correct choice. The correct choice is B.
86. C is correct, intravenous injection of human chorionic gonadotropin. You are looking for the FALSE answef,.
Injection of oxytocin (intravenously) would increase the circulating levels of oxytoiin. This would increas

Copyright @ by The Berkeley Review 266 The Berkeley


Specializing in MCAT
Biology Keproduction Er Development Section IV Answers

contractions. Choice A is true. Intravaginal administration of prostaglandins wouid increase contractions


by acting
on the uterus, just as if the prostaglandins were produced by the endometrium. (This is the idea behind oral
or
vaginal prostaglandin administration to allow abortion.) Choice B is true. Prostaglandins also work
orally (see
above). Choice D is true. Human chorionic gonadotropin (hCG) is produced durinf early pregnancy to
make sure
estrogen and progesterone levels remain high. Although it tapers ofi after about 10-weet<s, lt aione would
cause no
uterine contractions. Choice C is FALSE. The correct choice is C.
\ C is correct' administratio{ of inhibitors of prostaglandin production. Both oxytocin and prostaglandins must be
present for labor to occur. By eliminating one or the other from the equation, on" avoid an lubo. in which
the fetus is premature' A secretagogue encourages the secretion of iomething. This "un would not"uJy
be favorable in a
person in whom iabor threatens. Choice A is incorrect. An oxytocin agonist would act on the oxytocin
receptors just
as oxytocin would. Another unfavorable choice. Choice D is incorrect. RU-486 is used to promote
abortion. Choice
B is incorrect. Finally, inhibitors of prostaglandin production, such as indomethacin, can be used to prevent the
positive interactions between prostaglandins and oxytocin. Choice C is correct. The correct choice is C.
iq. D is correct, arachidonic acid. This is a bit of trivia, and you will either know it or not. Anyway, stearic is 1g:0,
myristic is 14:0, and linolenic is 18:2. only arachidonic has the required 20 carbons
120:$. ti is the p.".u.roi
prostaglandins. Choices A, B, and C are incorrect. Choice D is correct. The correct choice is D.
59. B is correct, I and III only. Peptide hormones do not cross the lipid bilayer directly. They usuaily interact
with
cellulal membrane receptors and signals are transferred to the intracellular medium by s"cond messengers. Choices
I
and III are coffect. If the hormon" enter the cell, then it cannot stimulate mRNA synthesis directly. Choice II is
incorrect. The correct choice is B. "un't

9{J. A is correct, delivery can occur without stretch signals from cervix. A paraplegic woman may not be able to
voluntarily contract her abdominal muscles to assist in labor. This is not a rLquiienient. Choice B is incorrect.
The
whole passage discusses the importance of the interactions between oxytocin and prostaglandins. They are required.
Choice C is incorrect. The smooth muscle of the uterus contracts without voiuntari stimulation. Choice D is
incorrect. What is missing in these women is the positive feedback (through afferent nerves; from cervical
stretching
that increases oxytocin secretion. choice A is correct. The correct choice is A.

91. B is correct' Gonadotropin releasing hormone (GnRH). Estrogen is under primarily under the control of leutenizing
hormone (LH) derived from the anterior pituitary. The production and releise of LH, in turn, is under the influencI
of gonadotropin releasing hormone (GnRH). Therefore, estrogen production and secretion is indirectlv stimulated
through the indirect effect of GnRH. The correct choice is B.

92. B is correct, testosterone synthesis. The seminiferous tubules are located in the male testes. These tubules are the
site of sperm production, or spermatogenesis. The important anatomical features of each seminiferous tubule are the
Leydig cells, the basement membrane, and the Sertoli cells. The Leydig cells are found between the seminiferous
tubuies, in the interstitial space. These cells respond to iuteinizing hormone (LH) and it helps regulate the conversion
of cholesterol to testosterone. Therefore, testosterone synthesis does NOT occur within the seminiferous tubuies.
Once synthesized, testosterone can cross the basement membrane and influence the Sertoli cells. The Sertoli cells
secrete androgen-binding protein which binds testosterone (an androgen). This helps to concentrate testosterone
within the seminiferous tubules. The Sertoli cells also respond to FSH, which helps to control spermatogenesis. The
correct choice is B.
93. D is correct, I, II, and III. The seminiferous tubules are under the influence of LH, FSH, and GnRH. GnRH, derived
from the hypothalamus, stimuiates the secretion of LH and FSH from the anterior pituitary. LH stimulates the
interstitial cells of Leydig to produce testosterone. FSH and testosterone, in turn, stimulate the deveiopment of the
seminiferous tubules and spermatogenesis. The correct choice is D.

9{. B is correct, LH. As mentioned in the previous answers, LH binds to receptors on the interstitial cells of Leydig and
stimulates the synthesis of testosterone from cholesterol. The correct choice is B.

95. D is correct, Insulin-iike growth factor I (IGF-I). The clue to answering this question comes from the question itself.
We are looking for the secretion of a hormone or factor that can indirectly act on muscle and bone tissue. The word
factor might clue us into what we are looking for. Insulin is secreted from the B-celts of the islets of Langerhans in
the pancreas. Insulin has a direct effect on the cellular uptake of glucose, fatty acids, and amino acids, and their

- rpyright @ by The Berkeley Review 267 The Berkeley Review


Specializing in MCAT Preparation
Biotogy Reproduction & Development Section IV Answers

subsequent conversion into_glycogen, triglycerides, and protein. Thyroxine (or T4, tetraiodothyronine)
is a hormone
that has a global effect on basal metabolic rate and is critical for nbrmal growth. Thyroxine iirectly
influences the
secretion of growth hormone and helps it to promote protein synthesis utid bon" growth. Epidermai growth
f'actor
of skin cells and promotes their growth. Insulin-like growtlifactor I (IGF-I), also called a
acts at the level
somatomedian, is simiiar to insulin in its structure. IGF-I is synthesized in the liver and acts
for growth
as a
hormone. In other words, GH stimulates the production of IGF-I which in turn promotes -"diuto.
things like frotein
synthesis, cellular division, and overall growth. The correct choice is D.

96. D is correct, hypothalamus. Growth hormone-is.synthesized by the anterior pituitary and is regulated by growth
hormone releasing hormone (GHRH) and growth.h?rTone inhibiling hormone (GHIH) from the hypothala*ui.
Th.
posterior pituitary releases antidiuretic hormone (ADH) and oxytocin. Bone and liver can react
to growth hormone.
but do not synthesize or release it. The correct choice is D.

97. D is correct, tumor cells do not have a hapioid chromosome number. We are looking for a statement
thaf support5
the idea that germ cell tumors arise from spematogonia. In the process of spermatog#esis,
spermatogonia will gir.e
rise to primary spermatocytes. In this process, the genetii complemint witi be reduced. In other
words.
spermatogonia have 46 chromosomes while primary spermatocytes have 23. Therefore, if tumor
cells did have their
origin in spermatogonia, they_should be diploid cells. The fact that tumor cells do not have a haploid
chromosome
number supports this theory. The correct choice is D.
98. B is correct, at least 90Vo of testicular cancers are found in the seminiferous tubules. The passage
tells us that onh
57o of testicular cancers do not originate from germ ceils. That means that aboutg5vo
do oiigina*te from germ celli.
Where are germ cells located? They are located in the seminiferous tubules of the teste. The
Jorrect choice is B.
99. D is correct, inf'ertile men run a greater risk for CIS. This answer could be anived at by a process of elimination.
Cryptorchidism is a risk factor. From the passage, we know that this condition arises from a developmental
error.
This supports our theory. The fact that tumors have been shown to deveiop in gestation supports the
theory that germ
cells tumors have their origins in fetal germ cells and have a misguided aevelopment. Furthermore,
the evidence thar
cancer genes act in fetal germ cells supports the theory that tumors come from fetal germ cells
and have a disrupte6
developmental fate. The only statement which does not support the theory is that inflrtile men run greater
a risk for
CIS- This really says nothing about misguided developmint and its connection to testicular cancer. The correct
choice is D.
100. D is correct, 92. This question is very straightforward. The number of chromosomes in a human haploid cell is 2_r
We are looking for a human cell that has a tetraploid number of chromosomes. Therefore,23 X 4 9l
chromosomes. The correct choice is D.
=
101. C is correct, testicular cancer is rare among twins. The passage informs us that testicular cancer has little basis 1r
genetics. We are looking for a statement that provides evidence for this notion. The fact that testicular cancer is raf;:
among identical twins is such a statement. If one twin developed testicular cancer, and the cancer had its basis ir
genetics, it is likely the other twin develops the cancer. Since testicular cancer is rare among twins, this statemenL.
supports the idea that testicular cancer has iittle basis in genetics. The correct choice is C.

102. D is correct, haploid adult from an unfertilized egg. This question is a strict definition type question that ca:
sometimes be found on the MCAT. Nothing in the passage directly tells us this definition. Often, these types o:
questions pertain to hormones. In this case, a developmental term was used. The correct choice is D.

Copyright @ by The Berkeley Review 264 The Berkeley Revier


Specializing in MCAT Preparation
ffifi€3Hogy
Section V A. The Endocrine Systern
l.
2.
Caiecholamine-Llormones
Peptide Hormones
3.
Endocrinology 4.
Steroid & Thyroid Hormones
RegulatoryMechanisms
& 5. The Pituitary Cland

Immunology The Immune $ystem


I. Cell Types
2. Antibody Structure
3. Antibody Action
4. T Cells
5.
' Humoral & Cellular Immunitv
---'* J
-

Practice Passages & Answers

,Ifu

n
Specidnztng in MCAT Preparation
Endocrinology & Immunotogy
Top lO Section Goals

Be familiar with catecholamine hormones and their actions.


CI? Catecholamines like dopamine, noreprnephrine, and epinephrine all contain a catechol ring (a benzene
ring with two adjacent hydroxyl groups). They act ai the'level of the cNS and pNS.

Be familiar with de hormones and their actions.


Peptide hormones contain two or more amino acids linked together by a peptide bond. These
hormones are secreted by a wide range of structures aid have a'va'riety of actions.

Be familiar the actions of the G and


Understand-what happens when a hormone like epinephrine binds to a cell surface receptor. Be
iamiiiar with the action of the C-protein and conversion of ATP to cAMP by adenylate iyclase.

At a fleneral level understand how phosphoryIation events activate Droteins.


A, classic example involves the phosphorylation of glycogen phosphorylase by protein kinase
A.
rhosphorylated glycogen phosphorylase is activated and can-convert glycogen to glucose.

Be familiar with steroid hormones and their actions.


The common plecursor to all steroid hormones is cholesterol. Steroid hormones are not stored after
their synthesis, but instead are used right away. Know the major groups of steroid hormones.

Be familiar with the mechanisms by which homeostasis is maintained.


Understand the concept o{ negarive feedback. Understand what is meant by endocrine, newoendocrine,
parac.rlne, ano autocrlne -regulatlon.

Know the hormones of the pituitarv.


It is.important to know.w_here the € majol hormones of the anterior and posterior pituitary are
synthesized and stored. Know why and how they are released into ttie circulat'ory sydtem.

Be aware of the different celt

leukocvtes are neutrophils,


neutroohils. eosinophils,
eosinonhils. basophiis,
ffi
involved with the immune
whlte blood cells/ or leukocytes, are involved in the body's immune response. The 5 different types
of leukocytes mrinorrrteq :nd
basonhils mcinocytes, lrimnh^"',ro. .
and ly?nphocytes

-?
Ogi Understand the basic structure and frrnction of an antibody.

Be familiar with the differences between humoral and cellular immunitv.


Understand the workings-of macrop_hages, MHC receptors, the different interleukins, the T cells and
B cells, the different interferons, and hdw they relate io foreign antigens.
Biology Endocrinology & lmmunology Catecholamine Hormones

CatechdIah.b'o0ilcs.:i:....l....]..;.'..
One way that cells can communicate with one another over long distances is
through extracellular products calied hormones. Hormones can generally be
divided into peptide hormones (e.g., insulin), amine hormones (e.g., epinephrine
or adrenaline which are classified as catecholamines), and steroids (e.g., the
male and fernale sex hormones). Hormones are released bv endocrine organs into
lhe blood and travel by way of the circulatory system to various target tissues.
\Vhen a particular endocrine cell is stimulated to release a hormone into the
':lood the
concentration of that hormone increases. Once the stimulation is
ierminated the hormone is no longer released the concentration falls back to
some normal resting level. Hormones generally have a very short lifetime in the
r1ood. Even though the time spent in the blood may be relativeiy short,
hormones can act within seconds or they can take hours and quite possibly dalrs
:o act on their target tissue. By comparison the action of the nervous system is
much faster.
l'Vhen a hormone acts on a target it can either do so at the level of the cell's
cell
membrane by binding to a specific receptor in that membrane or by passing
:hrough the membrane itself and binding with a specific target protein within the
'ell's cytoplasm. After the hormone has bound to its specific receptor that
:omplex undergoes a conformational change that allows for the synthesis of an
ntracelluiar messenger. This second messenger passes the information mediated
bv the hormone (i.e., the first messenger) to some specific reaction within the
:e11. For example, the second messenger might convey instructions that allow a
:articular set of reactions to release glucose into the biood or it might even act at
:re level of the gene to turn off or turn on gene expression.

Let's consider the action of the catecholamine epinephrine (adrenaline) on a


:r-pical hepatic (liver) cell. when the body is under some type of stress iike
:hysical exercise or even fright, an increased need for glucose arises. Glucose is
slored in the body in the form of glycogen and through a series of reactions can
:-.e mobilized and used as a source of energy during these times of stress.

Cnce a stress has been perceived the nervous system responds by signaling the
adrenal medulla (part of the adrenal gland which sits on top of the kidneys) to
:elease epinephrine into the extracellular fluid. Epinephrine diffuses into the
:1ood and is carried to the hepatic cells where it binds to a specific cell membrane
:eceptor called a B-adrenergic receptor. This action causes the activation of the
-nzyme adenylate cyclase (bound on the cvtoplasmic membrane surface) which
increases the concentration of the second messenger cyclic adenosine
monophosphate (cAMP) within the cell.

The binding of epinephrine, a water solubie hormone, to the cell surface receptor
and the synthesis of cAMP within the cytosol is coupled through the action of a
G protein. The name of this protein stems from the fact that it binds guanosine
triphosphate (GTP) and GDP. GTP and GDP are both nucleotides just like ATP
and ADP.

Initially, before epinephrine binds to its receptor, GDP is bound to a particular


subunit of the G protein. However, once epinephrine binds to its target receptor

Copyright O by The Berkeley Review 271 The Berkeley Review


Specializing in MCAT Preparation
Biology Endocrinology & Immunology Catecholamine llormones

this hormone-receptor complex interacts with the G protein and stimulates the
exchange of GDP for GTp. The G protein-GTp complex diffuses through the
membrane and activates the membrane bound adenylate cyclase enzyrile. In
turn, adenylate cyclase catalyzes the conversion of AT-p to .Rwtp. These events
are shown in the sequences (a) through (d) in Figure 5-1.

Hormone
(Epinephrine) s
Extracellular Space

(a) Mernbraneof
iLiVer C€ll ,

Cytosol
Inactive

Extracellular Space

(b) MernbraneOf
:Livei'cCitrr. ,

Cytosol

Extracellular Space

(c) IVlemli,fmb:,o-f
Adenylate . tivireeti..
Cyclase
Cytosol
Active

Extracellular Space

(d) Membianeo-f
LiVer,,GCll.
Cytosol

ATP cAMP + PP;

Figure 5.1
Action of adenylate cyclase.

After a brief period of time the G protein hydrolyzes the bound GTp to GDp and
Pi (not shown in Figure 5-1) thus returning the G protein back to its inactive
state (Figure 5-1a). However, during this period of activation (Figure 5-1d) the G
protein-GTP complex was able to activate many adenylate cyclaie enzymes thus
forming many molecules of cAMp. In other words, by the time G protein is

Copyright @ by The Berkeley Review 272 The Berkeley Review


Specializing in MCAT preparation
Biology Endocrinology & Immunology Catecholamine Hormones

ractivated the hormonal signal induced by epinephrine has been amplified


:ranv times.

Since cAMP is synthesized within the cytosol of the liver cell it can diffuse within
rat cytoplasm and interacts with a variety of molecules. One of the molecules
rat cAMP interacts with is a particular type of protein kinase called protein
kinase A (abbreviated as PKA). A protein kinase is simply an enzyme that
=ansfers the end phosphate (the gamma phosphate) of ATP to a specific amino
aid residue of a substrate protein. cAMP will bind to PKA and stimulate it to
:hosphorylate (add a phosphate group) an enzyme called glycogen
ghosphorylase. Once glycogen phosphorylase has become phosphorylated it is
:"orv an active enzyme and will catalyze the conversion of glycogen into glucose.
ItLis is shown in Figure 5-2. Glucose is released into the blood and travels
--rroughout the body by way of the circulatory system. [Recall that glycogen is
srmply the storage form of glucose in mammals. Also, there are other reactions
::rr-olved in this ampiification process. Only the important ones have been shown
rere.]

{<
cAMP ).PKA

Inactive
Glycogen
phosphorylase
I
Glycogen
g
Glucose

Figure 5.2
-{ctivation by phosphorylation.

Each enzyme in the cascade of reactions shown in Figure 5-2 promotes the
activation of many more molecules in the next step in the sequence. This is
exactly what we saw when epinephrine bound to its receptor. This rapid
amplification process allows the binding of just a few epinephrine molecules to
release grams of glucose into the blood. [In Figure 5-1 and Figure 5-2 the sites of
amplification are indicated by asterisks (*).1

Cholera is an intestinal disorder caused by a bacterium (Vibrio cholerae). The


major symptom of this disorder is diarrhea and if left untreated will result in
,revere dehydration and eventual death. This toxin binds to the active state of the
G protein and prevents the GTP from being hydrolyzed to GDP and P1. This
means that the adenylate cyclase enzyme is continually active and massive
amounts of cAMP are synthesized. cAMP causes the intestinal cells to secrete
idigestive) fluids.

Copyright @ by The Berkeley Review 273 The Berkeley Review


Specializing in MCAT Preparation
Biology Endocrinology & Immunology Peptide llormones

P-e$fiH
The water soluble peptide hormone gastrin stimulates the secretion of HCI and
pepsinogen from the_ stomach in response to stimulation from the vagus nerve
and partially digested protein.

Gastrin (the first messenger) binds to a specific ceII surface receptor in the plasma
membrane and activates a particular G protein (different tharrthe one in Figure
,!-3). The activated G protein interacts with the membrane enzyme phosphohlase
C (abbreviated as PLC) and induces that enzyme to hydroiyze phosphatiayt-
inositol-4,5-bisphosphate (PiP2) to inositol-1,4,5-triphosphate (ip:J -and 1',2-
diacyiglycerol (DAG).

Hormone
tGastrinl\
V
Extracellular Space

(a) Plasma,Memtiiane

Cytosol

(b)

Extracellular Space

(c)

Protein Protein-P
(inactive) (active)
Stimulates the
release of Ca2+ Ca2* Irr
\.2
HCI secretion
into the lumen
of the stomach

Figure 5-3
Activation of a receptor by a peptide hormone.

Next, the second messenger, IP3, which was released into the cytoplasm,
interacts with the endoplasmic reticulum and stimulates the release of Ca2e into

Copyright @ by The Berkeley Review 274 The Berkeley Review


Specializing in MCAT Preparation
Biology Dndocrinology & Immunology Peptide llormones

the cytoplarT by some unknown mechanism. Meanwhite DAG, diffusing


through the plasma membrane, interacts with a molecule called protein kinase d
and stimulates that kinase, with the help of Ca2 e just released from the
endopiasmic reticulum, to phosphorylate an unknown protein which in turn
causes HCI secretion into the lumen of the stomach. These mechanistic events are
outlined in (a) through (c) in Figure 5-3.

Insulin is a water soluble peptide hormone that binds to a specific trans-


membrane receptor in the cell membranes of liver, fat, and muscle cells. once
insulin binds to the receptor on the cell surface the cytoplasmic portion of the
receptor is converted into a tyrosine kinase that autophosphorylates the amino
acid tyrosine found within the cytoplasmic portion oi the te""ptor. This acts to
further enhance the activity of the tyrosine kinase. presumably the insulin
receptor can also internalize and (somehow) act as a second messenger. This
action, along with enhanced tyrosine kinase activity, leads to the internalization
of glucose into these cells (see Figure 5-4). The actual events in the insulin
signaling mechanism that leads to the uptake of glucose are somewhat obscure at
the present time. [Recail that if giucose is in abundance (after a meal), it can be
stored in the form of glycogen.l

Hormone
(Insulini
\
Extracellular Space

Plasma Nlembrane

2nd
Cytosol
Tyrosine kinase Internalization -----> Messenger
activity

Phosphorylation
activiry Glucose uptake

Figure 5.4
insulin signaling mechanism.

Copyright @ by The Berkeley Review 275 The Berkeley Review


Specializing in MCAT Preparation
Biology Endocrinology & Immunology Steroid & Thyroid Hormones

$',G-

These are also lipid solubte hormones which can pass through the cell's plasma
membrane and interact with a receptor either in the cytosol or in the nucleus.
Thyroid hormones, secreted from the thyroid gland, can diffuse across the
plasma membrane and into the nucleus where they bind with specific receptors.
The hormone receptor complex then activates transcription essential for certain
metabolic processes. Thyroid hormones help to regulate growth and
differentiation and they can stimulate the breakdowr-r of proteins, fats, and
glucose.

Steroid hormones all originate from a single precursor molecule, cholesterol. Two
important classes of steroid hormones are the androgens (male sex hormones
)
and estrogens (female sex hormones). Estrogery is lipid soluble, can
-hich
diffuse across the membrane of a target cell and bind with a specific receptor
with the cytoplasm. This steroid-receptor complex can then enter into the nucleus
where it influences the transcription of nRNA for certain protein products. The
general scheme for this action is shown in Figure 5-5.

Hormone-Receptor
Complex

Hormone
mRN A
@
D 'RNA 0
DNA
S
6)
Receptor -\-^,r-^r Proteins

=fl
oi:
OH c.) Nucleus

Figure 5-5
General action of a steroid hormone.

Copyright @ by The Berkeley Review 276 The Berkeley Review


Specializing in MCAT Preparation
Biology Endocrinology 8r Immunology R.egulatory Mechanisms

d b s
General Mechanisms
How is homeostasis maintained in the body? Homeostasis is simply the
maintenance of stability within the body. one way that homeo.iurl, l,
maintained is by negative feedback. ln this case there is some controlled variable
which is sensed by a particular sensor in the body. Through a variety of
mechanisms these sensors send input to control centers which in turn ,"g.,lut"
the controlied variable (see Figure 5-6). For example, if a certain produci was
being produced in a large quantity before negative feedback was initiated, then
after initiation of negative feedback a sma[er quantity would be produced.
Negative feedback reverses the output from elfectois such as gLnds and
muscles.

Controlled
-.);"#"
Variable
Fnt Sensor Mechanism
--\
Negative Feedback =>
Figure 5-6
\egative feedback.

Endocrine Regulation
thg endocrine system r,ve might have a grand secreting a hormone into the
]iblood. Thishormone, as we have seen, can infruence otheicells in the bodv. For
example, let's consider the regulation of blood glucose levels. \Arhy are the
tlood
levels of glucose so important? If glucose supplies to the brain ur"-i11terr.rpted
for
ar extended period of time, brain damage can resuit. Glucose turns out to be the
only fuel_utlrizedby the brain. [That is, -xcept during extended fasting in which
;ase the brain can use compounds called ketone bodles.] Thus, the controiled
i'ariable in this regulatory mechanism is the concentration of blood glucose.

B Ceils
-+ rnsurin es5 hil:ll::,t r----\
'
Uotake of:
Glucose
Blood glucose
increases
-
IGlucose]
stooo gru.os. fl
decreases I I
+ Blood Release of:
A cells Glucagon Liver &
-) Fat cells '-). Glucose
Fatty Acids
-
Figure 5-7
:ifects of glucose on the B-cells and u,-cells of the pancreas.

,lle pancreas secretes two hormones that are important in maintaining the
:roper levels of blood glucose. Both of these hormones are secreted from clJsters

3opyright @ by The Berkeiey Review 277 The Berkeley Review


Specializing in MCAT preparation
Biology Endocrinology & Immunology Regulatory Mechanisms

of specialized cells called the islets of Langerhans. Insulin is secreted by the


B
cells (or B cells) while glucagon is secreted by the cr cells (or A cells). After a
meal the levels of blood glucose begin to increase. This increase stimulates the
B
cells to release insulin into the blood. Insulin binds to specific receptors on liver,
fat, and muscle cells and through a complicated mechanis- pro*oi", the uptake
of glucose into these cells. This action tends to lower (by negative feedback) the
blood glucose levels. See Figure 5-7.

Conversely, if the levels of blood glucose levels begin to decrease below some
normal value, then the u cells are stimulated to secrete glucagon. This hormone
circulates in the blood and binds to specific receptors ot ii.ru. and fat cells.
Glucagon not only stimulates the liver cells to degrade glycogen to glucose but it
also stimulates the fat cells to release fatty acids. fatty iiias can be metab olized
and used in the Krebs cycle where they supply energy. This helps to alleviate the
need for glucose as an energy source. Through these negative feedback actions
the levels of glucose will begin to increase. SeeFigure 5-7.

Neuroendocrine Kegulation
In this case the hormone is not release for an endocrine cell but rather from a
nerve cell which releases its neurotransmitter in the form of a hormone into the
blood. For example, the adrenal medulla can receive sensory input from a
sympathetic nerve, which tells it to release epinephrine into the blood. Other
examples of neuroendocrine regulation involve the hypothalamus and the
pituitary gland. The pituitary gland can be divided -into the anterior and
posterior pituitary.

Paracrine Regulation
In paracrine regulation the chemical that acts as a signal is released from one cell
and influences a cell immediately adjacent to it (Figure 5-g). An example of such
a paracrine cell would be mast cells, which contain large amounts of histamine"
The substances released by the paracrine cell are generally dumped into the
extracellular space and not into the bloodstream. other examples o? a paracrine
signal would be neurohormones and neurotransmitters.

ffi
:;;"
[I-J/A-/ I
b a'.'
Paracrine I ^ -r\ a O
I
a
a
: :----- Signaring
compound Autocrine

Figure 5-B
Paracrine and autocrine regulation.

Autocrine Kegulation
In autocrine regulation cells can release certain chemicals which they can then
respond to themselves (Figure 5-8). For example, certain cells (like tumor cellsrl
can release growth factors which can then bind to specific receptors on the
membrane of that same cell. Thus, the cells that releasedthe growth hormone are
stimulated to grow. Prostaglandins, which are lipid-soluble chemicals, alsn
appear to show autocrine regulation.

Copyright @ by The Berkeley Review 27A The Berkeley Review


Specializing in MCAT Preparation
Biology Endocrinology & Immunology The Pituitary Gland

Posterior Pituitary
The posterior pituitary releases oxytocin and antidiuretic hormone (Figure 5-9).
These two hormones are synthesized in specific cells of the hypothalamus. The
major effect of oxytocin is to stimulate {emale uterine contraction while
antidiuretic hormone (ADH) stimulates water reabsorption in the kidneys and
also helps to increase the blood volume (pressure).

Hypothalmic Secretory
Neurons

v--
V,

Pituitary
Stalk

Capillariestr) Artery

Anterior
Pituitary
fr Capillaries

Posterior
Pituitary

TSH, ACTH, FSH, LH


Oxytocin
GH, PRL
ADH

Figure 5-9
The hormones released from the pituitary gland.

The ADH that is synthesized in the nerve cell bodies in the hypothalamus are
packaged into vesicles and transported down the axon to the terminal bouton in
the posterior pituitary. A nerve impulse (which propagated down the same axon)
causes the release of these hormones into a system of nearby capillaries. ADH
circulates in the biood and eventually reaches the kidneys where it stimulates
ivater and Na@ reabsorption.

The nerve cell bodies in the hypothalamus where ADH was synthesized can
sense a change in biood volume (and a change in the concentration of Nae). If
these cells sense a low biood volume, they release ADH into the blood. At the
Ievel of the kidneys ADH will stimulate the retention of as much water as
possibie. In other words, ADH prevents diuresis which is the excessive loss

Copyright @ by The Berkeley Review 279 The Berkeley Review


Specializing in MCAT Preparation
Biology Endocrinology & Immunology The Pituitary Gland

urine. By preventing diuresis water can be reabsorbed and the blood volume
increased.

Anterior Pituitary
pituitary can secrete six hormones (Figure 5-39). They are thyroid
The anterior
stimulating hormone (TSH), adrenocorticotropic hormone (ACTH), ronicte
stimulating hormone
!FSH), luteinizing hormone (LH), growth hormone (GH),
and proiactin (PRL). we will discuss the function of ihese hormones as we
proceed in the course.

These hormones are reguiatedby a second set of hormones which are stored in
hypothalamic nerves. For example,let's consider TSH. within the hypothalamus
are nerve cells that contain thyrotropin releasing hormone (TRH). when this
nerve is stimulated it secretes TRH into a set of capillaries which extend into the
anterior pituitary. TRH stimulates the synthesis and release of TSH from the
anterior pituitary into the biood. TSH binds to specialized receptors in the
thyroid gland and causes the release of thyroxine. Tiris hormone, when released
from the thyroid gland, influence metabolism and growth.

If the metabolic rate of the body is too low, TSH is released from the anterior
pituitary and stimulates the release of thyroxine from the thyroid gland. once
thyroxine increases metabolism to just the right level (homeostasis)1t acts back
on the nerve cells in the hypothalamus and the cells in the anterior pituitary and
inhibits the release of rRH and rsH, respectively. Again, this is ur,oih", example
of negative feedback.

Copyright @ by The Berkeley Review zao The Berkeley Review


Specializing in MCAT Preparation
Biology Endocrinology & Immunology Cell Types

Gellii S"'

Every day your body is subiect to countless attacks by microorganisms and


viruses which can cause a variety of diseases. Bacteria can enter itttJyoru system
and disrupt cellular function. One example that we have mentioned is massive
diarrhea caused by the bacterium Vibrio cholerae. The bacterial toxin from this
organism can cause over a liter of {luid to be lost every hour. Parasitic viruses can
also reap havoc on the body. Viruses are much smallr than bacteria and contain
a nucleic acid core (either RNA or DNA) surrounded by a protein coat.
viruses
lack their own metabolicrnachinery. once they invade ur-, orgunism they can
take
over the host's metabolic machinery for their own use und ge.rerate more
progeny viruses which, after the infected cell lyses, can spread and infect other
cells. However, there are certain conditions *il"t" the viral genome integrates
rnto the chromosome of the host cel1. In this case the genome can be
'iril genome
replicated when the host chromosome is replicated. If this viral removes
itseif from the host genome, it can take orr"i th" metabolic machlery of
the host
cell and produce more progeny viruses. Cancer causing viruses can insert their
genetic material in to host chromosomes as well. Viruses can
cause a variety of
ciseases ranging from smali pox and infiuenza to measles and the common
cold.
Ii we were to examine some blood uncler the microscope, we would find two
lifferent cells types-the erythrocytes (red blood cells) and the leukocytes (white
rlood cells). Erythrocytes in adults are produced in the maffow of thre sternum,
:ibs, and vertebrae. Leukocytes uru ptod.r""d partially in the tissues of the
lymph
,:nd partiy in bone marrow. we have already coniidered the erythrocytes'in
-:-revious sections. Let's now briefly consider the leukocytes.

Sere are 6 types of leukocytes found in the blood. The three types of leukocytes
rat we are interested in are the monocytes, neutrophils, andiymphocytes. "The
::.onocytes and neutrophils are considered to be phagocytes. .iheie ceils,
'"'ith mast cells and a variety of other along
ceil typei, participate in the immune
response.

\Iast Cells: These ceils are derived from leukocytes and then migrate out into the
r>uc> where
-sues wrtcre they
Lrrey resloe.
reside. \44ren
vvnen mast CelS ,iiuur" histamine
cells are stimulated they reiease
' hich acts on endothelial cells and causes an increased permeability to cells like
:,e neutrophils. This increased permeability ailows the neutrophils easy access to
--:.e surrounding
tissue in order to defend against foreign bacteria or viruses.

?hagocytes: As we have mentioned, monocytes and neutrophils are both


::-agocytes. when monocytes leave the blood through pores in the blood vessels
':-d enter into the tissues, they can be transform"d ir-tto macrophages.
I eutrophils circulate in the blood. Flowever, they too can leave the blood
*

*^:ough pores in the blood vessels and enter into the tissues. The macrophages
,:,d neutrophils are_lhe plimary ceII types that attack and destroy ior"ig'
, .:teria and viruses.
T"y do this by the process of phagocytosis, enguifing tie
,:eign invader by endocytosis. once inside the phagocyte iyscsomi-l ur,r1i*u,
, : hydrogen peroxide is released which can then d-egrade ihe foreign objects.

:eyright @ by The Berkeiey Review The Berkeley Review


Specializing in MCAT preparation
Biology 'Endocrinology & Immunology Cell Types

Lymphocytes: There are 2 types of l1'rnphocytes. Lymphocytes which are derived


from the thymus gland are called T lymphocytes (or T cells). The other type of
lymphocytes are called B lymphocytes (or B cells). Let's briefly consider each
type'

T lymphocytes: These cells are responsible for cell-mediated immunity. These


cells are responsible for the destruction of foreign microorganisms and other
such agents harmful to the body. There are 3 types of T lymphocytes which we
will be considering: (1) cytotoxic T cells (also called killer T cells), (2) helper T
cells, and (3) suppressor T cells.

B lymphocytes: These cells circulate in the blood and to the lymph organs like
the spleen and lymph nodes and are responsible for humoral mediated
immunity. [Humor is another way of saying fluid.] Upon infection the B
lymphocytes can differentiate into plasma cells which have the ability to
slmthesize and secrete antibodies. Antibodies are proteins that are synthesized
in response to an antigen that has been introduced into the body. An antigen is
simply a foreign substance (protein or polysaccharide) which has a high
molecular weight that has entered the body and induces a particular immune
response. Antigens interact with specific antibodies.

General Immune Kesponse


Let's consider a general response to the invasion of a bacterium or even a vims.
These invaders have on their surface an antigen. The cellular immune response
begins with a macrophage engulfing a foreign particle and phagocytizing it, The
antigenic fragments released into the cytosol of the macrophage are transported
to the macrophage's membrane where they bind to a specific surface proteiru
This specific surface protein is called a major histocompatibility compler
(MHC) protein (of the Class I typu). The antigen is being presented or
"displayed" on the surface of the macrophage.

Certain receptors on the class of T lymphocytes called cytotoxic T cells can


recognize this antigen-surface protein complex on the macrophage. When the
cytotoxic T cell binds to the antigen-Class I MHC protein complex of the
macrophage, a growth factor called interleukin-L is released by the macrophage-
The cytotoxic T cell itself releases a growth factor called interleukin-2
Interleukin-l, interleukin-2, and interleukins released by helper T cells
(discussed below) stimulate the slmthesis of more cytotoxic T cells. These kilier T
cells proliferate and bind to the invading foreign cells beadng the antigen and
induce them to lyse. This type of response is referred to as cell mediated
immunity.

On the surface of B lymphocytes are Class II MHC proteins and antibodies-


When a B lymphocyte finds an antigen that has specifically bound to its
antibody, it engulfs that antigen-antibody complex. After degrading the antigerr
antibody complex the B lymphocyte transports a portion of that antigen to
Class II MHC protein and "displays" the complex on the surface of i
membrane.

Helper T cells with the right receptors are able to bind to the antigen-Class
MHC protein complex. This binding stimulates the helper T cells to re
interleukins (a lymphokine). This stimulates the B lymphocytes to pro
and form plasma cells. The plasma cells in turn produce a vast amount

Copyright @ by The Berkeley Review 282 The Berkeley


Specializing in MCAT
Biotogy Endocrinology Er Immunology Cell Types
antibodies which are specific for the antigen. when
these circulating antibodies
bind to the antigen they act as a tag tnut rigg
the antigen-antibody comprex and-destroy"it. "rr".riutirg fnugocytes to engurf
This type of?elfoLr" i, referred to
as humoral mediated immunity.

The human immunodeficiency virus (HIV),


thought to be responsible for the
acquired immune deficiency syndrome (AIDS),
ac"ts at the levei of the h"rp";i
cells by infecting them.

l r:lright @ by The Berkeley Review 2a3 The Berkeley Review


Speciatizing in MCAT prelraration
Biology Endocrinology & Immunology Antibody Structure

ffi
Most human antibodies, or immunoglogulins (abbreviated as Ig), are composed
of 4 subunits arranged in a "Y" configuration as shown in Figure 5-10. There are 2
light chains and 2 heavy chains. These subunits are joined to one another br,
disulfide bonds. Within each heavy and light chain are variable domains and
constant domains.

At the terminal ends of the heavy and light chains are variabie (v) regions that
can differ in amino acid sequence from immunoglobulin to immunoglobulin. The
constant (C) regions of the heavy and light chains are found in the lower portion-.
of the immunoglobulin. The antigen binding site for a particular antibody i:
located at the end of the variable regions of the heavy and light chains.

Two other regions of the immunoglobulin are important. They are the diversitr-
(D) region and the joining (j) region. The genes that code for the I, D, and \-
regions of the variable domain greatly increase the diversity one finds among the
immunoglobulins.

Antigen
V = variable .il$
D = diversity ^$' Antigen
binding binding
site ; = joining site
^no*7

Light Chai n t
Constant
Heavy Chain Domains
)
Disulfide bond

B Cell Membrane

Figure 5-lO
Generalized antibody structure.

Classes of Antibodies
We have 5 classes of immunoglobulins which differ in the composition of
heavy chains. They are (in alphabetical order) IgA, IgD, IgE,IgG, and IgM.
functions are:

Copyright @ by The Berkeley Review 2a4 The Berkeley


Specializing in MCAT
Biotogy Endocrinology & Immunology Antibody Structure

IgA: Found in milk and helps to protect nursing infants.


IgD: Has an unknown function.
IgE: Binds to mast cells and is invoived in the allergic reaction.
IgG: The only antibody able to cross the placenta. iI is also the most abundant
and is produced within days after the IgM antibody is secreted.
IgM: Produced a few days after detection of an antigen and it is the first
antibody produced in response to an antigen.

Combinatorial Diversifi cation


The light chain contains at least 300 different genes that can determine
the
variable region and about 4 different genes thit can determine that joining
region. Therefore, there can be about (300) x (4) or -1.,200 different
combinations
for the light chain. The heavy chain contains about 1,000 different genes
for the
variable region, about 'l I different genes for the diversity region,
and about 4
different genes for the joining region. Thus, we find that tirere ire about (1,000)
(rz1 * 141 or 48,000 different combinations for
x
the heavy chain. If we associate all
the different possible light chains with all the different possible
heavy chains, we
wili get a combinatorial diversification of about (.1,,200) * (+S,OOO) or roughly
5.g x
107 different possibie antibody combinations. In other words,
the immune
system_can generate an antibody for practically any antigen
that invades the
body. [The point behind this is the enormous antiboJy diversity
that is
gene-rated. Depending on how the calculation is done and what
values are used
the diversity can be as high as 1.1 x 1011.1

Copyright @ by The Berkeley Review 2a5 The Berkeley Review


Specializing in MCAT preparation
Biology Endocrinology & Immunology Antibody Action

Antibodies do not destroy foreign antigens. They simply recognize and identify
them. Antibodies can do this is a number of ways. One method is by directly
blocking the foreign invader from gaining access to host cells. This is
accomplished by the antibodies binding to the antigens of, say, a virus as shown
in Figure 5-11.

Antigen

Figure 5.1I
Direct block.

Complement is a rather complicated system for disposing of invaders. The


essence of this process is as follows: An antibody has already recognized and
bound to a specific antigen on a bacterial cell that is considered an intruder. A
complement protein (a plasma protein) recognizes this antigen-antibody complex
and binds to the Fc domain of the antibody. After a series of reactions the
complement protein is activated and triggers an immune response. Further
reactions form a membrane attack complex (MAC) that inserts into the bacteriai
cell's membrane and forms a channel that lets water into the cell. The bacterial
cell swells with water and eventually lyses (bursts). See Figure 5-12.

Activation

Complement Complement
Protein Protein

Figure 5. l2
Complement.

Antibodies can bind to specific antigens on the surface of a bacterial cell and cod
the cell surface. Once the antibodies have attached to the bacterial ce*
phagocytes and/or killer T cells can bind to the terminal portion of the Fc
domain of the antibodies and begin to engulf the foreign invader as shor,rrr:mr
Figure 5-13.
Figure 5-15
Cell surface coating

Copyright @ by The Berkeley Review


Biology Endocrinology 6r Immunology T Cells

The receptors for T cells are composed of two polypeptide chains, each with a
constant and a variable domain. within the variable domain in each polypeptide
chain we find a variable (v) region and a joining (j) region. on one of the
polypeptide chain is a diversity (D) region. see Figlre s-r+. Thls is analogous
to
the immunoglobulin structure in that a great amount of (antigen uiiraingy
diversity can be generated from different combinations of the genes that produJe
ihese polypeptide chains.

Antigen binding
site

v*,ur.
Domain 1
fv Variable

V = variable
D = diversity
3 = joining
Lr
il Domain

Constant Domain

T Cell Membrane Cytoplasm

Figure 5-14
T Ce11 receptor.

Cytotoxic T cells have cell receptors like the one shown in Figure 5-15. If a virus
infects a host cell, then that virus will begin to take over tie host's metabolic
machinery. As this happens some of the iiral antigens are transported to the
surface of the host cell where they can complex *ith u Class I MHC protein
receptor. class I MHC receptors are found on almost every one of our celli.

MHC I receptor
antigen complex
MHC I receptor r-)

o-
&;
i)*:r'1
,Y{ Cytotoxic
\r_. T cell
f.-dffi^
Figure 5- l5
lvtotoxic T Cell interaction with MHC I receptor.

-he antigen-Class I MHC protein compiex is recognized by the cytotoxic T cell.


3inding occurs between the cytotoxic T cell and the antigen-bearing host cell as
=hown in Figure 5-15. The cytotoxic T cells induce lysis in the host cell contain

Jopyright @ by The Berkeley Review 247 The Berkeley Keview


Specializing in MCAT preparation
Biology Endocrinology & Immunology T Cells

the viral particles in order to prevent more progeny viral particles from
spreading.

Immature helper T cells recognize macrophages which have presented an


antigen on their Class II MHC protein receptors as shown in Figure 5-16.
Binding induces the macrophage to synthesize and reiease interleukin-l which
acts on the immature heiper T cell and causes it to synthesize and release
interleukin-2. Interleukin-2 further stimulates the immature helper T cell to
proliferate into a mature helper T ceil. The mature form of the helper T cell
secretes interleukin-2 which can activate cytotoxic T cells, B cells, and more
helper T cells.

MHC II receptor
antigen complex
MHC tr receptor r-)
2a

50>
dE
H,r
ry HeIper
T cell
O OI)
Vnal /
2- poteins-\r1
Activates:
cytotoxic T cells, B cells-
helper T cells
Interleukin- 1

Figure 5.16
Cytotoxic T Cell interaction with MHC I receptor

.r*

Copyright O by The Berkeley Review The Berkeley


Specializing in MCAT
Biology Endocrinology Er Immunology tlumoral & Celular Immunity

ffi*
Generalized Review
-
€t's summarize the basic events in humoral and cellular
immunity. The
:r:nbers in these steps will correspond to the numbers
in Figure S-17.

7. A virus enters the body by the blood and is engulfed by a macrophage.

on the surface of the macrophage are Class I MHC receptors and Class
II MHC receptors.
J. A Class II MHC receptor presents the virar antigen to the receptor
of a
helper T cell.

t-i This causes the macrophage to release interleukin_l (fl-l).


_i. IL-1 stimulates the helper T celis to proliferate.

{i. The helper T cell is stimulated to release IL_2 which enhances


proliferation of helper T cells.

A B cell with a Class rI MHC protein presents virar antigen to heiper T


cell.

IL-2 released from herper T celr stimulates B ceils to proriferate.

!B
B cells produce plasma cells and memory B cells. Memory
'remember" antigen and proliferate faster iuring
B cells
a future inlasion of
the same virus.

!3. Plasma cells secrete antibodies specific for the viral antigen.

I-llt- The.antibodies respond by direct brock, comprement, and


cell surface
coating.

rL-2 from the helper T cells stimulates cytotoxic T cells which have
bound to the class I MHC protein-antigen complex of an infected
cell
to lyse that infected cell. :ir .

L-i. lnterferon is secreted by the infected host cell and acts on the cytotoxic
T cell to help enhance the immune response.

n4- Cytotoxic T cell aiso make memory T cells (not shown) which will
proiiferate faster during a future invasion of the same virus.

ht O by The Berkeley Review 249


Biotogy Endocrinology & Immunology Humoral & Cellutar Immunity

t
#
\\

) (rr/-
\,",!liJ,"" Y /
Lf"_\
)" Y
\,v
Direct Block

6",;N
\u Complement
v Mast

{YP- ",
V Cells
s
Histamine
/\.
Antibodies s
Leaky
(10)
Endothelial
Cells
Cell Surface Coating

Figure 5.17
Review of humoral and cellular immunity.

Copyright @ by The Berkeley Review The Berkeley


Specializing in MCAT
Endocrinology
& Immlrlrology
To Go
15 Passages

I03 Questions

Time forAll Passages Taken Together as a practice Exam


125 Minutes

Passage Titles Questions


I. Antibody Structure | -7
U. Acquirer{ Imntunint 8-14
IIII. Major Histocompatibility Complex (MHC) T5 -21
IV. IgA Antibody Experiment 11 11
LL--I
V. Complement System 28-33
VI. MyastheniaGravis/AutoimmuneDisease 34-4A
VIII. Giucotu, Glucagon, & Insulin 4t-46
VUI. Calcium, PTH, Calcitonin, & Calcitriol 47 -54
IX. Erythroblaslosls Fetalis 55-61
X. Septic Shock 62-61
XI. Calcitonin & Osteoporosis 68-15
XII. Vertebrate Immune System 76-83
X[I. Insulin Receptor 84-90
XIV. Vitamin D3, PTH, & Calcitonin 91-96
XV. Sepsis Syndrome 97 - r03
Suggestions
The passages that foliow are designed to get you to think in a conceptual manner about the processes
of physiology at the organismal level. If you have a solid foundation in physiology, many of these
answers will be straightforward. If you have not had a pleasant experience with the topic, some of these
answers might appear to come from the void past the Oort field of the solar system.

Pick a few passage topics at random. For these initial few passages, do not worry about the time. Just
focus on what is expected of you. First, read the passage. Second, Iook at any diagrams, charts, or
graphs. Third, read each question and the accompanying answers carefully. Fourth, answer the
questions the best you can. Check the solutions and see how you did. \ trhether you got the answers right
or wrong, it is important to read the explanations and see if you understand (and agree with) what is
being explained. Keep a record of your results.

After you feel comfortable with the format of those initial few passages, pick another block of
passages and trythem. Be aware that time is going to become important. Generally, you wiil have about
1 minute and 15 seconds to complete a question. Be a little more creative in how you approach this next
group. If you feel comfortable with the outline presented above, fine. If not, then try different
approaches to a passage. For example, you might feel well versed enough to read the questions first and
then try to answer some of them, without ever having read the passage. Maybe you can answer some of
the questions by just looking at the diagrams, charts, or graphs that are presented in a particular passage.
Remember, we are not clones of one another. You need to begin to develop a format that works best for
you. Keeping a record of your results may be helpful.

The last block of passages might contain topics that are unfamiliar to you. Find a place where the
level of distraction is at a minimum. Get out your watch and time yourself on these passages, either
individually or as a group. It is important to have a feel for time, and how much is passing as you try to
answer each question. Never let a question get you flustered. If you cannot figure out what the answer is
from information given to you in the passage, or from your own knowledge-base, dump it and move on
to the next question. As you do this, make a note of that pesky question and come back to it at the end,
when you have more time. When you are finished, check your answers and make sure you understand
the solutions. Be inquisitive. If you do not know the answer to something, look it up. The solution tends
to stay with you longer. (For example, what is the Oort field?)

The estimated score conversions for 100 questions are shown below. At best, these are rough
approximations and should be used only to give one a feel for which ballpark they are sitting in.

Section V
Estimated Score Conversions
Scaled Score Raw Score

>72 86 - 100
10-11 79-85
8-9 65 -78
7 59-64
6 54-58
5 48-53
<4 0-47
Biology Antibody Structure Passage I

Passage I (Questions 1-7) Immunoglobulins fall into five classes (isotypes). The
approximate percentage of total antibody in the serum
In order to protect themselves from pathogens, animals immunoglobulin pool is indicated in Table 1.
have developed an array of protective mechanisms within
fieir immune system. Immunity can be divided into two Tabie l: Antibody Classes
ivpes: innate (nonspecific) and acquired (specific).
,l!gtyp" 7o of plasma Ig
Innate immunity affords protection against a multitude IgG 70-75
of different pathogens and is not dependent on specific IgA 15-20
:ecognition of foreign macromolecules. The skin, mucous IgM 10
:rembranes, and enzymes in secretions are all examples of IgD <i
nonspecific immunity. IgE = 0.005

Acquired immunity enhances the protection provided In humans IgG is the only antibody that can cross the
:''' innate immunity and is the result of an initial encounter placental membrane. IgA is primarily found in external
^ith a foreign macromolecule (immunization). If an secretions such as tears, saliva, colostrum, and milk. IgM
-nmune response is activated, the agent inducing that is the first antibody produced by rhe plasma cells during
:.sponse (an immunogen) will initiate a series of reactions an tmmune response and is the predominant antibody
:at lead to the production of antibodies (proteins) that are produced by the fetus during development. IgD has an
.:ecific against the inducing agent. The ability of the unknown function, but since it is found as a surface
,:ducing agent to combine with an antibody is calied protein in many B cells, it may play a role in B cell
;.trigeniciry, and an agent with this characteristic is called differentiation into a more mature form like plasma cells
::, antigen. Vaccinations and a recovery from a disease or memory B cells. IgE is thought to confer immunity to
--i.e the measles are examples of specific immunity. certain parasites, such as worms. IgE, can also bind to
basophils and mast cells where, in mast cells, it promotes
-\ntibodies are globuiar glycoproteins that participate the release of histamine, a chemical that causes piripheral
.: the immune response. These immunoglobulins (Ig) are vasodilation and increased capillary permeability, leading
::3sent in the y-globulin component of blood serum and to anaphylactic reactions.
':: produced by plasma cells. All antibodies are related to
. ,: another through a common motif (Figure 1). The fragment of the antibody that binds anrigen is the
antigen binding domain. The binding occurs between the
paratope of the antibody and the epitope (antigenic
determinant) of the antigen. An antigen may possess just a
single epitope on its surface, making it a unideterminant,
univalent antigen. Antigens may also be unideterminant
and multivalent, multideterminant and univalent, or multi-
determinant and multivalent. Antibody-antigen interaction
can result in a variety of outcomes, including precipitation
and agglutination. These complexes can be dissociited by
high salt concentration or either high or low pH.

1. Which of the fbllowing is NOT a componenr of the


nonspecific immune system?
Figure I

Key for IgG


A. Keratinized and epidermal cells.
B. Beating of epithelial cell cilia.
C = Constant region C. Placental transfer of antibody.
V = Variable region D. Normal body temperature.
L = Light chain
H = Heavy chain 2- Which of the following types of immunity would
Fab = antigen binding fragment result from a vaccination?
Fc = crystailizable fragment
I = Disulfide bonds A. Active immunity that is naturally acquired.
---!- = Papain cleaves B. Passive immunity that is artificially acquired.
Pepsin cleaves c. Active immunity that is artificially acquired.
D. Passive immunity that is naturally acquired.

: n nght @ by The Berkeley Review 293 The Berkeley Review


Specializing in MCAT Preparation
Antibody Structure Passage I

3. The binding of antibody to antigen occurs through 7. Lysozyme inhibits peptidoglycan synthesis in the
all of the foliowing EXCEPT; cell wall of gram negative bacteria. Which of the
following immunoglobulins would act agonistically
A. Covalent bonding. with lysozyme?
B. Hydrogen bonding.
C. Hydrophobic forces. A. IgG
D. Van der Waals forces. B. IgA
C. IgM
D. IgE

4. A paratope can best be described as that area ofthe:

A. antigen which reacts with the antibody.


B. antibody designated by the Vg regions of the
Fab.
C. antigen which exhibits a multideterminant and
univalent expression.
D. antibody which has similar dimensions as the
antigenic determinant.

5. Papain, a protease extracted from papaya plants,


cieaves primarily on the carboxyl side of lysine and
arginine amino acid residues. Treatment of IgG with
papain before IgG is exposed to a unideterminant,
multivalent antigen will result in:

A. cross-linking, because the antigen possesses


just one type of determinant but many of them.
B. an Fc fragment and a bivalent Fab fragment,
which can participate in cross-linking.
C. antigens that are not cross-linked, because
each Fab fragment is monovalent.
D. two monovalent Fab fragments, which can
participate in cross-linking.

6. Treponema pallidium, an organism that causes


syphilis, can cross the placental membrane and enter
into the fetal circulation. Which of the following
immunoglobulins would be expected to increase in
concentration in the blood serum ofthe fetus?

A. IgG
B. IgA
C. IgM
D. IgE

Copyright @ by The Berkeley Review The Berkeley


Specializing in MCAT
Biology Acquired Immunity Passage II

Passage II (Questions 8-14) 10. The following diagram indicates the primary and
secondary response to an antigen. IgG, a monomer,
Acquired immunity is the resistance to disease that an crosses the blood vessels easily, while
organism develops during its lifetime. Immunity may be
IgM, a
pentamer, does not cross as well.
acquired actively or passively. Active immunity occurs
rvhen an organism is attacked by a foreign substance or a
microorganism (an antigen). Antibodies and specialized
ivmphocytes are produced by this exposure that confer a
'memory" when the foreign 3
!
agent attacks again at a later
rime. Passive immunity occurs when an organism receives o! 6 100

pre-formed antibodies from another organism. e7


g>'
Acquired immunity may be achieved through natural '-Elo
or artificial means. For instance, a person may be exposed
^.=
to the chicken pox virus, develop a rash, and beiome
rmmune to chicken pox for the rest of his life. This
:rrmunity is actively acquired through a natural means.
Conversely, a new chicken pox vaccine would provide
tnificially acquired active immvnity. Vaccines doiontain
rricroorganisms that may be dead or weakened. The
0 7 t421 28 35 42
';ubstances in vaccines stimulate an active immune
49
Time (days)
:esponse. An example of passive natural immunity is the
naternal provision of antibodies to the fetus via the blood
Which of the following statements is FALSE?
md to the neonate via breastmilk. The baby has an
rmmature immune system, and these antibodies function
:or several months as the baby's system matures. An A. A "booster shot" for an adult exposure to
tetanus promotes formation of mainly IgG.
:rample of passive artificial immunity is receiving
B. Before the primary exposure, there are no
"atibodies from a rabbit that was immune to a spidei antibodies to a particular antigen.
- enom, as
a treatment for a spider bite.
C. Both IgG and IgM remain at high levels
following a "booster shot".
When a person is passively immune to a foreign
D. IgM provides the primary response to an
antigen.
agent, is this a permanent immunity?

A. Yes, receiving antibodies confers permanent


immunity. 11. A susceptible adult individual received pre-formed
B. Yes, making antibodies confers permanent antibodies isolated from the serum of in immune
immunity. individual. What type of acquired immunity
C. No, receiving antibodies confers transient this confer on the first individual?
does
immunity.
D. No, making antibodies confers transient A. Artificially acquired acrive immunity.
immunity. B. Artificially acquired passive immunity.
C. Naturally acquired active immunity.
q. A newborn child is accidentally kissed by her
D. Naturally acquired passive immunity.
cousin, who is contagious with measles. Why does
the newborn stay healthy?
12. The first vaccine by Edward Jenner in 1798
A. Maternal antibodies provided immunity to protected against smallpox. He used a related strain
measles in this situation.
B. The newborn child quickly made antibodies to
of microorganisms that caused cowpox as a
vaccination against the microorganisms of smallpox.
the measles virus. What type of immunity was conferred in this casl?
C. The cousin provided antibodies by kissing the
newborn child. A. Artificially acquired active immunity.
D. Paternal antibodies provided immunity to B. Artificially acquired passive immunity.
measles in this situation.
C. Naturally acquired active immunity.
D. Naturally acquired passive immunity.

-:pyright @ by The Berkeley Review 295 The Berkeley Review


Specializing in MCAT preparation
Biology Acquired Immunity Passage

13. Blood serum is subjected to electrophoresis in order


to separate the proteins. Antibodies are present in the
gamma globulin fraction. The following diagram
shows serum proteins following gel electrophoresis:

Direction of migration

Cathode Anode
(-) (+)

B
Globulins Albumin

In this diagram, which protein is the largest, based


on its migration pattern?

A. albumin
B. a-globulin
C. B-globulin
D. y-globulin

14. If a newborn is orphaned at birth, which process


would provide more antibodies for the child?

I. Feed the chiid breastmilk from another nursing


mother.
II. Seclude the child in a sterile hospital unit.
IfI. Vaccinate the child immediatelv to all
childhood diseases.

A. i only
B. I and II only
C. II and III only
D. I, II, and III

Copyright @ by The Berkeley Review The Berkeley Reviev


Specializing in MCAT Preparation
Biology Major llistocompatibility Complex (MHC) Passage III

Passage III (Questions 15-21) Endosomes are membrane vesicles that often contain
surface proteins and their associated ligands. Endosomes
Transplantation antigen proteins expressed on the are known to contain proteases, which are beiieved to
surface of a cell and recognized by the immune system degrade the invariant chain and allow the molecule to
are encoded by the major histocompatibility complex bind peptide. However, a small part of the invariant chain
(MHC) genes. The MHC genes are of two fundamental (known as CLIP) remains at the binding cleft until a DM
types, class I and class II. While mouse and human molecule (an MHC class Il-like molecule) enters the
populations carry more than one hundred forms of the endosome and binds CLIP, actively removing it from the
molecules, only between three and six of each class are MHC class II molecule. At this time, the MHC class II
expressed. molecule, bound with peptide, is placed on the cell
surface-
Both classes of MHC molecules are involved with
antigen processing, which includes the ingestion of
antigens, the fragmentation of antigens into peptides, and
the binding of these peptides to MHC molecules. The 15. The amino acid sequence of both the MHC class I
formation of the MHC-peptide complex is a critical event and MHC class II molecules should show:
in the effective elimination of intracellular parasites. The
peptides associated with MHC class I have invariably A. hydrophilicregions.
been found to originate from a cell's own proteins, while B. hydrophobicregions.
the peptides found bound to MHC class II are normally C. both hydrophobic and hydroapathetic regions.
located on the outer membrane. D. both hydrophilic and hydrophobic regions.

The MHC class I protein usually binds peptides that


are eight to nine amino acids long. The two ends of the
peptide chain bind to discrete binding sites located in the
cleft of the MHC class I molecule (Figure 1). The binding
cleft of an MHC class II molecule is similar in shape to 16. On the surface of a cell infected by a virus, the
that of an MHC class I molecule, but the MHC class II majority of MHC class I molecules have bound
molecule usually binds peptides in the middle of the cleft peptides that originate from:
(Figure 2).
A. the virus.
B. akillerTcell.
C. a B cell.
D. a host cell.

MHC ciass II
molecule 17. The two protein subunits that constitute the MHC
class II molecule are MOST likely to contain a:

Membrane A. Shine-Dalgarnosequence.
,\ B. signal peptide sequence.
C. pyrimidine-richsequence.
D. purine-richsequence.

MHC class II molecules have been shown to assemble 18. Endosomes are formed by which of the following
in the endoplasmic reticulum. However, immediately after processes?
their synthesis, the MHC ciass II subunits associate with a
third molecule known as the invariant chain. The A. Megacytosis
invariant chain inhibits peptides from binding to the MHC B. Exocytosis
class II molecules and causes the MHC class II molecules C. Endocytosis
to leave the Golgi complex and fuse with endosomes. D. Transcytosis

Copyright @ by The Berkeley Review 297 The Berkeley Review


Specializing in MCAT Preparation
Biology Major llistocompatibitity Complex (MflC) Passage III

19. The protein DM is structurally similar to:

A. the invariant chain.


B. MHC class lI.
C. CLIP.
D. MHC class I.

20. Which of the following statements is true regarding


peptides bound to MHC class I and MHC class II
proteins?

I. Peptides bound to MHC class I should exhibit


a greater size variation.
II. Peptides bound to MHC class II should exhibit
a greater size variation.
III. The amino acids binding to a cleft should be
conserved despite the variety of peptides the
cleft can bind.

A. I only
B. II only
C. I and III only
D. II and III only

2I. Which of the following is LEAST likely to be a


defense mechanism used by a pathogen to deter the
antigen processing system?

A. Suppression of MHC molecules early in an


infection
B. Production of molecules that bind to MHC
classI molecules in the ER and prevent cell
surface expression
C. Production of a transcription factor that
increases the transcription rate of the MHC
gene early in an infection
D. Suppression of the DM molecule

Copyright @ by The Berkeley Review 294 The Berkeley Review


Specializing in MCAT Preparation
Biology IgA Antibody Experiment namgn m

Passage IV (Questions 22-27) The graphs in Figure 2 represent the numb,er o: trg,\
secreting cells in mouse lung tissue I and I rl eek"
In the following experiment, researchers studied the following exposure to a virus carrier alone or a r-irus
role of the cytokine, interleukin-6, as a factor in the canier plus the IL-6 gene.
response of immunoglobulin A (IgA) to foreign molecules
in mice. IgA is the antibody group that is released from
epithelial surfaces in secretions such as saliva or breast
milk. IgA often represents a first line of defense against 22. Which of the following tissues are lined by epithelial
invading pathogens. tissue?

Experiment I L Mouth
The IgA responses of mice unable to make IL-6 (IL-6) II. Small intestine
and control mice (IL-6+) were studied. Ovalbumin, an egg IU. Urethra
protein, was added to the mouse intestinal mucosa.
Immunoglobulin response is depicted in Figure 1.
A. I only
B. I and II only
q) 50 C. II and III only
o
40
D. I, II, and III
C)
O
30
a_) 20
10
(-)
0
23. Which of the following statements is TRUE of
IgA IgM IgG IgE Figure 1?

Lr
Figure 1 A. IL-6- mice produce higher ievels of IgA than
IL-6+ mice.
I
Experiment 2 IL-6- mice produce lower levels of IgA than
IL-6- and IL-6+ mice were immunized with a virus IL-6+ mice.
construct carrying the mouse IL-6 gene. The virus was C. The presence of IL-6 did not affect the
theorized to insert the IL-6 gene into the DNA of the host production of IgA.
cells it infected. D. IL-6- mice respond strongly to ovalbumin.

3 100
Q9 IL.6+
bo ri
-x
Q( carrier
With virus With virus carrier IgA is composed of what type of molecules?
0!
LA \C)
alone plus IL-6 gene
;1 O A. Amino acids
B. Fatty acids
9n C. Sphingomyelins
t2l2 D. Phospholipids
Weeks following innoculation of mice

With virus carier


3o- 1oo plus IL-6 gene
Which statement is TRUE of Figure 2?
Ug'
:9q
A. The virus carrier alone transformed both
OO
(.) e strains of mice.
v)s
;aO
.tfF! B. Restoring IL-6 in the IL-6- mice improved IgA
Qh production.

€o C. IL-6- mice were hypersensitive to the virus


1212 D.
carrier.
IL-6+ mice did not respond to the virus carier.
Weeks following innoculation of mice

Figure 2
ti

Copyright O by The Berkeley Review 299 The Berkeley Review


iti Specializing in MCAT Preparation

&
Biology IgA Antibody Experiment Passage IV

26. The researchers did not report IL-6 concentrations in


the blood. Why is this the case?

A. Cytokines are hormones and act while passing


through the entire circulatory system.
B. Hormone concentrations cannot be measured
in blood.
C. IL-6 never leaves the cell that produces it.
D. Cytokines are local hormones and often act
without passing through the entire circulatory
system.

27. What term refers to the medical alteration of genes


to correct an inherited or acquired disease?

A. Gene therapy
B. Vaccination
C. Geneticimmunization
D. Pleiotropy

Copyright @ by The Berkeley Review 500 The Berkeley Kevier


Specializing in MCAT Preparatiot
Biology Complement System Passage V

Passage V (Questions 28-33) 29. The proteolytic cascade described in the passage:

Complement was given its name because it A. focuses the complement system away from cel1
complements the action of antibodies and is the principal membranes.
means by which antibodies defend vertebrates against B. does not take place in the alternate pathway.
most bacterial infections. A system of serum proteins are C. provides a means of amplification, ultimatelv
activated to form a membrane attack complex (MAC) leading to many MACs.
which forms holes in microorganisms. Complement also D. uses serine proteases at all serine residues in a
amplifies the defense system by dilating the blood vessels protein.
and attracting phagocytic cel1s to the site of infection.
Individuals who are complement deficient also suffer
from immune complex diseases, in which antibody- 30. The production of antibodies used in the
antigen complexes precipitate in small blood vessels in complement process would be greatly affected b;' a
disease of the:
skin, joints, and brain causing the destruction of tissue.

Complement consists of about 20 interacting proteins.


A. lymph nodes.
The components involved in reactions are known as C1- B. bone marrow.
C9, factor B, and factor D. The rest of the proteins are C. thymus.
involved in the regulation of this system. These proteins D. spleen.
circulate in the blood in an inactive form unless aitivated
directly by an invading microorganism or indirectly by an 31. The classical pathway is usually activated by IgG or
immune response. The final result of activation is igM antibodies bound to antigens on the surface of a
assemblage of the late complement components (C5-C9) microorganism. The Cl complex mosr likely binds
into a membrane attack complex. to the:

The early complement components are activated by A. antigenic determinant.


either antibodies bound to a microorganism or by B. variable region of the antibody.
polysaccharides on a microbial envelope. There are two C. cell membrane.
distinct pathways of eariy component activation. Cl, C2, D. constant region of the antibody.
and C4 belong to the classical pathway and is triggered by
antibody binding. Factor B and D belong to rhe alternative 32. As stated in the passage, protease cleavage acts to
pathway and are triggered by microbial polysaccharides. expose a membrane binding site on the larger
Both pathways wiil act on C3, a central component in the fragment. The most likely reason for this is to:
complement system.

4. avoid precipitation of complement proteins.


All early components and C3 are proenzymes that are B. have the larger fragment act as a diffusible
activated by each other through proteolytic cleavage. As signal in the bloodstream.
each proenzyme is cleaved, it is activated to generate a C. confine complement activation to the cell
serine protease which cleaves the next proenzyme in the surface where it began.
sequence. Each activated enzyme cleaves many molecules D. inhibit the next reaction in the cascade
of the next proenzyme in the chain. The cleavage sequence.
normally exposes a membrane binding site on the larger
fi'agment and liberates a small peptide fragment into the
blood stream. The C3 molecule is eventualiy cleaved, 33. During the proteolytic complement cascade, ser,eral
with its larger fragment binding both the cell membrane small biologically active fragments are generateci.
and C5. Activation of C5 will initiate the spontaneous One of the end results of these molecules' activir;, is
assembly of C5 through C9, forming the membrane attack an increase the permeability of local blood vessels.
complex. Which is the most likely explanation for such an
increase?

28. According to the passage, complement may A. These molecules stimulate the secretron of
normally help: histamine from T lymphocytes.
B. These molecules stimulate the secretion of
A. produceantigen-antibodycomplexes. histamine from basophils.
B. destroyantigen-antibodycompiexes. C. These molecules stimulate the secretion of
C. solubilizeantigen-antibodycomplexes. histamine from macrophage.
D. precipitateantigen-antibodycomplexes. D. These molecules stimulate the secretion of
histamine from erythrocytes.

Copyright @ by The Berkeley Review 501 The Berkeley Review


Specializing in MCAT Preparation
Biology Myasthenia Gravis/Autoimmune Diseases Passage VI

Passage VI (Questions 34-40) 35. Shown below is a diagram of a neuromuscular


junction. Which number indicates an acetylcholine
Myasthenia gravis is a rare, chronic, neuromuscular receptor?
disease. It is characterized by skeletal muscle weakness
and fatigability in response to repeated contraction.
Resting partially restores muscle stre;gth. The muscies of
the eyes, face, jaw, and neck are usually affected first. As
the disease progresses, weakness spreads to rhe
extremities and the diaphragm.
fn severe cases, all the
muscies are weakened.

Research indicates myasthenia gravis is an


autoimmune disorder in which antibodies are produced to
the acetylcholine receptor that is present at the
neuromuscular junction. Antibodies to the acetylcholine
receptor have been found in 85Vo of patients with
generalized myasthenia. Although the mlchanism for a
antibody production is unclear, one hypothesis is that a I ol
certain thymus cells that resemble musci" (myoid cells)
are.damaged by a virus. The virus may have a molecular
region that mimics part of the acetylcholine receptor, such
as-the herpes simplex virus. A virus may damage myoid
cells so that antibodies are produced against tt"- air"ttty.
By whatever mechanism, the viraf infection induces
antibody production.

The actual interaction between the antibody and the


receptor is not fully understood. The antibody may block A. I
the receptor, it may cause faster receptor breakdown, or it B.II
may promote complement-mediated damage. c.m
D.IV
Autoimmune diseases as a whole are relativelv
common. In the following table is a list of soml
autoimmune diseases and the antibodies produced in the 36. Which of the following is NOT an example of an
disease state: autoimmune disease?

A. Type II diabetes.
B. Addison's disease.
Disease Antibody Against Effects
C. Myasthenia gravis.
Type I D. Graves' disease.
B celis of pancreas Destroys B cells
diabetes

Graves' Thyroid stimuiating Stimulares TSH


disease hormone receptor
37. Which of the following is NOT a symprom of
receptor on thyroid
Graves' disease?
Multiple Myelin basic protein Disrupts
sclerosis (hypothesized) myelination A. Increased metabolic rate.
Glomerulo- Basement membrane B. Weight loss.
Destroys a variable
nephritis of glomular capillaries number of glomeruii C. Lethargy.
D. Hyperactivity.

34, Which drug could be given to counteract the effects


38. Secretion of which of the following from
of the antibody produced in myasthenia gravis? the
pancreas is halted by antibodies to the B cells?

A. An inhibitor of acetylcholinesterase. A. Glucagon.


B. An immunostimulant.
B. Insulin.
C. A paralytic agent, like curare.
C. Bicarbonate.
D. An inhibitor of acetylcholine synthase. D. Digestive enzymes.

Copyright @ by The Berkeley Review 302 The Berkeley Review


Specializing in MCAT preparation
Biology Myasthenia Gravis/Autoimmune Diseases Passage VI

How can a viral infection lead to an autoimmune


disease?

The virus resembles a "self'molecule, leading


to antibodies that cross-react with other body
molecules
II. The virus damages a cell so that unrecognized
cell proteins are released, causing antibody
production
III. The virus resembles a "non-selfl' molecule,
leading to antibodies that cross-react with
other body molecules

A. I only
B. I and II only
C. II only
D. II and III only

40. Often patients with autoimmune diseases are treated


with corticosteroids to reduce immune responses.
Which organ in the body produces corticosteroids?

A. Pancreas.
B. Adrenal gland.
C. Pituitary gland.
D. Thyroid gland.

Copyright @ by The Berkeley Review 303 The Berkeley Keview


Specializing in MCAT Preparation
Biology Glucose, Glucagon, & Insulin Passage Vtr

Passage VfI (Questions 4l-46) 41. During times of stress the importance of having
adequate levels of glucose, for utilization as energy,
The regulation and metabolism of carbohvdrates in the is dependent on hormonal secretions from the
body are controlled by the pancreas. The endocrine pancreas. Which choice below will BEST readv the
component of the pancreas, the islets of Langerhans, are body for stressful situations?
specifically responsible for carbohydrate control. These
small clusters of cells imbedded within the exocrine A. Increased levels of glucagon and insulin.
portion of the pancreas contain peptides with specific B. Increased levels of glucagon and decreased
hormonal activity.
levels of insulin.
C. Decreased levels of glucagon and decreased
Glucagon, secreted by the a cells (or A cells), liberates levels of insulin.
glucose from storage areas in the body, stimulates glucose
D. Decreased leveis of glucagon and increased
production, increases lipid concentration in the blood
Ievels of insulin.
stream by releasing free fatty acids, and increases the
production of ketones. u cells are stimulated to secrete
glucagon during increases in plasma amino acid levels,
cortisol secretion, exercise, and sympathetic nervous
system stimulation. Inhibition of glucagon is promoted by
increases in plasma glucose, ketone, free fatty acids, 42. Ketosis is developed from an increase in the
insulin, and somatostatin levels. conversion of free fatty acids to ketone bodies.
These ketone bodies are an important source of
Insulin is secreted by the B cells (or B cells) and energy in times of fasting. However, prolonged
functionally is important in increasing the storage of ketosis will lead to a plasma acidosis due to liberated
glucose, fatty acids, and amino acids in the cells of target
hydrogen ions from ketone bodies. A patient witb
tissues. Furthermore, insulin decreases the release of
glucose, mannose, amino acid, and glucagon plasma
acidosis will develop shortness of breath.
dehydration, hypervolemia, and hypotension. In
levels. Many intestinal hormones also stimulate insulin severe cases the acidosis and dehydration will
secretion. Parasympathetic stimulation of the cells will
B depress consciousness to the point of coma. Which
increase insulin secretion, while sympathetic stimulation
choice below will lead to the development of ketosis
will inhibit secretion. and acidosis?

Somatostatin is a peptide secreted by 5 cells (or D A. Increased levels ofglucagon and insulin.
cells) in the islets. Somatostatin inhibits the production of B. Increased levels of glucagon and decreased
both insulin and glucagon, and it acts as a regulator of levels of insulin.
islet secretion. C. Decreased levels of glucagon.
D. None of the above.
The effects ofinsulin and glucagon target very specific
regions of the body where their cellular actions occur.
Insulin's effects are generally associated with muscle and
adipose tissue, leukocytes, fibroblasts, and mammary
glands. Insulin does not directly affect brain and kidney
tissue, intestinal mucosa, and red blood cells. Glucagon's 43. Symptoms reported by a patient include weakness.
effects are targeted mainly on the liver. dizziness, confusion, and hunger. Furthermore, somr
tremors, palpitations, and nervousness are alsp
reported. These last symptoms are characteristic d
hypoglycemia. The development of these sympron$
is due to:

A. abnormal increases in insulin secretion.


B. abnormal decreases in insulin secretion.
C. abnormally high levels of plasma glucose.
D. abnormally low levels of free fatty acids.

Copyright @ by The Berkeley Review 3A4 The Berkeley


Specializing in MCAT
Biology Qlucose, Glucagon, & Insulin Passage YI

44. What pancreatic hormonal response is expected after


a heavy protein intake?

A. Increased levels of insulin and glucagon.


B. Increased levels ofinsulin and decreased levels
of glucagon.
c. Decreased levels ofinsulin and glucagon.
D. Decreased levels of insulin and increased
levels ofglucagon.

45. From the data reported on glucose utilization of


tissue and responsiveness of tissue to insulin, it may
be determined that the main function of insulin
secretion is:

A. increasing glucose uptake in the brain.


B. increasing glucose release from the liver.
C. access and storage of glucose in cells of the
peripheral tissues.
D. increasing glucose loss in the kidney.

46. Patients diagnosed as having diabetes mellitus are


said to be in "a state of starvation in the midst of
plenty." This analogy refers to:

A. extracellular glucose excess.


B. extracellularglucosedeficiency.
C. decreased effects of insulin on intestinal
mucosa uptake of glucose.
D. deficiency of fatty acids in neural tissue.

Copyright @ by The Berkeley Review 305 The Berkeley Keview


Specializing in MCAT Preparation
Biology Calcium, PTII, Calcitonin, Calcitriol Passage Vtr

Passage VIII (Questions 47-54) Calcitonin acts to inhibit osteoclast activity and reduce
bone resorption. This hormone also inhibits talcium and
, Calcium and phosphate both play an important role in phosphate reabsorption in the kidney and increases the
the mineralization of bone in vertebratei. Calcium is excretion ofthese ions in the urine.
obtained from the diet and is largely absorbed at the level
of the intestine. The intracellular concentration of calcium
is about 10-7 mol/I- while the extraceliular concentration
of calcium is about l0-3 mol/L. Blood calcium levels are 47. Administration of PTH leads to changes in plasma
primarily determined by bone metabolism and urinary calcium and phosphate concentrations. Whicli of the
excretion. following graphs BEST represents these changes?

Calcium metabolism is regulated through the actions A.


of parathyroid hormone (PTH), calcitonin, and vitamin D3 J
(choiecalciferol). PTH is synthesized and secreted by the o o
parathyroid glands, usually located in the central region of "o
p
the thyroid gland near the trachea. Calcitonin is o
synthesized and secreted from parafollicular (or C) cells
located in the thyroid gland. Cholcalciferol is synthesized O I
in an inactive form in the skin of animals from a Q

photolytic reaction involving UV light and a sterol -


derivative. In order for this inactive hormone to be 0 O Time (hours)
activated, it must first be hydroxylated in the liver and PTH added
then hydroxylated in the kidney. The activity of the B.
hydroxylase enzyme in the kidney is regulated and J
increased by PTH. o
tr
p
Bone is composed of an organic matrix consisting of o
collagen fibers and a ground substance composed of
extracellular fluid and proteoglycans. The collagin fibers o
6 B
help to give bone its great tensile strength, white tne U
ground substanca helps to control the deposition of t-
calcium salts, like hydroxyapatite (Ca5@O4):OH). The 0 0 Time (hours)
calcium salts help to provide for the great compressional PTH added
strength found in bone. C.
..1
The collagen matrix and ground substance is laid down o o
by bone cells called osteoblasts. The tissue which is E
Calcium fc
p
formed, called an osteoid, can enlrap some of the o
osteoblasts. Entrapped osteoblasts are called osteocytes. E
Phosphate
As the bone grows, calcium salts precipitate on the
collagen fibers. Bone is also undergoing resorption by (-)

cells called osteoclasts. -


0 fr Time (hours)
PTH causes calcium absorption from bone by PTH added
stimulating osteociastic activity and transiently inhibiting D.
osteoblastic activity. At the level of the kidney, pTH J
increases calcium absorption in the distal tubules and o
collecting ducts and greatly decreases the reabsorption of '6
p
phosphate at the proximal tubules. 6'

The activated form of cholecalciferol (1,25-(OH)Z-D/ o


has target receptors in the intestine, bone, and kidney (to ()
name but a few tissues). In the intestine this hormone -
promotes absorption of calcium and phosphate (following 0 G Time (hours)
as the counterion), while in bone it promotes resorption of PTH added
both calcium and phosphate. In the kidneys this hormone
promotes the reabsorption of both calcium and phosphate
so that little is excreted in the urine.

Copyright @ by The Berkeley Review 506 The Berkeley Revieu


Specializing in MCAT Preparation
Biology Calcium, PTII, Calcitonin, Calcitriol Passage Vtr

48. Which of the following structures BEST represents 49. Calcium and phosphate absorption in the intestines
vitamin D3? is stimulated by an increase in:

A. I. 1,25(OH)2D3
II. Calcitonin
III. PTH

A. I only
B. I and II only
C. III only
D. I and III only

50. In order for the secretion of calcitonin to have a


B. greater effect on the concentration ofcalcium ions in
the plasma, which of the following statements must
be true regarding osteoclast activity and plasma
calcium levels?

A. Increased osteoclast activity coupled with a


hypercalcemic plasma.
B. Decreased osteoclast activity coupled with a
hypercalcemic plasma.
C. Increased osteoclast activity coupled with a
hypocalcemic plasma.
D. Decreased osteoclast activity coupled with a
hypocalcemic plasma.

C.
51. Hypophosphatemic rickets is an Xlinked dominant
trait that leads to decreased levels of phosphate
reabsorption in the kidneys. Which pedigiee sho*n
below BEST represents this disease?

A. B.

':l1IT
rr+\-u+-l
fl-rl [i J-I-J-I
,,, u,
D. C. D.

Jopyright @ by The Berkeley Review 307 The Berkeley Review


Specializing in MCAT Preparation
Biology Calcium, PTII, Calcitonin, Calcitriol Passage Vtr

52. Familial hypophospharemia is BEST treated by diet


modification and supplying adequate amounts of:

A. calcium.
B. phosphate.
C. calcium and phosphate.
D. phosphate anA t,ZS1Ofq2O3.

53. Hypoparathyroidism is BEST characrerized by:

I. increased osteoblast activity.


Ir. increased osteoclast activity.
III. increased neural excitability.
IV. increased plasma calcium concentrations.

A. I only
B. II and III only
C. Itr only
D. I and fV only

54- In the graph shown below, all of the following


statements concerning the relationship between
PTH, calcitonin, and calcium are true EXCEpT:

o
o
c€

o
o o
o
o o
o
a :-

5
Total Ca2+ concentration in piasma

A. PTH is a hypercalcemic hormone.


B. calcitonin is a hypocalcemic hormone.
C. a positive linear relationship exists between
calcitonin secretion and the concentration of
plasma calcium.
D. a positive linear relationship exists between
PTH secretion and the concentration of plasma
calcium.

Copyright @ by The Berkeley Review The Berkeley Kevier


Specializing in MCAT
\
Biology Erythroblastosis Fetalis Passage IX

Passage IX (Questions 55-61) 56, Which of the following statements is TRUE?

Erythroblastosis fetalis (EF) is also known as A. The placenta allows passage of all blood
hemolytic disease of the newborn. Hemolysis is rupture of products from the mother to the fetus.
red blood ceils, so in hemolytic disease, anemia is B. The Rh factor is nor a component of the ABO
.ommon! due to the lysis of red blood cells. In the EF blood group system.
--ondition, maternal antigens cross the placental barrier, C. The mother makes red blood cells for the fetus
auack proteins on the surface of the red blood cells of the in the placenta.
r-etus, and lyse the cells. A very specific set of conditions D. Terminated pregnancies have no effect on
must exist for this disease to occur. future development of EF.

The mother must be Rhesus (Rh) factor negative, the


:etus musf be Rh factor positive, and the mother's immune
s\ stem must be sensitized to the Rh positive antigen
tlrough previous full-term pregnancy or abortion. The Rh
:actor antigen is transmitted as a dominant trait, so that 57. What preventive measure couid protect subsequent
rnly people who are homozygous recessive are Rh factor fetuses il an Rh-negative mother gave birth to an
regative. Rh-positive fetus?

Roughly 90Vo of the cases of EF result from A. Give the mother a blood transfusion with Rh-
':nsitivities to the D antigen on the Rh factor. When Rh positive biood.
:ositive blood enters the circulation of an Rh negative B. Give the fetus a blood transfusion with Rh_
lother, antibody formation against D may be induced. negative blood.
This exposure may be during an accidental infusion, C. Treat mother with a set of antibodies directed
iuring pregnancy, delivery, or during a miscarriage or against the anti-Rh antibodies.
"nortion. During a first pregnancy, there is usually little D. Treat fetus with antibodies against the anti-Rh
:rchange of fetal and maternal blood, except near the end antibodies.
- i the pregnancy or during delivery.

This time frame does not allow for antibody formation


. by an attack on the fetal biood cells. The
--'ilowed
:ioblem lies in subsequent pregnancies. Small amounts of
rtigen, even the amount in I mL of fetal blood entering 58. An infant wirh severe EF has jaundice, a yellow
.:e mother's circulation, promote rapid increases in hei coloring due to excess bilirubin, a breakdown
.r:i-D antibody titer. IgG is produced, and it can easily product of heme. In what tissue or organ is heme
:.oss the placental barrier into the fetal blood supply. degraded into bilirubin?
;','en in the ideal conditions for EF, sometimes
the diieaie
: res not manifest, due to variable physiological A. Spleen
::nditions. B. Bone marrow
C. Liver
D. All of the above

59. An Rh-positive mother is pregnant with an Rh-


negative fetus. Will the fetus develop EF?
55, A Rh-negative woman and a heterozygous Rh-
positive man have one child together. The woman A. No, there are no maternal antigens to the Rh
has never been pregnant before. What is the factor antigens.
likelihood that the child wiil be born with EF? B. Yes, the mother can still make antigens to the
Rh factor of the fetus.
A. l)AVo C. No, the fetal antibodies protect its red blood
B. l5Vo cells.
C. 50Vo D. No, but fetal antibodies attack the maternal red
D. jVo
blood cells.

- rpyright @ by The Berkeley Review 309 The Berkeley Review


Specializing in MCAT preparation
Biology Erythroblastosis Fetalis Passage IX

60. What variables could affect the severity of


hemolysis in an Rh-positive fetus whose mother is
Rh-negative?

I. Amount of blood transferred.


II. Sensitivity of mother to D antigen.
Iil. Number of pregnancies.

A. I only
B. I and II only
C. II and III only
D. I,II, and IIi

61. Which of the following clinical signs could be


consistent with a diagnosis of EF in a newborn?

I. High levels of bilirubin in the blood.


il. Low levels of hemoglobin in the blood.
ilI. Increased levels of erythrocytes.

A. I only
B. I and II only
C. II and III only
D. I, II, and III

Copyright @ by The Berkeley Review 5ro The Berkeley Review


Specializing in MCAT Preparation
Biology Septic Shock Passage X

Passage X (Questions 62-67) 64. A defect in which of the following cells would
inhibit the production of soluble anribodies?
Septic shock, a disease characterized by hemodynamic
derangements and multi-organ malfunction, has generally A. Mast cells
been associated with a gram-negative infection. However, B. Cytotoxic T cells
it is becomming increasingly clear that gram positive C. Plasma cells
organisms are equally responsible for sepsis. Many of D. Erythrocytes
these gram positive ogranisms release molecules known
as superantigens. These superantigens can induce T cell
proliferation without regard to the antigenic specificity of
ihe cell.
65. In an experiment, a sepsis patient is treated with
Acommon sign of septic shock is widespread anti-factor XII antibodies. After treatment. one
rctivation of coagulation leading to widespread would expect to see a:
rntravascular clotting. Microbial products activate Factor
-\I[, a molecule involved in b]ood clotting. Activation of A. total lack of intravascular clotting.
:ris factor initiates the intrinsic coagulation pathway and B. rise in the level of intravascular clotting.
.:so the bradykinin pathway. Bradykinin is a potent C. rise in the patient's blood pressure.
', asodilator and also increases the permeability of vascular D. decrease in the patient's blood pressure.
:ndothelial cells.

Cytokines, such as interleukin-1 and tumor necrosis


:.;tor, activate tissue factor III. This factor is found on the
- iler membrane of macrophage and endothelial cells, and 66. Bradykinin acts to increase the radius of a given
, ::mulates the extrinsic coagulatory pathway. blood vessel by a factor of 2. The flow of blood
through the vessel should increase by a factor of:

A. 2.
B. 4.
c. 8.
D. 16.
r,l- A Gram-positive cell differs from a Gram-negative
cell in that a Gram-positive cell:

-{. does not have an outer membrane on its cell


wall. 67. The activation of bradykinin most likely results in:
B. does have an outer membrane on its cell wall.
C. contains a thin peptidoglycan layer adjacent to A. hypotension in the patient.
the plasma membrane. B. cytokine release.
D. contains no peptidoglycan layer adjacent to the C. macrophageactivation.
plasma membrane. D. stimulation of the extrinsic coagulation
pathway.

\lacrophages destroy microorganisms through:

A. exocytosis, secreting toxins which eventually


form membrane attack complexes.
B. xxocytosis, surrounding the foreign particle
with a lipid bilayer which protects the host
organism.
C. endocytosis, engulfing foreign particles which
eventually will fuse with a lysosome.
D. endocytosis, engulfing foreign particles which
eventually will fuse with a peroxisome.

r' :.qht @ by The Berkeley Review 5l I The Berkeley Review


Specializing in MCAT Preparation
Biology Calcitonin and Osteoporosis Passage XI

Passage XI (Questions 68-75) 71. Which two amino acids form a disulfide bridse?

Calcitonin (CT) is a polypeptide hormone secreted by A. Methionine-threonine


the parafollicular cells of the thyroid gland in mammals. B. Methionine-methionine
32 amino acids make up the hormone, and a disulfide C. Cysteine-cysteine
bridge links residues 1 and 7. The entire CT molecule and D. Cystine-cysrine
the. disulfide bridge are required for full biological
activity.

CT works as an antagonist of parathyroid hormone 72. Which of the following choices is a probable
(PTH) In response to small increases in plasma calcium, explanation why salmon CT is 30 times more active
CT is released and acts on the kidnw and bone to in humans than human CT?
decrease the calcium level. In the bone *airix, osteoblasts
synthesize bone, and osteoclasts catabolize bone. The I. Salmon CT is more resistant to degradation by
main eff-ects of CT are ( i ) inhibition of osteoclasts and (2) human enzymes.
a transient increase in urinary calcium and phosphate. II. Salmon CT attaches more strongly to the DNA
of the osteoclasts.
Calcitonin is used pharmacologicaily as a treatment IfI. Salmon CT attaches more tightly to the CT
for osteoporosis. Saimon CT is commonly used. Although receptor.
salmon CT differs markedly from human CT, it is about
30 times more potent when used in non-ailergic humans A. I only
to treat osteoporosis. Treatment with CT is not without its B. I and III only
own side effects. CT treatment for osteoporosis increases C. II and III only
plasma PTH and requires simultaneous calcium D. I, II, and III
supplementation to avoid hyperparathyroidism.

58. Osteoporosis is a disorder ofbone characterized by a


73. If a person begins a calcium supplement regimen
decrease in bone quantity, most common in women
following menopause and in elderly men and and doubles calcium intake, whit would be the
response in CT secretion?
women. Which of the following conditions would
lead to the GREATEST decrease in bone quantity?
A. CT secretion increases.
A. Decreased osteoblast activity, decreased
B. CT secretion decreases.
osteoclast activity.
C. CT secretion remains unchansed.
B. Increased osteoblast activity, decreased
D. CT secretion halts completelyl
osteoclast activity.
C. Decreased osteoblast activity, increased
osteoclast activity. 74. How is salmon CT, a polypepride, administered?
D. Increased osteoblast activity, increased
osteoclast activity. A. Ingestion to avoid allergic reaction.
B. Injection to avoid hydrolysis.
C. Ingestion to avoid hydroiysis.
69. What is the most abundant mineral in the human D. Injection to avoid allergic reaction.
body?

A. Bn
B. Water 75. What are the symptoms of an allergic reaction it-
C. Calcium foreign protein in the bioodstream? "
D. Zinc
I. Flushing or reddening ofthe skin.
II. Skin welrs (hives).
70. Based on the passage, what is the role of pTH? IU. Difficultybreathing.

A. PTH increases plasma calcium. A. I only


B. PTH decreases plasma calcium. B. I and II only
C. PTH increases urinary calcium. C. III only
D. PTH increases urinary phosphate. D. I, II, and III

Copyright @ by The Berkeley Review 312 The Berkeley Rerieu


Specializing in MCAT preparatim
Biology Vertebrate Immune System Passage XII

Passage XII (Questions 76-83) 77. When certain types of antibodies bind to their tarset
cells, they attract a series of proteins coiiectivJly
In order to protect vertebrates from infection, the known as complement. These pioteins can then form
im,mune system has evolved both an antibody-based and a pores which allow small molecules to freely diffuse
cell-mediated response to foreign antigens. B cells, which across the plasma membrane. What effect would this
originate and develop in hemopoietic tissues (bone have on a target cell?
marrow and fetal liver), are responsible for producing and
secreting antibodies which bind to antigen particl6s. T
celis also originate in hemopoietic tissues but later
A. The ceil would die due to its inability to
initiate action potentials.
migrate to and mature in the thymus during early B. The celi would become hyperpolarized.
development. Cytotoxic T celis are mainly .esponsible for
mounting a cell-mediated defense by directlyiausing the
C. The cell would lyse due- to an upset water
balance.
death of infected celis. While B cells can be activat; by
the binding of extracellular antigen to special receptors on
D. The cell would shrink due to an upset water
balance.
the plasma membrane, T cells must come in direcicontact
r.r,ith infected ceils in order to become activated.

How the immune system differentiates between self


and foreign antigens has been the topic of much study.
Immunologists in the first half of thii century p.opor"d
two main theories:
78. Which of the foliowing statements would
Theory I consistent with BOTH Theory 1 and Theory 2?
be

Vertebrates inherit genes that encode receptors


ipresent on the surface of B and T cells) that are capable L A foreign cell line is injected into a mouse
of binding oniy foreign antigens. The immune system embryo. Further injections of the cell line into
doesn't react against host tissues because it geneiically the adult mouse do not elicit an immune
lacks the receptors which bind selfantigens. response.
il. Transplantation of organs between
Theory 2 monozygotic twins does not result in orsan
rejection by the immune system.
The.. immune system is inherently
capable of ilI. Cells transplanted between mice which are
responding to both self and foreign antigens, but it genetically identical (i.e., from the same inbred
becomes "tolerant" to self antigens during early strain) are tolerated by the new host's immune
development. Since foreign antigens aren't preseni during
system.
embryonic stages, the immune system does not deveiop i
tolerance to them.
A. I oniy
B. II only
C. II and III only
D. I, II, and III

76. According to the passage, removal of the thymus


from an adult human would most likely result in:

A. drastically decreased antibody-mediated


lmmune fesponse. 79. Tolerance to self antigens breaks down in the human
B. drastically decreased cell,mediated immune autoimmune disease myasthenia gravis, resulting in
response. the production of 4ntibodies against the patient's
C. drastically decreased antibody and cell_ skeletal muscle acetylcholine receptors. Wt ictr of
mediated immune response. the foliowing is a likely symprom of this disease?
D. littie or no change in the effectiveness of either
type of immune response. A. Irregularities in heart contraction.
B. Weakness and difficulty breathing.
C. Paralysis of the gastrointestinal tract.
D. Dementia.

Copyright @ by The Berkeley Revierv .'t^t The Berkeley Review


Specializing in MCAT preparation
Biology Vertebrate Immune System Passage XII

80. Recently activated B and T cells are examined via 83. Virally infected cells are usually killed by cytotoxic
electron microscopy. Which of the following would T cells. The T cells can most likely target:
be the most likely observations?
A. most types of cells in the body.
A. The B cells have a greater number of B. only cells lining the blood vessels.
mitochondria than the T cells. C. epithelial cells only.
B. The B cells have considerably more rough D. other blood cells onlv.
endoplasmic reticulum than the T cells.
C. Both the T and B cells lack nuclei.
D. No differences between the two types of cells
are revealed at the level of electron
microscopy.

81.. A sample of B cells is removed from an adult


mouse. A highly radioactive antigen X is added to
the B cells, killing the few that bind strongly
(<0.0lVo). The remaining B cells are injected into
mice whose own B cells were destroyed by
irradiation. These mice can now make no antibodies
to antigen X but do respond to other antigens. Which
of the following can most likely be concluded?

A. Each B cell is predetermined to bind a specific


antigen.
B. B cells can recognize new antigens and "learn"
to bind to them.
C. T cells must interfere with B cell binding.
D. Antigen X is normally non-immunogenic.

82. Secreted antibodies often act collectively to bind to


large target antigens and cross-link them into
insoluble masses which are easily phagocytosed.
Which of the following would NOT be consistent
with this phenomenon?

A. Antibodies each have two identical binding


sites.
B. Antigens can be bound by only one antibody at
a time.
C. Antibody binding sites are connected by a
flexible "hinge" region.
D. Large antigens contain multiple antibody-
binding sites.

Copyright @ by The Berkeley Review 314 The Berkeley Revier


Specializing in MCAT Preparatic
Biology Insulin Receptor Passage XItr

Passage XIII (Questions 84-90) 86. The response to glucagon is believed to involve the
production of cAMp as a secondary messenger.
The human insuiin receptor is an integral Which of rhe following graphs BEST describes"the
transmembrane protein located in the plasma membrane
sequence of events associated with the binding
of many celis throughour the body. Studies have shown of
glucagon to its receptor? fNote: CA cAMp; p:p
that a functional receptor consists of two alpha and two = =
protein phosphorylation; CR cellular response.l
beta subunits which are connected using disulfide =
bonding. The receptor consists of several domains, A.
inciuding one which binds insulin, a transmembrane C)

binding region, and an intracellular tyrosine kinase cg


()
phosphokinase (TPK) segment. b3
>t
How does ligand binding create intracellular changes <€o
.
which bring about the effects of the insulin hormone?
There is experimental evidence which suggests that the o
human
,insulin receptor is a insulin activited tyrosine
kinase. While many details remain unciear, the binding of
the hormone leads to both an autophosphorylation of two
tyrosine residues on the TpK and u ptorptorylation of B.
a
cytoplasmic constituenrs. In addition, the binding of d
O
insulin to irs receptor is believed to activate phosphohf,ase !i

C, which converts glycan phosphatidyl inositol pfrospiate .: I


to glycan inositol phosphate (GIp) and 2_diacylgiyceiol. a !r/
<fto
O
GIP is known to mimic certain insuiin responses and O

2-diacylglycerol, in the presence of elevated calcium


ievels, is known to activate protein kinase C. protein Time .-
kinase C is known to influence the activity of other
enzymes and metabolic pathways. C.
a)

.J
>.9
>t
84. Which of the following procedures is used on cells
to isolate the entire insulin receptor for experimental <reO
studies?

A. Acidic wash. Time -


B. Oxidizing wash.
C. Detergent wash. D.
D. Reducing wash. C)
dJ

i'H
>t
:o
85. Through genetic manipulations, the insulin binding <3 o
region of the insulin receptor (IR) was fused with the o
kinase region of the epidermal growth factor (EGF) A
receptor (also a ligand activated kinase). The
resulting hybrid receptor was transfected into a cell
and the addition of insulin resulted in a functional
EGF kinase. From this experiment, it can be
concluded that IR and EGF have similar: 87. In a type I, insulin dependent diabetes, the individual
alflicted wirh the disease most likely has:
A. ligand binding regions.
B. signaltransductionmechanisms. A. a mutated insulin binding site.
C. secondary messenger cascades. B. a mutated membrane anchoring site.
D. effects on carbohydrate, fat, and protein C. a normal insulin receptor.
metabolism. D. an inactive phospholipase C.

Copyright O by The Berkeley Review 315 The Berkeley Review


Specializing in MCAT preparation
Biology Insulin Receptor Passage XItr

88. Which of the following is consistent with the actions


of GIP?

A. Inhibition of hexokinase.
B. Inhibition of phosphofructokinase.
C. Stimulation of glycogen breakdown.
D. Stimulation of pyruvate dehydrogenase.

89. In cell #1, the addition of bivalent anti-insulin


receptor antibodies induces a response without the
addition of insulin. In ce17 #2, addition of a fragment
anti-receptor antibody (monovalent) induces no
response without insulin. To cell #2, antibodies
against the monovalent anti-receptor antibodies are
added. This will result in:

A. no response.
B. decreased glucose uptake .
C. increased CO2 production.
D. increased insulin binding.

90. To disrupt the dimer in order to study its


components, a researcher will most likely use:

A. oxidizingconditions.
B. reducingconditions.
C. high speed centrifugation.
D. high pH.

Copyright @ by The BerkeleY Review 316 The Berkeley


Specializing in MCAT
t
Biology Vitamin D3, PTH, & Calcitonin Passage )ilV

Passage XIV (Questions 9l-96) 93. PTH and CT secreting cells are found in which of the
following glands?
In all vertebrates the maintenance of calcium (Cu2@),
phosphate (PO+3e), and magnesium (Mg2o) homeostasis A. Pituitary gland
is governed primarily by the vitamin D3 derivative 1,25_ B. Adrenal gland
dihydroxycholecalciferol (1,25-(OH)2-D3), calcitonin C. Thyroid gland
(CT), and parathyroid hormone (pTH). D. Thymus gland

Humans obtain vitamin D3 either by ingestion or


through synthetic mechanisms that are initiated by cells in 94. Which of the following categories represent the three
the epidermis of the skin. Vitamin D3 is hydroxylated first hormones described in the passage?
in the liver and then is transported to the kidney where the
enzyme 1-hydroxylase converts it to the active form of the I. Amine
hormone, 1,25-(OH)2-D3. The activity of 1o-hydroxylase u. Polypeptide
durin g hypocalcemia and hypophospharemia. IIr. Steroid
1s -e$119ed
1,25-(OH)2-D3 passes inro a rarger cell ind then into the
nucleus where it complexes with a receptor protein that A. II only
has a DNA binding site. Gene transcripti,on can either be B. I and III only
enhanced or suppressed. Enhancement leads to the C. II and III only
synthesis of calcium binding proreins (calbindins) in D. III only
intestinal villi and crypt cells. Calbindin promotes the
uptake of Ca2@ from the intestinal lumen. A similar
mechanism allows for the absorption of phosphate and
95. Which of the following conditions besr describes
magnesium from the intestinal lumen.
patient with a deficiency in pTH secretion?

PTH is released from the chief cells of the parathyroid I. Hypocalcemia


glands while CT is released from the parafollicular cells il. Hypercalcemia
(or C cells) of the thyroid gland. Both hormones ur. Hyperphosphatemia
have
_senes encoded in the short arm of human chromosome l1
and are synthesized as a preprohormone from different A. I only
primary RNA transcripts. B. III only
C. II and III only
PTH acts on bone and on the distal tubule of the D. I and III only
iiidney to promote Ca2@ reabsorption and inhibits the
reabsorption of PO43e in the proximal tubule of the
96. Which of the following curves BEST represents the
oJdney. PTH also stimulates rhe synthesis of 1,25-(OH)2_
relationship between blood levels of parathyroid
D3 in the kidney. CT acts ro lower both plasma calcium
hormone (PTH) and calcironin (CT) in rerms of total
:nd plasma phosphate levels. plasma Ca2@ concentration levels?

A. B.
91. Ultraviolet light stimulates vitamin D3 production in
which of the following cell types?
c)
ats o
A. Hepatocytes o
o
B. Keratinocytes
C. Leukocytes low high low high
D. Lymphocytes
Total Plasma ICa2+1 Total Plasma [Ca2+1

0t The primary precursor for 1,25-(OH)2-D3 is located C. D.


in which one of the following organs?
o
A. Liver a -
B. Intestine Q U
C. Integument
low high low high
D. Kidney
Totai Plasma [Ca2+1 Total Plasma [Ca2+1

- rpyright @ by The Berkeley Review 317 The Berkeley Review


Specializing in MCAT preparation
Biology Sepsis S;rndrome Passage XV

Passage XV (Questions 97-103) 99. Which cell products are used free radical
reactions to destroy pathogens?
The normal response to an infection is a complex
system of activations and inactivations of the members of A. Chemical attractants.
the immune system. Neutrophils are the first to respond to B. Proteolytic enzymes.
an infection by squeezing through blood vessel walls and C. Cytokines.
destroying pathogens directly by the release of toxic D. Toxic oxygen products.
oxygen products, such as superoxide, hydrogen peroxide,
and nitric oxide, and by the release of proteolytic
enzymes. These white blood cells follow chemical
attractants given off by the pathogen. Monocytes, another 100. What is the chemical formula for nitric oxide?
type of white blood cell, arrive next. They may be
transformed into macrophages in the tissue and engulf A. NO
pathogens and cellular debris. Monocytes and B. HNO3
macrophages release cytokines, a powerful class of C. H2N202
chemicals that modulate responses of different members D. Noz
of the immune system. T cells and B cells are attracted
and activated by cytokines at the infection site. Another
set of cytokines are released to signal "stop" when the
pathogens are neutralized. This is the normal progression 10L. Researchers are attacking sepsis syndrorne from
ofevents when the body attacks an invading bacteria' many perspectives. In one approach, synthetic
receptors for tumor necrosis factor (TNF), a
However, in the U.S. each year, about 175,000 people cytokine, are injected to bind to TNF and halt the
die from sepsis syndrome, in which the sepsis process is cytokine cascade. What assumption does this model
actually amplified rather than terminated. The cytokines make?
for signal termination are not released correctly, leading
to an imbalance with the amplifying cytokines. This A. TNF halts the abnormal cytokine cascade of
attracts all the white blood cell participants to the area, sepsis syndrome.
which release more activating cytokines. Some of the B. TNF receptors enter cells by endocytosis and
substances that are released lead to increased permeability bind intracellular TNF.
of the blood vessels. In normal situations, this allows C. TNF is mainly present in the general
WBCs to more easily enter the infected tissue' In excess, circulation.
this leads to blood celi leakage and damage, and D. TNF moves from cell to only through gap
positively feeds back to attract more WBCs. Ultimately, if junctions.
this process is unchecked, blood vessels will deteriorate'
leading to tissue death, organ death, or even patient death.

102. What is the most abundant type of white blood cell?

A. Neutrophils
B. Erythrocytes
97. Which is the term for the process of white blood C. Leukocytes
cells squeezing through blood vessels? D. Macrophages

A. Mitosis
B. Diapedesis
C. Phagocytosis 103. What is the role of cytokines the inflammatory
D. Endocytosis process?

I. To activate WBCs.
98" What purpose do proteolytic enzymes serve in IL To deactivate WBCs.
celluiar defense? ilI. To kill bacteria directly.

A. Inactivate bacterial lipid signals. A. I only


B. Digest bacterial cell surface carbohydrates. B. I and II only
C. Destroy bacterial DNA. C. i and III only
D. Hydrolyze bacterial Proteins. D. I, II, and III

Copyright @ by The BerkeleY Review 5la The BerketeY Revien'


Specializing in MCAT PreParation
Biotogy Immunology & Endocrinology Section V Answers

immunity is what the individual has deveroped


fi*;::tT",1, in :::i:T1"'"T:::""i:,?:gl 1T11..1":",oecific)
,i;;";"";;;;;.;ffi;;;J'oj'*ili"'ii;'J.l:.T,gffi:li1'?:fi1
i:t "1':'5
l: *:"::::j-':rii:T*lL1ft'"4: i"v.. r1*"";il'"';;;;'. ff:i;'J"J#;
ffi;T;:T,"""X':*f:"""t,*115":::r::*:5:f-11llfl"" are rined ;i,h "? i,ii, .,",li.ili;"iii#J:?,?:::
:31,:l:::":j#i::k::Ti,:l. yyrrrvtr i::1.":!1.:,,ru." T,t:;,";#;;;5;
Dvurere trruLUus allu LIte gprlnella celrs th"-,;;"",';;;;1i;i;:iffii#.i]ji:
:H:,,,"5;,.,?1,,.:i"*'.:Y::,::i1:,::11:'-"1f,.^,j_li_",1i
called cilia that move particuiate matter over 119 lh.""pitheiiai
the epitheliar surface. In tr
the respiratory tract the mucous membianes of
:|;:'::::i:l,|::ll:le lfga
with cliated
"pitrrerium
thai;;;;,;;ffi;ffi;;i;Ji;ilffiiilH,"fi?ffi:'r:l
$:#Jlf#?J#::il?L?":;:llS",:"f;*jT_1.^"-Tjt9:11,t";il;;;;;il;ffi;".""p:,,"#|"'Jiifj':il11i
;;;;;il#il;;f"ilT"'i'Jl'*?lli"i#
ffiiTll"o:*::T:::?:1fi1::::::1. 9.Y1ry-:1q""i""'iri"'n"g"";)
i*-,""
3::J*::3T::li,:l'::i:i'.:l:'il:l:".:":t"::'o: F; il;;;;iil"'f*;ili'l]r;ff1TrTi;
il:T'j;'"iffi:':J:::l,T.lT
rrom mother to retus via the piacenta. rhe retus acquires
the mother. The correct choice is C. "v,iJ-l' ffiffi;;;"#tjil ri
2. c is correct' active immunity which is artificially acquired. In the first paragraph
acquired imrnunity is specific immunity. In the thud pu.ug.uph of the passage we see thar
disease like the measle.s are exampies of specific immunitlr.
*e see tnit valcinations and a recovery from a
i vacclnation i, ,i_fiy un immunization against a
pathogen' It is something that is irtfficiat[1' acquired. Theindividual
who receives'the ua".ination then begins to
active\' produce his or her own immunological agents agains, G *i"nuu,ed (weakened;
pathogen has been made innocuous, it still ietains'its *rig""i"iiy putt og"n. Even though the
1i"., io n".J,i.ry rio- a disease like the
measles is an example of. active immunity wiich is. #,r1"rry
ffii."d."pir"p.ri.
example of passive immunity which is artificially acquired. transter
antitoxin aoministration would be an
piacental membrane would be an example of pasiive immunity
ft tgG from trr" -otrr".-io the fetus through the
whichis naturally acquired. The correct choice is c.
A is correct' covalent bonding. In the 7th paragraph of-the passage
it says that antibody-antigen complexes can be
dissociated by high salt concentrations or higtt or low irH. iiir
"ith"r
antibody and an antigen are relatively weak. -"st mean that the binding forces between
All oithe bondi ana7o, uona;ng forces listed in rhe answer choices an
weak except for the covalent uona. we would not expect are
o .ouut"ni bond to b" u.oten b1, a change in
(physiological) pH or by treatment with a high salt concentration.
The correct choice is A.
4. D is correct' antibody which has similar dimensions as_the antigenic determinant. This question is
an understanding of a paratope. In simply asking for
7th paragraph.of. tt purlug" *e find rhat th; p#t";
_the
antibody' This allows us to immediatety eiiminate choices " A anic. Look is associared with the
at the structure'ot,n" tgc molecule in
Figure i of the passage' This antibody can bind two epitopes or
tn" *-" ,pecificity (unideterminant). The portion of
the antibody that binds the epitope resides near the N-ierminus
of both the vu'"i,a vi1"gion, of the Fab. The
epitope does notjust bind the vg regions alone. we can eliminate choice s. rh.
determinants is roughly equivalent to about 6 amino acids. Therefore,
u,r"*g" size of antigenic
the size of the paratope musr be simiiar in
dimensions' If this were not the case, then binding between irr"-"pitop"
una pu.utJf" woutd be unfavorable.
Remember, hydrophobic forces, van der waai forces, electrostatic
relatively close proximities between participating parties before they {orces, and hydrogen bonding all require
can be of any use. The correct choice is D.

C is correct, antigens that are not cross-linked, because each Fab fragment
is monovaient. In order to answer this
question, we must consider the structure of the IgG molecule in
Figure'l of the passag". N.r" irr" there are two Fab
domains' It is a bivalent structure. As it sLnds, this antibJdy could participate in cross-linking
unideterminant, multivalent antigen (see the diagram below). with a

However' after addition of the protea.se papain, the antibody is cleaved


into an Fc segment and two Fab segments.
The important point here is that the Fab iegments-are now indepenclent
of each ottr"r.'irr"f .un no* independently
bind to the antigen's epitope (see above). Cioss-linking wiil not iesurt.

In choice-A.the first part of the answer (cross-linking) is wrong, but the


second part is correct. This makes the whole
answer choice incorrect. In choice B the answer ii correct, "but for
treatment of IgG with pepsin, not papain. In
choice D two monovalent fragments are produced, but they cannot participate
in crJss-linkir! 1r" correct choice
is C.

,-opyright @ by The Berkeley Review 319 The Berkeley Review


Specializing in MCAT preparation
Biology Immunology E( Endocrinology Section V Answers

epitope

Before IgG is treated with papain. After IgG is treated with papain.

6. C is correct, IgM. In the 6th paragraph of the passage we find that IgG is the oniy antibody to cross the placental
membrane. Even though this antibody enters into fetal circulation and can confer fetal immunity to certain diseases,
it is IgM which is the first antibody produced during an immune response. Also, it is the predominant antibody
produced by the fetus during fetal development. The correct choice is C.

B is correct, IgA. In the 6th paragraph of the passage we see that IgA is primarily found in external secretions such
as tears, saliva, colostrum, and milk. This question is asking us to recall a little information about lysozyme. In
vertebrates, this enzyme is widely distributed in a variety of cells and secretions. The connection that needed to be
made was that both lysozyme and IgA are found in bodily secretions. The actions of the other immunoglobulins are
outlined in the passage. Ihe correct choice is B.

C is correct, no, receiving antibodies confers transient immunity. Passive immunity means receiving antibodies
from another organism. The recipient does not make antibodies in this situation. Thus, choices B and D are
incorrect. Injected foreign antibodies eventually degrade and are not replaced, since an immune response was not
triggered in the recipient. The effect is not permanent. It is transient. Eliminate choice A. The correct choice is C.

9. A is correct, maternal antibodies provided immunity to measles in this situation. The newborn has an immature
immune system. She is protected by passive immunity via maternal antibodies she received in the uterus and in
breastmilk, if she is fed that way. She cannot respond quickly to primary exposures of viruses until after about 1
month of age. Choice B is incorrect. Although antibodies may be present in saliva in an extremely small amount.
this is not how passive immunity is acquired. Choice C is incorrect. Paternal antibodies stay with the father. Choice
D is incorrect. The correct choice is A.

10. C is correct, both IgG and IgM remain at high levels following a "booster shot". We are looking for the FALSE
statement. Al1 of the questions can be answered by consulting the graph. A "booster shot" means receiving a
vaccination to boost the antibody effects ofan earlier vaccination. On the second (and further) exposures, we can see
in the figure that IgG provides the primary response. Choice A is TRUE. Before a person is exposed to an antigen.
there are no antibodies for that particular antigen. Choice B is TRUE. The primary response to an antigen is a rise in
IgM. This, as you can see in the figure, precedes a rise in IgG. Choice D is TRUE. Both IgG and IgM fall off in their
levels following each expose. They do not remain at high levels. Choice C is FALSE. Since we are looking for the
FALSE answer, choose C. The correct choice is C.

11. B is correct, artificially acquired passive immunity. Since the person receives antibodies from someone else who is
immune to the particular antigen, then it must be passive. Choices A and C are incorrect. Passive natural immuniq'
comes from a mother-baby relationship. This person is an adult, so it must be artificial. Choice D is incorrect. The
correct choice is B.

12. A is correct, artificially acquired active immunity. A vaccine provides artificially acquired immunity. Choices C
and D are incorrect. Active immunity is created by the immune response of the body to the antigens in the vaccine.

Copyright @ by The Berkeley Review 320 The Berkeley Revie*-


Specializing in MCAT Preparation
Biology Immunology 6r Endocrinology Section V Answers

when a second exposure occurs, the body has antibodies and lymphocytes ready
to attack. choice B is incorrect.
The correct choice is A.

13. D istorrect, y-globulin' A gei electrophoresis retards larger molecules,,while


allowing smaller ones to migrate more
quickly' The larger molecules are the slowest. The y-glo"bulin moved the
least in a given time, so they must be the
largest' For your information, the y-globulin fractio-n contains antibodies
immunity. The correct choice is D. -
which lan be used to confer passive

14. A is correct, I only. Breastmilk' especiaily colostrum, the breastmilk of several


days immediately following birth, is
important for all babies. This chiid coujd benefit from the antibody prorection
conferred by another mother,s
breastmilk' choice I is correct. Seclusion wouid protect the baby from exposure
to antigens, but it still would not
make more antibodies. when the babv's immune system matures, it needs
exposure t;;;i;"r, in order to make
antibodies' Eiiminate choice II' I'rmediate vaccinatron would not produce
considerably greater antibodies, since the
baby's immune system cannot function really effectively at birth. ihoi""
ilI is incorrect. The correct choice is A.

15. D is correct' both hydrophilic and hydrophobic regions. The first sentence
of the passage states that the proteins
which are encoded from the class I and class II MHC genes are expressed on the surfice
oFa cett. This should strike
a nerve in your brain. In particular, it should tell.youihat proteins
expressed on the surface of the membrane must
have both a hydrophilic region and a hydrophobic region rrre rrvarophilic region
most of the protein is sitting in the extracellular medium, which wili -urt in this case,
most like"ly be polar"*iriu"cause,
due to extracellular fluid.
The hydrophobic region is necessary because the protein is anchored
to the cell by the plasma membrane of the cell,
which is ofcourse lipid in nature. Therefore, the protein should contain.both
r,yar"pfi"oi"-ino r,ya.optilic region.
Knowing this information' choices A and B are rnerely incomplete. choice "
c is reounaant oecaus" hydroapathetic is
similar to hydrophobicity. The correct choice is D,

16. D is correct, host cellular in origin. This question is not really a trick q.uestion,
it simply requires a little thinking.
There are about one half of a million to one million^class I vigc proteins.
Th;r" ir ;;-;uy'you should know rhat
number, but you should tearize that there are a lot of these proteins
on the surface. Furthe#rore, the passage states
that the peptides found with class I MHC are invariably ttre celt's orn prot"inr.
when the cell is infected with virus,
the virai proteins/peptides will be presented to stimuiaie an immune
,"rponr". But this does not change the fact the
most class I MHC molecules will still have their own proteins being presented
on the cell surface. choices
do not make any sense because those cells are involved with the reffiition/destruction B and c
of cells infected with virus.
There is no evidence to suspect those cells will have their peptides pre"sented.
The correct choice is D.
[- B is correct' signal peptide sequence. This question requires a solid knowledge
of cellular action. It is stated in the
passage that class II MHC molecules have been shown io assemble in the endiplur*i. ."ti"uium.
the proteins get to the endoplasmic reticulum (ER). Proteins that ar;
This requires that
destined'for the ER tuu" u special sequence
called the signal peptide sequence which causes the ribosome they are
being translated on io uina to the ER. The
subsequent translation of the protein occurs, and the entire protein then finds
i-tself within ttre BR. consider the other
answers' Choice A is incorrect because a Shine-Dalgarno s"quen.e is a segment
of RNA found in prokaryotes that is
recognized by ribosomes as a place of binding. Therefore, choice A is lncorrect.
Choices C and D are incorrect
because the question asks about proteins, not DNA sequences. The correct choice is B.
c is correct, endocytosis. Recall that the ingestion of material by the invagination of the plasma
membrane is called
endocytosis. This is exactly the process that occurs for the rvrur4rrerr
straightforward
oi ttrese
formatiJn ur eriJ;;;;"r.
Lrl959 elt(]osomes. This
Inls 15i a Very
'Yqas question' Consider the
Yqvolrvu' vurrDruer other allswcls.
Lus uLrlEl Lfrolce A ls
answers. Choice is a non-senslcal
non-sensical answer and can automatically
automaticallv be
eliminated' choice B is the opposite of endocytosis, and would call for the expuision
of cellular material through the
plasma membrane' Transcyto-sis is the transport of material by
a vesicle from one side of ,h" fi";;;";"rd: ili.
usually occurs in epithelial cells. The correct choice is C.

B is correct, structurally similar to that of the class_Il MHC The passage states
that the CLIp part of the invariant
;hain is actively removed by binding to the DM molecule. The clip used to be bound
to the class II MHC molecule.
Based on this information, one could logically assume that the bu and
class II MHC molecule are structurally
similar, if they bind the same molecule. Considering the other answers, there is no
evidence for any of the
:iatements' The DM molecule is binding CLIP, which does not mean they are structurally
similar. it just means they

r,- *::.: O by The Berkeley Review 321 The Berkeley Review


Specializing in MCAT preparation
Biology Immunology & Endocrinology Section V Answers

form a bond. Choice D is incorrect because class I MHC is not involved with DM. Finally, DM is not likely similar
to the invariant chain. Remember, CLIP is merely a part of the invariant chain, and if DM is not like CLIP, there is
no reason to believe it will be like the invariant chain. The correct choice is B.

20. D is conect, II and III only. A given class I MHC molecuie will bind to a variety of peptides. However, the cleft
where the peptides bind does not change for a given class I molecule. In other words, the amino acids that make up
the cleft are invariant. The peptides that bind to the cleft can only do so if certain amino acids in their chain are able
to bind to the amino acids which make up the cleft. That is, amino acids at certain positions in the peptides must be
highly conserved if a large variety of peptides are able to bind to a class I molecule which does not change its amino
acid composition. Therefore, choice fII is correct. We find that choice II is conect based on the following logic:
Iooking at Figure 1 and Figure 2, we can see that in class I molecules, the peptides are bound in the cleft by their
ends. Therefore, the peptides can only be so long, and as stated in the passage, about eight or nine amino acids long.
However, with class II, the binding occurs in the middle, and therefore we will see more of a variety of lengths in the
peptides which bind. The peptides which bind to class II molecuies are not constricted to binding to the ends, so they
do not have to be of a certain length. The correct choice is D.

21. C is correct, production of a transcription factor which increases the transcription rate of the MHC gene early in an
infection. If there was a transcription factor that promoted the transcription of the MHC gene early in infection, this
would promote an immune response against the pathogen. This is clearly not a defense mechanism for the pathogen.
All of the other answers result in a depression of the immune response, and therefore can be categorized as a defense
mechanism used by a pathogen to disrupt the antigen processing system. The correct choice is C.

7) D is correct, I, II, and III. Any body surface that can contact the outside environment is lined with epithelial tissue.
The mouth, is, of course, lined with epithelium. Choice I is correct. The entire GI tract is essentially "outside the
body." It is completely lined with epithelial tissue. Choice II is correct. The urethra also has contact with the external
environment. It, too is lined with epithelium. Choice III is correct. The correct choice is D.

23. B is correct,IL-6- mice produce lower levels of IgA than IL-6+ mice. Read the bar graph. The IL-6-mice have
almost no IgA compared to the IL-6+ group. Choice A is incorrect. Choice C is incorrect. The response to
ovalbumin was almost nil in the IL-6- mice. Choice D is incorrect. The correct choice is B.

24. A is correct, amino acids. Immunoglobulins are proteins, made of amino acids. Fatty acids are part of many types oi
lipid molecules. Choice B is incorrect. Sphingomyelins are special fats that are found in nervous system tissue
Choice C is incorrect. Phospholipids are part of cell membranes. Choice D is incorrect. The correct choice is A.

?( B is correct, restoring IL-6 in the IL-6- mice improved IgA production. The virus carrier alone did not change the
genetic makeup of either strain of mice. Choice A is incorrect. The IL-6+ mice simply responded to a viral infection
by increasing IgA slightly. Choice D is incorrect. The IL-6- mice did not respond at all to the virus carrier. Thel
were hyposensitive. Choice C is incorrect. The correct choice is B.

26. D is correct, cytokines are local hormones and often act without passing through the entire circulatory systexn
Cytokines are special "local hormones" that do not always pass through the entire blood supply. Measurements maoi
on blood samples may not reflect what is happening at the tissue level. Choice A is incorrect. Hormone
concentrations can be measured in blood samples by radioimmunoassay. Choice B is incorrect. IL-6 can and does
leave the cell that produces it, like other cytokines. How could they interact with IgA if they never left the cell an;
IgA is present in secretions? Choice C is incorrect. The correct choice is D.

27. A is correct, gene therapy. You may have read about the first gene therapy trials in humans during the last year ff'
so. This was to alter genes in people with inherited, disease-causing genetic defects. Specific viruses carried the nem
genetic material. Hopefully, these viruses will infect the host's DNA with a good copy of their defective gene
Vaccination is just plain, old immunization with a modified form of a pathogen. Choice B is incorrect. Geneu;
immunization is a fake answer, and Choice C is incorrect. Pleiotropy having a gene that affects many differen:
characteristics in an individual. Choice D is incorrect. The correct choice is A.

Copyright @ by The Berkeley Review 322 The Berkeley Revieu


Specializing in MCAT Preparatior
Biology Immunology 8r Endocrinolo g.r Section \' {nsu-ers

28, c is correct, soiubilize antigen-antibody complexes. This question jusr re:*-.;. . :


The passage states that persons who are complement deficient sui-i.. 1....--
r:-:--_ ' .. .,i
antibody complexes form precipitates in the blood vessels of skin. joints anci 'n.-,.
cause tissue damage. One can assume from this statement that complemenr
:
piri s .
antibody complexes because when the compiement is not present, the precipitatis
torn,
29. C is correct, provides a means of amplification, uhimately leading to many
MACs. The ker ,.;. .:: ,.. .-:_: : -:: * , r
the word cascade. one should be familiar with the idea of biirogical;;;J;. .

pathway cleaves many molecuies of the next proenzyme in the chain.


nu.ru.,i',-.. -.._.,;-., _-,
The activario" --'
components consists of an amplifying proteolytic cascade where each molecuie "i*.-,.,.. --=.,.-. *-
activated uiifr.-U._*i".,_-.
sequence leads to the production of nany membrane attack complexes. -
The correct choice is C.
30. B is correct, bone marow. This answer is arrived at through one's knowledge
of immune system development. The
question is asking about what disease would affect the pioduction
of antibodies. This would clearly be the bone
marrow' The bone marrow is the site of B-cell birth and maturation. B-cells
u.. r"rfonriu;;;#;[i#;;;;;
antibodies' Therefore' a disease o{'the bone.marrow would greatly
affect B-cell maturation and thereby affect
antibody production. The thymus is the site of r-cell maturatioi, whiie
the ,pt""n unJ tn" ly,npr, nodes are what are
calied secondary lymphoid organs' where mature B-cells and r-ceils "hang
out" to r"aci rith to."lgn antigens. The
correct choice is B.

31. D is correct, constant region of the antibody. we know that there exists interaction/activation
complement component and either IgG or tgM. rhe question.is asking between the c1
what is the most likely means by which C1 is
activated' Cl does not change depending on the micioorganism. In
Jther words, we do not have a difrerent Cl for
different microorganisms. Howevet, ti do have diffeient IgGs and IgMs for different-microorganisms.
difference lies in the variable region of the antibody. That variable ,"g;oniinos The
microorganism' The antibodies also have a constant region, which does
t; ,h"-;;;i;;nic dererminanr of the
not change (except for class switching) or
depend on the particular microorganism. It most likely*tharthe cl,
which remain"s.onrlul|^riu recognize and be
activated by the constant region of the antibody. If this were not
the case, cl would itself have to change to
recognize the variable region of the antibody. The correct choice
is D.
2,) c is correct, confine complement activation to the cell surface where it began. The
early components and c3 are
proenzymes that are activated sequentially by limited proteolytic
cleavage. vihen each tr;nryn . in the sequence is
cleaved, it is activated to^generate a serin-e piotease *hich cleaues
the next protein in the sequence. Many cleavages
liberate a small peptide fragment and expose a membrane-binding
site on the larger fragment. The larger fragment
can now bind tightly to the target cell membrane and carry out"the
next reaction in ihe sequence. The reaction
cascade is constrained to the cell surface where it began, and where
we want it to be. The correct choice is C.

33. B is correct, these molecules stimulate the secretion of histamine from basophils.
The question tells us that an
increase is seen in the permeability of blood vessels. From the answers given,
we know this is due to histamine
secretion' This histamine that is secreted increases the permeability of local
blood vessels which will allow white
blood cells, antibodies. and more complement to enter tlie site of iniection. The question
becomes what cell secretes
histamine' The answer is that mast cills and basophils secrete histamine. Moit
are familiar with mast cells, but
basophils are a type of white blood cel1. They falt under the-category of granulocytes
secrete histamine to
help mediate inflammatory responses. The other types of cetti in the"uns*e.s
u."""Jtrr"y
rl*frj not involved in the
secretion of histamine. The correct choice is B.

34 A is correct,an inhibitor of acetylcholinesterase. Since the receptors are blocked


or not as numerous, an increase in
acetylcholine (ACh) should remove some of the effects. one way ro accomplish
this is to give an inhibitor of
acetylcholinesterase so the ACh is not degraded as quickly. This would give
higher of ACh at the
receptors' An immunostimulant would further complicate the problem by encouiaging "on""nt.utions
choice B is incorrect. A paralytic agent would add io the -or" antibody production.
-uril" weakness. choice cls incorrect. Decreasing the
amount of ACh would be a bad idea, so choice D is incorrect. The correct choice
is A.

Copyright @ by The Berkeley Review 323 The Berkeley Review


Specializing in MCAT preparation
Biology Immunology & Endocrinology Section V Answers

35. C is correct, III. The nerve is the top or presynaptic part, while the muscle is the bottom, or postsynaptic part.
Choice I indicates a vesicle containing ACh. Choice II indicated vesicle fused with the membrane and releasing
ACh into the synaptic cleft. Choice III is the acetylcholine receptor on the muscie cel1. Choice IV is a receptor on
the nerve cell. The correct choice is C.

36. A is correct, Type II diabetes. Type II diabetes is not caused by an autoimmune agent. It is mostly related to insulin
resistance. Choices B, C, and D are clearly autoimmune diseases from the passage. The correct choice is A.

5t. C is coruect, lethargy. The effect of stimulating the TSH receptor causes the thyroid to secrete more thyroid
hormone. This increases metabolic rate and can iead to hyperactivity in severe cases. Weight loss is probably
connected due to increased metabolic rate. The person is not lethargic, which means tired and sluggish. The correct
choice is C.

38. B is correct, insulin. The beta cells of the pancreas produce insulin. Lack of insulin is the defect in Type I diabetes.
You could figure this out from the table if you forgot. The other hormones are produced by the pancreas, but not in
the beta cells. The correct choice is B.

39. C is correct, I and II only. A virus can resemble the structure of a body molecule and lead to autoimmune attack.
Choice I is correct. A virus can damage a cell so that cell components that do not usually travel in the blood are
placed in the blood. This could also lead to an autoimmune response. Choice II is correct. Choice III is the opposite
ofchoice I and is incorrect. The correct choice is C.

40. B is correct, adrenal gland. The adrenal cortex secretes steroid hormones. The correct choice is B.

41. B is correct, increased levels of glucagon and decreased levels of insulin. It is important to remember that glucose is
needed by the body as an energy source. Thus, in times of stress, when the body needs energy to deal with the
stressor, glucose plasma levels must increase. Sympathetic stimulation of giucagon secretion will increase glycogen
breakdown and increase glucose plasma levels. This will make glucose available for tissues to utilize as an eneigy
source. The sympathetic system will inhibit insulin secretion. The correct choice is B.

42. B is correct, increased levels of glucagon and decreased levels of insulin. The major function of glucagon secretion
is to increase the amount of products in the plasma that can be utilized for energy production. During fasting, blood
glucose levels are low, stimulating glucagon secretion. Glucagon secretion will increase the release of free fatty
acids, which can be converted to ketone bodies and utilized for energy. However, if insulin levels are low, as in
diabetes, ketone utilization is low and ketone products increase in the plasma beyond normal levels leading to
ketosis and acidosis. The correct choice is B.

43. A is correct, abnormal increases in insulin secretion. Hypoglycemia, by its name, means "low blood glucose." Thus,
abnormal increases in insulin secretion will decrease levels ofblood glucose. Decreases in insulin secretion and high
levels of plasma glucose are signs of hyperglycemia. Low levels of free fatty acids may indicate increases in insuiin
secretion or decreases in glucagon secretion. The correct choice is A.

44. A is correct, increased levels of insulin and glucagon. Increases in plasma amino acid concentrations will stimulate
insulin secretion to promote protein production in cells. Furthermore, glucagon secretion from the pancreas is also
increased. Glucagon secretion is needed to prevent hypoglycemia after a heavy protein meal since amino acids also
stimuiate insulin secretion. The correct choice is A.

45. C is correct, access and storage of glucose in cells of the peripheral tissue. Insulin stimulates the storage of glucose
and eases the entry of glucose into tissue. The tissue affected most readily appears to be muscle and connective
tissue of the periphery. The brain does not need insulin for glucose uptake, as brain tissue is not affected by insulin.
Insulin stimulates glucose uptake and storage by the liver. Insulin will decrease glucose loss in the kidney by
decreasing plasma glucose levels and increasing uptake of glucose by the tubular cells of the kidney. The correct
choice is C.

Copyright @ by The Berkeley Review 324 The Berkeley Keview


Specializing in MCAT Preparation
Biology Immunology & Endocrinology Section V Ansners

46. A is correct' extracellular glucose excess. The lack ofinsulin secretion will decrease the uptake ofglucose
bi..celis.
Even with extracellular concentrations of glucose rising, cells cannot utilize the glucose U""uut"
it cannot enter the
cells. Therefore, tissues die of starvation, with high concentration of glucose*located just outside
the cell. The
correct choice is A.

47. B is correct, (see the graph below). In the 5th paragraph we read the following: "pTH causes calcium absorption
from bone by stimulating osteoclastic activity and transiently inhibiting osteoElastic activity. At the level of the
kidney, PTH increases calcium absorption in the distal tubules and collecting ducts and greatly decreases
the
reabsorption of phosphate at the proximal tubules." This means that at the tirne PTH is adininisiered
we would
expect to see an increase in plasma calcium levels and a decrease in plasma phosphate levels.
Since pTH is a
hormone and goes everywhere the blood goes, we would also expect to iee these effects happen
at about the same
time, which is exactly what we observe in the graph for choice B.

J
!
o o
E

o
E
O

U
F
0 0 Time (hours)
PTH added

We-would not expect to see the levels of phosphate or calcium lag behind one another. This immediately
allows us
to eliminate choices C and D. We can eliminate choice A simply because we know (from
the passage) that the levels
of plasma calcium will increase. The correct choice is B.

48. B is correct, (see the reaction below). The answer to this question is found in the first paragraph
of the passage. We
are told that vitamin D3 is also called cholecalciferol and that it was synthesized irom"a'sterol
derivative in a
photolytic reaction involving UV light. A sterol derivative is based on the ring structure of cholesterol.
A photolytic
reaction involves the breaking of bonds. Think of "lytic" as something being;'lysed" or broken. you
If co'mpare'the
four structures which are given as_answers, you will note that choicJs B, i, and D are rather similar
in that they
contain just three complete rings. Choice A contains four complete rings. One of the rings (called
the',B,,ring) in
choice A has been open up by the breaking of a bond. In this case that bond was broken "by itre action
of UV li-ght.
Since vitamin D3 is formed from a photolytic reaction, we can eliminate choice A as a possible answer.
turns out
[It
that choice A is called 7-dehydrocholesterol and is the precursor to vitamin D3.l

CH, cH,

UV radiation
(a photolytic reaction)

7-Dehydrocholesterol

Vitamin D3 (cholcalciferol)
We are left with choices B, C, and D. Itwas stated in the passage that vitamin D3 is the inactive form of the
hormone. It was activated by first hydroxylating it in the liver and then by hydroxylating it in the kidney. In other
words, the active form of vitamin D3 (called lc,25-dihydroxycholecalciferol (or 1,25-(OH)2-Df
)) has two more

lopyright @ by The Berkeley Review 325 The Berkeley Review


Specializing in MCAT Preparation
Biology Immunology & Endocrinology Section V Answers

hydroxyl groups on it than the inactive form. Therefore, we look for that structure which has not had two
hydroxylation events. Based on this analysis we can eliminate choices C and D. The correct choice is B.

49. D is correct, I and III


9!1y In the second paragraph it was mentioned that in order to synthesize the activated form
of vitamin D3 (1'25-(OH)z-Di, the hydroxyiase enzyme in the kidneys musr be activated (stimulated) by pTH.
Once the activated hormone is synthesized, calcium and phosphate can be absorbed across the mucosa membrane
of
the intestines. The phosphate ion follows the calcium ion across the membrane so electrical neutrality is maintained.
We can eliminate choice C because PTH by itself has no direct effect on either calcium or phosphaie absorption in
the intestines. We can also eiiminate choice B because calcitonin does not allow for intestinal abiorption of ialcium
or phosphate either. Even though choice A is a correct answer, it is not the BEST answer. In brder to get the
activated form of vitamin D, PTH is needed. Therefore, an increase in both PTH and subsequently an incrJase in
1,25-(oH)2-D3 is needed for absorption in the intestine. The correct choice is D.

s0. A is correct' increased osteoclast activity coupled with a hypercalcemic plasma. Be careful with what this question
is asking you to answer. The bottom line of what is being asked is: "What does it take in order to secrete calcitonin?',
Calcitonin acts to loser plasma calcium levels. In order to lower those ievels they must first be high. What leads to
high calcium levels? Increased osteoclast activity (i.e., bone resorption and the ieturn of calcium-and phosphate
to
the blood) and a hypercalcemic plasma (i.e., a plasma in which the level of calcium is above normal). These
two
conditions demand a decrease_in the levels of plasma calcium. The secretion of calcitonin by the parafollicular
cells
of the thyroid gland reduces the concentration of calcium in the blood. Calcitonin does this by inhibiting osteoclast
activity (i'e., inhibits bone resorption) and inhibiting calcium reabsorption at the level of the kidney. T-he correct
choice is A.

51. C is correct, (see the pedigree below). The characteristic to look for in an X-linked dominant trait is that alt of the
daughters and none of the sons of males who carry the trait are affected. This is exactly what we see in this
pedigree. Interestingly, the phosphate levels are not as low and the rickets are not u. i"u"." in an afflicted
heterozygous female as in the afflicted male.

rT'oTo

,,,
'+ +" r, tr-I
[I'rt i-l rrn ,,,
Autosomal Dominant Autosomal Recessive X-linked Dominant
Autosomal Dominant

If any son is affected or if any daughter is unaffected, then the inheritance is autosomal and not X-linked.
Remember, autosomal recessive is oniy expressed in homozygotes. The correct choice is C.

<t D is correct, phosphate and 1,25-(OH)z-D:.Familial hypophosphatemia refers to the fact that there is inadequate
phosphate in the plasma. As mentioned, this is an X-linked recessive disease and stems from the fact that
reabsorption of phosphate at the levei of the kidneys is impaired. The addition of caicium will not affect the levels of
phosphate and so we can eliminate choice A. Addition of phosphate alone will not make that much of a difference
because it will not have an adequate means by which to be absorbed. We can eliminate choice B. We cam also
eliminate choice C for the same reasoning. The addition of 1,25-(OH)2-D3 along with the phosphate insures
adequate absorption by the intestines and (presumably) an increased reabsorption at the level of the kidney. The
correct choice is D.

E2 C is correct, increased neural excitability. Lefs consider the list ofpossible answers. In order to do this we need to
know the meaning of hypoparathyroidism. The parathyroid giands secrete PTH and PTH allows for calcium
absorption from bone by stimulating the osteoclasts and transiently inhibiting osteoblasts. PTH stimulates calcium
reabsorbtion at the kidneys and indirectly allows for calcium absorption at the intestine through the action of the
active form of vitamin D. A hypoparathyroid gland is one that does not secrete sufficient amounts of PTH. As a

Copyright @ by The Berkeley Review 326 The Berkeley Review


Specializing in MCAT Preparation
Biology Immunology & Endocrinology Section V Answers

result the levels of calcium in the body decrease below normal values.
Not only is osteoclast activity decreased, but
because calcium levels are low, osteoblast activity is decreased
as wett. with this information we can eliminate
choices A, B, and D' we are left with choice c as the correct
answer. why would hypoparathyroidism lead to
increased neural activity? A decrease in calcium in the extracellular
fluid allows -o." ,o'airi.n to flow through the
membranes of nerve cells. some nerves, like those.in the peripheral
n"ruous ;t;4";,-;;;in to sponraneousiy
discharge' This will eventually give rise to tetany and if not
coriect immediut"ry'.ur r*Jio a"utt . The correct
choice is C.

54. D is correct, a positive linear reiationship exists between PTH secretion


and the concentration of plasma calcium.
This statement is false. In order to makJit rue we would say that: "an
inverse relationship exists between prH
secretion and the concentration of plasma caicium. we can see this
from rhe t.;ph ;;;;'b"jor. The first three
choices are all true as discussed in the passage and in the previous
unr*"rr. The correct choice is n.

tr so
o
cd
o
O
o o
O 5
o
o
L-
$

Total Ca2+ concentration in piasma

she is very untikery to have antigens ro rhe Rh


"".::,:"";::,"",:?l:.
113::::1",1:111:jTl :hild,
aborrion, ormiscarriaie e""" ti,""ei,
facror. Exposure
;;;;;:;;r;;;;ffiffii;#:
ffi:i,f:TT.r].1,:f*:,1:It,9.:ll":.t, ;J;;
+:o:li:""""1^"1fl llt^","111::llld,
rl: -?r*I musr arso r,uu" tr,"
In this case, she is probabry not sensitized. The correit choice
untiil;;il#;;;,Jffi;",:',1,ffi:ffi;.Y;
is D.

56. is n1t a component of the ABo blood group sysrem. The piacenra
l:^":j::"^':11"
some
ova'w things
r'urE;) and *l 1*11
4rru allows
iluuws passage
Passage to otheri.
otners. There
lhere ls
is not tndiscriminate
indiscrimi
acts as a barrier for
mixing of maternal and fetal blood. Nor does
the mother make the red blood cells for the fetus. Choices
A and C are incorrect. As we learned
lenrnerl in
in rha --,,
-ooo,-o any
the passage,
is taken to completion or not, can trigger antibody formarion in
lLllli"p; Yl*:t l,
fetuses. Choice D is incorrect. The correct choice is B.
Rh- ,;#;^;f;il$;'Jili
57. C is correct, treat mother with antibodies against the anti-Rh antibodies.
removed by other antibodies, then the mother is.not sensitized
If the antibodies can be trapped and
against Rh antigen for subsequent pregnancies.
Treating the fetus would not help future fetuses. choice D is inconEct.
In extreme circumstances, the fetus, not the
mother' receives a blood transfusion. This would not help future pregnancies,
either. The correct choice is C.
58. D is correct, all of the above. All three of these organs produce bitirubin,
a break-down product of heme. The liver
conjugates the bilirubin with glucuronic acid, so it is ioluble, and
secretes it into the gail bladder. The correct
choice is D.

59. A is correct, no, there are no maternal antigens to the Rh. factor antigens. The term positive means the antigen is
present on the RBC' The mother does not have antigens to herself.
ThJfetus is negativi,
on the RBCs to make antibodies against. choice B is incorrect. The newborn
roit"." are no Rh antigens
is not immunologically competent,
able to make antibodies, until aboul l month after birth. It relies on
the antibodies donateJd;" mother. Choices C
and D are incorrect. The correct choice is A.

60. D is correct, I, II, and III. Hemolysis in the Rh-positive fetus depends on the
amount of antibodyproduced by its
Rh-negative mother. If a large amount of blood has been transferred between
the fetus and the mother, then she is
more likely to produce anti-Rh factor antibodies. Choice I is correct. How
her antibody proau"tion responds to a
given stimulus is also a factor. choice II is correct. As the number of pregnanci"r
in"i"'ur"i her exposure to Rh

Copyright O by The Berkeley Review 327 The Berkeley Review


Specializing in MCAT preparation
Biology Immunology Er Endocrinology Section V Answers

positive blood increases, and the likelihood of developing antibodies increases. Choice III is correct. The correct
choice is D.

61. B is conect, I and II only. The primary problem in EF is lysis of red blood cells. This leads to anemia and lower
hemoglobin level. Choice II is correct. If RBCs are destroyed, this does not lead to an increased number.
Erythrocytes are RBCs. Choice III is incorrect. Bilirubin is a breakdown product of heme. It is elevated in neonates
and fetuses with EF. Choice I is correct. The correct choice is B.

62. A is correct, does not have an outer membrane on its cell wall. This question requires one to know a little about the
differences between Gram-positive and Gram-negative cells. A Gram-positive cell does not have an outer membrane
on its cell wall. This information eliminates choice B. Furthermore, a Gram-positive bacterium has a very thick
peptidoglycan layer adjacent to the plasma membrane. The correct choice is A.

63. C is corrrect, endocytosis, engulfing foreign particles which eventually will fuse with a lysosome. The question
requires us to use our knowledge about how macrophages are functioning. They are phagoiytes in the body along
with the neutrophils. The cells engulf microorganisms (this eliminates choices A and B) which then become
phagocytic vesicles. These vesicles fuse with the cell's lysosome (eliminating choice D), an organelle with contains
highly reactive molecules like superoxides. The fustion of thephagocytic vesicles with the lysosome exposes the
-
foreign particles to these caustic molecules, acting to destroy the microorganism. The correct choice is C.

64. C is correct, plasma cells. This question requires us to draw on our knowledge of the immune system and the cells
which comprise that system. Antibodies are associated with B cells, which are lymphocytes. Antibodies can be
found in two forms. One is the membrane bound form, attached to the B cell at its plasma membrane. When a B cell
becomes activated, it will proliferate. Some of the new B cells will produce the antibody (it will be the same
antibody) in a soluble form, enabling the antibody to circulate in the lymph and blood. B cells which make soluble
antibody are called plasma cells. Therefore, a defect in plasma cells would affect production of soluble antibodies.
The correct choice is C.

fl5. C is correct, rise in the patient's blood pressure. Anti-Factor XII antibodies will take out (render non-functional)
some of the available Factor XII. This will affect the prodirction of bradykinin and the intrinsic coagulatory pathway.
Since not all Factor XII is non-functional, some intrinsic pathway leading to coagulation may occur. We still have
the entire extrinsic pathway. Therefore, we should not see either a total loss (choice A) or a rise (choice B) in the
intravascular clotting of the sepsis patient. As stated above, we are attenuating the role of bradykinin and one of its
role is to increase the permeability of the vessel. Increasing the permeability leads to a loss of fluid and hypotension.
Since we are attenuating the role of bradykinin, we should therefore see a rise in the patient's blood pressure. The
correct choice is C.

66. D is correct, 16. The flow of blood through a vessel is proportional to the radius to the fourth power. If we see an
increase in the radius of a factor of two, the increase in flow is 24 = 16. The correct choice is D.

67. A is correct, hypotension in the patient. One should realize from the passage that bradykinin acts in two ways. It
increases the amount of blood flowing as a vasodilator, and it increases the permeability of the endothelial cells
making up the blood vessels. The increased permeability causes a lot of the plasma in the blood to leaves the vessel
(through osmotic forces ) and enter into the tissue. The result of this is that the patient becomes hypovolumeic which
will lead to a drop in blood pressuie. Therefore, bradykinin will lead to hypotension in the patient. The correct
choice is A.

68. C is correct, decreased osteoblast activity, increased osteoclast activity. When bone synthesis is low (decreased
osteoblast activity) and bone breakdown is high (increased osteoclast activity), the amount of bone decrease is the
greatest. The correct choice is C.

69. C is correct, calcium. The question asks for a mineral. Eliminate choice B, water, because water is a compound, not
a single atom. B 12 is a vitamin, not a mineral, so choice A is incorrect. Since calcium was mentioned in the passage,

Copyright @ by The Berkeley Review 324 The Berkeley Review


Specializing in MCAT Preparation
Biology Immunolotry & Endocrinology Section V Answers

consider it. Calcium is the major mineral making up bones. our body contains about 1 kg
of calcium. Although zinc
is part of many enzymes, our body contains only a I'ew grams of zini. The correct
choic! is C.
70. A is correct, PTH increases piasma calcium. The passage states the CT is an antagonist of pTH. This means
they
have opposite actions' CT,works to decrease plasma caicium, and PTH works to inc?ease piasma
calcium. Choice B
is incorrect' To increase plasma calcium, the kidney decreases urinary calcium and phosphate.
Choices C and D are
incorrect. The correct choice is A.

71. C is correct, cysteine-cysteine. This is a biochemistry trivia question. Two cysteine residues link
together via their
sulfur atoms to form a cystine molecule that contains a disuifide bridge. Cysteine is the singie amino
acid, while
cystine refers to a dimer of cysteine. The correct choice is C.

72- B is correct, I and III. Choice II is inconect. Since CT is


a polypeptide hormone, it does not enter the cell, but rather
acts on a receptor on the piasma membrane. Choice I
is conect. The salmon CT is less recognizable to the human
enzymes' so it resists degradation more and has a longer-iasting action. Choice III is corrlct.
Saimon CT has a
stronger interaction with the CT receptor and promotes a longer signal on the receptor. The
correct choice is ts.
73. A is correct, CT secretion increases. In the beginning of a supplement program, a person would have
increased
calcium levels. In response to these increased levels, CT secretion wouldlnciease'to Lring
the levels back down to
normal. The correct choice is A.

74. B is correct, injection to avoid hydrolysis. Since.CT is a polypeptide, it is subject to digestion (hydrolysis)
in the
stomach' Ingestion means to take orally, so choices A and C aie incorrecr. Th"r" is a"possibiiiiy
of un aitergic
reaction when proteins are injected, so choice D is incorrect. The correct choice is B.

t5. D is correct, I, II, and III. In a systemic allergic reaction, histamine, leukotrienes, and prostagiandins
are released
and lead to skin reddening, hives, and constriction of the bronchioles (leads to bieathing difficulty),
among other
symptoms. The correct choice is D.

76. D is correct, Iittle or no change in the effectiveness of either type of immune response. The question
asks about the
consequences of removing the thymus from an adult human. From the passage w" learn that
the bone marrow and later mature in the thymus d.uf1s early development.the thymus is
i cells are produced in
therefore crucial during
early development for the production of T cells, which are reiponsible for the cell-mediated response.
If the thymu!
is removed from a mature adult, however, there will be little or no impact on the immune system
since most T cell
formation has aiready occurred. The tempting answer is choice B, bui remember to read uil of th"
answer choices
before making a decision. A careful reading of the question in this case is also a must. The correct choice
is D.

C is correct' the cell would lyse due to an upset water balance. When pores are formed in the membrane of the
target cell, smali molecules (i.e. charged ions) are suddenly able to flow freely along their electrochemical gradients.
The effect of this would be to disrupt the osmotic baiance maintained by the cell. T;his balance is necessar! because
the large number of negatively charged macromolecules in the cell (proteins, nucieic acids, etc.) would normally
attract a large number of cations in order to balance the charge. The entry of such a large number of cations woulb
draw water into the cell with them, a catastrophe which would cause lysis, or bursting, 6f the plasma membrane. In
normal cells, this is prevented because the membrane is mostly impermeable to small iharged i,ons, and the action of
membrane transporters serves to maintain an appropriate osmotic balance. The action of tie complement proteins is
to poke holes in the membrane, allowing ions and water to rush into the cell, causing lysis; this is an efficient way of
killing invading microorganisms. Choice D is the opposite of what was just desciibed and is therefore incorrect.
Choice A is also wrong; an inability to initiate action potentials is not letiral to the cell. Additionally, most cells
which are attacked by the antibody/complement system are invading microorganisms which do not have anything to
do with the neuronal function of conducting action potentials. Choice B is wrong because if the cell is nor-itty
negativeiy charged inside, holes in the membrane would allow cations to rush in, leading to depolarization rather
than hyperpolarization. The correct choice is C.

78. C is correct, II and III only. Statement II states that organ transplants between monozygotic twins do not result in
immune rejection of the transplanted organ. This implies that the recipient's immune ryrt"m regards the new organ

Copyright @ by The Berkeley Review 329 The Berkeley Review


Specializing in MCAT preparation
Biology Immunology 6r Endocrinology Section V Answers

as being "self'" Since monozygotic twins are genetically identical (as opposed to dizygotic twins), this
statement is
consistent with Theory 1, which states that individuals lacks the genesio produce u iJtponr" against self antigBns.
Since both twins have the same genetic structure, they both lack the genes necessary to mount u-n im,nun" r"rionr"
against their own tissues; therefore, according to Theory I the transplanted organ is not rejected. Statement II is also
consistent with Theory 2, which in essence states that the immune system "learns" not to attack its own tissues early
in deveiopment. Since both twins had the exactly the same tissues (the result of their identical genes), both of their
immune systems "learned" not to attack the same tissues. This also explains why the transplanted organ was not
rejected" Statement III is, for all intents and purposes, an identical situation to that presented in Statement II. In this
case' geneticaily identical mice from the same inbred strain are able to tolerate iranspiants from each other. An
inbred strain, incidentally, is a strain of mice that has been inbred for many generations, resulting in siblings which
are genetically identical to one another. Statement I states that cells from a foreign cell line are injected into*a mouse
at the embryonic stage. When the mouse matures, it does not mount an immunJresponse to further injections of the
s_ame cells. This supports Theory 2 inthat it provides evidence that the immune system "learns"
during embryonic
development not to attack cells that are present at the time (i.e., self cells). ThL immune system is*fooled into
thinking the fbreign cells are self ceils. This contradicts Theory 1. According to this theory, the foreign cells would
still be attacked even if they were injected at the embryo stage" The correct choice is C.

79. B is correct, weakness and difficulty breathing. The question states that antibodies are produced against skeletal
muscle acetylcholine receptors. This would most likely result in muscle weakness (dueio an inabi-lity of muscle
fibers to receive signals from effector nerves). Difficulty breathing would also be likely because miny skeletal
muscles (i.e., the intercostals) are involved in the respiratory process. This problem ulro be solved via a
process of elimination. Answer choice A can be eliminated because the heart is made up of "ould
cardiac muscle. Since the
antibodies only attack skeletal muscle receptors, the heart should not be affected. Likewise, the smooth muscle of
the gastrointestinal tract should not be affected, ruling out answer choice C. Choice D is unlikely to be true as well.
Neurons in the brain should not be affected by skeletal muscie receptor antibodies, and therefore no dementia should
occur. The correct choice is B.

80. B is correct, the B cells have considerably more rough endoplasmic reticulum than the T cells. Since the function of
activated B cells is to produce and secrete large amounts of antibody, it would be likely that under electron
microscopy they would appear to have more rough endoplasmic reticulum than activated T ceils, which do not have
as great of a secretory role. Rough ER is where proteins that are to be secreted are mainly produced.
The "rough,,
appearance of this organelle is due to the presence of ribosomes dotting the convoluted membrane of the ER. Thise
ribosome produce the antibody proteins. Choice A is not necessarily a likely observation, as neither cell has an
especially great need for mitochondria relative to the other. Answer choice C ii also incorrect as T and B cells both
have nuclei. Do not confuse them with another type of blood cell which does lack a nucleus: the erythrocyte. Answer
choice D is incorrect because electron microscopy is the most powerful means of visualizing und as such is
very likely to detect differences berween B and r cells. The correct choice is B. ""llr
81. A is correct, each B cell is predetermined to bind a specific antigen. First, let's go over the experiment in the
question. A population of B cells is removed from an adult mouse and mixed with an effectively lethal antigen X.
The B cells that have the appropriate surface receptors bind the antigen are killed due to the radioactivity. Veiy few
B cells are killed, therefore very few out of the total population (<0.017o) recognize antigen X specifically. Whin the
surviving B cells are injected into an irradiated mouse (lacking B cells), the mouse is now capibl" of reiponding to
other antigens, but not antigen X. It can be implied that the reason for this was the death of the B cblls which
specifically recognized antigen X. The remaining population of B cells had no members which recognized antigen
X, though they could recognize other antigens. Therefore we can conclude that each B cell is predetermined to bind
a specific antigen. B cells can not recognize new antigens and "learn" to bind to them, ma*ing answer choice B
incorrect. Answer choice C is wrong because the above experiment implies nothing about T cell interference with B
cell binding. Answer choice D is wrong because an antigen, by its very definition (antibody generator), is
immunogenic. Therefore antigen X does elicit an immune response normally. The correct choice is A.

82. B is correct, antigens can only be bound by a single antibody at a time. The question is asking which condition
would not hold true if antibodies were capable of "teaming up" and cross-linking antigens together. We can
approach this problem by first theorizing how antibodies could accomplish this feat. First, ifantigens could be bound
by more than one antibody, and each antibody had two antigen binding sites connected by a flexible "hinge" region,
cross-linking could easily occur. Each antibody would bind two antigen particles, which would then bind an
additional two antibodies, which would then bind another two antigen particles, etc. The flexible hinge region would
allow greater flexibility during the cross-linking process. The above description would make answer choices A, C,

Copyright @ by The Berkeley Review 330 The Berkeley Review


Specializing in MCAT Preparation
\

Biology Immunology & Endocrinology Section V Answers

and D all consistent with the cross-linking phenomenon. This leaves choice B
as the answer. If antigens could only
be-bound-by a single antibody at a time, ihe above scheme would not be possible
and there could be no cross-
linking. The correct choice is B.

83. A is correct, most types of cells in the body. The key concept to remember here is that viruses
can inf'ect most types
ofcelis in the body. Since the question states that ceis infecied with viruses are usually killed
by cytotoxic T cells, it
logically follows that the T cells should be able to target most cells in the body. The other
unr*". choices mention
specific cell types and are therefore incorrect. The correct choice is A.

84. c- is conect, detergent wash. The passage informs us that the insulin receptor
is an integral transmembrane protein.
The receptor is thus embedded in the lipid bilayer and actually spans the entire layer
flom the extracellular to the
intracellular face. These proteins are tightly bound to the lipidbiiayer by hydroptrouic
foices ano harsh conditions
are needed to remove them' In particular, detergents must be useO. rh" ,"uron
is that these molecules are
amphipathic, containing both a polar and
non-polar region. The non-polar region binds to the hydrophobic regions
the membrane proteins., disruPting the lipid bilaye.. The polar region creates of
a water soluble micelle with the
membrane protein (hydrophobic portion) in the middle. The t"""p-to. is thus
isolated and ready to study. The
correct choice is C.

85. B is correct, transduction mechanisms. we know from the question that we have a
hybrid receptor which is made up
of
a insulin binding region and a EGF tyrosine kinase segmint. From the passage,
rL r.nor it ut the insulin receptor
is believed to be a insulin activated tyrosine kinase. Recall that ttre uinainf of insulin
leads to tyrosine kinase
activiry' The mechanism which brings about this activity is termed the signal tinrou.iion
puril*uy. with our hybrid
receptor, the binding of insulin leads to kinase activity in the EGF segment. This
information indicates a similar
signal transduction mechanism. The correct choice is B.

86. C is correct' The question tells us that the binding of glucagon will result in the production of the secondary
messenger cAMP' The values of cAMP should thus rise wittr tirne. Using this piece
of information, we can eliminate
choices A and B. The question then becomes one of knowing the sequence
of events. To answer this, one has to use
previous knowledge and/or a little common sense. The seco-ndary *"rr"ng"r, produced
lead to changes in the cell.
Recall that to lruiy bring about change for the cell, there must be some ctrige il pr"bi;r,-;;proreins
are the key to
function. one very common way to regulate proteins is through their phosph&ylation
ana ineii dephosphorylation. It
is the secondary messenger that promotes the phosphorylation of certain peptides,
converting these proteins into
their active forms. The. active forms of the pr-oteins bring about the change. From this
sequence of graphs can be seen. The correct choice is C.
i"f;-;ii;n; ;;-;;.;;;;

87. C is correct, a normal insllin receptor. The question refers to a type I, insulin dependent
diabetes. These individuals
do not produce insulin. That is why they are dependent. This should clue us inlo the fact that
their receptors more
than likely have nothing wrong with them. The problem is that these individuals have no beta
cells in ttre pancreas
which release insulin. These individuals take insulin and their body responds appropriately. From
this information,
we can conclude that the receptors are more than likely normal. The correct ctroice fu C. -

88. D is correct, stimulation of pyruvate dehydrogenase. We know from the passage that GIp mimics certain insulin
responses. Therefore, we should look for an insulin response. Choice D inlicates a stimulation
of pyruvate
dehydrogenase, the enzyme converting pyruvate into acetyl CoA. This would indicate an acceleration
of giycolysis
and the Krebs cycle. This is certainly one of the effects of insulin, as the hormone promotes the
use of glucose as a
source of fuel. The correct choice is D.

89. C is correct, increased CO2 production. The question tells us that the addition of bivalent anti-insulin receptor
antibodies induces a response without insulin. What will antibodies cause? The key word here js
bivalent. The
bivalent structure will induce crossJinking, leading to a clustering of insulin receptors. This conclusion is bolstered
by the next sentence of the question which states that monovalenl andbodies, *hi.h produce cross-linking,
do not induce a response in the absence of insulin. However, if we add antibodies "unnot
against these monovalent
antibodies/fragments, we should be able to induce crossJinking and thus clustering. Therefire, we should be able to
induce an insulin-like response, which involves the promotion of glucose u. u fu-"1. When glucose is oxidized, the
carbons are released as carbon dioxide. The correct choice is C.

Copyright @ by The Berkeley Review The Berkeley Review


Specializing in MCAT Preparation
Biology Immunology & Endocrinology Section V Answers

90. B is correct, reducing conditions. This question is answered from our understanding of the conditions which are
necessary to form or break disulfide bonds. When two cysteine residues are oxidized, the bond is formed and they
fotm cystine. In order to break the bond, cystine must be placed in reducing conditions. From the passage, we know
that the dimer insulin receptor is held with disulfide bonds. To isolate a component of the receptor for study,
reducing conditions would be used to break these disuifide connections. The correct choice is B.

91. B is correct, keratinocytes. Hepatocytes are cells found in the liver. The liver has numerous functions, such as the
regulation of metabolism of carbohydrates, lipids, and proteins. The liver stores glycogen and is the primary site of
gluconeogenesis. The liver is a major source of cholesterol in the body and is a mijor sto.uge site forlron. The liver
is also the site where many hormones are degraded and where many toxins are inaitivate d,. Leukocyte.r (white blood
cells) are a celluiar constituenf of blood, aiong with erythrocytes and platelets, and they play an important role in the
body's defensive system. Leukocytes come in five classes: neutrophils, eosinophils,'baiophils, monocytes, and
lymphocytes. Lymphocytes can be categorized as being either B-cells, which mature in the bone marow, oi T-cells,
which mature in the thymus. Lymphocytes participate in the immune response.

Keratinocytes are skin cells of the epidermis. This cell type (of all the cell types listed) is closest to the exposure
of
ultraviolet light and allows for the conversion of 7-dehydrocholesterol to pievitamin D3. In the skin previiamin D3
can be converted to vitamin D3, which is the precursor to the active form, 1,25-(OH2)-D3. The correct choice is
B.

92. C is correct, integument. In the passage it states that humans obtain vitamin D3 either by ingestion or through
synthetic mechanisms that are initiated by cells in the epidermis of the skin. If we ingest vitamin D3, it will end up
being absorbed by the cells of the intestine. However, vitamin D3 is not the primary precursor for t,is-ioH
2)-D3. in
the skin (integument) the primary precursor (7-dehydrocholesterol) reacts with UV light to form previtamin b3,
which in turn will form vitamin D3. The correct choice is C.

93. C is correct, thyroid gland. In the passage it states that PTH is referred to as parathyroid hormone. It would seem
logical that PTH would therefore be derived from the parathyroid gland. There are four parathyroid glands imbedded
within the thyroid gland. One pair is located in the superior poles of the bi-lobed thyroid gtanA, wnlte the orher pair
is located in the inferior poles. The parafollicular cells (C cells) are located within the thyroid gland itself. The
correct choice is C.

94. C is correct, II and III only (polypeptide and steroid). In the second paragraph of the passage we are told that 1,25-
(OH)2-D3 passes into a target cell and then into the nucleus where it .ompie*es with a releptor protein that has a
DNA binding site. This is the general mechanism for action of steroid. hormones. We can eliminate choice A as a
possible answer.

In the third paragraph we find that both PTH and CT have genes encoded in the short arm of human chromosome 1l
and are synthesized as a preprohormone from different primary RNA transcripts. If they are both synthesized as a
preprohormone from an RNA transcript, it must mean that amino acids are being linked together in peptide linkages
at the ribosome. In other words, a polypeptide is being synthesized. The mature PTH polypeptide contiins 84 amino
acids while the mature CT polypeptide contains 32 amino acids. These three hormones fail under the steroid and
polypeptide classes of hormones. The correct choice is C.

95. D is correct, I and III only (hypocalcemia and hyperphosphatemia). What does PTH do under normal
circumstances? Based on information in the passage we find that PTH acts on bone and on the distal tubule of the
kidney to promote Ca2o reabsorption and inhibits the reabsorption of PO43 e in the proximal tubule of the kidney.
PTH aiso stimulates the synthesis of 1,25-(OH)2-D3 in the kidney. We saw that 1,25-(OH)2-D3 promores the uprake
of Ca2o and PO43e from the intestinal lumen. If we did not have adequate levels of PTH secretion, we would
expect low levels of Ca2@ reabsorption (hypocalcemia) and a decrease in urinary POo3e excretion. A decrease in
urinary PO43e excretion means that PO43e is being reabsorbed by the proximal tubules of the kidney. This will
lead to high (hyperphosphatemia) plasma leveis of PO43 e. Th" correct choice is D.

Copyright O by The Berkeley Review 332 The Berkeley Review


Specializing in MCAT Preparation
Biotogy lmm unology & Endocrinology
Section V Answers

96. B is correcr it Be careful


[Ca23,. :. .: ,t_,'' ,,ues. what iswhen
.-. '_- .t --=:-
reading the graph. Arong the x-axis
going to happen when the plasma levels
we see the totar plasma
goin-t to '.," .- ' ' - :-: :' :: c:-= tn the plasma' How can we do this? we increase the of cad ;;;;;;we are
levels of pTH. In the fourth
nar. -r - !
1]qaq:1L!.. '-: :::!::: -: s'ries that PTH acts on bone and on the distal tubuie of the kidney to p.o,not" cuze
reabsor:,- , -
\\'e se- ,:. _:.; :., f;;ilr:;;:trTrto
increase the levels of PrH ,"in t;;;,o d""."ur". ir,i, i, exactly whar
".,-
o
o
I

low high
Total Plasma [Ca2+1

what happens when plasma Ca2 e levels are high?


The parafollicular cells begin to secrete
osteoclasts of bone tissue' As the calcium levelJare calcitonin which acts on
iowered 1oy a m"cirunismihat i, not .o-pi"teiy
calcitonin leveis begin to drop. This is exacrly wtrat understood) the
is trro*n )n'*r";;;il l;;;;il;;;:'rdJ'J;.rect
choice is B.

97.
iljl,i"liTl:111"*",,,,:",1'|}?::1,:"^1::i1:::lil.ven il the passage, rer,s use our tesr_raking
skius. choice A is
il,:"_T:;liiJil:.#i::l:": l*^l^::.""::,i:*.',l:i 111,;?;;ffiH;;ii,Tio"liiil'f:'Jii,'f i:;*'J:$,::
ilyf:.?'#T.il?"".1i,T1'Jil'":u:j:""x:l'^:*:::li'lr:::Tid!11'!''i'tr:ilru:#J:i'J'll,i,?l?li,iilii,
the co*ecr term for wBCs squeezing out is criaieded.
i;;;;"ii ifroi"" i, n.
98' D is correct' hydrolyze bacterial proteins' Think
of what enzymes do and how they are named,
lipids' so choice A is incorrect' An amvtas"_o;si;.o;i;#;;;;;;ffiffiil,1"t1'j, A lipase hydrolyzes
,o B is incorrecr. A
H:r:::."ffiXf,lf1'tt'e
DNA, so cho'ce c i' il;;;.; f.olur"
or proreoryric "rroi." enzyme hydrolyzes proreins. rhe
99' D is correct' toxic oxygen products. Toxic-oxygen
products are
form free radicals' The pasidge explains the cJmmuni.ution toxic because they are fiee radicals or decompose
to
The proteases directly. attack bacteria protein
,ot" or.ytoti*r, ,'o choice C as incorrect.
and hydrolyze i""at, buido not usl-rre. "rj-inur"
iaai"ats. Eliminate choice B.
Chemicai attractants signal neutrophils to move
to infection area, so eliminate ciroic" a. The
correct choice is D.
100' is nitric oxide' HNo3 is nitric acid. H2N2o2 is nitramide.
ililffiTl"TiJ" No2 is nirrogen dioxide. The

101' c is correct, TNF is mainly present in the general circulation. In the question,
abnormal cytokine cascade' Therefore choice A is incorrect. we are told that binding TNF halts the
usuatiy proteins ,r-rui cells by endocytosis are
degraded in lysosomes'.If tle receptors w-ere taken "rrt",
up in this .rrunn"., then they would be degraded and not able to
interact with TNF' choice B is incbnect. rf the rxr i"cepto.s
are injected in the general circularion and work,
TNF must be present in the general circulation. This means then
choice b l, .ort""t-uiJ-rr, contradict choice
correct choice is C. D. The

lo2' A is correct, neutrophils. By correct reading, we can eiiminate


choices B and c. Erythrocytes are red blood cells,
choice B is incorrect. T-eukocvte is simplv Inother name so
fo. rhi,J;iJo"d;i, #i;i": c is incorrect. Neutrophils
are 60-7}vo of the wBC population, *uling them
the mort abundant. The correct choice is A.

103' B is correct' I and-II only. cytokines do not kill bacteria directly, they serve as communicators
deactivate wBCs. This is stated in the passage. The to activate or
correct choice is B.

Copyright @ by The Berkeley Review 333 The Berkeley Review


Specializing in MCAT preparation
EBI{ELEY
R.E.V.l.D.W'"

PERIODIC TABLE OF THE ELETTENTS

1 2
H He
1.0 4.0
3 4 5 6 7 8 9 10
Li Be B C N o F Ne
6.9 9.0 10.8 12.O r4.0 16.0 19.0 20.2
l1 12 13 14 15 16 17 l8
Na Mg AI Si P s CI Ar
23.0 24.3 27.0 28.1 31.0 32-t 35.5 39.9
19 20 21 22 ZJ 24 25 26 2'7 28 29 30 31 3Z 33 34 35 36
K Ca Sc Ti v Cr Mn Fe Co Ni Cu Zn Ga Ge As Se Br Kr
39.1 40.1 45.0 47.9 50.9 52.0 54.9 55.8 58.9 58.7 63.s 65.4 69.7 72.6 74.9 79_O 79.9 83.8
3t 38 39 40 41 42 43 44 45 46 47 48 49 50 5l 52 53 54
Rb Sr Y Zr Nb Mo Tc Ru Rh Pd Ag Cd In Sn Sb Te I Xe
85.5 87.6 88.9 91.2 92.9 95.9 (e8) 101.1 102.9 106.4 107.9 t12.4 114.8 118.7 127.8 127.6 126.9 131.3
55 56 57. 72 t3 74 75 76 77 78 79 80 81 82 83 84 85 86
Cs Ba Lal Hf Ta w Re Os Ir Pt Au Irg TI Pb Bi Po At Rn
132.9 137.3 138.9 178.5 180.9 183.9 186.2 190.2 t92.2 195. I 197.0 200.6 2M.4 201.2 209.0 (20e) (210) (222)
8-l 88 104 105 106 1W 108 r09 r10 lll 1t2
Fr Ra i?u Rf Db Sg Bh Hs Mt Uun Uuu Uub
(223) 226.0 227.0 (261) (262) (263) (262) (26s) (266) (269\ (272) (27',|)

58 59 60 61 62 63 64 65 66 67 68 69 70 7l
Ce Pr Nd Pm Sm Eu Gd Tb Dy Ho Er Tm Yb Lu
140.1 140.9 r44.2 (145) 150.4 152.0 r5'7.3 158.9 r62.5 164.9 167.3 168.9 173.0 175.0

90 9l 92 93 94 95 96 97 98 99 100 101 102 103


Th Pa U Np Pu Am Cm BK cf Es Fm Md No Lr
232.0 (231) 238.0 (237) (244) (243) (247) (247) (251) (252) (2s7) (2s8) (2s9) (260)

Specializing in MCAT Preparation

You might also like